FAR 1 Flashcards

1
Q

The SEC is comprised of five commissioners, appointed by the President of the United States, and five divisions. Which of the following divisions is responsible for overseeing compliance with the securities acts?

A

The Division of Corporate Finance oversees the compliance with the securities acts and examines all filings made by publicly held companies.

How well did you know this?
1
Not at all
2
3
4
5
Perfectly
2
Q

The SEC is comprised of five commissioners, appointed by the President of the United States, and five divisions. divisions. Which of the following divisions is responsible for completing the investigation and takes appropriate actions when there is a violation of a securities law (except the Public Utility Holding Company Act).

A

The Division of Enforcement completes the investigation and takes appropriate actions when there is a violation of a securities law (except the Public Utility Holding Company Act). This division makes recommendations to the Justice Department concerning any punishments or potential criminal prosecution.

How well did you know this?
1
Not at all
2
3
4
5
Perfectly
3
Q

The SEC is comprised of five commissioners, appointed by the President of the United States, and five divisions. divisions. Which of the following divisions is responsible for overseeing the secondary markets, exchanges, brokers, and dealers.

A

The Division of Trading and Markets oversees the secondary markets, exchanges, brokers, and dealers.

How well did you know this?
1
Not at all
2
3
4
5
Perfectly
4
Q

The SEC is comprised of five commissioners, appointed by the President of the United States, and five divisions. divisions. Which of the following divisions is responsible for overseeing the investment advisers and investment companies under the Investment Company Act of 1940 and the Investment Advisers Act of 1940.

A

The Division of Investment Management oversees the investment advisers and investment companies under the Investment Company Act of 1940 and the Investment Advisers Act of 1940.

How well did you know this?
1
Not at all
2
3
4
5
Perfectly
5
Q

The SEC enforces the corporate registration requirements of the Securities Act of 1933 as one of its principal objectives. These requirements are intended to provide information that enables the SEC to:

A

The mission of the SEC is to protect investors; maintain fair, orderly, and efficient markets; and facilitate capital formation. In order to carry out the mandates in the Securities Act of 1933, the SEC is ensuring that investors are provided with adequate information on which to base investment decisions.

How well did you know this?
1
Not at all
2
3
4
5
Perfectly
6
Q

A nonaccelerated filer, as established by the U.S. Securities and Exchange Commission, includes companies with less than exactly what amount in public equity float?

A

A nonaccelerated filer is defined by the SEC as an entity with less than $75 million in public market value.

How well did you know this?
1
Not at all
2
3
4
5
Perfectly
7
Q

Which of the following reports would a company file to meet the U.S. Securities and Exchange Commission’s requirements for unaudited, interim financial statements reviewed by an independent accountant?

A

Form 10-Q is the form for quarterly filing by a public entity with securities listed in the United States.

How well did you know this?
1
Not at all
2
3
4
5
Perfectly
8
Q

What is the Form 10-K?

A

Form 10-K is the form for annual filing by a public entity with securities listed in the United States.

How well did you know this?
1
Not at all
2
3
4
5
Perfectly
9
Q

What is the form 14A Proxy Statement?

A

The proxy statement is the form by which management requests the right to vote through proxy for shareholders at meetings.

How well did you know this?
1
Not at all
2
3
4
5
Perfectly
10
Q

What is the Form S-1?

A

Form S-1 is the basic registration form for new securities to be listed in the United States.

How well did you know this?
1
Not at all
2
3
4
5
Perfectly
11
Q

Wood Co.’s dividends on noncumulative preferred stock have been declared but not paid. Wood has not declared or paid dividends on its cumulative preferred stock in the current or the prior year and has reported a net loss in the current year. For the purpose of computing basic earnings per share, how should the income available to common stockholders be calculated?

A

The dividends on the noncumulative preferred stock and the current-year dividends on the cumulative preferred stock should be added to the net loss.

the dividends subtracted in computing basic EPS are (1) the annual dividend commitment on cumulative preferred whether or not declared or paid, and (2) declared dividends on noncumulative preferred whether paid or not. The firm has negative income. This answer means that the dividends reduce the numerator further - beyond the loss. The final numerator amount is less than (more negative than) the loss. Also, arrear dividends are never included in EPS because they were subtracted in computing EPS in a previous year.

How well did you know this?
1
Not at all
2
3
4
5
Perfectly
12
Q

During the current year, Comma Co. had outstanding: 25,000 shares of common stock, 8,000 shares of $20 par, 10% cumulative preferred stock, and 3,000 bonds that are $1,000 par and 9% convertible. The bonds were originally issued at par, and each bond was convertible into 30 shares of common stock. During the year, net income was $200,000, no dividends were declared, and the tax rate was 30%.

What amount was Comma’s basic earnings per share for the current year?

A

One year of preferred stock dividends is subtracted from income in the numerator of EPS because the stock is cumulative. The amount of dividends declared does not affect the calculation. The bonds are not relevant because basic EPS does not assume conversion of the bonds. The calculation is: Basic EPS = [$200,000 - (8,000 × $20 × .10)]/25,000 = ($200,000 − $16,000)/25,000 = $7.36.

How well did you know this?
1
Not at all
2
3
4
5
Perfectly
13
Q

Chape Co. had the following information related to common and preferred shares during the year:

Common shares outstanding, 1/1 700,000
Common shares repurchased, 3/31 20,000
Conversion of preferred shares, 6/30 40,000
Common shares repurchased, 12/1 36,000

Chape reported net income of $2,000,000 at December 31. What amount of shares should Chape use as the denominator in the computation of basic earnings per share?

A

Weighted average shares outstanding are weighted by the number of months the shares were outstanding during the year. The easiest way to do this is to take each change in common stock and multiply by the number of months remaining - add the shares that increased shares outstanding and subtract shares that reduced shares outstanding.

Shares	Months	Wtd avg
700,000	12/12	700,000
- 20,000	9/12	         - 15,000
\+40,000	6/12	         +20,000
-36,000	1/12	           - 3,000
                                 702,000
How well did you know this?
1
Not at all
2
3
4
5
Perfectly
14
Q

Strauch Co. has one class of common stock outstanding and no other securities that are potentially convertible into common stock. During Year 1, 100,000 shares of common stock were outstanding. In Year 2, two distributions of additional common shares occurred:

On April 1, 20,000 shares of treasury stock were sold, and on July 1, a 2-for-1 stock split was issued.

Net income was $410,000 in Year 2 and $350,000 in Year 1.

What amounts should Strauch report as earnings per share in its Year 2 and Year 1 comparative income statements?

A

The 2-for-1 split in Year 2 does not substantively change the value of any shares outstanding. Without retroactive application, EPS would be cut roughly in half in Year 2 compared to Year 1. Yet there was little substantive change in the performance of the firm. For reporting in Year 2:

Weighted average shares, Year 1 = 100,000(2) = 200,000.

EPS, Year 1 = $350,000/200,000 = $1.75.

Weighted average shares, 2006 = [100,000 + 20,000(9/12)]2 = 230,000

EPS, 2006 = $410,000/230,000 = $1.78.

Had the Year 1 shares not been adjusted for the split, Year 1 EPS would be $3.50 = $350,000/100,000, or roughly double the EPS of Year 2. Without retroactive application, it would appear that the firm had a drastic reduction in EPS in Year 2. The retroactive application of the split ensures that the base on which EPS is computed uses the same measuring unit.

How well did you know this?
1
Not at all
2
3
4
5
Perfectly
15
Q

The treasury stock method of entering stock options into the calculation of diluted EPS:

A

Is called the treasury stock method because the proceeds from assumed exercise are assumed to be used to purchase treasury stock.

Firms may use the proceeds from the exercise of stock options for any purpose. However, to promote uniformity in reporting, and to reduce the dilution from exercise, the assumption is that the proceeds are used to purchase the firm’s stock on the market. This reduces the net number of new shares outstanding from assumed exercise.

How well did you know this?
1
Not at all
2
3
4
5
Perfectly
16
Q

A firm with a net income of $30,000 and weighted average actual shares outstanding of 15,000 for the year also had the following two securities outstanding the entire year: (1) 2,000 options to purchase one share of stock for $12 per share. The average share price during the year was $20, (2) cumulative convertible preferred stock with an annual dividend commitment of $4,500. Total common shares issued on conversion are 2,900. Compute diluted EPS for this firm.

A

The options and convertible preferred stock are potential common stock (PCS). First compute basic EPS as the basis for diluted EPS, and also as a benchmark for determining whether the two potential common stock securities are dilutive. Basic EPS = ($30,000 – $4,500)/15,000 = $1.70. The preferred dividend is subtracted from income because the preferred is cumulative. Then determine the numerator and denominator effects of the PCS to enter them into diluted EPS in the order of lowest ratio of numerator to denominator effect (n/d) first. The option’s numerator effect is zero; the denominator effect = 2,000 – (2,000)$12/$20 = 800. 2,000 shares would be issued upon exercise but under the treasury stock method the firm is assumed to apply the proceeds from exercise (2,000 × $12) and purchase shares of the firm’s stock for $20 each. Thus, the n/d for options = 0/800 = 0. The n/d for the convertible preferred stock is the ratio of dividends that would not have been declared if the stock converted, to the common shares assumed issued on conversion. n/d = $4,500/2,900 = $1.55. Enter the options into diluted EPS first, because the options have the lower n/d. DEPS tentative = ($30,000 − $4,500)/(15,000 + 800) = $1.61. The convertible preferred is dilutive because its n/d ratio of $1.55 is less than $1.61, the tentative or first-pass amount for diluted EPS. DEPS final = ($30,000 – $4,500 + $4,500)/(15,000 + 800 + 2,900) = $1.60.

How well did you know this?
1
Not at all
2
3
4
5
Perfectly
17
Q

LM Company has net income of $130,000, weighted average shares of common stock outstanding of 50,000, and preferred dividends for the period of $20,000. What is LM’s earnings per share of common stock?

A

Earnings per share (EPS) is calculated on net income available to the common stockholders, $130,000 − $20,000, or $110,000, divided by weighted average shares of common stock outstanding, 50,000. The EPS = $110,000 / 50,000 = $2.20

How well did you know this?
1
Not at all
2
3
4
5
Perfectly
18
Q

If everything else is held constant, earnings per share is increased by:

A

Purchase of treasury stock.

Earnings per share is calculated by dividing earnings (profit) available to common stockholders by weighted average number of shares of common stock outstanding. If the denominator is decreased by purchasing treasury stock, then the EPS result is increased.

How well did you know this?
1
Not at all
2
3
4
5
Perfectly
19
Q

Cott Co.’s four business segments have revenues and identifiable assets expressed as percentages of Cott’s total revenues and total assets as follows:

Revenues	Assets
Ebon	64%	          66%
Fair	         14%	           18%
Gel	         14%	            4%
Hak	\_\_      8%_	_ 12%_
                100%	 100%
               ======	======
Which of these business segments are deemed to be reportable segments?
A

Ebon, Fair, Gel, and Hak

FAS 131, issued in 1997, uses the term “operating segments” rather than business segments. All four meet at least one of the three criteria for a reportable segment. A segment needs to meet only one of these criteria to be reportable (that is, required to report income and other data separately).

The three criteria are (summarized):

segment revenue is 10% or more of total revenue for all reported operating segments,
segment profit or loss is 10% or more of total profit for those segments reporting a profit, or 10% of total loss for those segments reporting a loss, whichever is greater in absolute amount, and
segment assets are 10% or more of total assets of all operating segments. Thus, each segment meets at least one of the three criteria.

How well did you know this?
1
Not at all
2
3
4
5
Perfectly
20
Q

Grum Corp., a publicly owned corporation, is subject to the requirements for segment reporting.

In its income statement for the year ending December 31, 2004, Grum reported revenues of $50,000,000, operating expenses of $47,000,000, and net income of $3,000,000. Operating expenses include payroll costs of $15,000,000. Grum’s combined identifiable assets of all industry segments at December 31, 2004 were $40,000,000.

In its 2004 financial statements, Grum should disclose major customer data if sales to any single customer amount to at least

A

Under FAS 131 (1997), if revenues from transactions with a single customer amount to 10% or more of a firm’s total revenue, that fact must be disclosed, along with the total revenues from each such customer.

For this firm with revenues of $50,000,000, 10% of total revenues is $5,000,000.

How well did you know this?
1
Not at all
2
3
4
5
Perfectly
21
Q

Yellow Co. received a large worker’s compensation claim of $90,000 in the third quarter for an injury occurring in the third quarter. How should Yellow account for the transaction in its interim financial report?

A

Recognize $90,000 in the third quarter.

The worker’s compensation claim should be reported in the period incurred, the third quarter. This is a transaction that occurred in the third quarter and does not impact other quarters.

How well did you know this?
1
Not at all
2
3
4
5
Perfectly
22
Q

On January 16, Tree Co. paid $60,000 in property taxes on its factory for the current calendar year. On April 2, Tree paid $240,000 for unanticipated major repairs to its factory equipment. The repairs will benefit operations for the remainder of the calendar year. What amount of these expenses should Tree include in its third quarter interim financial statements for the three months ended September 30?

A

Interim reporting treats each interim period as an integral part of the annual period. The two expenditures, in this problem, benefit more than one quarter. Thus, the expenses are recognized in the periods benefited, rather than only in the period of expenditure. The property taxes benefit all four quarters; therefore, $15,000 ($60,000/4) is recognized each quarter. The repair benefits three quarters; therefore, $80,000 ($240,000/3) is recognized each quarter. The sum of the two amounts is $95,000 to be recognized in quarter three.

How well did you know this?
1
Not at all
2
3
4
5
Perfectly
23
Q

A planned volume variance in the first quarter, which is expected to be absorbed by the end of the fiscal period, ordinarily should be deferred at the end of the first quarter if it is:

A

Paragraph 14.d. of APB Opinion 28 states: “Purchase price variances or volume or capacity cost variances that are planned and expected to be absorbed by the end of the annual period, should ordinarily be deferred at interim reporting dates.”

The reason for the deferral is that, from the point of view of the entire reporting year, there will be no volume variance. The Opinion requires the integral view of interim reporting for most items - that an interim period is an integral part of the annual period. Recognizing, rather than deferring, the variance in the first quarter would cause the reporting of the first quarter results to be unrepresentative of the annual period of which it is a part.

How well did you know this?
1
Not at all
2
3
4
5
Perfectly
24
Q

Kell Corp. reported $111,000 of net income for the quarter ended September 30, 20X5. Additional information for the quarter:

  • A $60,000 gain from discontinued operation, realized on April 30, 20X5, was allocated equally to the second, third, and fourth quarters of 2005.
  • A $16,000 cumulative-effect adjustment (dr.) resulting from a change in inventory valuation method was recognized on August 2, 20X5. The new method was used for the quarter ended September 30. The $111,000 earnings amount does not reflect the cumulative effect.

In addition, Kell paid $48,000 on February 1, 20X5, for 20X5 calendar-year property taxes. Of this amount, $12,000 was allocated to the third quarter of 20X5.

For the quarter ended September 30, 20X5, Kell should report net income of:

A

The gain from discontinued operations should be recognized entirely in the second quarter. There is no meaningful basis on which to allocate the gain. It is a one-time occurrence. The cumulative effect is not recognized in income. The firm’s treatment of the property tax cost is correct. The cost relates to the entire year. Therefore, each quarter should bear 1/4 of the cost.

Third quarter net income = $91,000 = $111,000 − $60,000/3.

How well did you know this?
1
Not at all
2
3
4
5
Perfectly
25
Q

Which one of the following is not an other comprehensive basis of accounting?

A

Pure accrual basis accounting is not an other comprehensive basis of accounting. The concept of “other comprehensive basis” means a comprehensive basis of accounting other than pure (or full) accrual accounting.

How well did you know this?
1
Not at all
2
3
4
5
Perfectly
26
Q

Alco, Inc., a small manufacturing company, prepares its financial statements using its income tax basis of accounting. In December, 2012, it determined that an error had been made in the amount of rent expense reported in its 2011 tax return. How should Alco account for the amount of the rental expense error in its 2012 financial statements?

A

The amount of the rental expense error made in the tax return (and financial statements) of the prior period would be reported as a prior period adjustment in Alco’s 2012 financial statements.

How well did you know this?
1
Not at all
2
3
4
5
Perfectly
27
Q

If a company that is not a public business entity wants to apply the simplified hedge accounting approach to a cash flow hedge of a variable rate borrowing with a receive-variable, pay-fixed interest rate swap, which of the following is a condition that must be met?

A

The variable interest rate on the interest rate swap and the variable interest rate on the hedged borrowing are linked to the same index.

To qualify for simplified hedge accounting, one of the criteria is that both the swap and the hedged borrowing are linked to the same index.

How well did you know this?
1
Not at all
2
3
4
5
Perfectly
28
Q

A private company decided to adopt one of the standards issued by the Private Company Council. What are the requirements upon adoption of the PCC standard?

A

Apply the new standard on a prospective basis.

Adoption of a private company standard is done on a prospective basis.

How well did you know this?
1
Not at all
2
3
4
5
Perfectly
29
Q

Even though the SEC delegates the creation of accounting standards to the private sector, the SEC frequently comments on accounting and auditing issues. The main pronouncements published by the SEC are:

A

Financial Reporting Releases (FRR).

The main pronouncements published by the SEC are the Financial Reporting Releases (FRR) and the Staff Accounting Bulletins (SAB).

How well did you know this?
1
Not at all
2
3
4
5
Perfectly
30
Q

Which of the following is the annual report that is filed with the United States Securities and Exchange Commission (SEC)?

A

Form 10-K.

How well did you know this?
1
Not at all
2
3
4
5
Perfectly
31
Q

What is Form 8-K?

A

Form 8-K is used to report significant events that effect the company such as a business combination or bankruptcy.

How well did you know this?
1
Not at all
2
3
4
5
Perfectly
32
Q

What is Form 10-Q?

A

Form 10-Q is the quarterly report filed with the SEC.

How well did you know this?
1
Not at all
2
3
4
5
Perfectly
33
Q

What is Form S-1?

A

Form S-1 is filed with the SEC in a public offering.

How well did you know this?
1
Not at all
2
3
4
5
Perfectly
34
Q

A nonaccelerated filer, as established by the U.S. Securities and Exchange Commission, includes companies with less than exactly what amount in public equity float?

A

A nonaccelerated filer is defined by the SEC as an entity with less than $75 million in public market value.

How well did you know this?
1
Not at all
2
3
4
5
Perfectly
35
Q

Which of the following reports would a company file to meet the U.S. Securities and Exchange Commission’s requirements for unaudited, interim financial statements reviewed by an independent accountant?

A

Form 10-Q is the form for quarterly filing by a public entity with securities listed in the United States.

How well did you know this?
1
Not at all
2
3
4
5
Perfectly
36
Q

What is 14A Proxy Statement?

A

The proxy statement is the form by which management requests the right to vote through proxy for shareholders at meetings.

How well did you know this?
1
Not at all
2
3
4
5
Perfectly
37
Q

Which of the following is not a required component of the 10-K filing?

A

Product market share.

The market share of the company’s product is not a required disclosure. The company may chose to voluntarily present this information, but it is not a required disclosure.

How well did you know this?
1
Not at all
2
3
4
5
Perfectly
38
Q

Which regulation governs the form and content of financial statement disclosures?

A

Regulation S-X

Regulation S-X governs the form and content of financial statements and financial statement disclosures for publicly traded entities

How well did you know this?
1
Not at all
2
3
4
5
Perfectly
39
Q

What does the Sarbanes Oxley govern?

A

Sarbanes Oxley enhances corporate governance to mitigate financial accounting abuses.

How well did you know this?
1
Not at all
2
3
4
5
Perfectly
40
Q

What does S-K govern?

A

Regulation S-K governs the form and content of non-financial statement disclosures.

How well did you know this?
1
Not at all
2
3
4
5
Perfectly
41
Q

Which of the following is required by Regulation S-K to be included in the Management’s Discussion and Analysis (MD&A) that is part of the 10-K?

A

Discussion of risks and uncertainties.

The SEC requires that the MD&A provide a “discussion and analysis” on operating results, liquidity, and capital resources, trends, and risks and uncertainties. Where there are significant increases in sales, management must discuss the extent that price, volume, or new products contributed to the increase.

How well did you know this?
1
Not at all
2
3
4
5
Perfectly
42
Q

A company that is a large accelerated filer must file its Form 10-Q with the United States Securities and Exchange Commission within how many days after the end of the period?

A

40 days

A large accelerated filer is a company with worldwide market value of outstanding voting and nonvoting common equity held by nonaffiliates of $700 million or more. A large accelerated filer must file its 10Q within 40 days after quarter end.

How well did you know this?
1
Not at all
2
3
4
5
Perfectly
43
Q

True or False

A company can use either the purchase or the pooling method for business combinations.

A

False

How well did you know this?
1
Not at all
2
3
4
5
Perfectly
44
Q

True or False

Total revenues for a consolidated entity are calculated by adding the total revenue of the parent for the year and the total revenue of the subsidiary from the date of acquisition, less any intercompany sales.

A

True

How well did you know this?
1
Not at all
2
3
4
5
Perfectly
45
Q

True or False

Goodwill is amortized over a period of forty years.

A

False

How well did you know this?
1
Not at all
2
3
4
5
Perfectly
46
Q

True or False

Goodwill is classified and included with other intangible assets for purposes of measuring impairment.

A

False

How well did you know this?
1
Not at all
2
3
4
5
Perfectly
47
Q

True or False

Intangible assets with indefinite lives are not amortized.

A

True

How well did you know this?
1
Not at all
2
3
4
5
Perfectly
48
Q

True or False

The stockholder equity accounts in the consolidated balance sheet are those of the parent company.

A

True

How well did you know this?
1
Not at all
2
3
4
5
Perfectly
49
Q

True or False

The effect of unrealized profits and sales between the acquirer and acquiree is classified as an intercompany transaction and must be eliminated.

A

True

How well did you know this?
1
Not at all
2
3
4
5
Perfectly
50
Q

True or False

A newly identified intangible asset must meet the contractual legal or the separability criterion to be recognized as an intangible asset at the time of consolidation.

A

True

How well did you know this?
1
Not at all
2
3
4
5
Perfectly
51
Q

Multi Step Income Statement

A
Net Sales
COGS
=Gross Margin 
\+Operating Expenses
=Operating Margin
-Interest Income
\+Interest Expense
- Other Rev, Gains, Expenses, & Losses
=Pretax income from Continuing Operations
\+Income Tax Expense
-Income from continuing operations
-Income from discontinued operations (next tax)
=Net Income

EPCS
Income from continuing operations:
Income from continuing operations:
Net income:

How well did you know this?
1
Not at all
2
3
4
5
Perfectly
52
Q

Under GAAP, which of the following can be issued as the primary form of public financial statement disclosure for a parent and its subsidiaries?

A

Under GAAP, only consolidated financial statements may be issued as the primary form of public disclosure for a parent and its subsidiaries. Parent only statements and separate parent and subsidiary statements may not be issued in lieu of consolidated financial statements.

How well did you know this?
1
Not at all
2
3
4
5
Perfectly
53
Q

Which one of the following methods, if any, may a parent use on its books to carry an investment in a subsidiary that it will consolidate?

A

A parent may use the cost method, the equity method, or any other method on its books to carry an investment in a subsidiary that it will consolidate. The method that is used on its books will affect the consolidating process, but the final consolidated financial statements will be the same regardless of the method the parent uses on its books.

How well did you know this?
1
Not at all
2
3
4
5
Perfectly
54
Q

Aceco has significant investments in three separate entities. These investments are:

  1. 40% ownership of the voting stock of Kapco.
  2. 60% ownership of the voting stock of Placo.
  3. 100% ownership of the voting stock of Simco

Which of Aceco’s investments would be consolidated with Aceco in its consolidated financial statements?

A

Since Aceco owns controlling interest in Placo (60%) and in Simco (100%), each would be consolidated with Aceco. Kapco would not be consolidated, because Aceco does not have controlling interest in Kapco. In Aceco’s consolidated financial statements, Kapco would be shown as an investment.

How well did you know this?
1
Not at all
2
3
4
5
Perfectly
55
Q

Sun Co. is a wholly owned subsidiary of Star Co. Both companies have separate general ledgers and prepare separate financial statements. Sun requires stand-alone financial statements. Which of the following statements is correct?

A

Consolidated statements should be prepared by Star, the parent, and not by Sun, the subsidiary. Star has an investment in and control of Sun, which is the basis for preparing consolidated statements; Sun does not have an investment in, or control of, Star. Thus, there is no basis for Sun to prepare consolidated statements.

How well did you know this?
1
Not at all
2
3
4
5
Perfectly
56
Q

The preparation of consolidated statements likely will require the following information about the subsidiary’s assets and liabilities at the date of acquisition:

A

Both book values and fair values of a subsidiary’s assets and liabilities will need to be determined at the date of acquisition in order to prepare consolidated financial statements after a business combination.

How well did you know this?
1
Not at all
2
3
4
5
Perfectly
57
Q

Penn, Inc., a manufacturing company, owns 75% of the common stock of Sell, Inc., an investment company. Sell owns 60% of the common stock of Vane, Inc., an insurance company.

In Penn’s consolidated financial statements, should consolidation accounting or equity method accounting be used for Sell and Vane?

A

Consolidation used for both Sell and Vane.

If one looked just at Penn’s interest in Vane’s result of 45% (75% × 60%), one might say that the equity method would be appropriate.

However, because Sell owns 60% of Vane, it controls Vane and would need to consolidate Vane. Because Penn owns 75% of Sell, it controls Vane and would need to consolidate Sell, which consolidated Vane. Thus, all three would be consolidated, making this response correct.

How well did you know this?
1
Not at all
2
3
4
5
Perfectly
58
Q

An investor will report an investment in its financial statements using a different method than it uses to carry the investment on its books if its minimum ownership of the investee is:

A

50+%

If an investor owns 50+% (up to and including 100%) of an investee, it will normally carry the investment on its books using the cost method, the equity method, or some other method, but it will report the investment in its financial statements as a consolidated subsidiary. The method used on the investor’s books will be different than the method used to report the investment in financial statements.

How well did you know this?
1
Not at all
2
3
4
5
Perfectly
59
Q

Consolidated financial statements are typically prepared when one company has a controlling financial interest in another unless:

A

The subsidiary is in bankruptcy.

Currently, the only reasons allowable for not consolidating a majority-owned subsidiary is where control does not reside with the majority owner, making this the correct response.

How well did you know this?
1
Not at all
2
3
4
5
Perfectly
60
Q

Which of the following information that exists at the date of an acquisition will be needed to carry out the consolidating process?

I. Book values of a subsidiary’s assets and liabilities.
II. Fair values of a subsidiary’s assets and liabilities.
III. Parent’s cost of its investment in the subsidiary.

A

In order to prepare consolidated financial statements, the parent needs the book values and fair values of a subsidiary’s assets and liabilities at the date of the business combination as well as the Parent’s cost of its investment in the subsidiary.

How well did you know this?
1
Not at all
2
3
4
5
Perfectly
61
Q

Consolidated financial statements are based on the concept that:

A

In the preparation of financial statements, economic substance takes precedence over legal form.

The preparation of consolidated financial statements is based on the concept that economic substance takes precedence over legal form. In form, the corporations are separate legal entities, but in substance, they are under the common economic control of the parent’s shareholders.

How well did you know this?
1
Not at all
2
3
4
5
Perfectly
62
Q

A subsidiary, acquired for cash in a business combination, owned inventories with a market value different from the book value as of the date of combination. A consolidated balance sheet prepared immediately after the acquisition would include this difference as part of:

A

Inventories

The difference between (fair) market value and book value of inventories would be recognized by adjusting inventories to fair value on the consolidated balance sheet.

How well did you know this?
1
Not at all
2
3
4
5
Perfectly
63
Q

On November 30, 2004, Parlor, Inc. purchased for cash at $15 per share all 250,000 shares of the outstanding common stock of Shaw Co.

On November 30, 2004, Shaw’s balance sheet showed a carrying amount of net assets of $3,000,000. On that date, the fair value of Shaw’s property, plant, and equipment exceeded its carrying amount by $400,000.

In its November 30, 2004, consolidated balance sheet, what amount should Parlor report as goodwill?

A

Goodwill is the difference between the purchase price of $3,750,000 (250,000 × $15.00) and the fair value of the net assets ($3,000,000 + $400,000) or $350,000.

How well did you know this?
1
Not at all
2
3
4
5
Perfectly
64
Q

A subsidiary, acquired for cash in a business combination, owned equipment with a market value in excess of book value as of the date of combination. A consolidated balance sheet prepared immediately after the acquisition would treat this excess as:

A

Plant and Equipment

The excess of (fair) market value over book value of equipment would be recognized by writing up plant and equipment to fair value on the consolidated balance sheet.

How well did you know this?
1
Not at all
2
3
4
5
Perfectly
65
Q

Beni Corp. purchased 100% of Carr Corp.’s outstanding capital stock for $430,000 cash. Immediately before the purchase, the balance sheets of both corporations reported the following:

                                    Beni	        Carr Assets	                    $2,000,000	$750,000 Liabilities	            $   750,000	$400,000 Common stock	    1,000,000	        310,000 Retained earnings	\_\_250,000	\_\_40,000 Liabilities and 

stockholders’ equity $2,000,000 $750,000

On the date of purchase, the fair value of Carr’s assets was $50,000 more than the aggregate carrying amounts. In the consolidated balance sheet prepared immediately after the purchase, the consolidated stockholders’ equity should amount to:

A

$1,250,000

On the date of a business combination using acquisition accounting, the consolidated stockholders’ equity will exactly equal the parent company stockholders’ equity. This will continue to be the case as long as the parent company uses a complete equity method of accounting for the subsidiary.

How well did you know this?
1
Not at all
2
3
4
5
Perfectly
66
Q

Pride, Inc. owns 80% of Simba, Inc.’s outstanding common stock. Simba, in turn, owns 10% of Pride’s outstanding common stock.

What percentage of the common stock cash dividends declared by the individual companies should be reported as dividends declared in the consolidated financial statements?

A

Dividends declared by Pride 90%
Dividends declared by Simba 0%

Subsidiary dividends are never considered part of consolidated dividends. They are either eliminated in the consolidation entries or allocated to reducing noncontrolling interests. In this problem, 80% of Simba’s dividends will be eliminated as intercompany, and 20% will be allocated to reducing noncontrolling interest.

In addition, since 10% of the dividends of Pride never go outside the consolidated entity, they are not considered dividends of the consolidated entity either.

How well did you know this?
1
Not at all
2
3
4
5
Perfectly
67
Q

Rowe Inc. owns 80% of Cowan Co.’s outstanding capital stock. On November 1, Rowe advanced $100,000 in cash to Cowan. What amount should be reported related to the advance in Rowe’s consolidated balance sheet as of December 31?

A

$-0-

All intercompany receivables and payables should be eliminated in the preparation of a consolidated balance sheet so that no intercompany receivables/payables are reported. In this question, an eliminating entry should be made that eliminates (credits) Rowe’s receivable from Cowan against (debits) Cowan’s payable to Rowe, both for $100,000. Since the receivable/payable is between the affiliated entities, 100% (not 80%) of the intercompany amount should be eliminated.

How well did you know this?
1
Not at all
2
3
4
5
Perfectly
68
Q

On December 31, 2005, Grey, Inc. owned 90% of Winn Corp., a consolidated subsidiary, and 20% of Carr Corp., an investee in which Grey cannot exercise significant influence. On the same date, Grey had receivables of $300,000 from Winn and $200,000 from Carr.

In its December 31, 2005, consolidated balance sheet, Grey should report accounts receivable from affiliates of:

A

$200,000

A 90% owned subsidiary will be consolidated, and any intercompany receivables such as these are eliminated from both parties’ books in the consolidating process.

Therefore, the $300,000 receivable from Winn will not appear on the consolidated balance sheet at all. The $200,000 from Carr is a receivable from an affiliate (20% owned) and will need to be reported as such.

How well did you know this?
1
Not at all
2
3
4
5
Perfectly
69
Q

King, Inc. owns 70% of Simmon Co.’s outstanding common stock. King’s liabilities total $450,000, and Simmon’s liabilities total $200,000. Included in Simmon’s financial statements is a $100,000 note payable to King. What amount of total liabilities should be reported in the consolidated financial statements?

A

$550,000

The consolidated financial statements should reflect 100% of the assets and liabilities of the subsidiary less any intercompany balances. Therefore the balance on the consolidated balance sheet should be: $450,000 + 200,000 − 100,000 = $550,000.

How well did you know this?
1
Not at all
2
3
4
5
Perfectly
70
Q

Which one of the following is not a characteristic of intercompany bonds?

A

When bonds become intercompany, they are written off of the books of the issuing affiliate and the investing affiliate.

The liability and investment related to intercompany bonds are eliminated only on the consolidating worksheet. They are not written off the books of either the issuing or the investing affiliate. From the perspective of the separate companies, the liability and investment related to the bonds continue to exist, but for consolidated purposes, they have been constructively retired.

How well did you know this?
1
Not at all
2
3
4
5
Perfectly
71
Q

On December 31, 2008, Pico acquired $250,000 par value of the outstanding $1,000,000 bonds of its subsidiary, Sico, in the market for $200,000. On that date, Sico had a $100,000 premium on its total bond liability.

Which one of the following is the amount of premium or discount on Pico’s investment in Sico’s bonds?

A

$50,000 discount

The premium or discount on a bond investment is the difference between the par value of the bonds and the price paid for the bonds in the market. If the price paid is more than par value, there is a premium on the bond investment. If the price paid is less than par value, there is a discount on the bond investment. In this case, the price paid for the investment ($200,000) is less than the par value of the bonds ($250,000) by $50,000. Therefore, there is a $50,000 discount on Pico’s investment.

How well did you know this?
1
Not at all
2
3
4
5
Perfectly
72
Q

On December 31, 2008, Pico acquired $250,000 par value of the outstanding $1,000,000 bonds of its subsidiary, Sico, in the market for $200,000. On that date, Sico had a $100,000 premium on its total bond liability.

Which one of the following is the net carrying value of Sico’s total bond liability?

A

$1,100,000

A premium on a bond liability results from the sale of the bonds at a price in excess of par (face) value. Therefore, a premium would be added to par value to get net carrying value. Sico’s premium on its bond liability ($100,000) should be added to the par (or face) value of its bond liability ($1,000,000) to determine the net carrying value of the liability. Thus, the answer should be $1,000,000 par value + $100,000 premium = $1,100,000 net carrying value.

How well did you know this?
1
Not at all
2
3
4
5
Perfectly
73
Q

On December 31, 2008, Pico acquired $250,000 par value of the outstanding $1,000,000 bonds of its subsidiary, Sico, in the market for $200,000. On that date, Sico had a $100,000 premium on its total bond liability.

Which one of the following is the net amount of gain or loss that will be recognized by Pico in its December 31, 2008, consolidated financial statements as a result of its intercompany bonds?

A

$75,000

In the elimination of intercompany bonds, the intercompany bond liability at par will be eliminated against the intercompany bond investment at par. Therefore, the gain or loss recognized as a result of constructive retirement of intercompany bonds is the net of the premium or discount on the bond liability and the premium or discount on the bond investment. In this case, there is a total $100,000 premium on the bond liability, but because only one-fourth ($250,000/$1,000,000 = 1/4) of the bonds are intercompany, only one fourth of the premium is eliminated. Thus, $25,000 of premium on the bond liability (a credit) will be eliminated against the $50,000 discount on the bond investment (also a credit). As a result of eliminating the two credits ($25,000 + $50,000 = $75,000), a $75,000 gain on constructive retirement will be recognized.

How well did you know this?
1
Not at all
2
3
4
5
Perfectly
74
Q

On December 31, 2008, Pico acquired $250,000 par value of the outstanding $1,000,000 bonds of its subsidiary, Sico, in the market for $200,000. At that date, Sico had a $100,000 premium on its total bond liability.

Assume each company maintains its premium or discount in a separate account. Which one of the following will be the intercompany bond elimination entry made on the December 31, 2008 consolidating worksheet?

A

DR: Bonds Payable
Premium on Bonds Payable
Discount on Bond Investment

CR: Investment in Bonds
Gain on Constructive Retirement

The Investment in Bonds (a debit balance, so it will be credited) and the Bonds Payable (a credit balance, so it will be debited) will be eliminated against each other at par value ($250,000). The Discount on Bond Investment $50,000 (a credit balance, so it will be debited) and the Premium on Bonds Payable $25,000 (a credit balance, so it will be debited) will be eliminated resulting in a Gain on Constructive Retirement of $75,000, a credit balance.

How well did you know this?
1
Not at all
2
3
4
5
Perfectly
75
Q

Combined statements may be used to present the results of operation of:

A

Companies under common management: Yes
Commonly controlled companies: Yes

Combined financial statements are used (when consolidated statements are not appropriate) to show the aggregate results both for companies under common management and for companies under common control (and for unconsolidated subsidiaries).

How well did you know this?
1
Not at all
2
3
4
5
Perfectly
76
Q

Nolan owns 100% of the capital stock of both Twill Corp. and Webb Corp.

Twill purchases merchandise inventory from Webb at 140% of Webb’s cost. During Year 4, merchandise that cost Webb $40,000 was sold to Twill. Twill sold all of this merchandise to unrelated customers for $81,200 during Year 4. In preparing combined financial statements for Year 4, Nolan’s bookkeeper disregarded the common ownership of Twill and Webb.

By what amount was unadjusted revenue overstated in the combined income statement for Year 4?

A

$56,000

Since all the goods have been sold outside the combined entity, income recognition is correct.

However, sales and cost of goods sold have been recorded at two different points (i.e., the sale from Webb to Twill and the sale from Twill to outsiders). To the combined entity, Webb’s cost of merchandise (the original cost to the combined entity) is what is needed for cost of goods sold, and Twill’s sales (the amount the merchandise was sold for outside the combined entity) is needed for sales.

This means that the sale from Webb to Twill and the cost of goods recorded by Twill need to be eliminated. That amount is $56,000 (computed as $40,000 cost to Webb × transfer price to Twill of 140% of cost = $56,000).

How well did you know this?
1
Not at all
2
3
4
5
Perfectly
77
Q

In the preparation of combined financial statements, would the following issues be treated in the same way as when preparing consolidated financial statements or in a different way?

A

Minority Interest: Same
Foreign Operations: Same
Different Fiscal Periods: Same

According to ASC 810, if problems associated with minority interest, foreign operations, different fiscal periods, or income taxes occur in the preparation of combined financial statements, they should be treated in the same manner as in the preparation of consolidated financial statements. Therefore, all three items should be treated in the same manner as in consolidated statements.

How well did you know this?
1
Not at all
2
3
4
5
Perfectly
78
Q

Wood Co.’s dividends on noncumulative preferred stock have been declared but not paid. Wood has not declared or paid dividends on its cumulative preferred stock in the current or the prior year and has reported a net loss in the current year. For the purpose of computing basic earnings per share, how should the income available to common stockholders be calculated?

A

The dividends on the noncumulative preferred stock and the current-year dividends on the cumulative preferred stock should be added to the net loss.

In general, the dividends subtracted in computing basic EPS are (1) the annual dividend commitment on cumulative preferred whether or not declared or paid, and (2) declared dividends on noncumulative preferred whether paid or not. The firm has negative income. This answer means that the dividends reduce the numerator further - beyond the loss. The final numerator amount is less than (more negative than) the loss. Also, arrear dividends are never included in EPS because they were subtracted in computing EPS in a previous year.

How well did you know this?
1
Not at all
2
3
4
5
Perfectly
79
Q

During the current year, Comma Co. had outstanding: 25,000 shares of common stock, 8,000 shares of $20 par, 10% cumulative preferred stock, and 3,000 bonds that are $1,000 par and 9% convertible. The bonds were originally issued at par, and each bond was convertible into 30 shares of common stock. During the year, net income was $200,000, no dividends were declared, and the tax rate was 30%.

What amount was Comma’s basic earnings per share for the current year?

A

$7.36

One year of preferred stock dividends is subtracted from income in the numerator of EPS because the stock is cumulative. The amount of dividends declared does not affect the calculation. The bonds are not relevant because basic EPS does not assume conversion of the bonds. The calculation is: Basic EPS = [$200,000 - (8,000 × $20 × .10)]/25,000 = ($200,000 − $16,000)/25,000 = $7.36.

How well did you know this?
1
Not at all
2
3
4
5
Perfectly
80
Q

Jen Co. had 200,000 shares of common stock and 20,000 shares of 10%, $100 par value cumulative preferred stock. No dividends on common stock were declared during the year. Net income was $2,000,000. What was Jen’s basic earnings per share?

A

$9.00

Earnings per share is: (net income - preferred dividends)/common shares outstanding. Preferred stock dividends are $100 × 10% × 20,000 shares = $200,000. Earnings per share is (2,000,000-200,000)/200,000=$9 per share.

How well did you know this?
1
Not at all
2
3
4
5
Perfectly
81
Q

Balm Co. had 100,000 shares of common stock outstanding as of January 1. The following events occurred during the year:

4/1 Issued 30,000 shares of common stock.
6/1 Issued 36,000 shares of common stock.
7/1 Declared a 5% stock dividend.
9/1 Purchased as treasury stock 35,000 shares of its common stock. Balm used the cost method to account for the treasury stock.

What is Balm’s weighted average of common stock outstanding at December 31?

A

139,008

More than one approach is available to compute WA (each yields the same answer) but perhaps the easiest is to weight each item separately going forward to the end of the year. This approach yields 139,008 = 100,000(12/12) + 30,000(9/12) + 36,000(7/12) - 35,000(4/12). The beginning shares are outstanding the entire year (12/12). The next two items are weighted for the fraction of the year they are outstanding. Stock dividends and splits are retroactively applied to all items before their issuance - hence the multiplication by 1.05. The treasury shares are removed from the average for 4/12 of the year - these shares already reflect the stock dividend.

How well did you know this?
1
Not at all
2
3
4
5
Perfectly
82
Q

Chape Co. had the following information related to common and preferred shares during the year:

Common shares outstanding, 1/1 700,000
Common shares repurchased, 3/31 20,000
Conversion of preferred shares, 6/30 40,000
Common shares repurchased, 12/1 36,000

Chape reported net income of $2,000,000 at December 31. What amount of shares should Chape use as the denominator in the computation of basic earnings per share?

A

702,000

Weighted average shares outstanding are weighted by the number of months the shares were outstanding during the year. The easiest way to do this is to take each change in common stock and multiply by the number of months remaining - add the shares that increased shares outstanding and subtract shares that reduced shares outstanding.

Shares	Months	Wtd avg
700,000	12/12	700,000
- 20,000	9/12	        - 15,000
\+40,000	6/12	        +20,000
-36,000	1/12	          - 3,000
702,000
How well did you know this?
1
Not at all
2
3
4
5
Perfectly
83
Q

Strauch Co. has one class of common stock outstanding and no other securities that are potentially convertible into common stock. During Year 1, 100,000 shares of common stock were outstanding. In Year 2, two distributions of additional common shares occurred:

On April 1, 20,000 shares of treasury stock were sold, and on July 1, a 2-for-1 stock split was issued.

Net income was $410,000 in Year 2 and $350,000 in Year 1.

What amounts should Strauch report as earnings per share in its Year 2 and Year 1 comparative income statements?

A

Year 2 $ 1.78
Year 1 $ 1.75

For EPS purposes, stock dividends and splits are retroactively applied to all periods presented, and to all share changes within the year of the split or dividend.

For reporting in Year 2:
Weighted average shares, Year 1 = 100,000(2) = 200,000.

EPS, Year 1 = $350,000/200,000 = $1.75.

Weighted average shares, 2006 = [100,000 + 20,000(9/12)]2 = 230,000

EPS, 2006 = $410,000/230,000 = $1.78.

How well did you know this?
1
Not at all
2
3
4
5
Perfectly
84
Q

On January 31, 2004, Pack, Inc. split its common stock 2 for 1, and Young, Inc. issued a 5% stock dividend. Both companies issued their December 31, 2003, financial statements on March 1, 2004.

Should Pack’s 2003 earnings per share (EPS) take into consideration the stock split, and should Young’s 2003 EPS take into consideration the stock dividend?

A

Pack’s 2003 EPS - Yes
Young’s 2003 EPS - Yes

EPS is used primarily as an input to predictions of future earnings. The stock split and dividend cause the number of shares outstanding to increase, and thus affect the future earnings prospects on a per share basis. These events should be included in the computation of EPS even though they did not occur as of the balance sheet date. Financial statement users view the information as if it were current as of the date of publication.

How well did you know this?
1
Not at all
2
3
4
5
Perfectly
85
Q

A company reported net income available to common stockholders of $2,000,000 for the year ended December 31, year 2. The company had 1,500,000 shares of common stock outstanding as of January 1, year 2, and issued 500,000 additional shares of common stock on May 1, year 2. What amount is the company’s basic earnings per share for the year ended December 31, year 2?

A

$1.09

Basic earnings per share (EPS) is net income available to common shareholders divided by the weighted average shares outstanding. This question provided the net income so the task is to compute the weighted shares outstanding. The beginning shares outstanding are 1,500,000 plus the shares issued during the year of 500,000 × 8/12 (333,333), or 1,833,333. Therefore, basic EPS is $2,000,000 ÷ 1,833,333 = $1.09.

How well did you know this?
1
Not at all
2
3
4
5
Perfectly
86
Q

A company had 400,000 shares of common stock issued and outstanding on January 1, year 1, and had the following equity transactions for year 1:

Transactions Date
Issued 200,000 new shares for cash April 1

Issued new shares as a result of a 3-for-1 stock split
July 1

Purchased 300,000 shares treasury stock for cash
October 1

What should the company use as the denominator for the calculation of basic earnings per share for year ended December 31, year 1?

A

1,575,000

The weighted average shares outstanding are calculated as follows:

({[400,000+(200,000×9/12)]×3}−300,000×3/12)=1,575,000(1,574,999rounded)

How well did you know this?
1
Not at all
2
3
4
5
Perfectly
87
Q

LM Company has net income of $130,000, weighted average shares of common stock outstanding of 50,000, and preferred dividends for the period of $20,000. What is LM’s earnings per share of common stock?

A

$2.20

Earnings per share (EPS) is calculated on net income available to the common stockholders, $130,000 − $20,000, or $110,000, divided by weighted average shares of common stock outstanding, 50,000. The EPS = $110,000 / 50,000 = $2.20

How well did you know this?
1
Not at all
2
3
4
5
Perfectly
88
Q

If everything else is held constant, earnings per share is increased by:

A

Purchase of treasury stock.

Earnings per share is calculated by dividing earnings (profit) available to common stockholders by weighted average number of shares of common stock outstanding. If the denominator is decreased by purchasing treasury stock, then the EPS result is increased.

How well did you know this?
1
Not at all
2
3
4
5
Perfectly
89
Q

Why do preferred stock dividends appear in the calculation of earnings per share (EPS)?

A

Preferred stock dividends are subtracted from the earnings for the period in the calculation of earnings per share.

Earnings per share (EPS) is calculated on net income available to the common stockholders, divided by weighted average shares of common stock outstanding. The preferred dividends must be subtracted from the net income, as that amount is not available to the common stockholders.

How well did you know this?
1
Not at all
2
3
4
5
Perfectly
90
Q

AB Company reported earnings per share of $10.50 on income before discontinued operations, ($2.00) on income (loss) attributed to discontinued operations, and $8.50 on net income. Which EPS figure is more relevant to a potential investor?

A

$10.50

Potential investors and current investors are interested in the future earnings potential of the entity. Thus, they are interested in the earnings per share on continuing income, which would be the $10.50 per share. The EPS attributed to discontinued operations cannot be used in predicting future earnings, as they are one-time events.

How well did you know this?
1
Not at all
2
3
4
5
Perfectly
91
Q

For which of the following income statement sections is earnings per share calculated?

A

Income before discontinued operations.

Earnings per share (EPS) is calculated on income before discontinued operations, and net income.

How well did you know this?
1
Not at all
2
3
4
5
Perfectly
92
Q

The treasury stock method of entering stock options into the calculation of diluted EPS:

A

Is called the treasury stock method because the proceeds from assumed exercise are assumed to be used to purchase treasury stock.

Firms may use the proceeds from the exercise of stock options for any purpose. However, to promote uniformity in reporting, and to reduce the dilution from exercise, the assumption is that the proceeds are used to purchase the firm’s stock on the market. This reduces the net number of new shares outstanding from assumed exercise.

How well did you know this?
1
Not at all
2
3
4
5
Perfectly
93
Q

A firm with a net income of $30,000 and weighted average actual shares outstanding of 15,000 for the year also had the following two securities outstanding the entire year: (1) 2,000 options to purchase one share of stock for $12 per share. The average share price during the year was $20, (2) cumulative convertible preferred stock with an annual dividend commitment of $4,500. Total common shares issued on conversion are 2,900. Compute diluted EPS for this firm.

A

$1.60

Basic EPS = ($30,000 – $4,500)/15,000 = $1.70

The option’s numerator effect is zero; the denominator effect = 2,000 – (2,000)$12/$20 = 800. 2,000 shares would be issued upon exercise but under the treasury stock method the firm is assumed to apply the proceeds from exercise (2,000 × $12) and purchase shares of the firm’s stock for $20 each. Thus, the n/d for options = 0/800 = 0. The n/d for the convertible preferred stock is the ratio of dividends that would not have been declared if the stock converted, to the common shares assumed issued on conversion. n/d = $4,500/2,900 = $1.55. Enter the options into diluted EPS first, because the options have the lower n/d. DEPS tentative = ($30,000 − $4,500)/(15,000 + 800) = $1.61. The convertible preferred is dilutive because its n/d ratio of $1.55 is less than $1.61, the tentative or first-pass amount for diluted EPS. DEPS final = ($30,000 – $4,500 + $4,500)/(15,000 + 800 + 2,900) = $1.60.

How well did you know this?
1
Not at all
2
3
4
5
Perfectly
94
Q

The following information pertains to Ceil Co., a company whose common stock trades in a public market:

Shares outstanding at 1/1 100,000
Stock dividend at 3/31 24,000
Stock issuance at 6/30 5,000

What is the weighted average number of shares Ceil should use to calculate its basic earnings per share for the year ended December 31?

A

126,500

The stock dividend is considered to be outstanding since the beginning of the year. The weighted average is therefore:

100,000+24,000+ (5,000X6/12) = 126,500.

How well did you know this?
1
Not at all
2
3
4
5
Perfectly
95
Q

A public entity sells steel for use in construction. One of its customer’s accounts for 43% of sales, and another customer accounts for 40% of sales. What should the entity disclose in its annual financial statements about these two customers?

A

The amount of the entity’s revenue from each of the two customers.

The entity must disclose the amount of revenues received from a single customer that total 10% or more of total revenues.

How well did you know this?
1
Not at all
2
3
4
5
Perfectly
96
Q

Opto Co. is a publicly traded, consolidated enterprise reporting segment information. Which of the following items is a required enterprise-wide disclosure regarding external customers?

A

The fact that transactions with a particular external customer constitute more than 10% of the total enterprise revenues.

This is one of the disclosures required in FAS 131. The identity of the customer does not need to be disclosed, but the segment reporting the revenue must be identified. Such a segment would meet one of the three quantitative thresholds for reporting segment information. The three thresholds are 10% of revenue, income, and assets.

How well did you know this?
1
Not at all
2
3
4
5
Perfectly
97
Q

Which of the following types of entities are required to report on business segments?

A

Publicly traded enterprises

FAS No. 131 requires that a public business enterprise report financial and descriptive information about its reportable operating segments.

How well did you know this?
1
Not at all
2
3
4
5
Perfectly
98
Q

In financial reporting of segment data, which of the following must be considered in determining if an industry segment is a reportable segment?

A

Sales to unaffiliated customers - YES
Intersegment sales - YES

There are three possible quantitative tests to determine if a segment is reportable. If one or more of the tests is met, the segment is reported. One of these tests is the revenue test. This test determines if the segment’s revenue, which includes both sales to external (unaffiliated customers) and intersegment sales, is 10% or more of the combined revenue of all the company’s operating segments. Therefore, both items are considered.

How well did you know this?
1
Not at all
2
3
4
5
Perfectly
99
Q

Correy Corp. and its divisions are engaged solely in manufacturing operations. The following data (consistent with prior years’ data) pertain to the industries in which operations were conducted for the year ending December 31, 2005:

Industry Total revenue Operating profit Identifiable
assets at
12/31/89

A $10,000,000 $1,750,000 $20,000,000
B 8,000,000 1,400,000 17,500,000
C 6,000,000 1,200,000 12,500,000
D 3,000,000 550,000 7,500,000
E 4,250,000 675,000 7,000,000
F 1,500,000 225,000 3,000,000
$32,750,000 $5,800,000 $67,500,000

In its segment information for 2005, how many reportable segments does Correy have?

A

Five

Using the three quantitative thresholds (tests) from FAS 131 (Disclosures about Segments), the five following segments are reportable operating segments:

A, B, C and E meet the test: Reported revenue, including external and internal, is 10% or more of the combined revenue of all reported operating segments. 10% of $32,750,000 is $3,275,000. The revenues of A, B, C and E all exceed this amount.

D meets the test: Its assets (here $7,500,000 for D) are 10% or more of the combined assets of all operating segments (here $6,750,000 = .10 × $67,500,000).

F meets none of these tests.

Note: Some of the above segments meet more than one test. Only one needs to be met for a segment to be reportable.

How well did you know this?
1
Not at all
2
3
4
5
Perfectly
100
Q

Cott Co.’s four business segments have revenues and identifiable assets expressed as percentages of Cott’s total revenues and total assets as follows:

                  Revenues	Assets
Ebon	        64%	         66%
Fair	                 14%           18%
Gel	                 14%	            4%
Hak	\_\_               8%_	_12%_
                         100%	 100%
                         ======	======
Which of these business segments are deemed to be reportable segments?
A

Ebon, Fair, Gel, and Hak

FAS 131, issued in 1997, uses the term “operating segments” rather than business segments. All four meet at least one of the three criteria for a reportable segment. A segment needs to meet only one of these criteria to be reportable (that is, required to report income and other data separately).

The three criteria are (summarized):

segment revenue is 10% or more of total revenue for all reported operating segments,
segment profit or loss is 10% or more of total profit for those segments reporting a profit, or 10% of total loss for those segments reporting a loss, whichever is greater in absolute amount, and
segment assets are 10% or more of total assets of all operating segments. Thus, each segment meets at least one of the three criteria.

How well did you know this?
1
Not at all
2
3
4
5
Perfectly
101
Q

Grum Corp., a publicly owned corporation, is subject to the requirements for segment reporting.

In its income statement for the year ending December 31, 2004, Grum reported revenues of $50,000,000, operating expenses of $47,000,000, and net income of $3,000,000. Operating expenses include payroll costs of $15,000,000. Grum’s combined identifiable assets of all industry segments at December 31, 2004 were $40,000,000.

In its 2004 financial statements, Grum should disclose major customer data if sales to any single customer amount to at least

A

$5,000,000

Under FAS 131 (1997), if revenues from transactions with a single customer amount to 10% or more of a firm’s total revenue, that fact must be disclosed, along with the total revenues from each such customer.

For this firm with revenues of $50,000,000, 10% of total revenues is $5,000,000.

How well did you know this?
1
Not at all
2
3
4
5
Perfectly
102
Q

What information should a public company present about revenues from foreign operations?

A

Disclose separately the amount of sales to unaffiliated customers and the amount of intracompany sales between geographical areas.

Segment disclosure requires that companies disclose the amount of sales to unaffiliated customers by geographical region. They also require disclosure of intracompany sales between geographical areas. These cannot be aggregated but must be reported separately.

How well did you know this?
1
Not at all
2
3
4
5
Perfectly
103
Q

Terra Co.’s total revenues from its three business segments were as follows:

Segment Sales to
unaffiliated
customers Intersegment
sales Total
revenues
Lion $70,000 $30,000 $100,000
Monk 22,000 4,000 26,000
Nevi 8,000 16,000 24,000
—— —– ——
Combined $100,000 $50,000 $150,000
Elimination - (50,000) (50,000)
—— —– ——
Consolidated $100,000 $ - $100,000
======== ====== =======
Which business segment(s) is (are) deemed to be reportable segment(s)?

A

Lion, Monk, and Nevi

Under FAS 131, a segment is reportable if its sales (including intersegment sales) are at least 10% of total combined revenues (including intersegment sales) for all segments.

Total combined sales are $150,000. Thus, all three segments are reportable because the combined sales of each exceed $15,000.

How well did you know this?
1
Not at all
2
3
4
5
Perfectly
104
Q

Yellow Co. received a large worker’s compensation claim of $90,000 in the third quarter for an injury occurring in the third quarter. How should Yellow account for the transaction in its interim financial report?

A

Recognize $90,000 in the third quarter.

The worker’s compensation claim should be reported in the period incurred, the third quarter. This is a transaction that occurred in the third quarter and does not impact other quarters.

How well did you know this?
1
Not at all
2
3
4
5
Perfectly
105
Q

On January 16, Tree Co. paid $60,000 in property taxes on its factory for the current calendar year. On April 2, Tree paid $240,000 for unanticipated major repairs to its factory equipment. The repairs will benefit operations for the remainder of the calendar year. What amount of these expenses should Tree include in its third quarter interim financial statements for the three months ended September 30?

A

$95,000

Interim reporting treats each interim period as an integral part of the annual period. The two expenditures, in this problem, benefit more than one quarter. Thus, the expenses are recognized in the periods benefited, rather than only in the period of expenditure. The property taxes benefit all four quarters; therefore, $15,000 ($60,000/4) is recognized each quarter. The repair benefits three quarters; therefore, $80,000 ($240,000/3) is recognized each quarter. The sum of the two amounts is $95,000 to be recognized in quarter three.

How well did you know this?
1
Not at all
2
3
4
5
Perfectly
106
Q

How are discontinued operations that occur at midyear initially reported?

A

Included in net income and disclosed in the notes to interim financial statements.

Discontinued operations are not related to any other interim period. Therefore, it would be erroneous to allocate their financial statement effects to more than one interim period.

How well did you know this?
1
Not at all
2
3
4
5
Perfectly
107
Q

Bard Co., a calendar-year corporation, reported income before income tax expense of $10,000 and income tax expense of $1,500 in its interim income statement for the first quarter of the year. Bard had income before income tax expense of $20,000 for the second quarter and an estimated effective annual rate of 25%. What amount should Bard report as income tax expense in its interim income statement for the second quarter?

A

$6,000

Interim income tax expense equals the difference between (1) the total income tax through the end of the interim period at the estimated annual tax rate, and (2) the income tax expense recognized in previous interim periods of the same year. For the second quarter, income tax expense therefore is computed as ($10,000 + $20,000)(.25) − $1,500 = $6,000.

How well did you know this?
1
Not at all
2
3
4
5
Perfectly
108
Q

An inventory loss from a permanent market decline of $360,000 occurred in May Year 1. Cox Co. appropriately recorded this loss in May Year 1 after its March 31, Year 1, quarterly report was issued.

What amount of inventory loss should be reported in Cox’s quarterly income statement for the three months ended June 30, Year 1?

A

$360,000

Unless temporary, declines in the market value of inventory should be recognized in full in the interim period in which they occur.

They should not be deferred to a later period. In this way, the quarterly financial statement reports a significant event for that quarter.

This is an example of an exception to the overall view adopted by the APB with regard to interim reports: that interim reports should be an integral part of the annual period.

How well did you know this?
1
Not at all
2
3
4
5
Perfectly
109
Q

On June 30, 20X5, Mill Corp. incurred a $100,000 net loss from disposal of a business segment. Also, on June 30, 20X5, Mill paid $40,000 for property taxes assessed for the calendar year 20X5.

What amount of the foregoing items should be included in the determination of Mill’s net income or loss for the six-month interim period ended June 30, 20X5?

A

$120,000

The disposal loss cannot be allocated to interim periods. It does not relate to any interim period other than the one in which it occurred. Thus, it is recognized completely in the earnings for the six month period ended June 30.

The property tax is allocated to interim periods based on time expired. The $40,000 tax relates to the entire year of 20X5. With half of the year elapsed at June 30, half of the tax should be recognized in expense. The sum of the amounts to be recognized at June 30 is $120,000 ($100,000 + $20,000).

How well did you know this?
1
Not at all
2
3
4
5
Perfectly
110
Q

An inventory loss from a market price decline occurred in the first quarter, and the decline was not expected to reverse during the fiscal year.

However, in the third quarter, the inventory’s market price recovery exceeded the market decline that occurred in the first quarter.

For interim financial reporting, the dollar amount of net inventory should:

A

Decrease in the first quarter by the amount of the market price decline and increase in the third quarter by the amount of the decrease in the first quarter.

When interim period inventory market value declines are not considered temporary (not expected to reverse), they are recognized in the quarter in which the decline occurs. Later recoveries are recognized as gains to the extent of previous losses only. The inventory may not be marked up above cost.

How well did you know this?
1
Not at all
2
3
4
5
Perfectly
111
Q

On March 15, 20X4, Krol Co. paid property taxes of $90,000 on its office building for the calendar year 20X4.

On April 1, 20X4, Krol paid $150,000 for unanticipated repairs to its office equipment. The repairs will benefit operations for the remainder of 20X4.

What is the total amount of these expenses that Krol should include in its quarterly income statement for the three months ended June 30, 20X4?

A

$72,500

One-fourth of the property taxes should be recognized for the second quarter income statement: $22,500 = $90,000/4. Although the entire annual amount was paid in the first quarter, only 1/4 of the total annual amount should be recognized in each quarter. This allocation is based on benefits received (the benefits that flow from payment of property taxes). It is reasonable to assume that each quarter benefits the same amount.

The repair cost benefits three quarters on an equal basis because it was paid at the beginning of the second quarter. Therefore, 1/3 of the cost, or $50,000, should be reported in the income statement for the second quarter.

Thus, the total expense to be recognized in the second quarter is $72,500 ($22,500 + $50,000).

How well did you know this?
1
Not at all
2
3
4
5
Perfectly
112
Q

Due to a decline in market price in the second quarter, Petal Co. incurred an inventory loss. The market price is expected to return to previous levels by the end of the year. At the end of the year, the decline had not reversed. When should the loss be reported in Petal’s interim income statements?

A

In the fourth quarter only.

Temporary declines in inventory value are not recognized in the interim period in which they occur. This decline was expected to be temporary, i.e. it was expected to reverse. Therefore, it is not recorded until the fourth quarter, at which time the normal annual LCM valuation is applied because the decline had not reversed. Had the decline in the second quarter been deemed permanent, it would have been recognized in the second quarter.

How well did you know this?
1
Not at all
2
3
4
5
Perfectly
113
Q

For interim financial reporting, a company’s income tax provision for the second quarter of 20X4 should be determined using the:

A

Effective tax rate expected to be applicable for the full year of 2004 as estimated at the end of the second quarter of 20X4.

To ensure the most current information, an estimate of the applicable tax rate for the entire year is made at the end of each quarter. Also at the end of each quarter, the tax for the entire portion of the year elapsed is computed, including previous quarters of that year. Finally, the previous quarters’ tax is subtracted, yielding the income tax for the latest quarter.

How well did you know this?
1
Not at all
2
3
4
5
Perfectly
114
Q

ASC 270, Interim Reporting, concluded that interim financial reporting should be viewed primarily in which of the following ways?

A

As reporting for an integral part of an annual period.

The fundamental principle underlying interim reporting is that interim reports should be considered an integral part of the annual reporting period. This has important implications for interim reporting. There are exceptions to this principle, however.

How well did you know this?
1
Not at all
2
3
4
5
Perfectly
115
Q

A planned volume variance in the first quarter, which is expected to be absorbed by the end of the fiscal period, ordinarily should be deferred at the end of the first quarter if it is:

A

Favorable - Yes
Unfavorable - Yes

“Purchase price variances or volume or capacity cost variances that are planned and expected to be absorbed by the end of the annual period, should ordinarily be deferred at interim reporting dates.”

The reason for the deferral is that, from the point of view of the entire reporting year, there will be no volume variance. The Opinion requires the integral view of interim reporting for most items - that an interim period is an integral part of the annual period. Recognizing, rather than deferring, the variance in the first quarter would cause the reporting of the first quarter results to be unrepresentative of the annual period of which it is a part.

How well did you know this?
1
Not at all
2
3
4
5
Perfectly
116
Q

Kell Corp. reported $111,000 of net income for the quarter ended September 30, 20X5. Additional information for the quarter:

A $60,000 gain from discontinued operation, realized on April 30, 20X5, was allocated equally to the second, third, and fourth quarters of 2005.
A $16,000 cumulative-effect adjustment (dr.) resulting from a change in inventory valuation method was recognized on August 2, 20X5. The new method was used for the quarter ended September 30. The $111,000 earnings amount does not reflect the cumulative effect.
In addition, Kell paid $48,000 on February 1, 20X5, for 20X5 calendar-year property taxes. Of this amount, $12,000 was allocated to the third quarter of 20X5.

For the quarter ended September 30, 20X5, Kell should report net income of:

A

$91,000

The gain from discontinued operations should be recognized entirely in the second quarter. There is no meaningful basis on which to allocate the gain. It is a one-time occurrence. The cumulative effect is not recognized in income. The firm’s treatment of the property tax cost is correct. The cost relates to the entire year. Therefore, each quarter should bear 1/4 of the cost.

Third quarter net income = $91,000 = $111,000 − $60,000/3.

How well did you know this?
1
Not at all
2
3
4
5
Perfectly
117
Q

Which of the following statements, if any, concerning the modified cash basis of accounting is/are correct?

I. The modified cash basis of accounting employs some elements of accrual accounting.

II. To be acceptable, modifications to the cash basis of accounting must have substantial support in practice.

A

Both I and II.

The modified cash basis of accounting employs some elements of accrual accounting (Statement I) and the modifications to cash basis must have substantial support in practice (Statement II). Both statements are correct.

How well did you know this?
1
Not at all
2
3
4
5
Perfectly
118
Q

Which one of the following is not an other comprehensive basis of accounting?

A

Pure accrual basis.

Pure accrual basis accounting is not an other comprehensive basis of accounting. The concept of “other comprehensive basis” means a comprehensive basis of accounting other than pure (or full) accrual accounting.

How well did you know this?
1
Not at all
2
3
4
5
Perfectly
119
Q

Alco, Inc., a small manufacturing company, prepares its financial statements using its income tax basis of accounting. In December, 2012, it determined that an error had been made in the amount of rent expense reported in its 2011 tax return. How should Alco account for the amount of the rental expense error in its 2012 financial statements?

A

As a prior period adjustment.

The amount of the rental expense error made in the tax return (and financial statements) of the prior period would be reported as a prior period adjustment in Alco’s 2012 financial statements.

How well did you know this?
1
Not at all
2
3
4
5
Perfectly
120
Q

Which of the following items would be recognized in financial statements prepared using an income tax basis of accounting relating to permanent differences?

A

Nontaxable Income - YES
Nondeductible Expenses - YES

Both nontaxable income items (e.g., life insurance proceeds from the death of an officer) and nondeductible expenses (e.g., premium cost of life insurance on an officer) would be recognized in financial statements prepared using an income tax basis of accounting.

How well did you know this?
1
Not at all
2
3
4
5
Perfectly
121
Q

When a set of financial statements is prepared using the cash basis or the modified cash basis of accounting, which one of the following is least likely to be an appropriate financial statement title?

A

Income Statement

When the cash basis or the modified cash basis of accounting is used, the title Income Statement, which is appropriate when the accrual basis of accounting is used, should be replaced by the title Statement of Cash Receipts and Cash Disbursements. This helps distinguish that the statement is not based on full accrual accounting consistent with U.S. GAAP.

How well did you know this?
1
Not at all
2
3
4
5
Perfectly
122
Q

If a company that is not a public business entity wants to apply the simplified hedge accounting approach to a cash flow hedge of a variable rate borrowing with a receive-variable, pay-fixed interest rate swap, which of the following is a condition that must be met?

A

The variable interest rate on the interest rate swap and the variable interest rate on the hedged borrowing are linked to the same index.

To qualify for simplified hedge accounting, one of the criteria is that both the swap and the hedged borrowing are linked to the same index.

How well did you know this?
1
Not at all
2
3
4
5
Perfectly
123
Q

A private company decided to adopt one of the standards issued by the Private Company Council. What are the requirements upon adoption of the PCC standard?

A

Apply the new standard on a prospective basis.

Adoption of a private company standard is done on a prospective basis.

How well did you know this?
1
Not at all
2
3
4
5
Perfectly
124
Q

The Private Company Council has issued modified accounting for private companies for what aspect of Goodwill?

A

Goodwill amortization.

The PCC allows private companies to amortize goodwill over a period to not exceed 10 years.

How well did you know this?
1
Not at all
2
3
4
5
Perfectly
125
Q

The following information pertains to Jet Corp. outstanding stock for Year 1:

Common stock, $5 par value
Shares outstanding, 1/1/01 20,000
2-for-1 stock split, 4/1/Year 1 20,000
Shares issued, 7/1/Year 1 10,000

Preferred stock, $10 par value, 5% cumulative
Shares outstanding, 1/1/Year 1 4,000

What are the number of shares Jet should use to calculate Year 1 earnings per share?

A

45,000

The weighted average shares outstanding for this firm for Year 1 is: 45,000 = [20,000(2) + 10,000(1/2)]. The split affects only the shares issued before date of the split. The July 1 issuance is weighted only by 1/2 a year because the shares were outstanding only 1/2 a year. EPS is computed only on common stock outstanding. The preferred shares have no effect on the computation.

How well did you know this?
1
Not at all
2
3
4
5
Perfectly
126
Q

A company had the following outstanding shares as of January 1, year 2:

Preferred stock, $60 par, 4%, cumulative 10,000 shares

Common stock, $3 par 50,000 shares

On April 1, year 2, the company sold 8,000 shares of previously unissued common stock. No dividends were in arrears on January 1, year 2, and no dividends were declared or paid during year 2. Net income for year 2 totaled $236,000. What amount is basic earnings per share for the year ended December 31, year 2?

A

$3.79

Basic EPS = Net Income - Preferred Dividends / Weighted shares outstanding. The numerator is $236,000 - preferred dividends [($60 × 10,000) × .04 = 24,000] = $212,000. The denominator is 50,000 (12/12) + 8,000 (9/12) = 56,000 shares. $212,000 / 56,000 = $3.786 or $3.79.

How well did you know this?
1
Not at all
2
3
4
5
Perfectly
127
Q

What qualifies as a reportable operating segment?

A

To qualify as a segment, a component must meet one of the three criteria. For all three criteria, the segment must account for 10% or more of the combined amount for all operating segments. Reporting to the company-wide chief operating decision maker is also a requirement of an operating segment.

How well did you know this?
1
Not at all
2
3
4
5
Perfectly
128
Q

The following information pertains to revenue earned by Timm Co.’s industry segments for the year ending December 31, 2005:

    Segment	Sales 
     to unaffiliated 
    customers	    Intersegment sale  Total revenue Alo	    $5,000	       $3,000	              $8,000 Bix	       8,000	         4,000	               12,000 Cee	       4,000	                -	                4,000 Dil	      43,000	        16,000	              59,000 Combined	60,000	23,000	              83,000 Elimination	-	         (23,000)        	    (23,000) Consolidated	$60,000	         -	           $60,000
             ========	 ======	                ======= In conformity with the revenue test, Timm's reportable segments were
A

Only Bix and Dil.

To meet the revenue test, an operating segment must have total sales (including intersegment sales) of 10% or more of the combined segment sales (including intersegment sales). $83,000 is the test number.

Only Bix with $12,000 of total sales and Dil with $59,000 have sales in excess of $8,300 (.10 × $83,000).

How well did you know this?
1
Not at all
2
3
4
5
Perfectly
129
Q

A corporation issues quarterly interim financial statements and uses the lower cost or net realizable value to value its inventory in its annual financial statements. Which of the following statements is correct regarding how the corporation should value its inventory in its interim financial statements?

A

Inventory losses generally should be recognized in the interim statements.

Only temporary losses expected to be recovered are not recognized in interim periods. Because most inventory losses are permanent.

How well did you know this?
1
Not at all
2
3
4
5
Perfectly
130
Q

In general, an enterprise preparing interim financial statements should:

A

Use the same accounting principles followed in preparing its latest annual financial statements.

Interim financial statements generally should reflect the same accounting principles used in preparing annual financial statements. Interim periods are considered an integral part of the annual period, with some exceptions.

How well did you know this?
1
Not at all
2
3
4
5
Perfectly
131
Q

Farr Corp. had the following transactions during the quarter ended March 31, 20X5:

Loss on early extinguishment of debt $ 70,000

Payment of fire insurance premium for calendar year 20X5 100,000

What amount should be included in Farr’s income statement for the quarter ended March 31, 20X5?

A

Extinguishment loss - $ 70,000
Insurance expense - $ 25,000

In large measure, accounting principles for interim periods are the same as for annual periods. The extinguishment loss is a one-time event and is recognized entirely in the first quarter. The insurance payment covers an annual period. Thus, only 1/4 of the payment, or $25,000 ($100,000 × .25), is allocated to the first quarter.

How well did you know this?
1
Not at all
2
3
4
5
Perfectly
132
Q

Financial Statements prepared on a modified cash basis of accounting would contain items measured on which, if either, of the following bases?

A

Cash Basis - YES
Accrual Basis - YES

A modified cash basis of accounting would contain items (accounts) measured under both the cash basis of accounting and the accrual basis of accounting. The modified cash basis of accounting uses cash basis accounting modified to incorporate accrual basis accounting for certain types of transactions/events. Modifications must be logical and consistent with accrual basis accounting under U.S. GAAP.

How well did you know this?
1
Not at all
2
3
4
5
Perfectly
133
Q

In financial statements prepared on the income-tax basis, how should the nondeductible portion of expenses, such as meals and entertainment, be reported?

A

Included in the expense category in the determination of income.

Despite the fact that these expenses are not deductible for tax purposes, they are still business expenses and need to be included in the determination of income on the financial statements. In addition, the income tax return requires information on the total meals and entertainment expense in order to calculate the deductible amount.

How well did you know this?
1
Not at all
2
3
4
5
Perfectly
134
Q

Hahn Co. prepared financial statements on the cash basis of accounting. The cash basis was modified so that an accrual of income taxes was reported.

Are these financial statements in accordance with the modified cash basis of accounting?

A

Yes.

Under a strict cash basis of accounting, revenues and expenses are recorded only when cash is received or paid. Under a modified cash basis of accounting, certain accruals and/or deferrals are recorded for financial-statement purposes.

The most common modifications are the capitalization and amortization of long-lived assets and the accrual for income taxes (recognition of income tax expense and related liability).

How well did you know this?
1
Not at all
2
3
4
5
Perfectly
135
Q

The following is Gold Corp.’s June 30 trial balance:

Cash overdraft $ 10,000
Accounts receivable, net $ 35,000
Inventory 58,000
Prepaid expenses 12,000
Land held for resale 100,000
Property, plant, and equipment, net 95,000
Accounts payable and accrued expenses 32,000
Common stock 25,000
Additional paid-in capital 150,000
Retained earnings 83,000
_________ _________
$300,000 $300,000
======== ========
Additional information:

Checks amounting to $30,000 were written to vendors and recorded on June 29 resulting in a cash overdraft of $10,000. The checks were mailed on July 9.
Land held for resale was sold for cash on July 15.
Gold issued its financial statements on July 31.
In its June 30 balance sheet, what amount should Gold report as current assets?

A

$225,000

Current assets are those assets expected to be consumed or realized in cash within one year of the balance sheet date. There is no overdraft because the checks were not sent as of the balance sheet date. Thus, the balance sheet should disclose $20,000 in cash ($30,000 − $10,000).

The land held for resale is a current asset because it is expected to be sold in the next year (and the corroboration of this expectation was known before the issuance of the financial statements).

Cash	$ 20,000
Net accounts receivable	35,000
Inventory	58,000
Prepaid expenses	12,000
Land held for resale	100,000
Total current assets	$225,000
How well did you know this?
1
Not at all
2
3
4
5
Perfectly
136
Q

On October 31, Dingo, Inc. had cash accounts at three different banks. One account balance is segregated solely for a November 15 payment into a bond sinking fund. A second account, used for branch operations, is overdrawn. The third account, used for regular corporate operations, has a positive balance.

How should these accounts be reported in Dingo’s October 31 classified balance sheet?

A

The segregated account should be reported as a noncurrent asset, the regular account should be reported as a current asset, and the overdraft should be reported as a current liability.

The accounts are with different banks. Thus, the accounts cannot be offset against one another.

The overdraft is a liability because the bank honored a check or withdrawal causing the account to be negative. The firm owes the bank this amount.

The regular corporate account is part of the cash account, a current asset. The segregated account is a long-term investment. The cash in this asset is set aside for a specific purpose. There is no intent to use the cash for ordinary operating purposes.

How well did you know this?
1
Not at all
2
3
4
5
Perfectly
137
Q

The following information pertains to Grey Co. on December 31, 20X3:

Checkbook balance $12,000
Bank statement balance 16,000
Check drawn on Grey’s account, payable to a vendor, dated and recorded 12/31/X3 but not mailed until 1/10/X4 1,800

On Grey’s December 31, 20X3 balance sheet, what amount should be reported as cash?

A

$13,800

The correct cash balance is the balance per the checkbook ($12,000) plus the $1,800 check written to the vendor, for a total of $13,800.

This check reduced the balance in the checkbook but was not mailed. Thus, the amount remains in Grey’s cash balance at the end of the year. The bank statement balance is not the correct balance because information about transactions affecting cash near the end of the month, recorded by Grey, did not reach the bank by the cutoff date.

How well did you know this?
1
Not at all
2
3
4
5
Perfectly
138
Q

Cook Co. had the following balances on December 31, 20X4:

Cash in checking account $350,000
Cash in money market account 250,000
U.S. Treasury bill, purchased 12/1/X4, maturing 2/28/X5 800,000
U.S. Treasury bond, purchased 3/1/X4, maturing 2/28/X5 500,000
Cook’s policy is to treat as cash equivalents all highly liquid investments with a maturity of three months or less when purchased. What amount should Cook report as cash and cash equivalents in its December 31, 20X4, balance sheet?

A

$1,400,000

The first three items in the list are included in cash and cash equivalents. If no restrictions apply, cash in checking accounts ($350,000) is always included in cash. Per ASC Topic 305, cash equivalents are short-term, highly liquid investments that are readily convertible into cash and have maturities of three months or less from the date of purchase by the entity. Common examples are Treasury bills, commercial paper, and money market funds. In this case, the cash equivalents are the money market account ($250,000) and the Treasury bill ($800,000). Therefore, total cash and cash equivalents is $1,400,000 ($350,000 + $250,000 + $800,000). The maturity of the Treasury bond was at least 12 months (3/1/X4 to 2/28/X5) from the date of purchase; therefore, it should not be reported in cash and cash equivalents. The reason for the three-month rule is to minimize price fluctuations due to interest rate changes. A security with a fluctuating price is not “equivalent” to cash. One year is too long a time to expect interest rates to remain stable.

How well did you know this?
1
Not at all
2
3
4
5
Perfectly
139
Q

The following are held by Smite Co.:

Cash in checking account $20,000
Cash in bond sinking fund account 30,000
Postdated check from customer dated one month from balance sheet date 250
Petty cash 200
Commercial paper (matures in two months) 7,000
Certificate of deposit (matures in six months) 5,000

What amount should be reported as cash and cash equivalents on Smite’s balance sheet?

A

$27,200

The cash balance is $20,200: the sum of the checking account balance and the petty cash. Because it has a maturity of less than three months, the only cash equivalent is the $7,000 of commercial paper. The final sum of these two accounts is $27,200.

How well did you know this?
1
Not at all
2
3
4
5
Perfectly
140
Q

Alton Co. had a cash balance of $32,300 recorded in its general ledger at the end of the month, prior to receiving its bank statement. Reconciliation of the bank statement reveals the following information:

Bank service charge: $15

Check deposited and returned for insufficient funds check: $120

Deposit recorded in the general ledger as $258 but should be $285

Checks outstanding: $1,800

After reconciling its bank statement, what amount should Alton report as its cash account balance?

A

$32,192

The reconciliation should be as follows:
Book balance	    $32,300
Less bank fees	(15)
Less NSF check	(120)
Plus deposit transposition error (285 – 258)	27
Corrected book balance	$32,192
How well did you know this?
1
Not at all
2
3
4
5
Perfectly
141
Q

A bank reconciliation with the headings “Balance per Books” and “Balance per Bank” lists three adjustments under the former and four adjustments under the latter. The company makes separate adjusting entries for each item in the reconciliation that requires an adjustment. How many adjusting entries are recorded?

A

3

Only amounts adjusting the balance per books require an adjusting entry because only those amounts explain why the firm’s recorded cash balance is not the same as the true cash balance. Common adjustments of this type include bank service charges, notes collected, and interest. The firm cannot alter the bank balance.

How well did you know this?
1
Not at all
2
3
4
5
Perfectly
142
Q

Hilltop Co.’s monthly bank statement shows a balance of $54,200. Reconciliation of the statement with company books reveals the following information:

Bank service charge $10
Insufficient funds check 650
Checks outstanding 1,500
Deposits in transit 350
Check deposited by Hilltop and cleared by the bank for $125
but improperly recorded by Hilltop as $152
What is the net cash balance after the reconciliation?

A

$53,050

The reconciling items that need to be adjusted to the bank balance are: checks outstanding (−1,500) and deposit in transit (+350). The net cash after the reconciliation is: Bank balance $54,200 − 1,500 + 350 = $53,050. The bank service charge and insufficient funds are already reflected in the bank balance. The error is on Hilltop’s books, not on the bank statement, and therefore it does not need to be included in the reconciliation.

How well did you know this?
1
Not at all
2
3
4
5
Perfectly
143
Q

On June 1, 2005, Pitt Corp. sold merchandise with a list price of $5,000 to Burr on account. Pitt allowed trade discounts of 30% and 20%.

Credit terms were 2/15, n/40 and the sale was made FOB shipping point. Pitt prepaid $200 of delivery costs for Burr as an accommodation.

On June 12, 2005, Pitt received from Burr a remittance in full payment amounting to

A

$2,944

$5,000(1 - .30)(1 - .20)(.98) + $200 = $2,944.

The chain trade discounts are applied to each successive net amount as shown in the calculation, and the cash discount of 2% is then applied to the final invoice amount.

The cash discount applies because the payment was made within 15 days of purchase. The goods were shipped FOB shipping point. Therefore, title transferred to Burr at the shipping point, meaning Burr bears the shipping charges. Because Pitt prepaid them as an accommodation, Burr must reimburse Pitt for the $200, the last term in the calculation leading to $2,944.

How well did you know this?
1
Not at all
2
3
4
5
Perfectly
144
Q

Alfisol, Inc. offers sales discounts of 2% on all credit sales paid within 15 days. For year 1, gross credit sales totaled $150,000 and 75% of Alfisol’s customers took advantage of the discount. Under the net method

A

For cash receipts after the discount period, discounts not taken must be credited for $750.
Under the net method, sales are initially recorded net of discounts. Payments received after the discount period total $37,500 ($150,000 × 25%), and the amount of discounts forfeited is $750 ($37,500 × 2%). Under the net method, the entry to record these receipts is

Dr Cash 37,500
Cr. AR 36,750
Cr. Discounts not taken 750

How well did you know this?
1
Not at all
2
3
4
5
Perfectly
145
Q

Steven Corporation began operations in year 1. For the year ended December 31, year 1, Steven made available the following information:

Total merchandise purchases for the year $350,000
Merchandise inventory at December 31, year 1 $70,000
Collections from customers $200,000

All merchandise was marked to sell at 40% above cost. Assuming that all sales are on a credit basis and all receivables are collectible, what should be the balance in accounts receivable at December 31, year 1?

A

$192,000

The first step is to determine sales for year 1. The cost of goods sold in year 1 is $280,000 ($350,000 purchases less $70,000 ending inventory). Note that beginning inventory is zero because the company began operations in year 1. Since all merchandise was marked to sell at 40% above cost, year 1 credit sales are $392,000 ($280,000 cost of goods sold × 140%). The second step is to determine the ending balance in accounts receivable.

A/R

Dr. Beg 0
Dr. Credit Sales $392,000
Cr. Collections $ 200,000
End Balance 192,000

How well did you know this?
1
Not at all
2
3
4
5
Perfectly
146
Q

When the allowance method of recognizing uncollectible accounts is used, how would the collection of an account previously written off affect accounts receivable and the allowance for uncollectible accounts?

A

Accounts receivable - No Effect
Allowance for uncollectible accounts - Increase

When an account is written off, the journal entry is debit the allowance for uncollectible accounts and credit accounts receivable. If the account is subsequently collected, an entry is made to reinstate the account receivable by debiting accounts receivable and crediting the allowance for uncollectible accounts. A second entry is made for the cash collection which involves debiting cash and crediting accounts receivable. Therefore, there is no change in accounts receivable when a previously written-off account is collected; accounts receivable is debited for the reinstatement, and credited for the payment. However, when the previously written-off account is collected, there is an increase in the allowance for uncollectible accounts.

How well did you know this?
1
Not at all
2
3
4
5
Perfectly
147
Q

When the allowance method of recognizing bad debt expense is used, the entries at the time of collection of a small account previously written off would

A

Increase the allowance for doubtful accounts.

The solutions approach is to determine the journal entries necessary to (1) reestablish and (2) collect the account receivable.

The entry to reestablish the account would be
Accounts receivable xx
Allowance for doubtful accounts xx

The entry to record collection would be
Cash xx
Accounts receivable xx

The net effect is an increase in a current asset account, cash, and an increase in a contra asset account, allowance for doubtful accounts.

How well did you know this?
1
Not at all
2
3
4
5
Perfectly
148
Q

The following information relates to Jay Co.’s accounts receivable for 2004:

Accounts receivable, 1/1/04 $ 650,000
Credit sales for 2004 2,700,000
Sales returns for 2004 75,000
Accounts written off during 2004 40,000
Collections from customers during 2004 2,150,000
Estimated future sales returns at 12/31/04 50,000
Estimated uncollectible accounts at 12/31/04 110,000

What amount should Jay report for accounts receivable, before allowances for sales returns and uncollectible accounts, on December 31, 2004?

A

$1,085,000

The question is asking for the gross accounts receivable balance, before allowances for future sales returns, allowances, and uncollectible accounts:

AR 1/1 + Credit sales - Sales returns - Write-offs - Collections = AR 12/31

$650,000 + $2,700,000 − $75,000 − $40,000 − $2,150,000 = $1,085,000

How well did you know this?
1
Not at all
2
3
4
5
Perfectly
149
Q

Gibbs Co. uses the allowance method for recognizing uncollectible accounts. Ignoring deferred taxes, the entry to record the write-off of a specific uncollectible account

A

Affects neither net income nor working capital.

The entry is:
Allowance for uncollectible accounts xxxx
Accounts receivable xxxx

The allowance is contra to accounts receivable and thus the entry does not affect net accounts receivable. The entry does not affect current assets, working capital, or income. However, the entry does reduce gross accounts receivable. Thus, the answer “affects neither net income nor working capital” is the only possible correct answer. The answer “affects neither net income nor accounts receivable” is incorrect if “accounts receivable” is interpreted as gross accounts receivable.

How well did you know this?
1
Not at all
2
3
4
5
Perfectly
150
Q

Marr Corp. reported rental revenue of $2,210,000 in its cash basis federal income tax return for the year ended November 30, 2004. Additional information is as follows:

Rents receivable - November 30, 2004 $1,060,000
Rents receivable - November 30, 2003 800,000

Uncollectible rents written off during the fiscal year 30,000

Under the accrual basis, Marr should report rental revenue of

A

$2,500,000

The cash basis revenue in the tax return is the amount of rent collected for tax purposes.

beg. rent receivable + accrual revenue - collections -write-offs = end. rent receivable

$800,000 + accrual revenue - $2,210,000 - $30,000 = $1,060,000

accrual revenue = $2,500,000

How well did you know this?
1
Not at all
2
3
4
5
Perfectly
151
Q

During the year, Hauser Co. wrote off a customer’s account receivable. Hauser used the allowance method for uncollectable accounts. What impact would the write-off have on net income and total assets?

A

Net income - No Effect
Total assets - No Effect

Under the allowance method for uncollectible accounts there is no impact on the balance sheet or net income when the receivable is written off. The estimated uncollectible is recognized at the time of the sale; therefore, when the account is written, off the allowance and the accounts receivable are both reduced resulting in no effect on the income statement or balance sheet.

How well did you know this?
1
Not at all
2
3
4
5
Perfectly
152
Q

Adam Co. reported sales revenue of $2,300,000 in its income statement for the year ended December 31, 2005. Additional information was as follows:

                                                    12/31/04	12/31/05 Accounts receivable	              $500,000	$650,000 Allowance for uncollectible accounts	(30,000)	(55,000)

Uncollectible accounts totaling $10,000 were written off during 2005. Under the cash basis of accounting, Adam would have reported 2005 sales of

A

$2,140,000

Under the cash basis of accounting, sales equals cash collected from customers. An equation or T account may be used to determine this amount:

AR, beginning + Sales - Write-offs - customer collections = AR, ending
$500,000 $2,300,000 $10,000 ? = $650,000

Solving for the unknown (?) amount, customer collections equals $2,140,000.

This is the amount collected from customers, and is the amount that would be reported as sales under the cash basis method of accounting.

How well did you know this?
1
Not at all
2
3
4
5
Perfectly
153
Q

Bee Co. uses the direct write-off method to account for uncollectible accounts receivable.

During an accounting period, Bee’s cash collections from customers equal sales adjusted for the addition or deduction of the following amounts:

A

Accounts written off - Deduction
Increase in accounts receivable balance - Deduction

Under the direct write-off method, write-offs are credited directly to accounts receivable (AR). No allowance account is used. Under the terms of the question, accounts receivable increased during the year.

Increase in AR = sales - cash collections - write-offs

cash collections = sales - increase in AR - write-offs.

How well did you know this?
1
Not at all
2
3
4
5
Perfectly
154
Q

Orr Co. prepared an aging of its accounts receivable at December 31, 2005 and determined that the net realizable value of the receivables was $250,000. Additional information is available as follows:

Allowance for uncollectible accounts at 1/1/05 - credit balance $ 28,000
Accounts written off as uncollectible during 2005 23,000
Accounts receivable at 12/31/05 270,000
Uncollectible accounts recovery during 2005 5,000

For the year ended December 31, 2005, Orr’s uncollectible accounts expense would be

A

$10,000

Beginning allowance balance + uncollectible accounts expense - write-offs + recoveries = ending allowance balance

$28,000 + uncollectible accounts expense − $23,000 + $5,000 = ($270,000 − $250,000)

Uncollectible accounts expense = $10,000

Under the aging method, the ending allowance balance equals the difference between gross accounts receivable ($270,000) and net realizable value of accounts receivable ($250,000).

Write-offs decrease the allowance balance, and uncollectible accounts expense increases the allowance.

Recoveries also increase the allowance because the amount by which the allowance was decreased when the account was written off is reinstated on recovery.

How well did you know this?
1
Not at all
2
3
4
5
Perfectly
155
Q

On the December 31, 2005 balance sheet of Mann Co., the current receivables consisted of the following:

Trade accounts receivable $ 93,000
Allowance for uncollectible accounts (2,000)
Claim against shipper for goods lost in transit (Nov. 2005) 3,000
Selling price of unsold goods sent by Mann on consignment at 130% of cost (not included in Mann’s ending inventory) 26,000
Security deposit on lease of warehouse used for storing some inventories 30,000
Total $150,000
========

On December 31, 2005, the correct total of Mann’s current net receivables was

A

$94,000

Only the first three items are included in net receivables:

Trade accounts receivable $93,000
Allowance for uncollectible accounts (2,000)
Claim against shipper for goods lost in transit (Nov. 2005) 3,000
Net receivables $94,000

The claim for lost goods is a definite receivable. The firm has a current claim on another entity. The goods on consignment should be included in Mann’s inventory at cost, not in accounts receivable at sales value. They have not been sold. The security deposit is not included in current receivables because the firm will likely not receive this deposit back during the next fiscal year.

How well did you know this?
1
Not at all
2
3
4
5
Perfectly
156
Q

On December 1, 2005, Tigg Mortgage Co. gave Pod Corp. a $200,000, 12% loan.

Pod received proceeds of $194,000 after the deduction of a $6,000 nonrefundable loan origination fee. Principal and interest are due in 60 monthly installments of $4,450, beginning January 1, 2006. The repayments yield an effective interest rate of 12% at a present value of $200,000 and 13.4% at a present value of $194,000.

What amount of accrued interest receivable should Tigg include in its December 31, 2005 balance sheet?

A

$2,000

The term “accrued interest receivable” refers to the cash amount of interest due. The cash amount of interest due is based on the contractual interest rate and face value. The loan origination fee is a way of increasing the effective interest but it does not affect the cash interest component. The $2,000 accrued interest = (.12)(1/12)($200,000).

How well did you know this?
1
Not at all
2
3
4
5
Perfectly
157
Q

Frame Co. has an 8% note receivable, in the original amount of $150,000, dated June 30, 2003. Payments of $50,000 in principal plus accrued interest are due annually on July 1, 2004, 2005, and 2006.

In its June 30, 2005, balance sheet, what amount should Frame report as a current asset for interest on the note receivable?

A

$8,000

As of June 30, 2005, only one payment has been received (July 1, 2004). Thus, $100,000 of principal balance has been outstanding for an entire year as of the balance sheet date. Interest receivable on June 30, 2005 is thus $8,000 (.08 × $100,000).

How well did you know this?
1
Not at all
2
3
4
5
Perfectly
158
Q

Leaf Co. purchased from Oak Co. a $20,000, 8%, 5-year note that required five equal annual year-end payments of $5,009. The note was discounted to yield a 9% rate to Leaf. At the date of purchase, Leaf recorded the note at its present value of $19,485.

What should be the total interest revenue earned by Leaf over the life of this note?

A

$5,560

Total interest revenue is the amount received over the term of the note less the present value of the note: 5($5,009) − $19,485 = $5,560.

Leaf paid $19,485 for the note, and will receive 5($5,009) over the note term. The difference is interest revenue.

How well did you know this?
1
Not at all
2
3
4
5
Perfectly
159
Q

On January 2, 2003, Emme Co. sold equipment with a carrying amount of $480,000 in exchange for a $600,000 noninterest-bearing note due January 2, 2006.

There was no established exchange price for the equipment. The prevailing rate of interest for a note of this type on January 2, 2003, was 10%. The present value of $1 at 10% for three periods is 0.75.

In Emme’s 2003 income statement, what amount should be reported as gain (loss) on sale of machinery?

A

($30,000) loss.

The proceeds on sale are measured as the present value of the note because there is no established market value for the equipment. The loss on sale is computed as:

Carrying amount $480,000
Less proceeds on sale: $600,000(.75) = 450,000
Equals loss on sale $ 30,000
The proceeds do not reflect the entire $600,000, because the difference between the note’s face value of $600,000 and its present value is interest to be recognized over the term of the note.

How well did you know this?
1
Not at all
2
3
4
5
Perfectly
160
Q

Each of Potter Pie Co.’s 21 new franchisees contracted to pay an initial franchise fee of $30,000.

By December 31, 2005, each franchisee had paid a nonrefundable $10,000 fee and signed a note to pay $10,000 principal plus the market rate of interest on December 31, 2006 and December 31, 2007.

Experience indicates that one franchise will default on the additional payments. Services for the initial fee will be performed in 2006.

What amount of net unearned franchise fees would Potter report on December 31, 2005?

A

$610,000

The $610,000 net unearned fee revenue = (21)($30,000) − $20,000. This amount includes the notes received, but does not include the one expected uncollectible note.

The notes receivable balance, recorded along with the unearned revenue, will not reflect the note expected to be uncollectible. Bad debt expense is not recorded for this note because there has been no revenue recognized against which to match the expense.

How well did you know this?
1
Not at all
2
3
4
5
Perfectly
161
Q

On June 1, 2005, Yola Corp. loaned Dale $500,000 on a 12% note, payable in five annual installments of $100,000 beginning January 2, 2006. In connection with this loan, Dale was required to deposit $5,000 in a noninterest-bearing escrow account.

The amount held in escrow is to be returned to Dale after all principal and interest payments have been made. Interest on the note is payable on the first day of each month beginning July 1, 2005. Dale made timely payments through November 1, 2005. On January 2, 2006, Yola received payment of the first principal installment plus all interest due.

On December 31, 2005, Yola’s interest receivable on the loan to Dale should be

A

$10,000

Because the last interest payment was made on November 1, the interest for November and December is unpaid as of December 31, 2005. Therefore, two months of interest is receivable, as of December 31, 2005, for a total receivable of $10,000 = (2/12)(12%)($500,000). No principal payments have yet been made as of this date.

How well did you know this?
1
Not at all
2
3
4
5
Perfectly
162
Q

On December 30, 2005, Chang Co. sold a machine to Door Co. in exchange for a noninterest-bearing note requiring ten annual payments of $10,000. Door made the first payment on December 30, 2005. The market interest rate for similar notes at the date of issuance was 8%. Information on present value factors is as follows:

Period Present value
of $1 at 8% Present value of
ordinary annuity of $1 at 8%
9 0.50 6.25
10 0.46 6.71

In its December 31, 2005, balance sheet, what amount should Chang report as note receivable?

A

$62,500

The note receivable should be reported at the present value of the nine remaining payments. The first payment was made at the date of the sale. The remaining nine payments comprise an ordinary annuity as of December 31, 2005 because the next payment is due one year from that date.

Therefore, the present value and reported note value on that date is 6.25($10,000) = $62,500.

How well did you know this?
1
Not at all
2
3
4
5
Perfectly
163
Q

On December 31, 1999, Key Co. received two $10,000 noninterest-bearing notes from customers in exchange for services rendered. The note from Alpha Co., which is due in nine months, was made under customary trade terms, but the note from Omega Co., which is due in two years, was not. The market interest rate for both notes at the date of issuance is 8%. The present value of $1 due in nine months at 8% is .944. The present value of $1 due in two years at 8% is .857. At what amounts should these two notes receivable be reported in Key’s December 31, 1999, balance sheet?

A

Alpha - $ 10,000
Omega - $ 8570

The note from Alpha Co. is a short-term asset. It is reported at the face value of $10,000. The note from Omega is discounted as a single sum for two time periods at 8% to be reported at $10,000X.857=$8,570.

How well did you know this?
1
Not at all
2
3
4
5
Perfectly
164
Q

On March 31, 2005, Vale Co. had an unadjusted credit balance of $1,000 in its allowance for uncollectible accounts. An analysis of Vale’s trade accounts receivable on that date revealed the following:

Age	Amount	Estimated uncollectible
0 − 30 days	$60,000	5%
31 − 60 days	4,000	10%
Over 60 days	2,000	$1,400
What amount should Vale report as allowance for uncollectible accounts in its March 31, 2005, balance sheet?
A

$4,800

The sum of the products of the AR amounts and uncollectible percentages yield the required ending allowance balance (the third AR category’s estimated uncollectible has already been computed): $60,000(5%) + $4,000(10%) + $1,400 = $4,800.

How well did you know this?
1
Not at all
2
3
4
5
Perfectly
165
Q

For the year ending December 31, 2005, Beal Co. estimated its allowance for uncollectible accounts using the year-end aging of accounts receivable. The following data are available:

Allowance for uncollectible accounts, 1/1/05 $42,000
Provision for uncollectible accounts during 2005 (2% on credit sales of $2,000,000) 40,000
Uncollectible accounts written off, 11/30/05 46,000
Estimated uncollectible accounts per aging, 12/31/05 52,000
After year-end adjustment, the uncollectible accounts expense for 2005 should be

A

$56,000

The balance in the allowance for uncollectible accounts before the 2005 adjustment is: $42,000 beginning balance − $46,000 write-offs = -$4,000 (debit balance). The desired ending balance under aging is $52,000.

Therefore, the required increase to the account is $56,000, and this amount is uncollectible accounts expense. The aging method first determines the required ending balance in the allowance account based on the age of receivables, and THEN adjusts the allowance account to that balance.

The $40,000 provision is based on credit sales, and is thus to be ignored in the question.

How well did you know this?
1
Not at all
2
3
4
5
Perfectly
166
Q

Inge Co. determined that the net value of its accounts receivable on December 31, 2005, based on an aging of the receivables, was $325,000. Additional information is as follows:

Allowance for uncollectible accounts - 1/1/05 $ 30,000
Uncollectible accounts written off during 2005 18,000
Uncollectible accounts recovered during 2005 2,000
Accounts receivable at 12/31/05 350,000
For 2005, what would be Inge’s uncollectible accounts expense?

A

$11,000

This question requires a determination of the pre-adjustment balance in the allowance account, and the ending balance. The difference between these two amounts is the increase in the account needed, which is also the amount recognized as bad debt expense. The aging method first determines the required ending balance in the allowance account, and then places the amount needed to increase the account to this required balance into the allowance account.

The pre-adjustment allowance balance =

Beginning balance - Write-offs + Recoveries =

$30,000 − $18,000 + $2,000 = $14,000

The ending allowance balance =

$350,000 ending gross AR − $325,000 ending net value of AR = $25,000

Therefore, bad debt expense is the amount needed to bring the allowance balance up to the ending balance of $25,000. The increase needed is $11,000 ($25,000 − $14,000).

How well did you know this?
1
Not at all
2
3
4
5
Perfectly
167
Q

Hall Co.’s allowance for uncollectible accounts had a credit balance of $24,000 on December 31, 2003. During 2004, Hall wrote off uncollectible accounts of $96,000. The aging of accounts receivable indicated that a $100,000 allowance for doubtful accounts was required on December 31, 2004. What amount of uncollectible accounts expense should Hall report for 2004?

A

$172,000

Beginning allowance balance $24,000
Less write-off (96,000)
Equals pre-adjustment allowance balance (72,000) (debit)
The allowance balance normally is a credit. For the ending balance in the account after adjustment to be $100,000 credit, the account must be increased $172,000 by recognizing uncollectible accounts expense.

Under the aging method, uncollectible accounts expense equals the amount required to increase the allowance balance to the indicated total based on the aging of receivables.

How well did you know this?
1
Not at all
2
3
4
5
Perfectly
168
Q

Ward Co. estimates its uncollectible accounts expense to be 2% of credit sales. Ward’s credit sales for 2004 were $1,000,000. During 2004, Ward wrote off $18,000 of uncollectible accounts. Ward’s allowance for uncollectible accounts had a $15,000 balance on January 1, 2004. In its December 31, 2004 income statement, what amount should Ward report as uncollectible accounts expense?

A

$20,000

The credit sales method does not adjust the allowance balance to a required ending amount, but rather simply places the appropriate percent of sales into uncollectible accounts expense and the allowance account. 2% × $1,000,000 = $20,000.

How well did you know this?
1
Not at all
2
3
4
5
Perfectly
169
Q

Marr Co. had the following sales and accounts receivable balances, prior to any adjustments at year end:

Credit sales $10,000,000
Accounts receivable 3,000,000
Allowance for uncollectible accounts (debit balance) 50,000
Marr uses 3% of accounts receivable to determine its allowance for uncollectible accounts at year end. By what amount should Marr adjust its allowance for uncollectible accounts at year end?

A

$140,000

The amount of the adjustment to get the $50,000 debit balance to a $90,000 (3% × $3,000,000) credit balance is $140,000.

How well did you know this?
1
Not at all
2
3
4
5
Perfectly
170
Q

After being held for 40 days, a 120-day, 12% interest-bearing note receivable was discounted at a bank at 15%. The proceeds received from the bank equal

A

Maturity value less the discount at 15%.

The bank charges its discount (its fee) on the maturity value, which is the face value of the note plus 12% interest for 120 days. The bank charges 15% on this amount for the 80 remaining days in the note term. Thus, the proceeds equal the maturity value less its fee.

How well did you know this?
1
Not at all
2
3
4
5
Perfectly
171
Q

Ace Co. sold King Co. a $20,000, 8%, 5-year note that required five equal annual year-end payments. This note was discounted to yield a 9% rate to King. The present value factors of an ordinary annuity of $1 for five periods are as follows:

8% 3.992
9% 3.890
What should be the total interest revenue earned by King on this note?

A

$5,560

Total interest over the life of the note equals the total amount paid by Ace over the life of the note less the proceeds to Ace. The proceeds equal the present value of the payments at the 9% yield rate. The annual payment is found using the 8% rate because that rate is contractually set and determines the annual payment.

The annual payment P is found as: $20,000 = P(3.992). P = $5,010

Total interest revenue = total payments by Ace - proceeds to Ace

= 5($5,010) − $5,010(3.89) = $5,560.

How well did you know this?
1
Not at all
2
3
4
5
Perfectly
172
Q

Roth, Inc. received from a customer a one-year, $500,000 note bearing annual interest of 8%. After holding the note for six months, Roth discounted the note at Regional Bank at an effective interest rate of 10%. What amount of cash did Roth receive from the bank?

A

$513,000

Maturity value of the note: $500,000(1.08) $540,000
Less discount to the bank: $540,000(.10)(6/12) (27,000)
Equals proceeds to Roth $513,000
The bank charges its discount on the maturity amount, for the period it holds the note. In effect, it is charging interest on interest yet to accrue (for the last six months). This procedure is followed because the maturity value is the amount at risk.

How well did you know this?
1
Not at all
2
3
4
5
Perfectly
173
Q

On July 1, 2005, Lee Co. sold goods in exchange for a $200,000, 8-month, noninterest-bearing note receivable. At the time of the sale, the note’s market rate of interest was 12%.

What amount did Lee receive when it discounted the note at 10% on September 1, 2005?

A

$190,000

Six months remain in the note term at the date of discounting.

Maturity value of note: $200,000
Less discount: $200,000(.10)(6/12) (10,000)
Equals proceeds on note $190,000

How well did you know this?
1
Not at all
2
3
4
5
Perfectly
174
Q

Red Co. had $3 million in accounts receivable recorded on its books. Red wanted to convert the $3 million in receivables to cash in a more timely manner than waiting the 45 days for payment as indicated on its invoices. Which of the following would alter the timing of Red’s cash flows for the $3 million in receivables already recorded on its books?

A

Factor the receivables outstanding.

Factoring is a sale of receivables. This allows Red Co. to sell the receivables and receive cash immediately upon sale.

How well did you know this?
1
Not at all
2
3
4
5
Perfectly
175
Q

On November 1, 2004, Davis Co. discounted with recourse at 10%, a one-year, noninterest-bearing, $20,500 note receivable maturing on January 31, 2005.

What amount of contingent liability for this note must Davis disclose in its financial statements for the year ended December 31, 2004?

A

$20,500

The firm is contingent for the maturity amount, which for a noninterest-bearing note is the face value. If the maker of the note fails to pay the bank or financial institution with whom Davis discounted the note, Davis would be called on to pay the entire maturity amount.

How well did you know this?
1
Not at all
2
3
4
5
Perfectly
176
Q

Milton Co. pledged some of its accounts receivable to Good Neighbor Financing Corporation in return for a loan. Which of the following statements is correct?

A

Milton will retain control of the receivables.

In a pledge arrangement, the title remains with the originator, in this case with Milton Co.

How well did you know this?
1
Not at all
2
3
4
5
Perfectly
177
Q

On April 1, Aloe, Inc. factored $80,000 of its accounts receivable without recourse. The factor retained 10% of the accounts receivable as an allowance for sales returns and charged a 5% commission on the gross amount of the factored receivables. What amount of cash did Aloe receive from the factored receivables?

A

$68,000

The net cash received when the receivables were factored was $80,000 × .85 (100% - 10% - 5%) = $68,000.

How well did you know this?
1
Not at all
2
3
4
5
Perfectly
178
Q

Choose the correct accounting by the creditor for a loan impairment. Column (1): recognize a loss or expense upon recognizing the impairment. Column (2): rate of interest to use in computing the revised book value of the receivable after the impairment.

A

1 - Yes
2 - Original Effective Rate

A loan impairment is recorded by reducing the net book value of the receivable to the present value of probable future cash inflows, discounted at the original rate in the receivable. The original rate is used because the loan continues to exist. The loss to the firm is measured at the rate existing when the original loan was created. The difference between the book value and present value, at the date of recognizing the impairment, is recorded as an expense or loss. There is no reason to report overstated assets.

How well did you know this?
1
Not at all
2
3
4
5
Perfectly
179
Q

A creditor’s note receivable has a carrying value of $60,000 at the end of Year 1. Based on information about the debtor, the creditor believes the note is impaired and establishes the new carrying value of the note to be $25,000 at the end of Year 1. During Years 2 and 3, the debtor pays $14,000 on the note each year (total payments, $28,000). For Year 3, under which method of the two indicated is interest revenue recognized?

A

Interest Method - YES
Cost Recovery Method - YES

The interest method recognizes interest revenue each year until the note is collected because the note was written down to present value when the impairment was recorded. The estimated future cash flows to be received include interest, which is recognized over the remaining term of the note. The cost recovery method recognizes interest revenue only after cash equal to the new carrying value is collected. During Year 3, total collections surpassed the $25,000 new carrying value. $3,000 of interest revenue is recognized under this method in Year 3 ($28,000 − $25,000).

How well did you know this?
1
Not at all
2
3
4
5
Perfectly
180
Q

Under IFRS, a cash generating unit (CGU) is:

A

The smallest group of assets that generates independent cash flows from continuing use.

A CGU is the smallest group of assets that can be identified that generates cash flows independently of the cash flows from other assets.

How well did you know this?
1
Not at all
2
3
4
5
Perfectly
181
Q

When a note receivable is determined to be impaired,

A

A loss or expense is recognized as equal to the difference between the note carrying value and the present value of the cash flows expected to be received.

A note is considered to be impaired if the present value of remaining cash flows is less than book value, using the rate in the note. This is caused by an expected delay in timing of cash flows or reduction in amount of cash flows compared with the original agreement. The creditor makes the determination that the note is impaired and writes the note down to present value. A loss is recorded for the decline in carrying value to present value.

How well did you know this?
1
Not at all
2
3
4
5
Perfectly
182
Q

During a reporting period, a computer manufacturing company used raw materials of $50,000, had direct labor costs of $75,000, and factory overhead of $30,000. Other expenses were for advertising of $5,000, staff salaries of $10,000, and bad debt of $3,000. The company did not have a beginning balance in any inventory account. All goods manufactured during the period were sold during the period. What amount was the company’s cost of goods sold during the reporting period?

A

$155,000

Since there is no beginning or ending inventory, the cost of goods sold equals the cost of materials used ($50,000), the direct labor costs ($75,000), and the ($30,000) or $155,000. The other costs, advertising, staff salaries, and bad debt expense, are not part of cost of goods sold.

How well did you know this?
1
Not at all
2
3
4
5
Perfectly
183
Q

On June 1, Year 2, Archer, Inc. issued a purchase order to Cotton Co. for a new copier machine. The machine requires one month to produce and is shipped f.o.b. destination on July 1, Year 2, and is received by Archer on July 15, Year 2. Cotton issues a sales invoice dated July 2, Year 2, for the machine. As of what date should Archer record a liability for the machine?

A

July 15, Year 2

The term f.o.b. destination means that title transfers to the buyer when it arrives at the destination. A liability is recorded when the title transfers.

How well did you know this?
1
Not at all
2
3
4
5
Perfectly
184
Q

Delar Co. completed its year-end physical count of inventory. The inventory was valued at first-in, first-out (FIFO) costs and totaled $500,000. Delar subsequently noted the following two items:

1,000 units of inventory with a FIFO cost of $10 each were shipped and billed to a customer, f.o.b. destination. These items were included in the physical count.
6,000 units at a FIFO cost of $5 each were held on consignment for one of its suppliers, but were excluded from the physical count.
What amount should Delar report as inventory at year end?

A

$500,000

Goods on consignment are excluded from ending inventory and goods shipped f.o.b. destination are included. There is no adjustment need to the $500,000 inventory value.

How well did you know this?
1
Not at all
2
3
4
5
Perfectly
185
Q

West Retailers purchased merchandise with a list price of $20,000, subject to trade discounts of 20% and 10%, with no cash discounts allowable.

West should record the cost of this merchandise as

A

$14,400

This is a chain discount and the correct recorded cost is $20,000(1 - .20)(1- .10) = $14,400. Each successive discount in a chain discount is applied to the previous net amount.

How well did you know this?
1
Not at all
2
3
4
5
Perfectly
186
Q

On October 20, 2005, Grimm Co. consigned 40 freezers to Holden Co. for sale at $1,000 each and paid $800 in transportation costs.

On December 30, 2005, Holden reported the sale of 10 freezers and remitted $8,500. The remittance was net of the agreed 15% commission.

What amount should Grimm recognize as consignment sales revenue for 2005?

A

$10,000

Consignment sales revenue is the revenue recognized on consignment sales.

In this case, total consignment revenue is 10 × $1,000 = $10,000. The commission and transportation costs are expenses that reduce earnings on consignment revenues, but they do not affect total revenues to be recognized.

How well did you know this?
1
Not at all
2
3
4
5
Perfectly
187
Q

The following information applied to Fenn, Inc. for 2005:

Merchandise purchased for resale $400,000
Freight-in 10,000
Freight-out 5,000
Purchase returns 2,000

Fenn’s 2005 inventoriable cost was

A

$408,000

Merchandise purchased for resale $400,000
Freight-in 10,000
Purchase returns (2,000)
Total inventoriable cost $408,000

Freight-out is a delivery expense. It is not inventoried because the goods have reached salable condition before incurring this cost. Only costs that contribute to preparing inventory for sale are inventoried.

How well did you know this?
1
Not at all
2
3
4
5
Perfectly
188
Q

On December 28, 2005, Kerr Manufacturing Co. purchased goods costing $50,000. The terms were FOB destination. Some of the costs incurred in connection with the sale and delivery of the goods were as follows:

Packaging for shipment $1,000
Shipping 1,500
Special handling charges 2,000

These goods were received on December 31, 2005. In Kerr’s December 31, 2005 balance sheet, what amount of cost for these goods should be included in inventory?

A

$50,000

Kerr will pay only $50,000 for the goods. None of the other costs listed are incurred by Kerr. Rather, the seller will incur those costs.

Even the shipping costs are borne by the seller because the terms are FOB destination. This means that title does not transfer to the buyer (Kerr) until the goods reach the destination. The seller owned the goods in transit and therefore incurred the transportation cost. Kerr’s recorded cost is $50,000.

How well did you know this?
1
Not at all
2
3
4
5
Perfectly
189
Q

The following items were included in Opal Co.’s inventory account on December 31, 2004:

Merchandise out on consignment, at sales price, including 40% markup on selling price $40,000

Goods purchased, in transit, shipped FOB shipping point 36,000

Goods held on consignment by Opal 27,000

By what amount should Opal’s inventory account at December 31, 2004 be reduced?

A

$43,000

The merchandise out on consignment is included in inventory at selling price. But inventory must be measured at cost. $40,000 = cost + .40($40,000). Thus, cost = $24,000. Therefore, inventory should be reduced by the $16,000 of markup on the merchandise out on consignment.

The goods held on consignment should be removed from the inventory because these goods do not belong to Opal.

Hence, the total reduction from inventory is $43,000 ($16,000 + $27,000). The goods in transit are properly included in inventory because they were shipped FOB shipping point, which means the goods belong to Opal when the goods reach the common carrier at the shipping point.

How well did you know this?
1
Not at all
2
3
4
5
Perfectly
190
Q

Herc Co.’s inventory on December 31, 2005 was $1,500,000, based on a physical count priced at cost, and before any necessary adjustment for the following:

Merchandise costing $90,000, shipped FOB shipping point from a vendor on December 30, 2005, was received and recorded on January 5, 2006.
Goods in the shipping area were excluded from inventory although shipment was not made until January 4, 2006. The goods, billed to the customer FOB shipping point on December 30, 2005, had a cost of $120,000.
What amount should Herc report as inventory in its December 31, 2005, balance sheet?

A

$1,710,000

The correct ending inventory balance is $1,710,000 ($1,500,000 + $90,000 + $120,000).

The $90,000 of merchandise is included because it was shipped before year-end and the title was transferred to Herc at the shipping point (before year-end). The $120,000 also is included because the goods have not been shipped. The FOB designation is irrelevant because the goods have not yet reached a common carrier.

How well did you know this?
1
Not at all
2
3
4
5
Perfectly
191
Q

The following information pertains to Deal Corp.’s 2004 cost of goods sold:

Inventory, 12/31/03 $ 90,000
2004 purchases 124,000
2004 write-off of obsolete inventory 34,000
Inventory, 12/31/04 30,000

The inventory written off became obsolete due to an unexpected and unusual technological advance by a competitor. In its 2004 income statement, what amount should Deal report as cost of goods sold?

A

$150,000

Beginning inventory	$ 90,000
Plus purchases	124,000
Less write-off	(34,000)
Less ending inventory	(30,000)
Equals cost of goods sold	$150,000
The write-off cannot be counted in cost of goods sold because it is a decrease in inventory not associated with sales.
How well did you know this?
1
Not at all
2
3
4
5
Perfectly
192
Q

The following information pertained to Azur Co. for the year:

Purchases	$102,800
Purchase discounts	10,280
Freight-in	15,420
Freight-out	5,140
Beginning inventory	30,840
Ending inventory	20,560

What amount should Azur report as cost of goods sold for the year?

A

$118,220

Cost of goods sold is determined (in a periodic inventory system) as:

Beginning Inventory $ 30,840

+ Net Purchases $107,940

= Goods Available for Sale $138,780

  • Ending inventory 20,560

= Cost of Goods Sold $118,220 Note net purchase is computed as

+ Purchases $102,800

  • Net Purchases 107,940

= Goods Available for Sale $138,780

  • Purchases Discounts (10,280)

+ Freight-in 15,420

Net Purchases includes any purchase discounts (or allowances) and other cost of getting the goods in place and condition for resale, including freight-in. Freight-out (to customers) is a selling cost. Therefore, Azur Co.’s cost of goods sold would be:

How well did you know this?
1
Not at all
2
3
4
5
Perfectly
193
Q

The following information was taken from Cody Co.’s accounting records for the year ended December 31, 2005:

Decrease in raw materials inventory $ 15,000
Increase in finished goods inventory 35,000
Raw materials purchased 430,000
Direct labor payroll 200,000
Factory overhead 300,000
Freight-out 45,000

There was no work-in-process inventory at the beginning or end of the year. Cody’s 2005 cost of goods sold is

A

$910,000

The correct answer is $910,000:

Raw materials purchased	$430,000
Plus decrease in raw materials	15,000*
Direct labor	200,000
Factory overhead	300,000
Less finished goods increase	(35,000) **
Cost of goods sold	$910,000

*The decrease in raw materials is added to the amount purchased resulting in the cost of materials incorporated into production. In other words, $15,000 of materials purchased in 2005 were placed into production in 2005. The total cost of materials brought into production in 2005 equals $445,000.

** The increase in finished goods represents costs incurred in the current period to finish inventory that was not sold in the current period. Therefore, these costs must be removed in determining cost of goods sold.

Freight-out is not a manufacturing cost but rather is a distribution cost.

Therefore, freight-out is not inventoried.

There is no change in work-in-process inventory to affect the calculation.

How well did you know this?
1
Not at all
2
3
4
5
Perfectly
194
Q

Beck Co.’s inventory of trees is as follows:

Beginning Inventory		10 trees @ $ 50
March 4	purchased	6 trees @ 55
March 12	sold	8 trees @ 100
March 20	purchased	9 trees @ 60
March 27	sold	7 trees @ 105
March 30	purchased	4 trees @ 65

What was Beck’s cost of goods sold using the last in, first out (LIFO) perpetual method?

A

$850

The perpetual method recognizes each purchase and sale at the time it occurs. The total value of cost of sales using perpetual LIFO is ((6 × $55) + (2 × $50) + (7 × $60)) = $850, as shown in the table below.

Trees remaining Cost of trees sold

Begin Inv 10 @ $ 50
10 @ $50
March 4 purchased 6 @ 55

10 @ $50

6 @ 55

March 12 sold 8 @ 100 8 @ 50

6 @ $55 = $330

2 @ $50 = 100

March 20 purchased 9 @ 60

8 @ 50

9 @ 60

March 27 sold 7 @ 105

8 @ 50

2 @ 60

7 @ $60 = 420

March 30 purchased 4 @ 65
8 @ 50

2 @ 60

4 @ 65

Total cost = $850

How well did you know this?
1
Not at all
2
3
4
5
Perfectly
195
Q
Units	Unit cost	Total cost	Units on hand
Balance on 1/1	2,000	$1	$2,000	2,000
Purchased on 1/8	1,200	3	3,600	3,200
Sold on 1/23	1,800			1,400
Purchased on 1/28	800	5	4,000	2,200
Nest uses the LIFO method to cost inventory. What amount should Nest report as inventory on January 31 under each of the following methods of recording inventory?
A

Under the LIFO (last-in, first-out) inventory method, goods sold are assumed to be the most recently acquired goods (at their related costs). Therefore, goods remaining (ending inventory) are assumed to be the earliest acquired goods (at their related costs). If the perpetual LIFO inventory method is used, when goods are sold, they are assumed to be the goods acquired just prior to the sale.

Thus, Nest’s sale of 1,800 units on 1/23 would have consisted of the 1,200 units acquired 1/8 and 600 units (of the 2,000) in beginning inventory. Ending inventory on January 31 would be:

1,400 units of beginning inventory @ $1 each = $1,400
800 units purchased 1/28 @ $5 each = 4,000
2,200 units in ending inventory reported @ = $5,400
If the periodic LIFO inventory method is used, ending inventory (and cost of goods sold) are determined only at the end of the period. Therefore, Nest’s sale of 1,800 units on 1/23 would have consisted of (by assumption at the end of the period) 800 units acquired on 1/28 and 1,000 units (of the 1,200) acquired on 1/8. Ending inventory on January 31 would be:

200 units of the 1,200 purchased 1/8 @ $3 = $ 600
2,000 units (all) of beginning inventory @ $1 = 2,000
2,200 units in ending inventory reported @ = $2,600

How well did you know this?
1
Not at all
2
3
4
5
Perfectly
196
Q

During periods of inflation, a perpetual inventory system would result in the same dollar amount of ending inventory as a periodic inventory system under which of the following inventory valuation methods?

A

FIFO - Yes
LIFO - No

Under a perpetual inventory system, the cost of goods sold (COGS) is determined at the time of each sale. In a perpetual FIFO inventory system, the cost of each sale (COGS) would be based on the cost of the earliest acquired goods on hand at the time of the sales. The cost of the most recently acquired goods would remain in ending inventory. In a perpetual LIFO inventory system, the cost of each sale (COGS) would be based on the cost of goods acquired just prior to the sale. The cost of the earlier acquired goods would remain in inventory.

Under a periodic inventory system, the costs of goods sold (COGS) and ending inventory are determined only at the end of the period. In a periodic FIFO inventory system, the cost of sales for the period (COGS) would be based on the cost of the earliest acquired goods available during the period. The cost of the most recently acquired goods would remain in ending inventory. In a periodic LIFO inventory system, the cost of sales for the period (COGS) would be based on the last goods acquired during the period. The cost of the earliest acquired goods would remain in ending inventory. The above descriptions can be summarized as follows for determination of COGS:

Inventory System/Method Cost of goods sold determined using FIFO Cost of goods sold determined using LIFO
Perpetual Earliest goods acquired Latest goods acquired prior to each sale
Periodic Earliest goods acquired Latest goods acquired during the period
Since cost of goods sold for the period would be the same under both perpetual FIFO and periodic FIFO, ending inventory would be the same under both perpetual FIFO and periodic FIFO. Cost of goods sold (and ending inventory) would not be the same under perpetual LIFO as under periodic LIFO because the perpetual system recognizes cost of goods sold based on the cost of goods acquired just prior to each sale, whereas the periodic system recognizes cost of goods sold based on cost of goods acquired prior to the end of each period.

How well did you know this?
1
Not at all
2
3
4
5
Perfectly
197
Q

Ashe Co. recorded the following data pertaining to raw material × during January 2005:

Units
Date	Received	Cost	Issued	On Hand
1/1/05 Inventory		$8.00		3,200
1/11/05 Issue			1,600	1,600
1/22/05 Purchase	4,800	$9.60		6,400

The moving-average unit cost of × inventory on January 31, 2005 is

A

$9.20

Units beginning inventory remaining at year-end (3,200 − 1,600)$8 = $12,800
Plus 1/22 purchase: 4,800($9.60) = 46,080
Ending inventory $58,880
Ending unit cost: $58,880/6,400 = $9.20
The moving average method costs issues at the unit cost of goods on hand at that point. Thus, the issue was costed at $8.00 per unit. The cost per unit changes with each purchase.

How well did you know this?
1
Not at all
2
3
4
5
Perfectly
198
Q

Generally, which inventory costing method approximates most closely the current cost for each of the following?

A

Cost of goods sold - LIFO
Ending inventory - FIFO

LIFO assumes the sale of the most recent purchases first and thus results in cost of goods sold that is the most current value. FIFO assumes the sale of the earliest purchases first (and beginning inventory before any purchases) and thus results in ending inventory that is the most current value. FIFO is sometimes called LISH: last in still here.

How well did you know this?
1
Not at all
2
3
4
5
Perfectly
199
Q

When the FIFO inventory method is used during periods of rising prices, a perpetual inventory system results in an ending inventory cost that is

A

The same as in a periodic inventory system.

FIFO produces the same results for periodic and perpetual systems. FIFO always assumes the sale of the earliest goods acquired. Therefore, unlike LIFO periodic, goods can never be assumed sold before they are acquired.

Cost of goods sold and ending inventory are the same under FIFO for both a periodic and a perpetual system.

How well did you know this?
1
Not at all
2
3
4
5
Perfectly
200
Q

A company decided to change its inventory valuation method from FIFO to LIFO in a period of rising prices. What was the result of the change on ending inventory and net income in the year of the change?

A

Ending inventory - Decrease
Net income -Decrease

Ending inventory would decrease because under LIFO, the latest items purchased (and therefore the most costly) are considered sold, leaving the earliest items purchased (and therefore the least costly) in inventory. This is opposite to the effect under FIFO.

The same is true for net income because now, under LIFO, cost of goods sold is increased relative to FIFO because the cost of the latest and most costly items are considered sold first.

How well did you know this?
1
Not at all
2
3
4
5
Perfectly
201
Q

Which inventory costing method would a company that wishes to maximize profits in a period of rising prices use?

A

FIFO

FIFO assumes the sale of the earliest goods first. With rising prices, the earliest goods reflect the lowest prices. Therefore, cost of goods sold under FIFO is the lowest of the cost flow assumptions. With the lowest cost of goods sold, gross margin and income are the highest among the available cost flow assumptions (LIFO and average being the others).

How well did you know this?
1
Not at all
2
3
4
5
Perfectly
202
Q

In 2005, Cobb adopted the dollar-value LIFO inventory method.

At that time, Cobb’s ending inventory had a base-year cost and an end-of-year cost of $300,000. In 2006, the ending inventory had a $400,000 base-year cost and a $440,000 end-of-year cost.

What dollar-value LIFO inventory cost would be reported in Cobb’s December 31, 2006, balance sheet?

A

$410,000

The price level index for 2006 is 1.1 ($440,000/$400,000). Ending 2006 DV LIFO inventory equals the beginning inventory DV LIFO plus the increase in inventory at base-year dollars converted to 2006 prices:

Ending DV LIFO = Beginning DV LIFO + (increase at base-year dollars)(1.1)

= $300,000 + ($400,000 − $300,000)(1.1) = $410,000.

How well did you know this?
1
Not at all
2
3
4
5
Perfectly
203
Q

In January, Stitch, Inc. adopted the dollar-value LIFO method of inventory valuation. At adoption, inventory was valued at $50,000. During the year, inventory increased $30,000 using base-year prices, and prices increased 10%. The designated market value of Stitch’s inventory exceeded its cost at year-end. What amount of inventory should Stitch report in its year-end balance sheet?

A

$83,000

Beginning inventory of $50,000 is at base-year dollars and the current year increase of $30,000 is also at base-year dollars. The current year layer must be converted to current year costs ($30,000 × 1.10) = $33,000. Ending dollar value LIFO is the beginning dollar value LIFO (in this case it was adopted in January so the beginning inventory must be $50,000) plus the current year layer of $33,000 or $83,000. Note that the sentence “The designated market value of Stitch’s inventory exceeded its cost at year end” is a distracter. It is simply stating that there is not an issue with the lower of cost or market since cost is lower.

How well did you know this?
1
Not at all
2
3
4
5
Perfectly
204
Q

Bach Co. adopted the dollar-value LIFO inventory method as of January 1, 2006.

A single inventory pool and an internally computed price index are used to compute Bach’s LIFO inventory layers. Information about Bach’s dollar-value inventory follows:

Inventory
Date at base-year cost at current-year cost
1/1/06 $90,000 $90,000
2006 layer 20,000 30,000
2007 layer 40,000 80,000

What was the price index used to compute Bach’s 2007 dollar-value LIFO inventory layer?

A

1.33

The question provides the ending inventory for 2007 at current cost by layer. The sum of the current cost column ($200,000) is the current cost of the entire inventory at the end of 2007. The sum of the base-year cost for the three years is $150,000. Hence, under this assumption, the ratio of current cost for the total inventory at the end of 2007 to the base-year cost is 1.33 ($200,000/$150,000). This index is then multiplied by the 2007 layer in base-year dollars to derive the increment to DV LIFO ending inventory.

How well did you know this?
1
Not at all
2
3
4
5
Perfectly
205
Q

Brock Co. adopted the dollar-value LIFO inventory method as of January 1, 2003. A single inventory pool and an internally computed price index are used to compute Brock’s LIFO inventory layers. Information about Brock’s dollar-value inventory follows:

Inventory
_________________________________
Date at base-year cost at current-year cost at
dollar-
value LIFO
1/1/03 $40,000 $40,000 $40,000
2003 layer 5,000 14,000 6,000
__________ __________ __________ __________
12/31/03 45,000 54,000 46,000
2004 layer 15,000 26,000 ?
__________ __________ __________ __________
12/31/04 $60,000 $80,000 ?
========= ========== ==========
What was Brock’s dollar-value LIFO inventory on December 31, 2004?

A

$66,000

The ending inventory is used to construct the internal price index. At the end of 2004, the ratio of current cost to base-year cost for ending inventory is $80,000/$60,000 = 1 1/3 or 4/3. This ratio is applied to the 2004 layer at base-year cost, yielding $20,000 ($15,000 × 4/3). This amount is the increase to DV LIFO. Therefore, ending 2004 DV LIFO is $66,000 ($46,000 + $20,000).

How well did you know this?
1
Not at all
2
3
4
5
Perfectly
206
Q

Lyon Co. estimated its ending inventory using a method based on the financial statements of prior periods in order to prepare its quarterly interim financial statements. What type of inventory system and method of estimating ending inventory is Lyon using?

A

The gross profit method can be used to estimate interim inventory, and the gross profit method is a periodic inventory system.

How well did you know this?
1
Not at all
2
3
4
5
Perfectly
207
Q

A flash flood swept through Hat, Inc.’s warehouse on May 1. After the flood, Hat’s accounting records showed the following:

Inventory, January 1	$ 35,000
Purchases, January 1 through May 1	200,000
Sales, January 1 through May 1	250,000
Inventory not damaged by flood	30,000
Gross profit percentage on sales	40%

What amount of inventory was lost in the flood?

A

$55,000

The gross margin method of estimating inventory is used to solve this problem. The cost of inventory lost cannot be identified by count but it can be estimated.

First, an estimate of cost of goods sold is subtracted from the cost of goods available on the date of the flood yielding the total amount of inventory that would have been present on May 1.

Second, the amount of inventory not lost is subtracted from the May 1 estimated total inventory. The result is an estimate of the amount lost.

With gross profit being 40% of sales, cost of goods sold must be 60% of sales, on average. Therefore, the estimate of cost of goods sold is $150,000 (.60 × $250,000). Beginning inventory ($35,000) + Purchases ($200,000) = Goods available = $235,000. Subtracting $150,000 of cost of goods sold yields $85,000 of inventory on May 1 ($235,000 − $150,000).

With $30,000 of inventory still accounted for, the amount of lost inventory at cost is $55,000 ($85,000 − $30,000).

How well did you know this?
1
Not at all
2
3
4
5
Perfectly
208
Q

The following two inventory items were purchased as a group in a liquidation sale for $1,000.

   Replacement	Carrying Value
    Item	Cost	On Seller's Books A	$400	            $390 B	   700	              755

The firm purchasing the inventory records item A at what amount?

A

$364

When items are purchased as a group, the total cost of the group is allocated to the individual items based on fair value. Replacement cost is the appropriate value to use in this case. The total replacement cost of the items is $1,100 ($400 + $700). Therefore, Item A is allocated 4/11 of the purchase cost, or $364 = ($400/$1,100)$1,000.

How well did you know this?
1
Not at all
2
3
4
5
Perfectly
209
Q

When marking up a specific line of household items for resale, a retailer computes its markup as 40% of cost. For purposes of estimating ending inventory using the gross margin method, what percentage is applied to sales when estimating cost of goods sold?

A

71

The gross margin method applies the cost to sales ratio to sales in order to derive an estimate of cost of goods sold. Subtracting the resulting estimate of cost of goods sold from the cost of goods available for sale yields an estimate of ending inventory without counting the items. This firm determines the selling price to be 140% of cost because the markup is 40% of cost. Cost plus markup yields selling price. Therefore, the cost to sales ratio is 1.00/1.40 or .71.

How well did you know this?
1
Not at all
2
3
4
5
Perfectly
210
Q

A firm’s inventory was destroyed by fire on August 14 of the current year. Fortunately, the firm had insurance to cover the loss. However, most of the inventory records were also destroyed in the fire. The average gross margin percentage is 40%, beginning inventory was $200,000, and $1,000,000 of purchases had been made through August 13. The firm had recorded sales of $1,200,000 through that date. Estimate the cost of the inventory lost in the fire.

A

$480,000

The gross margin method estimates the cost of inventory at the time of the fire as follows: Beginning inventory $200,000 + $1,000,000 Purchases = Ending inventory + Cost of goods sold. The estimate of cost of goods sold is found by multiplying the cost to sales ratio and sales. The gross margin percentage plus the cost to sales ratio is 1; therefore, cost/sales is .60 (= 1 - .40). Estimated cost of goods sold is $720,000 (= .60($1,200,000)). The equation is Beginning inventory $200,000 + Purchases $1,000,000 = Ending inventory + Cost of goods sold $720,000. Solving for ending inventory yields $480,000.

How well did you know this?
1
Not at all
2
3
4
5
Perfectly
211
Q

How does the retail inventory method establish the lower-of-cost-or-market valuation for ending inventory?

A

By excluding net markdowns from the cost-to-retail ratio.

Although the result is approximate, by excluding net markdowns from the denominator of the cost-to-retail ratio, the ratio is a smaller amount, resulting in a lower ending inventory valuation.

How well did you know this?
1
Not at all
2
3
4
5
Perfectly
212
Q

Data for a firm using the FIFO-LCM retail inventory method is as follows:

                                          Cost	        Retail Beginning inventory	     $ 300	        $ 467 Net purchases	              1,200	        2,000 Net additional markups		                   100 Net markdowns		                         (300) Sales		                                       $1,700

Compute cost of goods sold.

A

$1,169

Ending inventory at retail = $567 (= $467 + $2,000 + $100-$300-$1,700). The cost-to-retail ratio includes both the beginning inventory amounts, purchases and net markdowns. C/R = $300 + $1,200/($467 + $2,000 + $100) = .5843. Ending inventory at cost = $567(.5843) = $331. Cost of goods sold = $300 + $1,200 - $331 = $1,169.

How well did you know this?
1
Not at all
2
3
4
5
Perfectly
213
Q

Union Corp. uses the conventional retail method of inventory valuation. The following information is available:

                                        Cost	      Retail Beginning inventory	 $12,000	    $ 30,000 Purchases	                  60,000	       110,000 Net additional markups		                10,000 Net markdowns		                       20,000 Sales revenue		                       90,000

If the lower of cost or market rule is disregarded, what would be the estimated cost of the ending inventory?

A

$19,200

The cost to retail ratio under FIFO is: [$12,000 + $60,000/($30,000 + $110,000 + $10,000)] = .48.

Ending inventory at retail is $30,000 + $110,000 + $10,000 − $20,000 − $90,000 = $40,000.

Ending inventory at cost, therefore, is .48($40,000) = $19,200.

How well did you know this?
1
Not at all
2
3
4
5
Perfectly
214
Q

The retail inventory method includes which of the following in the calculation of both cost and retail amounts of goods available for sale?

A

Purchase returns.

The retail method measures beginning inventory and net purchases at both cost and retail. It then applies the average relationship between cost and retail (based on beginning inventory and purchases) to ending inventory at retail to determine ending inventory at cost.

Purchase returns reduce net purchases at both cost and retail because returns represent amounts included in gross purchases that are not available for sale.

How well did you know this?
1
Not at all
2
3
4
5
Perfectly
215
Q

Choose the correct inclusions to the cost-to-retail ratio computation under the dollar-value LIFO retail method.

A

Beginning Inventory - No
Net Markdowns - YES

DV LIFO retail uses the FIFO (not LCM) cost-to-retail ratio. Under LIFO, a layer added during a period should reflect only the cost and retail amounts pertaining to that period. Thus, beginning inventory amounts are not used in calculating the ratio. Also, because LIFO may contain inventory layers for several preceding periods, excluding net markdowns is not an effective way to accomplish the LCM valuation objective. Thus, net markdowns are included in the cost to retail computation.

How well did you know this?
1
Not at all
2
3
4
5
Perfectly
216
Q

A firm uses the dollar value LIFO retail method and has $2,000 in beginning inventory at retail at the beginning of the current year. The base year equivalent of this amount is $1,600. The base year index is 1.00. The beginning inventory reported in the Balance Sheet is $800. During the current year, the firm purchased $12,000 of inventory at cost and marked that up to $40,000. Sales for the year were $28,000. The relevant ending price index is 1.60. What amount does this firm report as inventory in its Balance Sheet at the end of the current year?

A

$4,232

EI retail, current index = $2,000 + $40,000-$28,000 = $14,000

EI retail, base = $14,000/1.6 = $8,750

Increase in EI retail, base = $8,750-$1,600 = $7,150

Increase in EI retail, current = $7,150(1.6) = $11,440

C/R (use FIFO, not LCM) = $12,000/$40,000 = .30

Increase in EI, cost = .30($11,440) = $3,432

EI, cost = $800 + $3,432= $4,232

How well did you know this?
1
Not at all
2
3
4
5
Perfectly
217
Q

Information for a firm using the dollar value (DV) LIFO retail method follows. The cost to retail (C/R) is provided along with price level indices. The data reflects the use of the method through year one.

            Retail	        Retail	             DV LIFO Layer	Base	Index	Current	  C/R	Cost Base	$200	1.00  	$200	.40	        $80 year one	  80	         1.10	            88	.34	        $30

For year two, ending inventory at retail (by count) totaled $450. The ending price-level index for the year was 1.15. The cost-to-retail ratio was .42. What is the ending inventory for financial reporting purposes for this firm?

A

$164

The DV LIFO retail process applies the DV LIFO method to retail dollars, and then deflates the retail layer added, now reflecting current prices, to cost, using the cost-to-retail ratio. The calculations are:

Ending inventory, retail, at base = $450(1.00/1.15) = $391

Increase in retail, current = $111(1.15/1.00) = $128

Increase in cost = $128(.42) = $54

Ending inventory at cost = ($80 + $30) + $54 = $164

How well did you know this?
1
Not at all
2
3
4
5
Perfectly
218
Q

Moss Co. has determined its December 31, 20X4 inventory to be $400,000 on a FIFO basis. Information pertaining to that inventory follows:

Estimated selling price $408,000
Estimated cost of disposal 20,000
Normal profit margin 60,000
Current replacement cost 360,000

Moss records losses that result from applying lower of cost or net realizable value. On December 31, 20X4, what should be the net carrying value of Moss’ inventory?

A

$388,000

Lower of cost or net realizable value applies to inventories that are carried at FIFO or Average cost. Net realizable value is selling price less cost of disposal. In this case it is $408,000 − $20,000 = $388,000.

How well did you know this?
1
Not at all
2
3
4
5
Perfectly
219
Q

The replacement cost of an inventory item is below the net realizable value and above the net realizable value less the normal profit margin. The original cost of the inventory item is below the net realizable value less the normal profit margin.

Under the lower of cost or market method, the inventory item should be valued at

A

Original cost.

In LCM, market value is replacement cost if replacement cost is between the ceiling value (net realizable value) and the floor value (net realizable value less normal profit margin).

This is the situation in this question. The original cost is below the floor value. Thus, market exceeds cost and the item is recorded at cost (lower of cost or market).

How well did you know this?
1
Not at all
2
3
4
5
Perfectly
220
Q

Kahn Co., in applying the lower of cost or market method, reports its inventory at replacement cost. Which of the following statements are correct?

The original cost is greater
than replacement cost

The net realizable value, less a
normal profit margin, is greater
than replacement cost

A

The original cost is greater
than replacement cost - YES

The net realizable value, less a
normal profit margin, is greater
than replacement cost - NO

Under LCM, the market value of inventory is the middle of three figures (in amount):

replacement cost
net realizable value
net realizable value less normal profit margin.
If the middle figure (market) is less than cost, then the inventory is reported at market. The inventory in this question is reported at replacement cost, which means that replacement cost is market value and replacement cost is less than cost. Also, replacement cost is the middle of the three figures (or tied with one of the other two).

Net realizable value less normal profit margin could not exceed replacement cost because that would imply that replacement cost is the lowest of the three figures, which contradicts the fact that replacement cost is market value.

Therefore, in terms of the question,

(1) original cost is greater than replacement cost, and
(2) net realizable value less normal profit margin is not greater than replacement cost.

How well did you know this?
1
Not at all
2
3
4
5
Perfectly
221
Q

At the end of the year, Ian Co. determined its inventory to be $258,000 on a LIFO (last in, first out) basis. The current replacement cost of this inventory was $230,000. Ian estimates that it could sell the inventory for $275,000 at a disposal cost of $14,000. If Ian’s normal profit margin for its inventory was $10,000, what would be its net carrying value?

A

$251,000

The “ceiling” for LCM (lower of cost or market) valuation is $261,000 net realizable value ($275,000 selling price less $14,000 disposal cost). The “floor” is net realizable value less normal profit margin or $251,000 ($261,000 − $10,000). Replacement cost of $230,000 is below the floor so “market” value is the floor, or middle, of the three amounts ($251,000). This amount is less than cost of $258,000. Therefore, the lower of cost or market valuation is $251,000.

How well did you know this?
1
Not at all
2
3
4
5
Perfectly
222
Q

The original cost of an inventory item is above the replacement cost. The inventory item’s replacement cost is above the net realizable value. Under the lower of cost or market method, the inventory item should be valued at

A

Net realizable value.

Inventory must be carried at lower of cost (such as LIFO or market. Market is replacement cost subject to a ceiling and floor. The ceiling for replacement cost is net realizable value (selling price less cost to complete) and the floor is net realizable value less normal profit margin. Use simple numbers to help solve this abstract question. In this question original cost (assume = 100) is greater than market ((replacement cost) assume = 80). Market (80) is greater than net realizable value (assume = 70). Market is subject to a ceiling of net realizable value (70). In this case the inventory would be valued at net realizable value.

How well did you know this?
1
Not at all
2
3
4
5
Perfectly
223
Q

The replacement cost of an inventory item is below the net realizable value and above the net realizable value less a normal profit margin. The inventory item’s original cost is above the net realizable value. Under the lower of cost or market method, the inventory item should be valued at

A

Replacement cost

The easiest way to answer a question like this is to make up simple numbers. The following simple numbers were made up to fit the abstract information in the question. Lower of cost or market states you record the inventory at the lower of original cost or market value (replacement cost) within the range of a ceiling and a floor. The numbers below show that replacement cost is lower than original cost and within the floor and ceiling. Replacement cost is the correct answer.

Original cost $10
Net realizable value 9
Replacement cost 8
NRV less normal PM 7

How well did you know this?
1
Not at all
2
3
4
5
Perfectly
224
Q

When an inventory overstatement in year one counterbalances in year two, this means:

A

A prior period adjustment is recorded if the error is discovered in year two.

Counterbalancing simply means that the effect of the inventory error in the second year is opposite that of the first year. Discovery in year two provides an opportunity for the firm to correct year two beginning retained earnings, which is overstated by the error in year one. The overstatement of inventory in year one caused cost of goods sold to be understated and income overstated in year one. The prior period adjustment, dated as of the beginning of year two, is a debit to retained earnings for the after-tax effect of the income overstatement in year one. Inventory is credited for the amount of the overstatement. This allows year two to begin with corrected balances.

How well did you know this?
1
Not at all
2
3
4
5
Perfectly
225
Q

If ending inventory for 20x5 is understated because certain items were missed in the count, then:

A

Net income for 20x5 will be understated and CGS for 20x6 will be understated.

Use the equation BI + PUR = EI + CGS. When EI is understated, CGS must be overstated to maintain the equation. Net income, therefore, is understated (20x5). Then next year, BI is also understated because BI for 20x6 is EI for 20x5. Using the equation, if BI is understated, CGS is also understated to maintain the equation.

How well did you know this?
1
Not at all
2
3
4
5
Perfectly
226
Q

Bren Co.’s beginning inventory at January 1, 2005 was understated by $26,000, and its ending inventory was overstated by $52,000. As a result, Bren’s cost of goods sold for 2005 was:

A

Understated by $78,000.

The effect of the beginning-inventory error is to understate cost of goods sold $26,000. The effect of the ending-inventory error is to understate cost of goods sold $52,000. The total effect then is to understate cost of goods sold $78,000.

These effects are analyzed by using the equation:

Beginning inventory + Purchases-Ending inventory = Cost of goods sold

For example, if beginning inventory is understated, then the right hand side of the equation (cost of goods sold) must also be understated by the same amount.

How well did you know this?
1
Not at all
2
3
4
5
Perfectly
227
Q

Losses on purchase commitments are recorded at the end of the current year when:

A

The contractual cost of the inventory in an irrevocable purchase contract exceeds the current cost.

Both qualities are required for a loss to be recognized. The firm must honor a contract in a later period by paying more than current cost and, thus, is in a loss position at the end of the current year.

How well did you know this?
1
Not at all
2
3
4
5
Perfectly
228
Q

At the end of 20x4, a firm recognized a loss on a contractual commitment to purchase inventory for $60,000. The value of the inventory at the end of 20x4 is $52,000. When the inventory was actually purchased in 20x5, its value had risen to $62,000. Choose the correct statement concerning reporting in 20x5.

A

The inventory is recorded at $60,000.

The maximum recorded value of the inventory is $60,000, which is the contractual amount and, also, the cost. If the firm can sell the inventory for more than $60,000, then gross margin will be recognized. The value of the inventory more than fully recovered, but gains are limited to the amount of previously recognized losses, which in this case, is $8,000.

How well did you know this?
1
Not at all
2
3
4
5
Perfectly
229
Q

A corporation entered into a purchase commitment to buy inventory. At the end of the accounting period, the current market value of the inventory was less than the fixed purchase price, by a material amount. Which of the following accounting treatments is most appropriate?

A

Describe the nature of the contract in a note to the financial statements, recognize a loss in the Income Statement, and recognize a liability for the accrued loss.

The firm has committed to a fixed price but must recognize the loss in the period the decline in price occurred, much like under lower-of-cost-or-market. Inventory is not reduced because the firm has not purchased the inventory under contract. There is no asset to reduce, but the decrease in net assets is accomplished by recording the liability for the portion of the purchase price that has no value.

How well did you know this?
1
Not at all
2
3
4
5
Perfectly
230
Q

In October of year one, a firm committed to a purchase of inventory at a total cost of $26,000. The contract is irrevocable and specifies a delivery date in March of year two. At the end of year one, the market value of the inventory under contract is worth $23,000 at current cost. Choose the correct reporting for the year one financial statements:

A

A liability of $3,000 is reported in the Balance Sheet.

The firm has committed to a purchase for a total cost of $26,000, but at year-end, the value of the item to be received is $3,000 less. The firm cannot postpone the loss and liability recognition because the reduction in the firm’s earnings and net assets has already occurred. The economic events causing the loss have occurred as of the Balance Sheet date.

How well did you know this?
1
Not at all
2
3
4
5
Perfectly
231
Q

As of December 31, Year 2, a company has an inventory item that was originally purchased for $80 in Year 1. The inventory item was written down to its net realizable value of $60 as of December 31, Year 1. As of December 31, Year 2, the inventory item had a net realizable value of $75 and a replacement cost of $65. Normal profit margins for this company are 20%. Under IFRS, what is the carrying amount of the inventory item as of December 31, Year 2?

A

$75

Under IFRS, the inventory would be carried at the lower of cost or NRV. The NRV at the end of Year 2 is $75.

How well did you know this?
1
Not at all
2
3
4
5
Perfectly
232
Q

At the end of year 1, a company reduced its inventory cost from $100 to its net realizable value of $80. As of the end of year 2, the inventory was still on hand, and its net realizable value increased to $150. Under IFRS, what journal entry should the company record for year 2 to properly report the inventory value?

A

Debit inventory for $20 and credit expense for $20.

Under IFRS, inventory is carried at the lower of cost or net realizable value. Recovery of previous write-downs is allowed. Therefore, in year 2, the company can recover the previous write-down of $20 ($100 – 80) but cannot write the inventory above the original cost. The entry to record the recovery is a debit to inventory and credit to expense or cost of goods sold for $20.

How well did you know this?
1
Not at all
2
3
4
5
Perfectly
233
Q

The following information relates to a company’s year-end inventory:

Inventory cost $ 910
Selling price of inventory $1,000
Normal profit margin 10% of selling price
Current replacement cost $740
Cost of completion and disposal $100

Under IFRS, what is the company’s year-end inventory balance?

A

$900

IFRS reports inventory at lower of cost or net realizable value. Net realizable value is the selling price less the cost to complete or dispose of the inventory. The net realizable value is the selling price of $1,000 less the cost to complete of $100, or $900. Net realizable value is less than the cost ($910) so the inventory is reported at $900.

How well did you know this?
1
Not at all
2
3
4
5
Perfectly
234
Q

A company issued a purchase order on December 15, Year 1, for a piece of capital equipment that costs $100,000. The capital equipment was shipped from the vendor on December 31, Year 1, and received by the company on January 5, Year 2. The equipment was installed and placed in service on February 1, Year 2. On what date should the depreciation expense begin?

A

February 1, Year 2

Depreciation expense should begin on the date that the asset is placed into service and therefore, contributing to the generation of revenues. The depreciation expense should begin on February 1, Year 2.

How well did you know this?
1
Not at all
2
3
4
5
Perfectly
235
Q

A corporation issued debt to purchase 10 acres of land for development purposes. Expenditures related to this purchase are as follows:

Description	Amount
Purchase price	$1,000,000
Real estate taxes in arrears	15,000
Debt issuance costs	2,000
Attorney fee—title search on land	5,000

The company should record its acquisition of the land in its financial statements at a value of

A

$1,020,000

The costs capitalized to the land are all costs to get the land ready for use (development). Those costs are: the cost of the land, the real estate taxes in arrears, and the attorney fee for the title search.

debt issue costs are a financing cost

How well did you know this?
1
Not at all
2
3
4
5
Perfectly
236
Q

Newt Co. sold a warehouse and used the proceeds to acquire a new warehouse. The excess of the proceeds over the carrying amount of the warehouse sold should be reported as a(an):

A

Part of continuing operations.

The gain or loss on the sale of an asset is part of continuing operations as it is expected that a company will sell existing assets from time to time as the assets are replaced.

GAAP does not allow the gain or loss on the sale of the old asset to adjust the cost of the new asset.

The sale of a single asset is not a segment of the business; therefore, the warehouse is not a discontinued operation.

The FASB eliminated the category for extraordinary gains and losses.

How well did you know this?
1
Not at all
2
3
4
5
Perfectly
237
Q

Talton Co. installed new assembly line production equipment at a cost of $185,000. Talton had to rearrange the assembly line and remove a wall to install the equipment. The rearrangement cost was $12,000 and the wall removal cost was $3,000. The rearrangement did not increase the life of the assembly line but it did make it more efficient. What amount of these costs should be capitalized by Talton?

A

$200,000

This response includes all the costs to get the equipment ready for use. The rearrangement costs and the wall removal costs were needed to put the equipment into use. The rearrangement costs made the production more efficient.

How well did you know this?
1
Not at all
2
3
4
5
Perfectly
238
Q

On December 1, 20X5, East Co. purchased a tract of land as a factory site for $300,000. The old building on the property was razed and salvaged materials resulting from demolition were sold.

Additional costs incurred and salvage proceeds realized during December 20X5 were as follows:

Cost to raze old building $25,000
Legal fees for purchase contract and to record ownership 5,000
Title guarantee insurance 6,000
Proceeds from sale of salvaged materials 4,000

In East’s December 31, 20X5 Balance Sheet, what amount should be reported as land?

A

$332,000

The correct answer, $332,000, equals: $300,000 + $25,000-$4,000 + $5,000 + $6,000.

The net cost to raze the old building ($21,000) is capitalized to land because it is a cost necessary to bring the land into its intended condition. The legal fees and title guarantee cost, likewise, must be incurred to avoid future legal problems, and thus contribute to the value of the land.

How well did you know this?
1
Not at all
2
3
4
5
Perfectly
239
Q

On October 1, 20X4, Shaw Corp. purchased a machine for $126,000 that was placed in service on November 30, 20X4. Shaw incurred additional costs for this machine, as follows:

Shipping $ 3,000
Installation 4,000
Testing 5,000

In Shaw’s December 31, 20X4 Balance Sheet, the machine’s cost should be reported as:

A

$138,000

Every cost listed is a cost necessary to place the asset into its intended condition and location. This is the general rule for capitalizing costs to plant assets.

Thus, all four costs are capitalized to the machine yielding a final capitalized value of $138,000 ($126,000 + $3,000 + $4,000 + $5,000).

How well did you know this?
1
Not at all
2
3
4
5
Perfectly
240
Q

Derby Co. incurred costs to modify its building and to rearrange its production line. As a result, an overall reduction in production costs is expected. However, the modifications did not increase the building’s market value, and the rearrangement did not extend the production line’s life.

Should the building modification costs and the production line rearrangement costs be capitalized?

A

Building modification costs - YES
Production line rearrangement costs - YES

The criterion for capitalizing post-acquisition costs is not whether the market value of the overall asset is increased. Rather, the criteria are (1) increase in useful life or (2) increase in productivity or efficiency including cost reduction.

An overall reduction in production costs meets the second criterion. Therefore, both costs are capitalized rather than immediately expensed.

How well did you know this?
1
Not at all
2
3
4
5
Perfectly
241
Q

Lano Corp.’s forestland was condemned for use as a national park. Compensation for the condemnation exceeded the forestland’s carrying amount. Lano purchased similar, but larger, replacement forestland for an amount greater than the condemnation award.

As a result of the condemnation and replacement, what is the net effect on the carrying amount of the forestland reported in Lano’s Balance Sheet?

A

The amount is increased by the excess of the replacement forestland’s cost over the condemned forestland’s carrying amount.

The two transactions are not related. The land account is decreased by the book value of the land condemned and increased by the cost of the land purchased. The relative magnitudes of the book values are shown below:

award > book value of condemned land

cost of new land > award

Therefore: cost of new land > book value of condemned land

Thus, the land is increased by the net amount: cost of new land-book value of old land

How well did you know this?
1
Not at all
2
3
4
5
Perfectly
242
Q

On December 1, 20X5, Boyd Co. purchased a $400,000 tract of land for a factory site. Boyd razed an old building on the property and sold the materials it salvaged from the demolition.

Boyd incurred additional costs and realized salvage proceeds during December 20X5 as follows:

Demolition of old building $50,000
Legal fees for purchase contract and recording ownership 10,000
Title guarantee insurance 12,000
Proceeds from sale of salvaged materials 8,000

In its December 31, 20X5, Balance Sheet, Boyd should report a balance in the land account of:

A

$464,000

Land purchase price	$400,000
Plus demolition of old building	50,000
Less salvage proceeds	(8,000)
Plus title insurance	12,000
Plus legal fees	10,000
Equals recorded land cost	$464,000

The net demolition cost is included in land because it is a cost required to prepare the land for its eventual use.

How well did you know this?
1
Not at all
2
3
4
5
Perfectly
243
Q

Merry Co. purchased a machine costing $125,000 for its manufacturing operations and paid shipping costs of $20,000. Merry spent an additional $10,000 testing and preparing the machine for use.

What amount should Merry record as the cost of the machine?

A

$155,000

All three costs are included for a total of $155,000. Both the shipping and testing costs are necessary to place the asset into its intended location and condition for use. This is the criterion for capitalizing costs on acquisition of plant assets.

How well did you know this?
1
Not at all
2
3
4
5
Perfectly
244
Q

Oak Co., a newly formed corporation, incurred the following expenditures related to land and building:

County assessment for sewer lines $ 2,500
Title search fees 625
Cash paid for land with a building to be demolished 135,000
Excavation for construction of basement 21,000
Removal of old building $21,000 less salvage of $5,000 16,000

At what amount should Oak record the land?

A

$154,125

The amounts necessary to get the land ready for its intended purpose attach themselves as a part of the total cost of the land. This would be the: $2,500+625+135,000+16,000=$154,125

How well did you know this?
1
Not at all
2
3
4
5
Perfectly
245
Q

A plant asset under construction by a firm for its own use was completed at the end of the current year. The following costs were incurred:

Materials $60,000
Labor 30,000
Incremental overhead 10,000
Capitalized interest 20,000

The asset has a service life of 10 years, estimated residual value of $10,000, and will be depreciated under the double declining balance method. At completion, the asset was worth $105,000 at fair value. What amount of depreciation will be recognized on the asset in total over its service life?

A

$95,000

The sum of the four listed costs is $120,000, which exceeds fair value of $105,000. Therefore, the asset is capitalized at $105,000, the lesser of the two amounts. Subtracting the $10,000 residual value yields $95,000 depreciable cost-the total depreciation over the life of the asset.

How well did you know this?
1
Not at all
2
3
4
5
Perfectly
246
Q

Immediately after a note payable was signed, its present value was $30,000. This note and $20,000 cash were used to acquire a used plant asset at the beginning of the current year. The interest rate implied in the note is 6%. Total interest payments due on the note over its term amount to $4,000. The term exceeds one year. No payments on the note are due during the current year. What amount of interest expense is recognized for the first year (current year) on this note, and what amount is capitalized to the plant asset account?

A

Interest Expense - $ 1,800
Capitalized Amount - $ 50,000

The interest expense recognized for the first year is .06($30,000) = $1,800. Although no interest is paid, interest is accrued, increasing the carrying value of the note. The asset is capitalized at $50,000, the sum of cash down payment and present value of the note. The interest over the note term is not capitalized because it does not assist in the process of placing the asset into its intended condition and location.

How well did you know this?
1
Not at all
2
3
4
5
Perfectly
247
Q

Plant assets are occasionally acquired by means other than by paying cash. Choose the correct statement about such acquisitions.

A

If a building is acquired by issuing an amount of stock that is significant in relation to the amount of stock outstanding before the exchange, the fair value of the building should be used to initially debit the building account.

The more objective or readily determinable value is used for recording the building. If the number of shares is significant in relation to the total shares outstanding, the stock price will be affected by the increase in the shares outstanding resulting from the purchase. The more objective value is the appraised value of the building.

When debt is incurred to purchase a plant asset, only the principal value of the debt (its present value) is capitalized. The interest subsequently paid is not considered part of the cost of the equipment because it does not contribute to the asset’s value in use.

How well did you know this?
1
Not at all
2
3
4
5
Perfectly
248
Q

A firm began the construction of its new manufacturing facility in January of 20x2. The following expenditures were made on construction in that year:

Jan. 1 $40,000
Mar. 1 120,000
Oct. 31 96,000

Debt outstanding the entire year:

6%, $60,000 construction loan
4%, $90,000 note payable not related to construction
6%, $90,000 note payable not related to construction

Compute interest to be capitalized using the weighted average method.

A

$8,190

Average accumulated expenditures is $156,000 = $40,000 + $120,000(10/12) + $96,000(2/12). This method uses the average interest rate on all interest bearing debt, weighted by principal. That rate is the quotient of the interest on all the debt divided by the principal on all the debt. The rate = ($3,600 + $3,600 + $5,400)/$240,000 = .0525. Interest capitalized = (.0525)$156,000 = $8,190.

How well did you know this?
1
Not at all
2
3
4
5
Perfectly
249
Q

Two approaches are available for applying interest rates to average accumulated expenditures for the purpose of capitalizing interest. These approaches are called the specific method and the weighted average method. In some cases, these approaches yield the same results. Two situations may be encountered in practice for a specific period:

(1) average accumulated expenditures exceed total interest bearing debt (principal) and
(2) the interest rates on all interest bearing debt instruments are the same.

Which situation yields the same results for the two approaches?

A

both (1) and (2).

When average accumulated expenditures exceeds interest bearing debt, all interest for the period is capitalized because all debt could have been avoided if the construction had not taken place. Also, if the interest rates on all debt are the same, then the two approaches yield the same results because, ultimately, only one interest rate is applied to average accumulated expenditures for computing capitalized interest.

How well did you know this?
1
Not at all
2
3
4
5
Perfectly
250
Q

Debt is frequently incurred when plant assets are acquired. For example, debt may be incurred on the purchase of plant assets. Debt may also be incurred during the construction of plant assets. How is the interest in these two cases treated for financial reporting?

A

Debt for purchase - Expensed
Debt during construction - Capitalized

Interest on debt incurred when purchasing a plant asset, is incurred after the asset has reached its intended condition and location. Therefore, it is expensed as incurred. Debt incurred during the construction of plant assets is considered avoidable and also incurred before the asset has reached its intended condition and location. Therefore, it is capitalized to the asset in the same way material, labor, and overhead are capitalized. The interest is expensed as part of depreciation during the service life of the asset.

How well did you know this?
1
Not at all
2
3
4
5
Perfectly
251
Q

Papa Company acquired land with an office building on it from its subsidiary, Sonny Company, for $110,000. Prior to the sale, Sonny’s carrying value of the land was $60,000 and its net carrying value of the building was $50,000. At the time of the transaction, Papa appropriately determined that the land had a fair value of $75,000 and the building had a fair value of $35,000. At what amount should Papa record the land and building on its books at the date of the transaction?

A

Land - $ 75,000
Building - $ 35,000

Papa should record the land and building on its books at the appropriately determined fair value at the date of the transaction. The prior carrying values on Sonny’s books are not relevant to the amounts at which Papa should record the assets on its books, but are relevant to the amounts that should be reported in the consolidated financial statements.

How well did you know this?
1
Not at all
2
3
4
5
Perfectly
252
Q

At the beginning of the year, Cann Co. started construction on a new $2 million addition to its plant. Total construction expenditures made during the year were $200,000 on January 2, $600,000 on May 1, and $300,000 on December 1. On January 2, the company borrowed $500,000 for the construction at 12%. The only other outstanding debt the company had was a 10% interest rate, long-term mortgage of $800,000, which had been outstanding the entire year. What amount of interest should Cann capitalize as part of the cost of the plant addition?

A

$72,500

First calculate the Average Accumulated Expenditures (AAE). This gives you the amount of borrowing from which to calculate avoidable interest ($625,000). Next calculate avoidable interest ($72,500) and actual interest (($500,000 × 12%) + ($800,000 × 10%) = $140,000). The amount that can be capitalized is the lesser of the avoidable interest or actual interest. The amount that can be capitalized is $72,500.

AAE	 	 
200,000	12/12	200,000
600,000	8/12	400,000
300,000	1/12	25,000
 	 	625,000

Avoidable interest
500,000 12% 60,000
125,000 10% 12,500
72,500

How well did you know this?
1
Not at all
2
3
4
5
Perfectly
253
Q

During Year 1, Bay Co. constructed machinery for its own use and for sale to customers. Bank loans financed these assets both during construction and after construction was complete.

How much of the interest incurred should be reported as interest expense in the Year 1 Income Statement?

A

Interest incurred for machinery for own use -
Interest incurred after completion

Interest incurred for machinery held for sale - All interest incurred

Interest during construction on assets constructed for a firm’s own use is capitalized until construction is complete. Thus, only the interest incurred after completion is expensed.

Interest is capitalized on the construction of assets for sale only if the assets are large, individual, discrete projects, such as ships or real estate developments. The equipment constructed for sale does not appear to be a discrete item in that sense and, thus, none of the interest is capitalized. It is all expensed.

How well did you know this?
1
Not at all
2
3
4
5
Perfectly
254
Q

Average accumulated expenditures for year five on a construction project amounted to $70,000. The total cash invested in the project by the end of year five, was $160,000. During year six, the firm spent another $240,000 (total) on the project, uniformly throughout the year. Compute average accumulated expenditures for year six.

A

$280,000

Average accumulated expenditures is the amount of debt for the annual period that could have been avoided. In this case, the firm has $160,000 already invested in the project at the beginning of year six. That amount represents $160,000 in debt, that could have been avoided for year six if the firm had not been involved in the construction project. The expenditures during year six were incurred evenly. Average accumulated expenditures therefore = $160,000(12/12) + $240,000/2 = $280,000. Also, [$160,000 + ($160,000 + $240,000)]/2 = $280,000.

How well did you know this?
1
Not at all
2
3
4
5
Perfectly
255
Q

A firm has spent the last two years constructing a building to be used as the firm’s headquarters. At the end of the first year of construction, the balance of building under construction was $400,000, which includes capitalized interest. During year two, the firm paid $240,000 to the contractor on March 1, and $600,000 on October 1. The building was not finished by the end of the second year. The firm had one loan outstanding all year, an 8%, $3,000,000 construction loan. Compute capitalized interest for year two.

A

$60,000

Average accumulated expenditures for the second year = $400,000(12/12) + $240,000(10/12) + $600,000(3/12) = $750,000. Interest capitalized = .08($750,000) = $60,000. Note that the interest capitalized in year one is compounded in year two because year one capitalized interest is included in average accumulated expenditures for the second year.

How well did you know this?
1
Not at all
2
3
4
5
Perfectly
256
Q

A company with a June 30 fiscal year-end entered into a $3,000,000 construction project on April 1 to be completed on September 30. The cumulative construction-in-progress balances at April 30, May 31, and June 30 were $500,000, $800,000, and $1,500,000, respectively. The interest rate on company debt used to finance the construction project was 5% from April 1 through June 30 and 6% from July 1 through September 30. Assuming that the asset is placed into service on October 1, what amount of interest should be capitalized to the project on June 30?

A

$11,666

Interest is capitalized on the project’s average expenditures times the interest during that period. Key here is to use the 5% annual interest over the three months (April, May, and June).
Average expenditures:
500,000 + 800,000 + 1,500,0003 = 933,333

Interest rate for 3-month period: .05 × 3/12 × .0125

Capitalized interest: 11,666

How well did you know this?
1
Not at all
2
3
4
5
Perfectly
257
Q

On June 18, 20X5, Dell Printing Co. incurred the following costs for one of its printing presses:

Purchase of collating and stapling attachment $84,000
Installation of attachment 36,000
Replacement parts for overhaul of press 26,000
Labor and overhead in connection with overhaul 14,000

The overhaul resulted in a significant increase in production. Neither the attachment nor the overhaul increased the estimated useful life of the press. What amount of the above costs should be capitalized?

A

$160,000

All four costs should be capitalized because they result in an increase in the productivity of the asset. Costs that increase EITHER the life OR productivity are capitalized. Either type of increase results in enhanced asset values. $160,000 is the sum of the four costs listed.

How well did you know this?
1
Not at all
2
3
4
5
Perfectly
258
Q

Many years after constructing a plant asset, management spent a significant sum on the asset. Which of the following types of expenditures should be capitalized in this instance:

(1) an expenditure for routine maintenance that increases the useful life compared with deferring the maintenance,
(2) an expenditure that increases the useful life of the asset compared with the original estimate assuming normal maintenance at the required intervals,
(3) an expenditure that increases the utility of the asset.

A

(1) NO
(2) YES
(3) YES

Post-acquisition expenditures, which increase the useful life (assuming normal maintenance) or the utility (usefulness or productivity) of the asset, are capitalized. Such expenditures provide value for more than one year. The original useful life of an asset assumes regular maintenance. Therefore, regular maintenance does not increase the intended useful life of the asset.

How well did you know this?
1
Not at all
2
3
4
5
Perfectly
259
Q

On January 1, 20X5, Dix Co. replaced its old boiler. The following information was available on that date:

Carrying amount of old boiler $ 8,000
Fair value of old boiler 2,000
Purchase and installation price of new boiler 100,000
The old boiler was sold for $2,000.

What amount should Dix capitalize as the cost of the new boiler?

A

$100,000

The disposal of the old boiler and purchase of the new boiler are separate transactions. The loss on disposal has no effect on the capitalized cost of the new boiler, which is recorded at its $100,000 purchase cost.

How well did you know this?
1
Not at all
2
3
4
5
Perfectly
260
Q

A building suffered uninsured fire damage. The damaged portion of the building was refurbished with higher quality materials. The cost and related accumulated depreciation of the damaged portion are identifiable. To account for these events, the owner should:

A

Capitalize the cost of refurbishing, and record a loss in the current period equal to the carrying amount of the damaged portion of the building.

When the cost and accumulated depreciation of a component or portion of a larger asset is identifiable, and that component or portion is replaced, the replacement is treated as two separate transactions:

(1) disposal of the old component (for zero proceeds in this case, due to the fire damage) and
(2) purchase of the new component.

Thus, a loss equal to the book value of the old component is recognized for (1) and the amount paid to purchase the new component is capitalized as a separate purchase for (2).

How well did you know this?
1
Not at all
2
3
4
5
Perfectly
261
Q

List the non accelerated methods of

depreciation.

A

Straight-line method
Service hours method
Units of output method

How well did you know this?
1
Not at all
2
3
4
5
Perfectly
262
Q

What type of allocation is depreciation

considered?

A

Systematic and rational allocation of

capitalized asset cost to time periods

How well did you know this?
1
Not at all
2
3
4
5
Perfectly
263
Q

How do we calculate the annual
straight-line depreciation amount of an
asset?

A

(Cost − Salvage value) / Useful life

How well did you know this?
1
Not at all
2
3
4
5
Perfectly
264
Q

Depreciation is included in overhead
and allocated to production based on
machine hours or direct labor for what
type of asset?

A

Manufacturing assets

How well did you know this?
1
Not at all
2
3
4
5
Perfectly
265
Q

How do we calculate depreciation

based on service hours?

A

Depreciation rate × Service hours used
Depreciation rate = (Cost −
Salvage value) / Estimated hours

How well did you know this?
1
Not at all
2
3
4
5
Perfectly
266
Q

Define “book value.”

A

Original cost less accumulated

depreciation to date

How well did you know this?
1
Not at all
2
3
4
5
Perfectly
267
Q

Ott Co. purchased a machine at an original cost of $90,000 on January 2, Year 1. The estimated useful life of the machine is 10 years, and the machine has no salvage value. Ott uses the straight-line method to calculate depreciation. On July 1, Year 10, Ott sold the machine for $5,000. What is the amount of gain or loss on the disposal of the machine?

A

$500 gain

The gain or loss on the disposal of an asset is the difference between the net book value (NBV) and the selling price. The annual depreciation of the machine is $9,000 ($90,000/10 years). The asset was held for 9.5 years, so accumulated depreciation is $85,500. NBV is $4,500 ($90,000 less accumulated depreciation of $85,500). The sale is a gain of $500 because the selling price is greater than the NBV ($5,000 − $4,500).

How well did you know this?
1
Not at all
2
3
4
5
Perfectly
268
Q

In year 6, Spirit, Inc. determined that the 12-year estimated useful life of a machine purchased for $48,000 in January year 1 should be extended by three years. The machine is being depreciated using the straight-line method and has no salvage value. What amount of depreciation expense should Spirit report in its financial statements for the year ending December 31, year 6?

A

$2,800

This is a change in estimate and is handled currently and prospectively by allocating the remaining book value at the beginning of year 6 over the revised estimate of remaining years at that point. Through the beginning of year 6, the asset has been used five years. Therefore, seven years remain in the original book value. The book value at the beginning of year 6 is 7/12 × $48,000 or $28,000. The remaining useful life of seven years is extended to 10. Therefore, depreciation expense for year 6 is $28,000 × 1/10 or $2,800.

How well did you know this?
1
Not at all
2
3
4
5
Perfectly
269
Q

Net income is understated if, in the first year, estimated salvage value is excluded from the depreciation computation when using the

A

Straight-line method - YES
Production or use method - YES

When salvage value is excluded from the computation of depreciation, excessive depreciation is recognized each year under BOTH methods. Therefore, income is understated for both methods.

Annual depreciation under straight-line is:

(1/n)(cost-salvage)

where n is the number of years in the useful life.

Annual depreciation under the production method is:

(current year production/tot.est.production)(cost-salvage)

In both cases, if salvage value is excluded from the computation, depreciation is overstated because cost, rather than depreciable cost, is used as the basis for depreciation.

How well did you know this?
1
Not at all
2
3
4
5
Perfectly
270
Q

Zahn Corp.’s comprehensive Balance Sheet at December 31, 2005 and 2004 reported accumulated depreciation balances of $800,000 and $600,000, respectively. Property with a cost of $50,000 and a carrying amount of $40,000 was the only property sold in 2005.

Depreciation charged to operations in 2005 was:

A

$210,000

The accumulated depreciation on the property sold was $10,000 ($50,000 cost less $40,000 carrying value). The sale of property requires that the accumulated depreciation on the property be removed from the accounts.

Thus, the $10,000 amount is a decrease in accumulated depreciation. With an overall increase of $200,000 in accumulated depreciation during the period ($800,000-$600,000), depreciation must have been $210,000 ($200,000 + $10,000).

How well did you know this?
1
Not at all
2
3
4
5
Perfectly
271
Q

What factor must be present to use the units of production (activity) method of depreciation?

A

Total units to be produced can be estimated.

Without an estimate for total units to be produced, depreciation could not be computed. Annual depreciation under this method is:

(Cost-salvage value)/(Total estimated production).

The quantity in square brackets is the rate of depreciation per unit.

How well did you know this?
1
Not at all
2
3
4
5
Perfectly
272
Q

On January 1, 20X5, Brecon Co. installed cabinets to display its merchandise in customers’ stores. Brecon expects to use these cabinets for five years.

Brecon’s 20X5 multi-step Income Statement should include:

A

One-fifth of the cabinet costs in selling, general, and administrative expenses.

With a five year life, 1/5 of the cost of the cabinets is expensed as depreciation. The cabinets are not involved in the manufacturing of the goods. Rather, they are used to help sell the merchandise.

Thus, the depreciation is not included in cost of goods sold; rather, it is included in selling, general, and administrative expenses.

How well did you know this?
1
Not at all
2
3
4
5
Perfectly
273
Q

In which of the following situations is the units of production method of depreciation most appropriate?

A

An asset’s service potential declines with use.

This method is most appropriate when the service potential of an asset can be estimated reliably in terms of a physical variable, such as miles to be driven, or number of units of output that can be produced by the asset.

Over time, as more units are produced, the service potential of the asset declines because the total number of units that can be produced is finite. Over time, the number of units that can be produced by the asset in the future declines. The primary causative agent for depreciation under the units of production method is, thus, the actual use of the asset in production.

How well did you know this?
1
Not at all
2
3
4
5
Perfectly
274
Q

On January 2, Year 1, Lem Corp. bought machinery under a contract that required a down payment of $10,000, plus 24 monthly payments of $5,000 each, for total cash payments of $130,000.

The cash-equivalent price of the machinery was $110,000. The machinery has an estimated useful life of 10 years and estimated salvage value of $5,000. Lem uses straight-line depreciation.

In its Year 1 Income Statement, what amount should Lem report as depreciation for this machinery?

A

$10,500

The capitalized cost of the equipment is $110,000, not the total of the cash payments to be made. The latter amount includes interest.

Thus, annual depreciation is $10,500:

($110,000-$5,000)/10.

How well did you know this?
1
Not at all
2
3
4
5
Perfectly
275
Q

Ichor Co. reported equipment with an original cost of $379,000 and $344,000 and accumulated depreciation of $153,000 and $128,000, respectively, in its comparative financial statements for the years ended December 31, 20X5 and 20X4.

During 20X5, Ichor purchased equipment costing $50,000 and sold equipment with a carrying value of $9,000.

What amount should Ichor report as depreciation expense for 20X5?

A

$31,000

Net equipment at end of 20X4: $344,000-$128,000 = $216,000
Equipment purchase 50,000
Book value of equipment sold (9,000)
Depreciation in 20X5 ?
Equals net equipment at end of 20X5: $379,000-$153,000 = $226,000
Solving for depreciation yields $31,000 depreciation for 20X5.

How well did you know this?
1
Not at all
2
3
4
5
Perfectly
276
Q

What depreciation method does

not use salvage value?

A

Double-declining balance

How well did you know this?
1
Not at all
2
3
4
5
Perfectly
277
Q

How do we calculate the rate

used in double declining balance?

A
Straight-line rate: Number of
years divided into 1 (i.e., if 5
years, 1 / 5 = 20%).
Twice the straight-line rate:
20% × 2 = 40%
How well did you know this?
1
Not at all
2
3
4
5
Perfectly
278
Q

When is the inventory method of

depreciation used?

A

When the inventory items are
smaller homogeneous groups of
assets and individual records for
the assets are not maintained

How well did you know this?
1
Not at all
2
3
4
5
Perfectly
279
Q

What depreciation method is

used for group/composite assets?

A

The straight-line method to
groups rather than individual
assets

How well did you know this?
1
Not at all
2
3
4
5
Perfectly
280
Q

List the type of costs capitalized

for natural resources.

A

Acquisition
Exploration
Development

How well did you know this?
1
Not at all
2
3
4
5
Perfectly
281
Q

What costs are included in the full
costing method for exploration
costs?

A

All costs of exploring for the
resource are capitalized to the
natural resources account.

How well did you know this?
1
Not at all
2
3
4
5
Perfectly
282
Q

Define “depletion.”

A

Refers to the allocation of the
cost of the natural resource to
inventory

How well did you know this?
1
Not at all
2
3
4
5
Perfectly
283
Q

List the methods of accounting

for exploration costs.

A

Successful efforts

Full costing

How well did you know this?
1
Not at all
2
3
4
5
Perfectly
284
Q

List the depletion rate formula.

A

(Natural resources account
balance − residual value)/(Total
estimated units)

How well did you know this?
1
Not at all
2
3
4
5
Perfectly
285
Q

What costs are included in the
successful efforts method for
exploration costs?

A

Only the costs of successful
exploration efforts are capitalized
to the natural resources account.

How well did you know this?
1
Not at all
2
3
4
5
Perfectly
286
Q

What is the classification of
natural resources on the balance
sheet?

A

Non current asset

How well did you know this?
1
Not at all
2
3
4
5
Perfectly
287
Q

List the general test for

impairment.

A

Book value > Recoverable cost

How well did you know this?
1
Not at all
2
3
4
5
Perfectly
288
Q

When is a held-for-sale asset

impaired?

A

It is impaired when book value
exceeds its fair value less cost to
sell at the end of the reporting
period.

How well did you know this?
1
Not at all
2
3
4
5
Perfectly
289
Q

How is the amount of impairment

loss on asset in use determined?

A

The amount by which the
carrying value of the asset
exceeds its market value

How well did you know this?
1
Not at all
2
3
4
5
Perfectly
290
Q

How is the amount of impairment
loss on assets held for disposal
determined?

A

Fair value less cost to sell

How well did you know this?
1
Not at all
2
3
4
5
Perfectly
291
Q

How are assets grouped for

impairment testing?

A

Assets are grouped at the lowest
possible organizational level
where cash flows can be
identified.

How well did you know this?
1
Not at all
2
3
4
5
Perfectly
292
Q

What items must be reported for

impairment losses?

A

Report the loss as part of ordinary
income and also disclose:

The asset impaired.

The events leading to
impairment.

The amount of the
impairment loss.

The method of determining
fair value, including interest
rate.

How well did you know this?
1
Not at all
2
3
4
5
Perfectly
293
Q

List the impairment tests for

assets in use.

A

Sum net future cash flows
from asset (recoverable
cost)

If sum > book value, no
impairment

If sum < book value,
impairment

How well did you know this?
1
Not at all
2
3
4
5
Perfectly
294
Q

How are assets grouped for
impairment testing under
International Financial Reporting
Standards (IFRS)?

A

At the “cash-generating unit”

level

How well did you know this?
1
Not at all
2
3
4
5
Perfectly
295
Q

Are reversals of impairment
allowed under International
Financial Reporting Standards
(IFRS)?

A

Yes, they are allowed.

How well did you know this?
1
Not at all
2
3
4
5
Perfectly
296
Q

Under International Financial
Reporting Standards (IFRS_, how
is the impairment loss presented
if the asset is carried at fair value?

A

Any impairment loss would be
classified out of other
comprehensive income and into
earnings.

How well did you know this?
1
Not at all
2
3
4
5
Perfectly
297
Q

Vore Corp. bought equipment on January 2, 20X4 for $200,000. This equipment had an estimated useful life of five years and a salvage value of $20,000. Depreciation was computed by the 150% declining balance method.

The accumulated depreciation balance at December 31, 20X5 should be:

A

$102,000

Depreciation in 20X4 = $200,000(1.50/5) = $ 60,000
Depreciation in 20X5 = ($200,000-$60,000)(1.50/5) = 42,000
Accumulated depreciation balance at the end of 20X5 $ 102,000
The declining balance class of depreciation method does not deduct salvage value when computing depreciation although care must be taken not to depreciate the asset below salvage value. Also, the rate of depreciation applied to book value is the percentage of the method (150% in this case) divided by the useful life of the asset. Double declining balance, for example, is 200%/n or 2/n where n = useful life.

How well did you know this?
1
Not at all
2
3
4
5
Perfectly
298
Q

On April 1, 20X4, Kew Co. purchased new machinery for $300,000. The machinery has an estimated useful life of five years, and depreciation is computed by the sum-of-the-years’-digits method.

The accumulated depreciation on this machinery at March 31, 20X6 should be:

A

$180,000

$180,000, the correct answer, equals $300,000[(5 + 4)/(5 + 4 + 3 + 2 + 1)].

Two full years of depreciation have been recorded, and the SYD method uses the number of years left at the beginning of each year as the numerator of the fraction used in depreciation. At the beginning of the first and second years, five and four years of the asset’s life remained, respectively. The denominator is the sum of the digits up to the asset’s useful life (5).

How well did you know this?
1
Not at all
2
3
4
5
Perfectly
299
Q

A fixed asset with a five-year estimated useful life and no residual value is sold at the end of the second year of its useful life.

How would using the sum-of-the-years’-digits method of depreciation, instead of the double declining balance method of depreciation, affect a gain or loss on the sale of the fixed asset?

A

Gain - Decrease
Loss - Increase

Under SYD, total depreciation through the first two years is [(5 + 4)/(1 + 2 + 3 + 4 + 5)]Cost = (9/15)Cost.

Therefore, book value remaining is (6/15)Cost = .4Cost.

Depreciation, year one = (2/5)Cost = .4Cost
Depreciation, year two = (2/5)(Cost-Depreciation, year one)
= (2/5)[Cost-(2/5)Cost]
=.4[Cost-.4(Cost)]
= .4(.6Cost) = .24 Cost

Total depreciation for the two years is therefore .4(Cost) + .24(Cost) = .64(Cost). Book value remaining is (1-.64)Cost = .36 Cost.

The asset has a larger book value under SYD after two years. For a given amount of proceeds on disposal, the larger book value under SYD causes any gain on disposal to be smaller than under DDB and any loss greater than under DDB. In other words, the gain decreases and the loss increases, relative to DDB.

How well did you know this?
1
Not at all
2
3
4
5
Perfectly
300
Q

Spiro Corp. uses the sum-of-the-years’ digits method to depreciate equipment purchased in January 20X3 for $20,000. The estimated salvage value of the equipment is $2,000, and the estimated useful life is four years.

What should Spiro report as the asset’s carrying amount as of December 31, 20X5?

A

$3,800

The carrying amount (book value) of a depreciable asset is its original cost less accumulated depreciation. Under sum-of-the-years’ digits method of calculating depreciation expense (and, therefore, accumulated depreciation), the net depreciable cost (original cost less estimated salvage value) is multiplied by a factor consisting of:

Numerator = the number of years the current year is from the end of the life of the asset

Denominator = the sum of numbers (digits) for each year in the life of the asset

For Spiro, the net depreciable cost is $20,000-$2,000 = $18,000. Since the equipment has an estimated useful life of four years, the sum of the digits for each year would be 1 + 2 + 3 + 4 = 10, the denominator for calculating each year’s depreciation. Depreciation for the four years would be:

Thus, at the end of 20X5 the carrying amount is $3,800, which also can be calculated as salvage value 2,000 + (1/10 × $18,000) = $2,000 + $1,800 = $3,800.

How well did you know this?
1
Not at all
2
3
4
5
Perfectly
301
Q

A depreciable asset has an estimated 15% salvage value. Under which of the following methods, properly applied, would the accumulated depreciation equal the original cost at the end of the asset’s estimated useful life?

A

Straight-line - NO
Double-declining balance - NO

Salvage value is the portion of the asset’s cost not subject to depreciation. Total depreciation, under any method, is limited to depreciable cost (cost less salvage value). The declining balance methods do not subtract salvage when computing depreciation. Care must be taken to avoid depreciating an asset beyond salvage value.

How well did you know this?
1
Not at all
2
3
4
5
Perfectly
302
Q

South Co. purchased a machine that was installed and placed in service on January 1, 20X4 at a cost of $240,000. Salvage value was estimated at $40,000. The machine is being depreciated over 10 years by the double declining balance method. For the year ended December 31, 20X5, what amount should South report as depreciation expense?

A

$38,400

Depreciation in 2004 = $240,000(2/10) = $48,000

Depreciation in 20X5 = ($240,000-$48,000)(2/10) = $38,400

The DDB method’s rate is always twice the straight-line rate, or 2/useful life. The method does not subtract salvage value when computing depreciation, but it also does not reduce book value below salvage value. The depreciation in any year is the rate times the beginning net book value of the asset.

How well did you know this?
1
Not at all
2
3
4
5
Perfectly
303
Q

On January 1, Year 1, Crater, Inc. purchased equipment having an estimated salvage value equal to 20% of its original cost at the end of a 10-year life. The equipment was sold December 31, Year 5, for 50% of its original cost.

If the equipment’s disposition resulted in a reported loss, which of the following depreciation methods did Crater use?

A

Straight-line.

The asset was sold when 1/2 of its useful life was expired. (The asset was used 5 years and had an original useful life of 10 years.) If an asset is sold at a loss, then the book value at the date of sale exceeds the proceeds from sale by the amount of the loss. Let C = original cost, and BV = book value at date of sale.

Then BV-proceeds = loss Proceeds = .50C according to the question data.

Thus, BV-.50C = loss. Thus, BV must exceed 50% of the original cost because BV-.50C is a positive number.

The only method from among those listed in the answer alternatives that leaves a BV greater than 50% of original cost after 50% of the useful life has expired is the SL method. The book value after the fifth year under SL is C-(C-.2C)(5/10) = .6C.

DDB’s book value after five years is much less than 50% of original cost because it is an accelerated method. The same holds for SYD. And under composite methods of depreciation, individual assets do not have a separately recorded book value. When sold, accumulated depreciation is debited for the difference between original cost and proceeds. No gain or loss is recognized. Thus, the composite method could not apply in this question.

How well did you know this?
1
Not at all
2
3
4
5
Perfectly
304
Q

Choose the best association of terms in the natural resources accounting area with the conceptual framework.

A

Successful efforts method-definition of asset.

The successful efforts method capitalizes only the cost of exploration efforts that locate the resource. As such, only those efforts that yield a probable future benefit are capitalized. This is a direct application of the asset definition, which requires that an asset have a probable future benefit.

How well did you know this?
1
Not at all
2
3
4
5
Perfectly
305
Q

A firm began a mineral exploitation venture during the current year by spending (1) $40 million for the mineral rights; (2) $100 million exploring for the minerals, one-fourth of which were successful; and (3) $60 million to develop the site. Management estimated that 20 million tons of ore would ultimately be removed from the property. Wages and other extraction costs for the current year amounted to $10 million. In total, 2 million tons of ore were removed from the deposit in the current year. The entire production for the period was sold. Compute cost of goods sold under the successful efforts method.

A

$22.5 million

The depletion rate = [$40 + (.25)($100) + $60]/20 = $6.25/ton. Depletion = 2,000,000($6.25/ton) = $12,500,000. Because all the ore removed was sold, cost of goods sold includes the entire amount of depletion and the extraction costs. Cost of goods sold = $12,500,000 + $10,000,000 = $22,500,000. Note, that extraction costs is included in inventory (and therefore, cost of goods sold), but not in the deposit (and therefore, not in depletion).

How well did you know this?
1
Not at all
2
3
4
5
Perfectly
306
Q

A firm began a mineral exploitation venture during the current year by spending (1) $40 million for the mineral rights; (2) $100 million exploring for the minerals, one-fourth of which were successful; and (3) $60 million to develop the site. Management estimated that 20 million tons of ore would ultimately be removed from the property. Wages and other extraction costs for the current year amounted to $10 million. In total, 2 million tons of ore were removed from the deposit in the current year. The entire production for the period was sold. What amount of depletion is recognized during the current year under the full costing method?

A

$20 million

The depletion rate = ($40 + $100 + $60)/20 = $10/ton. Depletion = 2,000,000($10/ton) = $20,000,000. Depletion for a period is the cost of the deposit allocated to the inventory removed for the period. In this case, the entire amount is included in cost of goods sold because there is no ending inventory. However, if there had been ore left at the end of the period, the $10/ton rate would have been applied to the units remaining. That would not change the answer to the question, however.

How well did you know this?
1
Not at all
2
3
4
5
Perfectly
307
Q

On December 31, 20X4, a building owned by Pine Corp. was totally destroyed by fire. The building had fire insurance coverage up to $500,000.

Other pertinent information as of December 31, 20X4, follows:

Building, carrying amount $520,000
Building, fair market value 550,000
Removal and cleanup cost 10,000
During January 20X5, before the 20X4 financial statements were issued, Pine received insurance proceeds of $500,000. On what amount should Pine base the determination of its loss on involuntary conversion?

A

$530,000

The sum of the carrying value ($520,000) and removal/cleanup cost ($10,000) is the amount to compare to the insurance proceeds when computing the loss. The fire caused the latter costs to be incurred; therefore, the cleanup costs should be included in the loss.

The fair value of the property would not figure into the recorded loss because the building account does not reflect this amount.

How well did you know this?
1
Not at all
2
3
4
5
Perfectly
308
Q

On July 1, 20X4, one of Rudd Co.’s delivery vans was destroyed in an accident. On that date, the van’s carrying value was $2,500.

On July 15, 20X4, Rudd received and recorded a $700 invoice for a new engine installed in the van in May 20X4, and another $500 invoice for various repairs. In August, Rudd received $3,500 under its insurance policy on the van, which it plans to use to replace the van.

What amount should Rudd report as gain (loss) on disposal of the van in its 20X4 income statement?

A

$300

The gain of $300 is the difference between the insurance proceeds and the sum of the carrying value of the van plus the cost of the new engine. The repair cost is expensed. It does not increase the value of the van. $300 = $3,500 − $2,500 − $700.

How well did you know this?
1
Not at all
2
3
4
5
Perfectly
309
Q

In January 20X2, Winn Corp. purchased equipment at a cost of $500,000.

The equipment had an estimated salvage value of $100,000, an estimated 8-year useful life, and was being depreciated by the straight-line method. Two years later, it became apparent to Winn that this equipment suffered a permanent impairment of value.

In January 20X4, management determined the carrying amount should be only $175,000, with a 2-year remaining useful life, and the salvage value should be reduced to $25,000.

In Winn’s December 31, 20X4, balance sheet, the equipment should be reported at a carrying amount of:

A

$100,000

The post-impairment carrying value is $175,000 at the beginning of 20X4. This amount is the new basis for depreciation. Depreciation in 20X4 is $75,000 [($175,000 − $25,000)/2].

The revised salvage value is used. After deducting the 20X4 depreciation expense of $75,000 from the $175,000 beginning book value, the ending 20X4 book value is $100,000.

How well did you know this?
1
Not at all
2
3
4
5
Perfectly
310
Q

Which of the following conditions must exist in order for an impairment loss to be recognized?

A

The test for impairment for an asset in use is whether the carrying value (book value) is less than its recoverable cost. An asset’s recoverable cost is the sum of its estimated net cash inflows projected for its remaining life.

When book value > recoverable cost, the carrying value is not recoverable. In other words, the asset is booked at more than the sum of its future net cash inflows.

For example, if an asset’s carrying value is $100 and its recoverable cost is $80, then its carrying value is not recoverable (only $80 is recoverable). The AMOUNT of the loss recognized is the difference between carrying value and fair value, but that difference is not used for TESTING whether an asset is impaired.

That difference is not the condition leading to the impairment loss.

How well did you know this?
1
Not at all
2
3
4
5
Perfectly
311
Q

Last year, Katt Co. reduced the carrying amount of its long-lived assets used in operations from $120,000 to $100,000 in connection with its annual impairment review. During the current year, Katt determined that the fair value of the same assets had increased to $130,000. What amount should Katt record as restoration of previously recognized impairment loss in the current year’s financial statements?

A

$0

Recovery of impairment losses is prohibited under U.S. GAAP.

How well did you know this?
1
Not at all
2
3
4
5
Perfectly
312
Q

Four years ago on January 2, Randall Co. purchased a long-lived asset. The purchase price of the asset was $250,000, with no salvage value. The estimated useful life of the asset was 10 years. Randall used the straight-line method to calculate depreciation expense. An impairment loss on the asset of $30,000 was recognized on December 31 of the current year. The estimated useful life of the asset at December 31 of the current year did not change. What amount should Randall report as depreciation expense in its income statement for the next year?

A

$20,000

The net book value of the asset at the time of impairment was $150,000: $250,000 cost less $100,000 accumulated depreciation (4 years of depreciation at $25,000 a year). After the impairment of $30,000, the net book value is $120,000 ($150,000 − 30,000). The remaining life is 6 years and annual depreciation is $20,000.

How well did you know this?
1
Not at all
2
3
4
5
Perfectly
313
Q

An asset group is being evaluated for an impairment loss. The following financial information is available for the asset group:

Carrying value $100,000,000
Sum of the undiscounted cash flows 95,000,000
Fair value 80,000,000
What amount of impairment loss, if any, should be recognized?

A

$20,000,000

Determination of impairment for an asset held in use is a two-step process. First the carrying value (CV) is compared to the recoverable cost (undiscounted cash flows). Since the CV is more than the recoverable cost, the second step must measure the impairment loss. The impairment loss is measured as the difference between CV and fair value (FV). The CV is $100 million and the FV is $80 million so the impairment loss is $20 million.

How well did you know this?
1
Not at all
2
3
4
5
Perfectly
314
Q

Restoration of the carrying value of a long-lived asset is permitted under IFRS if the asset’s fair value increases subsequent to recording an impairment loss for which of the following?

A

Held for use - YES
Held for disposal - YES

Under IFRS the impairment loss can be recovered if the asset is held for use or disposal.

How well did you know this?
1
Not at all
2
3
4
5
Perfectly
315
Q

Under IFRS the test for asset impairment is to compare the carrying value of the asset to its recoverable amount. Which of the following is the recoverable amount according to IFRS?

A

The greater of fair value less cost to sell or value in use.

The greater of fair value less cost to sell or value in use is the recoverable amount according to IFRS.

How well did you know this?
1
Not at all
2
3
4
5
Perfectly
316
Q
Under International Financial
Reporting Standards (IFRS), what
two methods can be used to
adjust accumulated
depreciation?
A
  1. The proportional method

2. The reset method

How well did you know this?
1
Not at all
2
3
4
5
Perfectly
317
Q

What happens during the reset

method?

A
Accumulated depreciation is
reset to zero by closing it to the
building account, and then the
building is adjusted for the
revaluation.
How well did you know this?
1
Not at all
2
3
4
5
Perfectly
318
Q

Under International Financial
Reporting Standards (IFRS), how
is interest during construction
accounted for?

A

It is expensed or capitalized.

How well did you know this?
1
Not at all
2
3
4
5
Perfectly
319
Q
Where is revaluation surplus
reported under International
Financial Reporting Standards
(IFRS) until the property, plant,
and equipment (PPE) is sold?
A

It is reported in equity.

How well did you know this?
1
Not at all
2
3
4
5
Perfectly
320
Q

Under International Financial
Reporting Standards (IFRS), is
revaluation of property, plant,
and equipment (PPE) allowed?

A

Yes, revaluation is allowed

How well did you know this?
1
Not at all
2
3
4
5
Perfectly
321
Q
How frequently do companies
have to review depreciation
policies under International
Financial Reporting Standards
(IFRS)?
A

Companies have to review the

policies annually

How well did you know this?
1
Not at all
2
3
4
5
Perfectly
322
Q

During a period of rising prices, this method results in a higher net income.

A

FIFO

How well did you know this?
1
Not at all
2
3
4
5
Perfectly
323
Q

This inventory cost flow assumption is prohibited under IFRS.

A

LIFO

How well did you know this?
1
Not at all
2
3
4
5
Perfectly
324
Q

Method that uses historical sales margins to estimate ending inventory.

A

Gross profit

How well did you know this?
1
Not at all
2
3
4
5
Perfectly
325
Q

Method that is appropriate when there is a relatively small number of significant dollar value items in inventory.

A

Specific identification

How well did you know this?
1
Not at all
2
3
4
5
Perfectly
326
Q

Average cost must be calculated each time additional inventory is purchased.

A

Moving average

How well did you know this?
1
Not at all
2
3
4
5
Perfectly
327
Q

Method that averages the cost of all items on hand and purchased during the period.

A

Weighted average

How well did you know this?
1
Not at all
2
3
4
5
Perfectly
328
Q

This method results in the lowest ending inventory in a period of rising prices.

A

LIFO

How well did you know this?
1
Not at all
2
3
4
5
Perfectly
329
Q

Method that uses a price index to measure changes in inventory.

A

Dollar-value LIFO

How well did you know this?
1
Not at all
2
3
4
5
Perfectly
330
Q

If used for tax purposes, this method must also be used for financial reporting purposes.

A

LIFO

How well did you know this?
1
Not at all
2
3
4
5
Perfectly
331
Q

The cost of goods sold balance is the same whether a perpetual or periodic inventory system is used.

A

FIFO

How well did you know this?
1
Not at all
2
3
4
5
Perfectly
332
Q

Understate ending inventory

A

Current-year income - Understate
Ending retained earnings balance, current year - Understate
Income of next year - Overstate
Ending retained earnings balance, next year - No Effect

How well did you know this?
1
Not at all
2
3
4
5
Perfectly
333
Q

Purchase and receive goods in current period, but record purchase in next period; goods are included in inventory.

A

Current-year income - Overstate
Ending retained earnings balance, current year - Overstate
Income of next year - Understate
Ending retained earnings balance, next year - No Effect

How well did you know this?
1
Not at all
2
3
4
5
Perfectly
334
Q

Goods purchased FOB shipping point were in transit at year−end and not included in inventory; purchase was recorded.

A

Current-year income - Understate
Ending retained earnings balance, current year - Understate
Income of next year - Overstate
Ending retained earnings balance, next year - No Effect

How well did you know this?
1
Not at all
2
3
4
5
Perfectly
335
Q

Goods shipped to a customer FOB destination were in transit at year−end and were included in inventory. The sale was not recorded.

A

Current-year income - No Effect
Ending retained earnings balance, current year - No Effect
Income of next year - No Effect
Ending retained earnings balance, next year - No Effect

How well did you know this?
1
Not at all
2
3
4
5
Perfectly
336
Q

On June 30, Almond Co.’s cash balance was $10,012 before adjustments, while its ending bank statement balance was $10,772. Check number 101 was issued June 2 in the amount of $95 but was erroneously recorded in Almond’s general ledger balance as $59. The check was correctly listed in the bank statement at $95.

The bank statement also included a credit memo for interest earned in the amount of $35 and a debit memo for monthly service charges in the amount of $50.

What was Almond’s adjusted cash balance on June 30?

A

$9,961

The adjusted cash balance is computed as $10,012 − corrected #101 amount ($95 − $59) + $35 interest − $50 service charge = $9,961. Check #101 was recorded for $59 but should have been recorded for $95.

How well did you know this?
1
Not at all
2
3
4
5
Perfectly
337
Q

When the allowance method of recognizing uncollectible accounts is used, the entries at the time of collection of a small account previously written off would

A

Increase the allowance for uncollectible accounts.

The entries are:

DR: Accounts receivable
CR: Allowance for uncollectibles
DR: Cash
CR: Accounts receivable

The first entry reinstates the amount of allowance used up when the account was originally written off. The normal balance in the account is a credit. The first entry increases the account.

How well did you know this?
1
Not at all
2
3
4
5
Perfectly
338
Q

When the allowance method of recognizing bad debt expense is used, the allowance would decrease when a(an)

A

Specific uncollectible account is written off.

The allowance account is increased when estimated uncollectible accounts expense is recognized. The allowance records the expected reduction in net accounts receivable until accounts are written off.

Then, when accounts actually become uncollectible and are written off, the allowance is decreased because it is no longer needed. The identity of the specific uncollectible account is known and that account is also decreased.

How well did you know this?
1
Not at all
2
3
4
5
Perfectly
339
Q

When the allowance method of recognizing uncollectible accounts is used, the entry to record the write-off of a specific account

A

Decreases both accounts receivable and the allowance for uncollectible accounts.

The entry is:

DR: Allowance for uncollectible accounts XXXX
CR: Accounts receivable XXXX
Both accounts are decreased.

The entry identifies the specific accounts receivable written off. That reduction takes the place of the earlier estimate, which created the allowance account in the first place.

How well did you know this?
1
Not at all
2
3
4
5
Perfectly
340
Q

Tinsel Co.’s balances in allowance for uncollectible accounts were $70,000 at the beginning of the current year and $55,000 at year end. During the year, receivables of $35,000 were written off as uncollectible. What amount should Tinsel report as uncollectible accounts expense at year end?

A

$20,000

To determine the amount of uncollectible expense (bad debt expense) use T accounts. Solve for ?? = 20,000

Allowance for uncollectible accounts
70,000 Beg balance
Write-offs 35,000 Bad debt expense

55,000 End balance

How well did you know this?
1
Not at all
2
3
4
5
Perfectly
341
Q

In its December 31 balance sheet, Butler Co. reported trade accounts receivable of $250,000 and related allowance for uncollectible accounts of $20,000.

What is the total amount of risk of accounting loss related to Butler’s trade accounts receivable, and what amount of that risk is off-balance sheet risk?

A

Risk of accounting loss - 230,000
Off-balance sheet risk - 0

This question requires an understanding of two accounting concepts:

  1. Risk of accounting loss on accounts receivable (credit risk). This is the risk of loss resulting from not collecting amounts due from sales made on credit, and is the total amount of loss that Butler would suffer if those who owe it failed to make any payments and the receivables proved to be of no value. Since Butler’s net carrying value of accounts receivable is $230,000 ($250,000 − $20,000), that is the amount of risk of accounting loss.
  2. Off-balance sheet risk: This is the amount of risk of loss that does not show on the balance sheet. Since all of Butler’s net accounts receivable show on the balance sheet, there is no off-balance sheet risk associated with the accounts receivable.
How well did you know this?
1
Not at all
2
3
4
5
Perfectly
342
Q

Which method of recording uncollectible accounts expense is consistent with accrual accounting?

A

Allowance - YES
Direct write-off - NO

The allowance method recognizes the estimate of bad debt expense (uncollectible accounts expense) in the year of sale. This is an example of accrual accounting, which measures expenses when incurred and revenues when earned.

The direct write-off method recognizes bad debt expense in the year of write-off, which may be after the year of sale. The direct write-off method is not consistent with accrual accounting.

How well did you know this?
1
Not at all
2
3
4
5
Perfectly
343
Q

The following information pertains to Tara Co.’s accounts receivable on December 31, Year 4

Days outstanding Amount Estimated % uncollectible
0-60 $120,000 1%
61-120 90,000 2%
Over 120 100,000 6%
$310,000
========

During Year 4, Tara wrote off $7,000 in receivables and recovered $4,000 that had been written off in prior years. Tara’s December 31, Year 3, allowance for uncollectible accounts was $22,000. Under the aging method, what amount of allowance for uncollectible accounts should Tara report on December 31, Year 4?

A

$9,000

The data on write-offs and recoveries is not relevant. The aging method computes a required ending allowance balance based on the aging schedule. That required ending balance is the sum of the products of the receivables in each age category and the uncollectible percentage: $120,000(.01) + $90,000(.02) + $100,000(.06) = $9,000.

The write-offs and recoveries do affect the preadjustment allowance balance and therefore the amount of uncollectible accounts expense to recognize. In this case, the preadjustment balance is $19,000 ($22,000 − $7,000 + $4,000), which means no uncollectible accounts expense would be recognized in 2004 because the preadjustment balance is more than sufficient (exceeds the $9,000 required balance).

How well did you know this?
1
Not at all
2
3
4
5
Perfectly
344
Q

Mare Co.’s December 31, Year 5, balance sheet reported the following current assets:

Cash $ 70,000
Accounts receivable 120,000
Inventories 60,000
Total $250,000
========
An analysis of the accounts disclosed that accounts receivable consisted of the following:

Trade accounts $ 96,000
Allowance for uncollectible accounts (2,000)
Selling price of Mare’s unsold goods out on consignment, at 130% of cost, not included in Mare’s ending inventory 26,000
_________
Total $120,000
========
On December 31, Year 5, the total of Mare’s current assets is

A

$244,000

Corrected total current assets is computed as follows:

$250,000 − $26,000 + $26,000/1.30 = $244,000.

The only adjustment needed is to remove the unrecognized gross margin on the unsold inventory out on consignment.

Although the cost of the inventory of $20,000 ($26,000/1.30) is incorrectly classified in accounts receivable, that misclassification does not affect the total for current assets because accounts receivable is part of current assets. Subtracting the $26,000 removes the inventory at selling price. Adding the $26,000/1.30 term adds the cost of the inventory back to current assets. Inventory is reported at cost, not selling price, because the items are unsold. Including them at selling price would imply profit recognition before sale.

How well did you know this?
1
Not at all
2
3
4
5
Perfectly
345
Q

Rue Co.’s allowance for uncollectible accounts had a credit balance of $12,000 on December 31, Year 1. During Year 2, Rue wrote-off uncollectible accounts of $48,000. The aging of accounts receivable indicated that a $50,000 allowance for uncollectible accounts was required on December 31, Year 2. What amount of uncollectible accounts expense should Rue report for Year 2?

A

$86,000

The preadjusted ending 2003 allowance balance is a $36,000 debit ($12,000 cr. beginning balance − $48,000 dr. from write-offs). When accounts are written off, the allowance is debited and accounts receivable is credited. The aging schedule indicates that a $50,000 ending credit allowance balance is required. Therefore, $86,000 of uncollectible accounts expense must be recognized to change the allowance balance from $36,000 dr. to $50,000 cr. An equation or T account approach also can be used to analyze the allowance account: Beginning balance $12,000 - write-offs $48,000 + uncollectible accounts expense (?) = Ending balance $50,000. Solving for uncollectible accounts expense yields $86,000.

How well did you know this?
1
Not at all
2
3
4
5
Perfectly
346
Q

Under the allowance method of recognizing uncollectible accounts, the entry to write-off an uncollectible account

A

Has no effect on net income

The entry is

DR: Allowance for uncollectible accounts XX
CR: Accounts receivable XX

This entry decreases the allowance because the purpose for which the account was created has now been realized (an uncollectible account). The entry has no effect on income because neither account in the entry is an income account.

How well did you know this?
1
Not at all
2
3
4
5
Perfectly
347
Q

Delta, Inc. sells to wholesalers on terms of 2/15, net 30. Delta has no cash sales but 50% of Delta’s customers take advantage of the discount. Delta uses the gross method of recording sales and trade receivables. An analysis of Delta’s trade receivables balances on December 31, revealed the following:

Age	                       Amount	    Collectible
0 − 15 days	      $100,000	        100%
16 − 30 days	         60,000	        95%
31 − 60 days	          5,000	        90%
Over 60 days	          2,500	       $500
                               $167,500
                              =========

In its December 31 balance sheet, what amount should Delta report for allowance for discounts?

A

$1,000

Only the accounts in the 0 − 15 day age category can take the discount, because the discount period ends 15 days after the sale (2/15, n30).

The discount percentage is 2% (2/15, n30). One-half of the customers take the discount. Therefore, the expected discounts to be taken after December 31 are: (.5)($100,000)(.02) = $1,000. This expected discount amount reduces net sales and net accounts receivable for the current year because it is based on current-year sales.

How well did you know this?
1
Not at all
2
3
4
5
Perfectly
348
Q

When the allowance method of recognizing uncollectible accounts is used, how would the collection of an account previously written off affect accounts receivable and the allowance for uncollectible accounts?

A

Accounts receivable - No Effect
Allowance for uncollectible accounts - Increase

The collection reverses the reduction in the allowance account when the specific account was written off. There is no net change in gross accounts receivable because the account was collected.

The two journal entries often given for this transaction are:

(1) dr. Accounts receivable,
cr. Allowance;
(2) dr. Cash;
cr. Accounts receivable.

The first entry reinstates the allowance. The offsetting debits and credits for accounts receivable in both two entries provide an internal record of the transaction.

How well did you know this?
1
Not at all
2
3
4
5
Perfectly
349
Q

Marr Co. had the following sales and accounts receivable balances prior to any adjustment at year end:

Credit sales $10,000,000
Accounts receivable 3,000,000
Allowance for uncollectible accounts 50,000

Marr uses 3% of accounts receivable to determine its allowance of uncollectible accounts at year-end. By what amount should Marr adjust its allowance for uncollectible accounts at year-end?

A

$40,000

When uncollectible accounts are based on receivables, the adjustment is the amount needed to bring the allowance up to the required amount based on those receivables. With $3,000,000 in receivables, the required ending allowance balance is $90,000 (.03 × $3,000,000). There is $50,000 in the allowance account before adjustment. Therefore, $40,000 must be added. That is the amount debited to bad debt expense, and credited to the allowance account.

How well did you know this?
1
Not at all
2
3
4
5
Perfectly
350
Q

On August 15, Year 1, Benet Co. sold goods for which it received a note bearing the market rate of interest on that date. The four-month note was dated July 15, Year 1.

Note that the principal, together with all interest, is due November 15, Year 1.

When the note was recorded on August 15, which of the following accounts increased?

A

Interest receivable.

The note was received one month into its term. Like a bond issued between interest dates and which collects accrued interest from the bondholder since the most recent interest payment date, this note is recorded with interest receivable for one month.

Benet earns only three months of interest revenue because that is the length of time it will hold the note.

How well did you know this?
1
Not at all
2
3
4
5
Perfectly
351
Q

On December 31, Year 1, Jet Co. received two $10,000 notes receivable from customers in exchange for services rendered. On both notes, interest is calculated on the outstanding principal balance at the annual rate of 3% and payable at maturity.

The note from Hart Corp., made under customary trade terms, is due in nine months and the note from Maxx, Inc. is due in five years. The market interest rate for similar notes on December 31, Year 1 was 8%. The compound interest factors to convert future values into present values at 8% follow:

Present value of $1 due in nine months: .944

Present value of $1 due in five years: .680

At what amounts should these two notes receivable be reported in Jet’s December 31, Year 1, balance sheet?

A

Hart - 10,000
Maxx - 7820

The 9-month note is reported at face value ($10,000) because current notes need not be measured at present value. The 5-year note is reported at $7,820, the present value of the future cash flows. The five years of interest is payable at maturity.

$7,820 = $10,000 + $10,000(.03)(5 years)], which is the present value of the note plus the present value of the 3% interest.

How well did you know this?
1
Not at all
2
3
4
5
Perfectly
352
Q

On Merf’s April 30, Year 1, balance sheet, a note receivable was reported as a noncurrent asset and its accrued interest for eight months was reported as a current asset. Which of the following terms would fit Merf’s note receivable?

A

Principal is due August 31, Year 2. Interest is due August 31, Year 1 and August 31, Year 2.

Although the question does not indicate when the principal is due, the candidate must choose one of the answer alternatives. This answer is the only possible answer.

Accrued interest for eight months at April 30 indicates that the end of the twelfth month is August 31. Under this answer alternative, interest is payable at least once per year, thus interest is due August 31, Year 1, four months after the balance sheet date, and again in one year.

Generally, the principal amount of a note is due when the last interest payment is due. Therefore, because the note is classified noncurrent, the principal would be due on an August 31 at least one year after April 30, Year 1, the date of the current balance sheet. Thus, this answer is a possible answer. None of the other answers are possible.

How well did you know this?
1
Not at all
2
3
4
5
Perfectly
353
Q

Pie Co. uses the installment sales method to recognize revenue. Customers pay the installment notes in 24 equal monthly amounts, which include 12% interest.

What is an installment note’s receivable balance six months after the sale?

A

The present value of the remaining monthly payments discounted at 12%.

The question does not specify the exact meaning of the term “note receivable balance.” When the term “gross” is not applied, it is safe to assume that the balance referred to is the net balance, that is, net of interest yet to be recognized.

Notes are reported at present value, which is the amount net of interest yet to be recognized. However, note balances under the installment method include deferred gross margin yet to be realized, because deferred gross margin is subtracted as a separate line item.

Thus, the question is referring to the notes receivable balance exclusive of interest yet to be recognized, but inclusive of deferred gross margin yet to be realized. The note’s balance is the present value of the remaining payments. This is a two-year note. Therefore, valuation at present value is required. The note’s valuation is the present value of the remaining payments at the original discount rate.

How well did you know this?
1
Not at all
2
3
4
5
Perfectly
354
Q

Estimates of price-level changes for specific inventories are required for which of the following inventory methods?

A

Dollar-value LIFO.

DV LIFO is based on price level indices. The ending inventory is determined at current cost, and then reduced to the price level existing at the base-year (the year LIFO was adopted). The ending inventory measured in base-year dollars is compared to beginning inventory measured in base-year dollars. The difference is the increase in inventory measured in base-year dollars. This difference is then raised to the current-year price level and added to beginning inventory DV LIFO, yielding ending inventory DV LIFO.

Thus, price-level changes are used throughout this method.

Price-level changes are used as a means of estimating the ending inventory. Individual item costs are not maintained or used in the valuation of inventory.

How well did you know this?
1
Not at all
2
3
4
5
Perfectly
355
Q

On January 2 of the current year, LTTI Co. entered into a three-year, non-cancelable contract to buy up to 1 million units of a product each year at $.10 per unit with a minimum annual guarantee purchase of 200,000 units. At year end, LTTI had only purchased 80,000 units and decided to cancel sales of the product. What amount should LTTI report as a loss related to the purchase commitment as of December 31 of the current year?

A

$52,000

This amount represents the amount of the minimum guaranteed amount ($60,000 {200,000 units a year × 3 years × $.10}) less what was already purchased ($8,000 {80,000 units × $.10}) = $52,000.

How well did you know this?
1
Not at all
2
3
4
5
Perfectly
356
Q

A company determined the following values for its inventory as of the end of its fiscal year:

Historical cost $100,000
Current replacement cost 70,000
Net realizable value 90,000
Net realizable value less normal profit margin 85,000
Fair value 95,000

Under IFRS, what amount should the company report as inventory on its Balance Sheet?

A

$90,000

Since historical cost is greater than any of the other values, the question is to what value should the inventory be marked down? This answer is correct because it is the net realizable value and the IFRS requires the lower of cost or net realizable value.

How well did you know this?
1
Not at all
2
3
4
5
Perfectly
357
Q

A company manufactures and distributes replacement parts for various industries. As of December 31, year 1, the following amounts pertain to the company’s inventory:

Item Cost NRC SP Cost/sell or
dispose Norm PM

Blades $41,000 $ 38,000 $ 50,000 $ 2,000 $15,000
Towers 52,000 40,000 54,000 4,000 14,000
Generators 20,000 24,000 30,000 2,000 6,000
Gearboxes 80,000 105,000 120,000 12,000 8,000

What is the total carrying value of the company’s inventory as of December 31, year 1, under IFRS?

A

$191,000

Inventory under IFRS is reported at the lower of cost or net realizable value (NRV) where NRV is the selling price less the cost to complete or dispose. The table below calculates the NRV for each inventory item.

NRV Cost Lower of

                                                                Cost or NRV Blades	50,000 − 2,000 = 48,000	41,000	41,000 Towers	54,000 − 4,000 = 50,000	52,000	50,000 Generators30,000 − 2,000 = 28,000	20,000	20,000 Gearboxes120,000 − 12,000 = 108,00080,000	80,000 Total	 	 	191,000
How well did you know this?
1
Not at all
2
3
4
5
Perfectly
358
Q

A manufacturer has the following per-unit costs and values for its sole product:

Cost 10.00
Current replacement cost 5.50
Net realizable value 6.00
Net realizable value less normal profit margin 5.20

In accordance with IFRS, what is the per-unit carrying value of inventory in the manufacturer’s statement of financial position?

A

6.00

IFRS requires that inventory be reported at the lower of cost or net realizable value. Net realizable value is defined by IAS 2 as “the estimated selling price in the ordinary course of business less the estimated costs of completion and the estimated costs necessary to make the sale.” In this question, NRV is lower than cost, therefore the inventory should be reported at NRV of 6.00.

How well did you know this?
1
Not at all
2
3
4
5
Perfectly
359
Q

Under IFRS, when an entity chooses the revaluation model as its accounting policy for measuring property, plant, and equipment, which of the following statements is correct?

A

When an asset is revalued, the entire class of property, plant, and equipment to which that asset belongs must be revalued.

When remeasurement to fair value is used, it must be applied to the entire class or components of PPE.

How well did you know this?
1
Not at all
2
3
4
5
Perfectly
360
Q

A company has a parcel of land to be used for a future production facility. The company applies the revaluation model under IFRS to this class of assets. In year 1, the company acquired the land for $100,000. At the end of year 1, the carrying amount was reduced to $90,000, which represented the fair value at that date. At the end of year 2, the land was revalued, and the fair value increased to $105,000. How should the company account for the year 2 change in fair value?

A

By recognizing $10,000 in profit or loss and $5,000 in other comprehensive income.

Under IFRS an increase in an assets fair value above original cost are recorded in a revaluation surplus account and any decreases in an assets fair value below the original cost are recorded as losses to the income statement. Therefore, the 10,000 decrease in year 1 would have been recorded as a loss to the income statement and the 15,000 increase in year 2 would be recorded as a 10,000 gain to the income statement and 5,000 gain in revaluation surplus (OCI).

How well did you know this?
1
Not at all
2
3
4
5
Perfectly
361
Q

On January 1, year one, an entity acquires a new piece of machinery for $100,000 with an estimated useful life of 10 years. The machine has a drum that must be replaced every five years and costs $20,000 to replace. Also included in the cost of the machine is an inspection fee of $8,000. Continued operations of the machine requires an inspection every four years after purchase. The company uses the straight-line method of depreciation. Under IFRS what is the depreciation expense for year one?

A

$13,200

Under IFRS the components of the asset must be depreciated over their estimated useful life. Therefore, the $100,000 cost is broken down into the following components:

Depreciable value	  Life	Depreciation
$72,000	                  10 yr.	$7,200
20,000	                   5 yr.	 4,000
8,000	                    4 yr.	 2,000
                                               $13,200
How well did you know this?
1
Not at all
2
3
4
5
Perfectly
362
Q

A transportation company purchased a passenger bus for $100,000 on January 1, year 1. The company expects the bus to be used for 20 years if it follows a maintenance schedule of replacing the engine after 10 years and replacing the seats every 8 years. It estimates that the current cost to replace the engine is $25,000 and the current cost to replace the seats is $10,000. The company uses straight-line depreciation, and the bus has no residual value. The company considers any component equal to or greater than 10% of the overall cost to be significant. Under IFRS, how much depreciation expense should the company recognize for the bus for the year ended December 31, year 1?

A

$7,000

Under component depreciation, the asset is separated into component parts, and each part is depreciated over its useful life. The bus would be separated into three parts and depreciated as follows:
Bus	         65,000	÷ 20 years =	3,250
Engine	25,000	÷ 10 years =	2,500
Seats	10,000	÷ 8 years =	1,250
Total	100,000	 	                  7,000
How well did you know this?
1
Not at all
2
3
4
5
Perfectly
363
Q

On December 31, 2005, Vey Co. traded equipment with an original cost of $100,000 and accumulated depreciation of $40,000 for productive equipment with a fair value of $60,000.

In addition, Vey received $30,000 cash in connection with this exchange. There is commercial substance to the exchange.

What should be Vey’s carrying amount for the equipment received at December 31, 2005?

A

$60,000

When there is commercial substance to the exchange, the acquired asset is measured at fair value. In this case, the value is $60,000 as given in the problem. This amount also equals the fair value of assets given up in the exchange. The implied fair value of the asset exchanged is $60,000 + $30,000 cash received, or $90,000. The fair value of assets given up is therefore $90,000 less $30,000 cash received, or $60,000. The full journal entry for the exchange is: dr. plant asset 60,000; dr. accumulated depreciation, 40,000; debit cash 30,000; credit plant asset 100,000; credit gain, 30,000. The gain equals the difference between the fair value of the asset exchanged (90,000) and its book value (60,000).

How well did you know this?
1
Not at all
2
3
4
5
Perfectly
364
Q

On January 1, Feld traded a delivery truck and paid $10,000 cash for a tow truck owned by Baker. The delivery truck had an original cost of $140,000, accumulated depreciation of $80,000, and an estimated fair value of $90,000. Feld estimated the fair value of Baker’s tow truck to be $100,000. The transaction had commercial substance. What amount of gain should be recognized by Feld?

A

$30,000

The book value of the delivery truck is $60,000 ($140,000 − $80,000). Its fair value is $90,000. A gain of $30,000 is therefore implied. Cash was paid and the exchange had commercial substance. Therefore, the gain is fully recognized. If the exchange lacked commercial substance, no gain would be recognized.

How well did you know this?
1
Not at all
2
3
4
5
Perfectly
365
Q

Which of the following statements describes the proper accounting for losses when nonmonetary assets are exchanged for other nonmonetary assets?

A

A loss is recognized immediately, because assets received should not be valued at more than their cash-equivalent price.

The loss recognized on the exchange equals the book value of the asset transferred less its fair value at the time of the exchange. The fair value is less than book value. The amount recorded for the asset acquired is its fair value, or the fair value of the asset transferred plus or minus cash paid or received, whichever is more reliably determinable. By using the lower fair value of the asset transferred to measure the value of the asset acquired, the asset acquired will not be overstated above its fair value.

How well did you know this?
1
Not at all
2
3
4
5
Perfectly
366
Q

Which of the following statements correctly describes the proper accounting for nonmonetary exchanges that are deemed to have commercial substance?

A

It recognizes gains and losses immediately.

Gains and losses from nonmonetary exchanges that have commercial substance are recognized immediately.

How well did you know this?
1
Not at all
2
3
4
5
Perfectly
367
Q

Bensol Co. and Sable Co. exchanged similar trucks with fair values in excess of carrying amounts. In addition, Bensol paid Sable to compensate for the difference in truck values.

The exchange has commercial substance.

As a consequence of the exchange, Sable recognizes

A

A gain equal to the difference between the fair value and carrying amount of the truck given up

With commercial substance, the exchange is measured at fair value. The full gain is recognized and is equal to the difference between the fair value of the asset given up and its book value.

For example, assume the following values: new asset fair value 20, old asset fair value 26, cash received 6, old asset cost 30, old asset accumulated depreciation 9. The full entry is: dr. New Truck 20; dr. Accumulated Depreciation 9; dr. Cash 6; cr. Old Truck 30; cr. Gain 5.

The gain equals the old asset’s fair value of 26 and its book value of 21 (30 − 9).

How well did you know this?
1
Not at all
2
3
4
5
Perfectly
368
Q

During 2004, Beam Co. paid $1,000 cash and traded inventory, which had a carrying amount of $20,000 and a fair value of $21,000, for other inventory in the same line of business with a fair value of $22,000.

The exchange was made to facilitate sales to their respective customers.

What amount of gain (loss) should Beam record related to the inventory exchange?

A

$0

There is a $1,000 gain inherent in the transfer of the old (Beam’s) inventory item (fair value of $21,000 - carrying amount of $20,000). If Beam’s item were sold, gross profit of $1,000 would result. However, under GAAP, exchanges of inventory made to facilitate sales are an exception to fair value measurement. Therefore, no gain or loss is recognized and the inventory received is valued at the book value of the inventory given up plus cash paid, for a total of $21,000. This amount is $1,000 less than the new inventory’s fair value because the $1,000 gain is disallowed.

How well did you know this?
1
Not at all
2
3
4
5
Perfectly
369
Q

Yola Co. and Zaro Co. are fuel oil distributors. To facilitate the delivery of oil to their customers, Yola and Zaro exchanged ownership of 1,200 barrels of oil without physically moving the oil. Yola paid Zaro $30,000 to compensate for a difference in the grade of oil.

On the date of the exchange, cost and market values of the oil were as follows:

                         Yola Co.	Zaro Co. Cost	              $100,000	$126,000 Market values	120,000	150,000

In Zaro’s income statement, what amount of gain should be reported from the exchange of the oil?

A

$0

This exchange was made to facilitate a sale of inventory to customers.

Under FAS 153, this is one of the exceptions to measuring exchanges of nonmonetary assets at market value. In this case, the exchange is measured at book value; no gain or loss is recognized. The oil received by Zaro would be measured (debited) at book value ($126,000) less cash received ($30,000), or $96,000. Cash would be debited for $30,000. The oil exchanged would be credited for $126,000.

No gain or loss is recognized.

How well did you know this?
1
Not at all
2
3
4
5
Perfectly
370
Q

On June 30, 2005, Finn, Inc. exchanged 2,000 shares of Edlow Corp. $30 par value common stock for a patent owned by Bisk Co. The Edlow stock was acquired in 2003 at a cost of $50,000.

At the exchange date, Edlow common stock had a fair value of $40 per share, and the patent had a net carrying amount of $100,000 on Bisk’s books.

Finn should record the patent at

A

$80,000.

The patent is valued at the market value of the stock exchanged, which is $80,000 ($40 × 2,000 shares).

In general, the recorded value of purchased intangibles is the value of the consideration given or the value of the intangible, whichever is more reliable. The book value of the seller is not a reliable substitute for market value. There is no reliable amount given for the market value of the patent.

How well did you know this?
1
Not at all
2
3
4
5
Perfectly
371
Q

A company exchanged land with an appraised value of $50,000 and an original cost of $20,000 for machinery with a fair value of $55,000. Assuming that the transaction has commercial substance, what is the gain on the exchange?

A

$30,000

The gain on an exchange of nonmonetary assets is based on the fair value and book value of the asset exchanged. The land with a fair value of $50,000 is given for machinery. The company is using the land as legal tender. The gain will be the difference between the book value and the fair value of the asset given or $50,000 − $20,000 = $30,000.

How well did you know this?
1
Not at all
2
3
4
5
Perfectly
372
Q

Bayberry Co. has an asset with a cost of $200,000 and accumulated depreciation of $120,000. Driftwood Co. has an asset with a cost of $250,000 and accumulated depreciation of $160,000. Both assets have a fair value of $100,000. Bayberry and Driftwood find it mutually advantageous to exchange assets, and the exchange results in improved future cash flows for both companies. What amount, if any, is Bayberry’s gain on the exchange?

A

$20,000

The gain on the exchange would be the difference between the carrying value of the asset exchanged and the fair value of that asset. Bayberry’s carrying value is $80,000 and the fair value is $100,000; therefore, the gain to Bayberry is $20,000.

How well did you know this?
1
Not at all
2
3
4
5
Perfectly
373
Q

Under International Financial Reporting
Standards (IFRS), what two methods can be
used to adjust accumulated depreciation?

A
  1. The proportional method

2. The reset method

How well did you know this?
1
Not at all
2
3
4
5
Perfectly
374
Q

What happens during the reset method?

A

Accumulated depreciation is reset to zero by
closing it to the building account, and then the
building is adjusted for the revaluation.

How well did you know this?
1
Not at all
2
3
4
5
Perfectly
375
Q

Under International Financial Reporting
Standards (IFRS), how is interest during
construction accounted for?

A

It is expensed or capitalized.

How well did you know this?
1
Not at all
2
3
4
5
Perfectly
376
Q

Where is revaluation surplus reported under
International Financial Reporting Standards
(IFRS) until the property, plant, and equipment
(PPE) is sold?

A

It is reported in equity.

How well did you know this?
1
Not at all
2
3
4
5
Perfectly
377
Q

Under International Financial Reporting
Standards (IFRS), is revaluation of property,
plant, and equipment (PPE) allowed?

A

Yes, revaluation is allowed.

How well did you know this?
1
Not at all
2
3
4
5
Perfectly
378
Q

How frequently do companies have to review
depreciation policies under International
Financial Reporting Standards (IFRS)?

A

Companies have to review the policies

annually.

How well did you know this?
1
Not at all
2
3
4
5
Perfectly
379
Q

What is the fair value of acquired assets if cash

is paid in exchange?

A

Fair value of acquired asset = Fair value of asset

exchanged + Cash paid

How well did you know this?
1
Not at all
2
3
4
5
Perfectly
380
Q

List the characteristics of an exchange that

indicate commercial substance

A

The amount of cash paid or received on
exchange is significant in relation to the
fair value of the assets exchanged.

The functions of the assets exchanged
are different.

How well did you know this?
1
Not at all
2
3
4
5
Perfectly
381
Q

Define “nonmonetary asset.”

A

An asset that does not have a fixed nominal or
stated value, as is the case with cash, accounts
receivable, and other monetary assets

How well did you know this?
1
Not at all
2
3
4
5
Perfectly
382
Q

In a nonmonetary exchange, under what
circumstances is fair value not used to value an
asset?

A

Fair value of either asset is not
determinable.

Exchange is made to facilitate a sale.

Exchange lacks commercial substance.

How well did you know this?
1
Not at all
2
3
4
5
Perfectly
383
Q

What is the preferred valuation for an acquired

asset in a nonmonetary exchange?

A

The fair value of assets given in the exchange

How well did you know this?
1
Not at all
2
3
4
5
Perfectly
384
Q

When do you use list price?

A

List price should not be used for fair value

because they are notoriously inflated.

How well did you know this?
1
Not at all
2
3
4
5
Perfectly
385
Q

What is the fair value of acquired assets if cash

is received in exchange?

A

Fair value of acquired asset = Fair value of asset

exchanged – Cash received

How well did you know this?
1
Not at all
2
3
4
5
Perfectly
386
Q

Solen Co. and Nolse Co. exchanged trucks with fair values in excess of carrying amounts. In addition, Solen paid Nolse to compensate for the difference in truck values.

The exchange lacks commercial substance.

As a consequence of the exchange, Solen recognizes

A

Neither a gain nor a loss.

Solen has an implied gain given that the fair value of its asset exceeds its book value. But when there is no commercial substance, such gains are recognized only when cash is received. Solen paid cash on the exchange.

How well did you know this?
1
Not at all
2
3
4
5
Perfectly
387
Q

Slad Co. exchanged productive assets with Gil Co. and, in addition, paid Gil $100,000 cash. The following information pertains to this exchange:

Assets Carrying amts Fair values
Relinquished by Gil $75,000 $140,000
Relinquished by Slad 40,000 40,000

In Slad’s books, the assets acquired should be recorded at what amount?

A

$140,000

The entry is:

Asset (new) 140,000
Asset (old book value) 40,000
Cash 100,000

Slad has no gain because the fair value and carrying value of its asset (old) are the same. Slad has given up total consideration worth $140,000 at fair value, for an asset worth $140,000. There is no unrecognized gain on similar assets to diminish the recorded value of the new asset.

The entry is the same regardless of whether the exchange has commercial substance because the fair value of assets exchanged equals their book value in total ($140,000). There is no implied gain or loss.

How well did you know this?
1
Not at all
2
3
4
5
Perfectly
388
Q

May Co. and Sty Co. exchanged nonmonetary assets. The exchange did not culminate an earning process for either May or Sty (the exchange lacked commercial substance). May paid cash to Sty in connection with the exchange.

The book value of the asset exchanged exceeded its fair value for both firms. Therefore, a loss on the exchange should be recognized by

A

May - YES
Sty - YES

The fair value of each asset is less than book value implying that both firms have a loss. Losses are recognized in full regardless of whether there is commercial exchange.

How well did you know this?
1
Not at all
2
3
4
5
Perfectly
389
Q

Amble, Inc. exchanged a truck with a carrying amount of $12,000 and a fair value of $20,000 for a truck and $5,000 cash. The fair value of the truck received was $15,000.

At what amount should Amble record the truck received in the exchange assuming the exchange lacks commercial substance?

A

$15,000

There is an implied gain of $8,000, the difference between the $20,000 fair value of the old asset and its $12,000 book value. Because the proportion of cash received is 25% ($5,000/$20,000), the entire gain is recognized and the acquired asset is recognized at fair value ($15,000).

Note that the answer would be the same had there been commercial substance.

How well did you know this?
1
Not at all
2
3
4
5
Perfectly
390
Q

In an exchange of plant assets, Transit Co. received equipment with a fair value equal to the carrying amount of equipment given up. Transit also contributed cash.

The exchange lacks commercial substance.

As a result of the exchange, Transit recognized

A

A loss equal to the cash given up.

The fair value of the new asset equals the old asset’s book value. Because cash was paid, the fair value of the old asset is less than the fair value of the new asset.

Therefore, the fair value of the old asset is also less than the old asset’s book value resulting in a loss.

Using dollar amounts, assume the fair value of the new asset is $10. The book value of the old asset is also $10 by assumption. Assume Transit paid $2 cash.

Then the fair value of the old asset is $8 implying a loss of $2, the amount of cash paid. Even without commercial substance, losses are recognized in full.

How well did you know this?
1
Not at all
2
3
4
5
Perfectly
391
Q

Slate Co. and Talse Co. exchanged similar plots of land with fair values in excess of carrying amounts. In addition, Slate received cash from Talse to compensate for the difference in land values.

The exchange lacks commercial substance.

As a result of the exchange, Slate should recognize

A

A gain in an amount determined by the ratio of cash received to total consideration.

When commercial substance is lacking, gains are recognized in proportion to the amount of cash received.

How well did you know this?
1
Not at all
2
3
4
5
Perfectly
392
Q

In a barter transaction where advertising services provided are exchanged for advertising services received, under which of the following situations can the advertising provider recognize revenue for the services performed? Assume the accounting is under IFRS guidelines.

A

When there is a nonbarter transaction for similar advertising services that can be reliably measured with a different counterparty

The fair value of the advertising services provided can be reliably measured by reference to a nonbarter transaction for similar advertising with a different counterparty (SIC Interpretation 31, para 5).

How well did you know this?
1
Not at all
2
3
4
5
Perfectly
393
Q

Which of the following is not requirement for an asset to be categorized as a plant asset?

A

Have a useful life of at least three years.

A useful life of at least three years is NOT a requirement for classification of a plant asset. The plant asset must have a useful life extending more than one year beyond the Balance Sheet date.

How well did you know this?
1
Not at all
2
3
4
5
Perfectly
394
Q

Which of the following is a required footnote disclosure on property, plant, and equipment?

A

Range of useful lives of plant assets.
Depreciation methods of plant assets.
Accumulated depreciation related to plant assets.

All items listed are required disclosures: useful life, depreciation methods, and the accumulated depreciation of plant asset. Read through select disclosures of the financial statements of real companies-this will help reinforce the disclosure requirements and jog your memory because you will remember reading about the disclosure.

How well did you know this?
1
Not at all
2
3
4
5
Perfectly
395
Q

Theoretically, which of the following costs incurred in connection with a machine purchased for use in a company’s manufacturing operations would be capitalized?

A

Insurance on machine while in transit. - YES
Testing and preparation of machine for use. - YES

Both costs should be capitalized because they are costs necessary to place the asset into its intended use and location. Neither should be expensed as incurred.

How well did you know this?
1
Not at all
2
3
4
5
Perfectly
396
Q

Campbell Corp. exchanged delivery trucks with Highway, Inc. Campbell’s truck originally cost $23,000, its accumulated depreciation was $20,000, and its fair value was $5,000. Highway’s truck originally cost $23,500, its accumulated depreciation was $19,900, and its fair value was $5,700. Campbell also paid Highway $700 in cash as part of the transaction. The transaction lacks commercial substance. What amount is the new book value for the truck Campbell received?

A

$3,700

When a transaction lacks commercial substance and cash is paid, the new asset is recorded at the book value of the old asset plus any cash given. Campbell has the same economic position as before the exchange - a different truck used in the same manner and $700 less cash. The new truck is the BV of the old asset ($3,000) plus the cash paid ($700) or $3,700.

How well did you know this?
1
Not at all
2
3
4
5
Perfectly
397
Q

Charm Co. owns a delivery truck with an original cost of $10,000 and accumulated depreciation of $7,000. Charm acquired a new truck by exchanging the old truck and paying $2,000 in cash. The new truck has a fair value of $5,000 at the time of the exchange. What amount of gain or loss should Charm recognize?

A

$0

This is an exchange with no commercial substance because the exchange was one truck for another. When cash is given and there is no commercial substance, a gain is not recognized. This transaction is an even exchange in value. Charm gave the old truck with a net book value of $3,000 ($10,000 – 7,000) plus $2,000 in cash, or a total of $5,000. The consideration given of $5,000 equals the fair value of the new truck, $5,000.

How well did you know this?
1
Not at all
2
3
4
5
Perfectly
398
Q

A company has a long-lived asset with a carrying value of $120,000, expected future cash flows of $130,000, present value of expected future cash flows of $100,000, and a market value of $105,000. Under IFRS what amount of impairment loss should be reported?

A

$15,000

This response is the difference between carrying value and recoverable amount. According to IFRS the recoverable amount is the greater of fair value less cost to sell ($105,000) or value in use ($100,000). Value in use is the discounted cash flows. Therefore, this asset is has an impairment of $15,000 because the recoverable amount is $105,000 and the carrying value is $120,000.

How well did you know this?
1
Not at all
2
3
4
5
Perfectly
399
Q

A company has a long-lived asset with a carrying value of $120,000, expected future cash flows of $130,000, present value of expected future cash flows of $100,000, and a market value of $105,000. What amount of impairment loss should be reported?

A

$0

The recoverable cost (expected future cash flows) of $130,000 exceeds the $120,000 book value. Therefore, the asset is not impaired, and no loss is recorded. Although both the market value and present value of the future cash flows are less than book value, as long as the nominal sum of future cash flows ($130,000) exceeds book value, no impairment is recorded. The firm is expected to recover its book value.

How well did you know this?
1
Not at all
2
3
4
5
Perfectly
400
Q

Restorations of carrying value for long-lived assets are permitted if an asset’s fair value increases subsequent to recording an impairment loss for which of the following?

A

Held for use - No
Held for disposal -Yes

If an asset is held for disposal, previous losses can be recovered. The logic is that the recovery will be realized in the near future if the asset is in the process of being disposed. In contrast, an asset held for use CANNOT recover previous impairment because there is no certainty regarding the ultimate realization of those losses.

How well did you know this?
1
Not at all
2
3
4
5
Perfectly
401
Q

Gown, Inc. sold a warehouse and used the proceeds to acquire a new warehouse. The excess of the proceeds over the carrying amount of the warehouse sold should be reported as a(an):

A

Part of continuing operations.

The excess of proceeds over the carrying value increases the net assets of the firm, is recorded as an ordinary gain, and is included in income from continuing operations. The purchase of the new warehouse is an unrelated transaction.

How well did you know this?
1
Not at all
2
3
4
5
Perfectly
402
Q

Carr, Inc. purchased equipment for $100,000 on January 1, 20X2. The equipment had an estimated 10-year useful life and a $15,000 salvage value. Carr uses the 200% declining-balance depreciation method. In its 20X3 Income Statement, what amount should Carr report as depreciation expense for the equipment?

A

$16,000

The 200% declining balance depreciation method is also called the double declining balance method or DDB. Because this is a declining balance method, the book value at the beginning of 20X3 must be computed, and that is affected by depreciation in 20X2. For 20X2, depreciation under DDB is 2/10 × $100,000 or $20,000. Note that salvage value is not subtracted when computing depreciation because the “declining balance” is book value. For 20X3, depreciation is 2/10 × ($100,000-$20,000) = $16,000 because the book value at the beginning of 20X3 is reduced by 20X2 depreciation.

How well did you know this?
1
Not at all
2
3
4
5
Perfectly
403
Q

A building suffered uninsured water and related damage. The damaged portion of the building was refurbished with upgraded materials. The cost and related accumulated depreciation of the damaged portion are identifiable.

To account for these events, the owner should:

A

Capitalize the cost of refurbishing and record a loss in the current period equal to the carrying amount of the damaged portion of the building.ca

When the portion of an asset that is removed from a larger asset has identifiable costs and accumulated depreciation amounts, those amounts are removed from the books. The difference between these two amounts is the carrying value of the damaged portion of the larger asset. There is no insurance. Therefore, the carrying value of the damaged portion is written off as a loss. The replacement assets are capitalized at cost. The entries are:

Portion removed New materials
Loss Asset
Accumulated depreciation Cash
Asset

How well did you know this?
1
Not at all
2
3
4
5
Perfectly
404
Q

A firm is constructing a warehouse for its own use and purchased the land for the site immediately before beginning construction. Interest is capitalized on which of the following:

A

Warehouse - YES
Land - No

The average accumulated expenditures for purposes of capitalizing interest during construction of the warehouse includes the land cost, but the interest is capitalized to the warehouse only. The land is not under construction.

How well did you know this?
1
Not at all
2
3
4
5
Perfectly
405
Q

Sun Co. was constructing fixed assets that qualified for interest capitalization. Sun had the following outstanding debt issuances during the entire year of construction:

$6,000,000 face value, 8% interest.

$8,000,000 face value, 9% interest.

None of the borrowings were specified for the construction of the qualified fixed asset. Average expenditures for the year were $1,000,000. What interest rate should Sun use to calculate capitalized interest on the construction?

A

8.57%

Neither debt issuances were identified as the construction loan. Therefore, the interest rate must be determined based on the weighted average of the interest on all of the debt outstanding during the year. The calculation is as follows:

           $6,000,000 × .08 =	$480,000
           $8,000,000 × .09 =	$720,000 Totals	$14,000,000	                 $1,200,000 $1,200,000 / $14,000,000 =	8.57%
How well did you know this?
1
Not at all
2
3
4
5
Perfectly
406
Q

A manufacturing firm purchased used equipment for $135,000. The original owners estimated that the residual value of the equipment was $10,000. The carrying amount of the equipment was $120,000 when ownership transferred. The new owners estimate that the expected remaining useful life of the equipment was 10 years, with a salvage value of $15,000. What amount represents the depreciable base used by the new owners?

A

$120,000

The purchase price of the asset acquired less its salvage value is the asset’s depreciable cost. In this case, total depreciation on the asset is limited to $120,000 ($135,000 purchase price-$15,000 salvage value). The cost to the seller and the previous salvage value are not relevant to the new owner.

How well did you know this?
1
Not at all
2
3
4
5
Perfectly
407
Q

A state government condemned Cory Co.’s parcel of real estate. Cory will receive $750,000 for this property, which has a carrying amount of $575,000. Cory incurred the following costs as a result of the condemnation:

Appraisal fees to support a $750,000 value $2,500
Attorney fees for the closing with the state 3,500
Attorney fees to review contract to acquire replacement property 3,000
Title insurance on replacement property 4,000

What amount of cost should Cory use to determine the gain on the condemnation?

A

$581,000

The total value to be compared to the amount received from the government in computing the gain:

Carrying amount $575,000
Plus appraisal fees to support a $750,000 value
2,500
Plus attorney fees for the closing with the state 3,500
Equals total cost to compare to $750,000 received from state $581,000

The second and third items in the above list essentially reduce the net proceeds from the state and thus decrease the gain. The $3,000 and $4,000 amounts pertaining to the replacement property are not associated with the existing property and do not affect the gain on its condemnation.

How well did you know this?
1
Not at all
2
3
4
5
Perfectly
408
Q

Papa Company acquired land with an office building on it from its subsidiary, Sonny Company, for $110,000. Prior to the sale, Sonny’s carrying value of the land was $60,000 and its net carrying value of the building was $50,000. At the time of the transaction, Papa appropriately determined that the land had a fair value of $75,000 and the building had a fair value of $35,000. At what amount should the land and building be reported on Papa’s consolidated statements prepared immediately after the transaction?

A

Land - $ 60,000
Building - $ 50,000

Even though there was no profit or loss on the intercompany transaction, it resulted in amounts being redistributed between the depreciable asset office building and the non-amortizable asset land, which would result in different amounts of depreciation expense than if the transaction had not occurred. Therefore, the intercompany transaction must be “eliminated” so that the consolidated statements would show land at $60,000 and buildings at $50,000. (Sonny also would need to assess the building for possible impairment.)

How well did you know this?
1
Not at all
2
3
4
5
Perfectly
409
Q

Young Corp. purchased equipment by making a down payment of $4,000 and issuing a note payable for $18,000. A payment of $6,000 is to be made at the end of each year for three years. The applicable rate of interest is 8%. The present value of an ordinary annuity factor for three years at 8% is 2.58, and the present value for the future amount of a single sum of one dollar for three years at 8% is .735. Shipping charges for the equipment were $2,000, and installation charges were $3,500. What is the capitalized cost of the equipment?

A

$24,980

The capitalized cost is the sum of the down payment, present value of the note payments, and the shipping and installation charges. $4,000 + $6,000(2.58) + $2,000 + $3,500 = $24,980. The present value of the three payments required on the note is capitalized, which excludes the interest included in those payments. The two charges are capitalized because they were incurred to place the asset into its intended condition and location.

How well did you know this?
1
Not at all
2
3
4
5
Perfectly
410
Q

Land was purchased to be used as the site for the construction of a plant. A building on the property was sold and removed by the buyer so that construction on the plant could begin.

The proceeds from the sale of the building should be:

A

Deducted from the cost of the land.

The proceeds from the building removed and sold reduce the cost of the land to the buyer. Had the building been razed, the net razing cost would be added to the land. Compared to the latter situation, the case in the problem results in a cost savings.

How well did you know this?
1
Not at all
2
3
4
5
Perfectly
411
Q

What is the required accounting treatment

when an investor has control of an investee?

A

Treat as a subsidiary and consolidate investee
with investor (consolidated financial
statements)

How well did you know this?
1
Not at all
2
3
4
5
Perfectly
412
Q

What is the basis for general guidelines for
determining the level of influence over an
investee?

A

The nature and extent of ownership

How well did you know this?
1
Not at all
2
3
4
5
Perfectly
413
Q

What is the basis for general guidelines for
determining the level of economic influence
over an equity investee?

A

The nature and extent of ownership rights

How well did you know this?
1
Not at all
2
3
4
5
Perfectly
414
Q

List the investor’s considerations in selecting

the correct accounting for an investment.

A

The nature of the investment
The extent of the investment
Management’s intent

How well did you know this?
1
Not at all
2
3
4
5
Perfectly
415
Q

List the investor’s considerations in selecting

the correct accounting for an investment.

A
Whether the investment is a debt or
equity security
Whether there is readily determinable fair
value
Management's intent on how long the
investment will be held
How well did you know this?
1
Not at all
2
3
4
5
Perfectly
416
Q

Identify the three possible levels of influence

over an investee for accounting purposes.

A
  1. Not significant
  2. Significant influence but not control
  3. Control
How well did you know this?
1
Not at all
2
3
4
5
Perfectly
417
Q

Define “debt securities.”

A

Securities representing the right of the creditor
to receive from the debtor a principal amount
at a specified future date and to receive
interest as payment for providing use of funds

How well did you know this?
1
Not at all
2
3
4
5
Perfectly
418
Q

Define “equity securities.”

A

Securities representing ownership or the right

to acquire ownership interest

How well did you know this?
1
Not at all
2
3
4
5
Perfectly
419
Q

List the guidelines for determining no

significant influence in an investment.

A
The investment is:
In debt securities;
In nonvoting stock;
Temporary in nature;
Less than 20% ownership of voting stock.
How well did you know this?
1
Not at all
2
3
4
5
Perfectly
420
Q

Which one of the following is not considered an equity investment for investment accounting purposes?

A

Redeemable preferred stock

Redeemable preferred stock is not considered an equity security for investment accounting purposes. Redeemable preferred stock, also known as callable preferred stock, may be reacquired by the issuing corporation under prescribed conditions.

421
Q

In which one of the following circumstances would an investor most likely have control of an investee?

A

The investor owns more than 50% of the voting common stock of an investee.

When an investor owns more than 50% of the voting common stock of an investee, in the absence of constraining conditions (e.g., investee in bankruptcy), the investor has controlling interest in the investee.

422
Q

Which one of the following is least likely to be a factor in determining how an investment in debt or equity securities is accounted for and reported in financial statements?

A

The method of payment used to acquire the investment

The method of payment used to acquire an investment does not help determine the correct accounting treatment of the investment. While the method of payment determines what will be “credited” upon acquisition of the securities, it will not enter into the subsequent accounting treatment of the investment.

423
Q

Which, if any, of the following grants the investor ownership rights?

A

Bond investment - No
Common stock investment - Yes

An investment in the bonds of another entity does not give the investor an ownership interest, but an investment in the common stock of another entity does give the investor an ownership interest. An investment in the bonds of another entity establishes a debtor-creditor relationship, not an ownership relationship.

424
Q

In the absence of other relevant factors, what minimum level of voting ownership is considered to give an investor significant influence over an investee?

A

20%

The minimum level of voting ownership considered to give an investor significant influence over an investee is 20%. In the absence of other relevant factors, an investor is considered to have significant influence over an investee if it owns 20%–50% of the voting securities of the investee.

425
Q

Which of the following statements is true concerning the correct accounting for equity investments?

An investor must account for (measure) all equity investments using fair value.

An investor may elect to account for (measure) some equity investments at fair value.

A

II only.

An investor may elect to use fair value to account for or measure some investments that otherwise would be accounted for using the equity method (Statement II). However, an investor is not required to use fair value to account for all equity investments (Statement I).

426
Q

When are equity securities carried at fair value?

A

When there is no significant influence and fair

value is readily determinable

427
Q

Which condition indicates that fair value is

readily determinable?

A
1. Prices or bid-and-ask quotations are
currently available on a securities
exchange or in the over-the-counter
(OTC) market if the OTC prices are
publicly reported.
2. Prices or quotations are in a foreign
market that has the breadth and scope of
the U.S. markets.
3. Prices or quotations for investments on
the fair value per share is published
based on current transactions.
428
Q

Define equity securities.

A

Any security that represents ownership interest
or the right to acquire or dispose of ownership
interest.

429
Q

What is “look through” with respect to valuing

an equity security?

A

Look-through means that the investor would
“look through” the form of the investment to
the nature of the securities held by the
investee. Look through is not allowed in
determining the nature of the investment

430
Q

Paxton Corporation purchased 100 shares of Swedberg Company’s common stock. The purchase is for $40 per share plus brokerage fees of $280. Paxton’s entry to record the investment would include

A. Dr. Cash $4,00 Cr. Investment Swedberg $4,000
B. Dr.Investment Swedberg $4,280 Cr.Cash $4,280
C. Dr.Investment Swedberg $4,000
Dr. Brokerage Fee Expense 280
Cr. Cash $4,280
D. Dr. Investment Swedberg $4,000 Cr.Cash $4,000

A

Row B

Dr.Investment Swedberg $4,280
Cr.Cash $4,280

The investment in Swedberg is initially measured and recorded at the price paid for the shares including the brokerage fees. The subsequent measurement of an equity investment is at fair value.

431
Q

On March 14, Apple Corporation purchased 6,000 shares of Pear Inc. for $25 per share plus a $340 brokerage fee. On June 30, when the shares were trading at $27, Apple prepared an adjustment to fair value and recorded the annual dividend of $0.40 per share. On August 14, Apple sold 4,000 shares of Pear for $29 per share less a brokerage fee of $225. The journal entry at the date of sale would include

A

A debit to cash for $115,775.

432
Q

Which of the following is not an indication that an equity security has readily determinable fair value?

A

Prices must be estimated based on similar securities in inactive markets.

Prices estimated based on similar securities in an inactive market are an indication that the fair value of the security is not readily determinable.

433
Q

On December 31, Ott Co. had investments in equity securities as follows:

                  Cost	Fair value	Lower of cost 
                                                     or fair value Man Co.	      $10,000	$ 8,000	$ 8,000 Kemo, Inc.	9,000	11,000	9,000 Fenn Corp.	11,000	9,000	9,000
                $30,000	$28,000	$26,000
                     ======	 ======	====== Ott's December 31 Balance Sheet should report the equity securities as
A

$28,000.

Investments in equity securities are reported at fair value ($28,000).

434
Q

When an investor does not exert influence over the investee and accounts for an equity investment at fair value, cash dividends received by the investor from the investee should normally be recorded as

A

Dividend income.

When there is no influence over the investee and the equity investments carried at fair value, cash dividends are recorded as dividend income.

435
Q

What major transactions or events would cause
the carrying amount of an investment to
change when the cost method is used to
account for the investment?

A

The carrying amount of the investment would
change when there is an observable
transaction for a similar or identical security.

436
Q

Equity investments can be carried at cost when

A

Fair value is not readily determinable

437
Q

Where on the financial statements are
impairment losses on equity investments
recorded?

A

In the income statement

438
Q

How are dividends recorded for equity

investments carried at cost?

A

As dividend income

439
Q

Assume an entity is holding an equity security where there is not a readily determinable fair value. Which of the following is not a factor to consider in the evaluation of potential impairment?

A

The costs associated with gathering data on similar investments, researching valuation methodologies, and the cost to hire a valuation consultant

The costs associated with determining fair value are not taken into consideration when assessing whether fair value is readily determinable. The costs listed in this response are the typical and reasonable costs that the entity would incur to determine fair value.

440
Q

How is an impairment loss recognized on the financial statements for a cost method equity investment?

A

In current earnings

Impairment losses on equity investments carried at cost are recognized in current earnings.

441
Q

Which of the following describes a factor when an entity can invoke the practicability exception to using fair value when reporting an equity security?

A

There are no bid-and-ask quotations available on a securities exchange or in a publicly reported over-the-counter market.

If there are no sources of fair value quotations, including no bid-and-ask quotations in an exchange of over-the-counter markets, then the entity can elect the practicability exception and use the cost method.

442
Q

In Year 1, Sloco purchased an equity security for $40,000 and determined that the security had no readily determinable fair value. At the end of Year 2, there were observable price changes in a similar security that indicated that the fair value of Solco’s investment had declined to $36,000. Because of recovery in market conditions in Year 3, there were observable price changes in the similar security, indicating that the value of Sloco’s investment is $41,000. What amount, if any, would Sloco recognize as an impairment (loss) or gain in Year 2 and Year 3?

Year 2	Year 3
A. $ 0	$ 0
B. ($4,000)	$ 0
C. ($4,000)	$4,000
D. ($4,000)	$5,000
A

Row D.

The impairment loss of $4,000 would be recognized in Year 2, and the gain for recovery of $5,000 in Year 3 would also be recognized. If there are observable transactions from which to determine fair value, then that information should be used to measure the investment.

443
Q

On December 31, Ott Co. had investments in equity securities as follows:

% Owned Investment Cost Fair value 12/31
3% Man Co. $10,000 $ 11,000
19% Kemo, Inc. 9,000 Not readily
available
5% Fenn Corp. 11,000 7,000

Ott’s December 31 balance sheet should report the equity securities as

A

$27,000.

Investments in equity securities are reported at fair value if the control is not significant and there is a readily determinable fair value. If the control is not significant and there is not a readily determinable fair value, then the entity may elect to use the cost method. In this situation, only the investment in Kemo Inc. qualifies to be reported using the cost method.

444
Q

Identify the three major equity method items

recognized each period by an investor.

A
1. Recognize investor's share of investee's
net income/loss.
2. Recognize investor's share of investee's
dividends declared.
3. Recognize adjustment to share of
investee's net income/loss for
"depreciation/amortization" of amount
allocated to excess of fair value over
book value.
445
Q

Under what conditions will an investment give
the investor significant influence, but not
control, over the investee?

A

When an investor owns 20% to 50% of the
voting equity securities of an investee and
there are no impediments to the investor
exercising its voting rights to influence the
investee’s operating and financial policies.
Investments in nonvoting equity securities
(e.g., preferred stock) or in debt securities do
not convey influence

446
Q

At the time an investor makes an investment
that gives it significant influence over an
investee, what information must the investor
determine in order to use the equity method of
accounting?

A

At the time of investment, the investor must
determine:
Book value of assets and liabilities of
investee.
Fair value of assets and liabilities of
investee.
Allocation of any difference between cost
of investment and fair value of investee’s
assets and liabilities

447
Q

When an investor has significant influence over
the operating and financial policies of an
investee, what method must be used to
account for the investment in the investee?

A

The investment must be carried on the
investor’s books and reported in the investor’s
financial statements using the equity method
of accounting.

448
Q

At the time an investment gives the investor
significant influence, but not control, over an
investee, how will any difference between the
cost of the investment and the book value of
the investee’s assets and liabilities be
allocated?

A
As an adjustment to the investee's assets and
liabilities to fair value, then
If cost of investment > fair value of
investee's net assets, to goodwill; or
If cost of investment < fair value of
investee's net assets, to gain.
449
Q

What is the required accounting if a change in
an investor’s level of ownership results in a loss
of significant influence, but the entire
investment is not disposed of?

A

The investor must cease using the equity
method of accounting and begin accounting for
the investment at fair value if fair value is
readily determinable. The investment will be
adjusted to fair value at the date significant
influence is lost, and any difference between
fair value and the prior equity-based carrying
amount will be recognized as a gain or loss in
current income

450
Q

Which of the following kinds of investments can result in the investor obtaining significant influence over an investee?

A

Equity investments - Yes
Debt investments - NO

An investment in equity securities of another entity gives the investor an ownership interest and, therefore, the ability to vote in corporate elections. An investment in the debt of another entity does not give the investor an ownership interest or the right to vote in corporate elections. An investment in the debt of another entity establishes a debtor-creditor relationship, not an ownership relationship.

451
Q

When an investor acquires sufficient voting common stock of an investee so that it has significant influence, which, if any, of the following kinds of investee data must the investor “capture” at the time the investment is made?

A

Book values of assets & liabilities - YES
Fair values of assets & liabilities - YES

At the time an investor acquires sufficient voting stock of an investee to give it significant influence over the investee, it must “capture” both the book values and fair values of the investee’s assets and liabilities in order to apply the equity method of accounting to the investment.

452
Q

On October 1, 200X, Catco acquired 12% of the common stock of Dexco. The firms had no other relationships or transactions. On January 1, 200Y, Catco acquired an additional 18% of Dexco’s common stock. There were no other transactions or relationships between the firms during 200Y. What method(s) of accounting would Catco have used for the investment during each of the following periods?

A

October 1–December 31, 200X - Fair Value
January 1–December 31, 200Y - Equity Method

Because Catco owned only 12% of Dexco’s common stock during the period October 1–December 31, 200X, it would not have been able to exercise significant influence over Dexco and would have used the fair value method. Because the purchase of an additional 18% of Dexco’s common stock would have given it a total of 30%, in the absence of other factors, it would be presumed to have significant influence over Dexco and, therefore, would have used the equity method during the period January 1–December 31, 200Y.

453
Q

In which one of the following cases is an investor most likely to use the equity method to carry and report an investment in an investee?

A

Investor is a manufacturing firm that owns 25% of the voting stock of a consulting firm.

An investor that owns 25% of the voting stock of an investee, in the absence of evidence to the contrary, is presumed to be able to exercise significant influence over the investee. The fact that the investor and the investee are in different lines of business generally does not mitigate the influence the investor is able to exercise.

454
Q

Larkin Co. has owned 25% of the common stock of Devon Co. for a number of years, and has the ability to exercise significant influence over Devon. The following information relates to Larkin’s investment in Devon during the most recent year:

Carrying amount of Larkin’s investment in Devon at the beginning of the year $200,000
Net income of Devon for the year 600,000
Total dividends paid to Devon’s stockholders during the year 400,000

What is the carrying amount of Larkin’s investment in Devon at year end?

A

$250,000

Larkin’s investment in Devon at year-end would be computed as the carrying amount of the investment at the beginning of the year ($200,000) + Larkin’s share of Devon’s reported net income for the year ($600,000 × .25 = $150,000)–Larkin’s share of Devon’s dividends paid during the year ($400,000 × .25 = $100,000), or $200,000 + $150,000 = $350,000-$100,000 = $250,000, the correct answer.

455
Q

A company has a 22% investment in another company that it accounts for using the equity method. Which of the following disclosures should be included in the company’s annual financial statements?

A

The company’s accounting policy for the investment

The investor must disclose the accounting policy for the investee. It is possible for the investor to use equity method accounting or elect the fair value option to account for the investee. The users of the financial statement need to know the basis for the equity accounting and if the investment included intercompany profits or other items that could impact the carrying value.

456
Q

Lee, Inc. acquired 30% of Polk Corp.’s voting stock on January 1, Year 1 for $100,000. During Year 1, Polk earned $40,000 and paid dividends of $25,000.

Lee’s 30% interest in Polk gives Lee the ability to exercise significant influence over Polk’s operating and financial policies. During Year 2, Polk earned $50,000 and paid dividends of $15,000 on April 1 and $15,000 on October 1.

On July 1, Year 2, Lee sold half of its stock in Polk for $66,000 cash.

Before income taxes, what amount should Lee include in its Year 1 Income Statement as a result of the investment?

A

$12,000

$12,000 = .30($40,000). Under the equity method, the investor recognizes its share of investee earnings in its own income. The equity method is used because Lee has significant influence over Polk.

The $7,500 amount is incorrect because it is Lee’s share of Polk’s dividends (.30 × $25,000). Under the equity method, dividends are a return of capital to the investor.
nificant influence over Polk.

457
Q

Lee, Inc. acquired 30% of Polk Corp.’s voting stock on January 1, year 1, for $100,000.

During year 1, Polk earned $40,000 and paid dividends of $25,000. Lee’s 30% interest in Polk gives Lee the ability to exercise significant influence over Polk’s operating and financial policies. During year 2, Polk earned $50,000 and paid dividends of $15,000 on April 1 and $15,000 on October 1.

On July 1, year 2, Lee sold half of its stock in Polk for $66,000 cash.

The carrying amount of this investment in Lee’s December 31, year 1 Balance Sheet should be

A

$104,500.

$104,500 = $100,000 + .30[$40,000–$25,000] because the equity method is used by Lee, who has significant influence over Polk.

Under the equity method, the investor recognizes its share of undistributed earnings in the investee, since acquiring the investment, in its income.

458
Q

When the equity method is used to account for investments in common stock, which of the following affects the investor’s reported investment income?

A

Goodwill amortization related to purchase - NO
Cash dividends from investee - NO

Under the equity method of accounting for an investment, neither amortization of goodwill nor dividends from the investee affect the investor’s investment income. Goodwill resulting from an investment in another entity (i.e., the excess of the cost of the investment over the investor’s share of the fair value of the investee’s identifiable assets) is not amortized. Dividends from the investee are not recognized as income; rather, they reduce the investment account.

459
Q

Sage, Inc. bought 40% of Adams Corp.’s outstanding common stock on January 2, Year 1 for $400,000. The investment gave Sage significant influence over Adams.

The carrying amount of Adams’ net assets at the purchase date totaled $900,000. Fair values and carrying amounts were the same for all items except for plant and inventory, for which fair values exceeded their carrying amounts by $90,000 and $10,000, respectively. The plant has an 18-year life. All inventory was sold during Year 1. Goodwill, if any, is expected to have a useful life of 40 years. During Year 1, Adams reported net income of $120,000 and paid a $20,000 cash dividend.

What amount should Sage report in its Income Statement from its investment in Adams for the year ended December 31, Year 1?

A

$42,000

This is a question on the full equity method.

Goodwill on the purchase = price of investment - 40%
(fair value of Adams’ net assets)

= $400,000 -.40($900,000 + $90,000 + $10,000)
= $400,000 - $400,000 = 0 (there is no goodwill to amortize)

Investment income:

40% of Adams’ income: .40($120,000) $48,000

Less 40% of excess of fair value of inventory over book value at acquisition. The inventory is sold, and therefore, the cost of goods sold of the investor must be increased by this amount: .40($10,000)
(4,000)

Less depreciation on 40% of the excess of fair value over book value of equipment: .40($90,000)/18
(2,000)

Equals amount Sage reports as investment income $42,000

460
Q

In its financial statements, Pulham Corp. uses the equity method of accounting for its 30% ownership of Angles Corp. At December 31, Year 2, Pulham has a receivable from Angles.

How should the receivable be reported in Pulham’s Year 2 financial statements?

A

The total receivable should be disclosed separately.

Although the equity method is often called a “one-line” consolidation, intercompany receivables remain separate from the investment account. Intercompany profit or loss is eliminated but that affects the income recognized by Pulham, not the receivable.

461
Q

Green Corp. owns 30% of the outstanding common stock and 100% of the outstanding noncumulative nonvoting preferred stock of Axel Corp. Green’s 30% ownership of common stock gives it significant influence over Axel.

In 2004, Axel declared dividends of $100,000 on its common stock and $60,000 on its preferred stock. Green exercises significant influence over Axel’s operations.

What amount of dividend revenue should Green report in its Income Statement for the year ended December 31, 2004?

A

$60,000

Only the dividends received on the preferred stock are recognized as revenue: $60,000 = 100% × ($60,000). The common stock investment is accounted for under the equity method, which treats all dividends received as a return of capital. Dividends reduce the investment account under this method.

462
Q

Pal Corp.’s 2004 dividend income included only part of the dividend received from its Ima Corp. investment. The balance of the dividend reduced Pal’s carrying amount for its Ima investment. This reflects that Pal accounts for its Ima investment by the:

A

Fair Value method, and only a portion of Ima’s 2004 dividends represent earnings after Pal’s acquisition.

The portion of the dividend reducing the investment carrying value is a liquidating dividend. A liquidating dividend occurs when the investee pays more income than was earned during the period the investor owned the shares of the investee.

For example, assume that Pal held 1% of Ima’s outstanding stock from January 1-December 31 of 2004 only. Ima earned $40,000 during 2004 but paid $50,000 in dividends ($10,000 coming from earnings before 2004). Pal would receive $500 dividends in total (1%), but only $400 are attributable to earnings during the period Pal was a shareholder. Thus, $100 of the dividend is attributable to income earned by Ima before Pal became an investor. From Pal’s viewpoint, this is a return of a portion of Pal’s investment, a liquidating dividend.

Under the cost method, liquidating dividends are treated as a reduction in the investment account whereas normal dividends are treated as income. The wording of the question implies that dividends are otherwise treated as income. Thus, the equity method could not be the method used by the firm.

Under the equity method, all dividends are treated as a reduction in the investment account. No dividends received are treated as income under the equity method.

463
Q

Information pertaining to dividends from Wray Corp.’s common stock investments for the year ended December 31, 2017 are as follows:

On September 8, 2017, Wray received a $50,000 cash dividend from Seco, Inc., in which Wray owns a 30% interest. A majority of Wray’s directors are also directors of Seco.

On October 15, 2017, Wray received a $6,000 dividend from King Co. Wray owns a 5% in King Co.

Wray owns a 2% interest in Bow Corp., which declared a $200,000 cash dividend on November 27, 2017 to stockholders of record on December 15, 2017, payable on January 5, 2018.

What amount should Wray report as dividend income in its Income Statement for the year ended December 31, 2017?

A

$10,000

This answer is correct because it includes the King and Bow dividend.

464
Q

Peel Co. received a cash dividend from a common stock investment. Should Peel report an increase in the investment account if it accounts for the investment as held-for-trading or uses the equity method of accounting?

A

Held-for-trading - No
Equity method - No

A dividend never increases the investment account under any accounting method.

Under the cost method, the dividend is recorded as revenue. Under the equity method, the dividend is recorded as a decrease in the investment account.

465
Q

Pare, Inc. purchased 10% of Tot Co.’s 100,000 outstanding shares of common stock on January 2, Year 1, for $50,000.

On December 31, Year 1, Pare purchased an additional 20,000 shares of Tot for $150,000. There was no goodwill as a result of either acquisition, and Tot had not issued any additional stock during Year 1. Tot reported earnings of $300,000 for Year 1.

What amount should Pare report in its December 31, Year 1, Balance Sheet as investment in Tot?

A

$200,000

Once the investor has acquired a sufficient percentage of the stock to use the equity method, it is prospectively applied. In this question, the equity method becomes the required method only at the very end of the year, and Pare would begin applying the equity method beginning on December 31. The ending balance of the investment account is: $50,000 (original investment) + $150,000 (second investment).

466
Q

Grant, Inc. acquired 30% of South Co.’s voting stock for $200,000 on January 2, 2015.

Grant’s 30% interest in South gave Grant the ability to exercise significant influence over South’s operating and financial policies. During 2015, South earned $80,000 and paid dividends of $50,000. South reported earnings of $100,000 for the six months ended June 30, 2016, and $200,000 for the year ended December 31, 2016. On July 1, 2016, Grant sold half of its stock in South for $150,000 cash. South paid dividends of $60,000 on October 1, 2016.

Before income taxes, what amount should Grant include in its 2015 Income Statement as a result of the investment?

A

$24,000

Grant uses the equity method because it has significant influence over South. Under the equity method, the investor recognizes its share of the investee earnings in its own income.

Thus, Grant will recognize $24,000 (.30 × $80,000) as equity in the earnings of South in its own income.

467
Q

What investments are classified as availablefor-sale?

A

Any debt investments not classified as either
held-to-maturity or held-for-trading. The
available-for-sale category is the default
category if an investment in debt does not
meet the requirements of either held-tomaturity
or held-for-trading

468
Q

How are available-for-sale investments
accounted for and reported in financial
statements?

A

Recognize interest income.

Amortize discount or premium, if any, on
debt securities.

Adjust investment to fair value at balance
sheet date with any gain/loss reported as
an item of other comprehensive income.

469
Q

How are available-for-sale investments

reported in the balance sheet?

A

At fair value as either current or noncurrent

assets (based on the entity’s policy).

470
Q

What amounts are included in a gain or loss
recognized on the sale of an available-for-sale
investment?

A
The gain or loss recognized on the sale of an
available-for-sale investment includes
The difference between the carrying
value of the investment and its selling
price; and
Any unrealized gain or loss in
accumulated other comprehensive
income related to the securities sold.
471
Q

What amounts should be included in the initial

recording of a held-for-trading investment?

A

Purchase price of the debt security

Directly related cost of acquisition (e.g.,
brokerage fee, transfer fee, etc.)

472
Q

List the criteria for held-for-trading securities.

A
  1. Applies to investments in debt securities.
  2. Investor buys for the purpose of selling in
    the near term.
473
Q

How are held-for-trading investments carried

and reported?

A

At fair value, with changes in fair value reported

in current income

474
Q

Where are unrealized holding gains and losses

on investments held-for-trading reported?

A
In income (income statement) as part of
income from continuing operations
475
Q

In year 1, a company reported in other comprehensive income an unrealized holding loss on a debt investment classified as available-for-sale. During Year 2, these securities were sold at a loss equal to the unrealized loss previously recognized. The reclassification adjustment should include which of the following?

A

The unrealized loss should be credited to the other comprehensive income account.

The unrealized loss would be credited to the other comprehensive income account to reclassify the holding loss as a realized loss in the income statement for Year 2. For purposes of illustration, assume the available-for-sale (AFS) securities were originally purchased for $5 and that the loss during Year 1 was $1. The related entries would be:

Purchase: DR. AFS Securities $5
CR. Cash $5

Year 1 DR. OCI (holding loss) $1
CR. AFS Securities $1

Year 2: DR. Cash $4
CR. AFS Securities $4

DR. Loss on AFS Securities $1 (Income Statement)
CR. OCI (holding loss) $1 (B/S, Accumulated OCI)

The last entry (above) reclassifies the holding loss to recognize a realized loss on sale.

476
Q

On January 1 of the current year, Barton Co. paid $900,000 to purchase two-year, 8%, $1,000,000 face value bonds that were issued by another publicly traded corporation. Barton plans to sell the bonds in the first quarter of the following year. The fair value of the bonds at the end of the current year was $1,020,000. At what amount should Barton report the bonds in its balance sheet at the end of the current year?

A

$1,020,000

For investments in debt securities other than those intended to be held-to-maturity, the fair value method is applied. $1,020,000 is the fair value of the investment in bonds and is the appropriate amount for reporting the investment.

477
Q

On December 29, 2017, BJ Co. sold a debt security investment that had been purchased on January 4, 2016. BJ owned no other securities. An unrealized loss was reported in the 2016 Income Statement. A realized gain was reported in the 2017 Income Statement.

Was the debt security classified as available-for-sale (AFS), and did its 2016 market price decline exceed its 2017 market price recovery?

AFS 2016 market price decline exceeded 2017
market price recovery
A. Yes Yes
B. Yes No
C. No Yes
D. No No

A

Row D

The security cannot be classified as available-for-sale because the unrealized gains and losses are recognized in the Income Statement. Unrealized gains and losses on available-for-sale securities are recognized in owners’ equity, not earnings. The second part of the question is somewhat ambiguous. The 2016 price decline could exceed or be exceeded by the 2017 price recovery. The loss in the first year is not related in amount and does not constrain the realized gain in the second year. The way to answer the question is to read the right column heading as implying that the earlier price decline must exceed the later price recovery. With that interpretation, the correct answer is no. For example, assume a cost of $10 and a market value of $4 at the end of the first year. An unrealized loss of $6 is recognized in earnings. During the second year, the security is sold for $12. A realized gain of $8 is recognized—the increase in the market value from the end of the first year to the sale in the second year. Thus, the market decline in the first year did not exceed the recovery in Year 2. (It could have exceeded the recovery in Year 2, but there is no requirement that it must.)

478
Q

At December 31, Hull Corp. had the following debt securities that were purchased during the year, its first year of operations:

                          Cost     Fair value	Unrealized G/(L) Held-for-trading:			 Security A	   $ 90,000  $ 60,000	$(30,000) Security B	      15,000	20,000	  5,000 Totals	         $105,000$ 80,000	$(25,000) ========	========	======== Available-for-sale:			 Security Y	$ 70,000	$ 80,000	$ 10,000 Security Z	90,000	45,000	(45,000) Totals	     $160,000	$ 125,000	$(35,000) ========	========	======== At December 31, adjustments to fair value should be established with a corresponding charge against
Income	Stockholders' equity
A.	$60,000	$0
B.	$30,000	$45,000
C.	$25,000	$35,000
D.	$25,000	$0
A

Row C

The $25,000 decline in value (unrealized loss) on trading securities is recognized in earnings for the year. The $35,000 decline in value (unrealized loss) on securities available-for-sale is recognized in owners’ equity, bypassing earnings. The reason for the difference in accounting treatment is that trading securities are held for short-term price appreciation. If the value of the trading portfolio increases or decreases, that gain or loss should be recognized in earnings consistent with the purpose for holding the investments. Securities available-for-sale are held for purposes other than short-term price appreciation. Thus, the increases and decreases in the portfolio market value may not be indicative of the intent of holding the securities. Recognition in earnings each year may cause unwarranted volatility in earnings.

479
Q

For a debt securities portfolio classified as available-for-sale, which of the following amounts should be included in the period’s net income?

I. Unrealized temporary losses during the period
II. Realized gains during the period
III. Changes in the valuation allowance during the period

A

II only

Only realized gains (from sale or reclassification) on available-for-sale securities are recognized in income for the period. The unrealized changes in fair value are recorded in owners’ equity.

480
Q

Jent Corp. purchased bonds at a discount of $10,000. Jent classified the bonds as available-for-sale and subsequently sold them at a premium of $14,000. At the time of the sale, $2,000 of the discount had been amortized. What amount should Jent report as gain on the sale of bonds?

A

$22,000

The book value at the date of sale was $8,000 below face value ($10,000 original discount – $2,000 amortization). The fair value of the bonds at date of sale was $14,000 above face value ($14,000 premium). Thus, the difference between the price of the bonds at sale and the book value was $22,000 ($8,000 + $14,000). That difference is the gain on sale.

481
Q

Which of the following is true with respect to impairment of available-for-sale securities?

A

If the decline in fair value is considered to be other-than-temporary, the unrealized losses in OCI are reclassified to earnings.

When the decline in fair value is considered to be other-than-temporary, the unrealized losses in OCI are reclassified to earnings.

482
Q

A short-term marketable debt security was purchased on September 1, Year 1, between interest dates. The next interest payment date was February 1, Year 2. On the balance sheet at December 31, Year 1, the debt security should be carried at

A

Fair value.

A short-term marketable debt security would be carried in a trading portfolio. Securities in trading portfolios are carried at fair value.

483
Q

Beach Co. determined that the decline in the fair value (FV) of an investment was below the amortized cost and permanent in nature (other-than-temporary). The investment was classified as available-for-sale on Beach’s books. Beach Co. does not elect the fair value option to account for these securities. The controller would properly record the decrease in FV by including it in which of the following?

A

Earnings section of the income statement

An available-for-sale security is valued at fair value at the balance sheet date, and any temporary decline in value is recorded in other comprehensive income for the period. However, because the decline was permanent (not temporary in nature), the available-for-sale security should be written down to fair value, and the amount of the write-down should be recorded in the income statement as a loss. Subsequent increases in the fair value of the available-for-sale security would be included in other comprehensive income in the year of the increase.

484
Q

Cook Company had the following investment portfolio of debt securities that were purchased during Year 2.

Stock Classification Cost / Fair Value 12-31-Y2
Company R Available-for-sale $30,000 $32,000
Company S Trading $42,000 $46,000
Company T Available-for-sale $15,000 $18,000

Cook elects to use the fair value option for reporting the investment in Company R. Which of the following statements is true?

A

Cook will report an unrealized gain on securities for $6,000 on the Year 2 income statement.

The fair value option may be elected on an instrument-by-instrument basis. Therefore, it is permissible for Cook to elect the fair value option for its investment in Company R. The investment in Company R would be revalued to fair value at the end of the period, and the resulting unrealized gain or loss would be recognized in earnings for Year 2. The investment in Company S is a trading security, and any unrealized gain is also recognized in earnings for the period. Therefore, Cook would report an unrealized gain of $2,000 for the investment in Company R and $4,000 for the investment in Company S, for a total gain of $6,000 on the Year 2 income statement. The unrealized gain on Investment T security would be reported under the provisions of ASC Topic 320 and included in other comprehensive income for the period.

485
Q

List the criteria for a held-to-maturity

classification.

A

The investment is a debt security

Investor has intent to hold to maturity.

Investor has ability to hold to maturity

486
Q

Under what conditions can a debt security sold

before maturity be considered held-tomaturity?

A

The sale is near enough to the maturity date so
that interest rate risk is substantially
eliminated.
The sale occurs after the investor has collected
a substantial portion (at least 85%) of the
principal outstanding at acquisition date.

487
Q

What amounts should be included in the initial

recording of a held-to-maturity investment?

A

Purchase price of security
Directly related cost of acquisition (e.g.,
brokerage fee, transfer fee, etc.)
Accrued interest, if any, is not included in
the cost of the investment.

488
Q

At what cost are held-to-maturity securities

carried and reported?

A

At amortized cost

489
Q

What method is used to amortize a premium or

discount on a security?

A

The effective interest method or straight-line

method if not materially different

490
Q

How is interest earned on held-to-maturity

investments reported in the income statement?

A

Reported as another income item in the income

statement

491
Q

Where are held-to-maturity investments

reported on the statement of cash flows?

A

As an investing activity

492
Q

The premium associated with an investment in
a callable bonds is amortized over what
period?

A

The premium is amortized to the earliest call

date.

493
Q

An investor purchased a bond as a long-term investment between interest dates at a premium. At the purchase date, the cash paid to the seller is

A

More than the face amount of the bond.

The amount paid by buyer would have been more than the face amount of the bond and would include the premium and the interest accrued since the last interest date.

494
Q

On January 1, 2014, Purl Corp. purchased, as a long-term investment, $500,000 face value Shaw, Inc. 8% bonds for $456,200. The bonds were purchased to yield 10% interest. Purl has the positive intent and ability to hold the bonds until maturity on January 1, 2020. The bonds pay interest annually on January 1, and Purl uses the interest method of amortization.

What amount (rounded to nearest $100) should Purl report on its December 31, 2015, balance sheet for this long-term investment?

A

$468,000

A held-to-maturity (HTM) investment purchased at a discount increases in value as maturity approaches, at which time the book value of the investment must be the face value of the investment. During the life of an HTM investment, the investor carries and reports the investment at amortized cost.

The interest and amortization entries for the two years 2014 and 2015 that lead to the correct ending balance at December 31, 2015, are:

December 31, 2014:
DR: Interest receivable .08($500,000) 40,000
DR: Discount on HTM bonds 5,620
CR: Interest revenue .10($456,200) 45,620

December 31, 2015:
DR: Interest receivable .08($500,000) 40,000
DR: Discount on HTM bonds 6,182
CR: Interest revenue .10($456,200 + $5,620) 46,182

Thus, the ending net investment balance at December 31, 2015, is $456,200 + $5,620 + $6,182 = $468,002, or $468,000 (rounded to the nearest $100 as required by the problem).

495
Q

On October 1, 2014, Park Co. purchased 200 of the $1,000-face-value, 10% bonds of Ott, Inc., for $220,000, including accrued interest of $5,000. The bonds, which mature on January 1, 2021, pay interest semiannually on January 1 and July 1. Park used the straight-line method of amortization and appropriately recorded the bonds as held-to-maturity.

On Park’s December 31, 2015, balance sheet, the bonds should be reported at

A

$212,000.

Held-to-maturity investments in bonds are reported at amortized cost. The discount or premium at purchase is amortized during the term of the bonds so that the carrying value is equal to face value at maturity. This is the amount to be received at maturity. The purchase price, exclusive of accrued interest, is $215,000 ($220,000 – $5,000). Accrued interest is not included in the investment carrying value. The premium paid on the bonds is $15,000 because the face value of the bonds is $200,000 (200 × $1,000). The term of holding the bonds is from October 1, 2014, to January 1, 2021, a period of six years and three months, or 75 months. The period from purchase to the December 31, 2015, balance sheet is 15 months. Amortization of the premium reduces the investment carrying amount because only face value, which is less than the amount paid for the investment, will be received at maturity. Therefore, the ending 12/31/15 investment carrying value is $212,000 = $215,000 – ($15,000 × 15/75 = $3,000).

496
Q

An investor purchased a bond classified as a held-to-maturity investment between interest dates at a discount. At the purchase date, the carrying amount of the bond is more than the:
Cash paid to seller Face amount of bond
A. No Yes
B. No No
C. Yes No
D. Yes Yes

A

Row B.

When a bond is purchased at a discount, the price paid is less than face value. Any cash paid to the seller for accrued interest is debited to interest receivable, not to the bond investment. Thus, the carrying value is the portion of the total amount paid attributable to the total bond price, exclusive of accrued interest. The carrying value must be less than the cash paid to the seller, which includes accrued interest.

497
Q

On July 1, 2014, York Co. purchased, as a held-to-maturity investment, $1,000,000 of Park, Inc.’s 8% bonds for $946,000, including accrued interest of $40,000. The bonds were purchased to yield 10% interest. The bonds mature on January 1, 2021, and pay interest annually on January 1. York uses the effective interest method of amortization.

On its December 31, 2014, balance sheet, what amount should York report as investment in bonds?

A

$911,300

Initial investment cost: $946,000 – $40,000 = $906,000

Interest revenue for 2014: $906,000(.10)(1/2 year) = $45,300
Less cash interest for 6 months: $1,000,000(.08)(1/2) = (40,000)

Equals amortization of discount (increases investment)
5,300

Investment in bonds balance at the end of 2014 $911,300

The initial investment cost or balance excludes accrued interest. The bonds were purchased at the halfway point in the interest period. York must pay for half a year’s interest and will receive a full year’s interest on January 1, 2015. The interest revenue for the year is based on the effective yield of 10%. The difference between interest revenue and the cash interest earned for the second half of 2014 is the growth in the value of the bond over time. The book value of the bond investment at maturity will be $1,000,000. Thus, the discount amortization increases the investment carrying value each year until it reaches $1,000,000.

498
Q

The credit losses associated with the impairment of debt securities are separated in which of the following circumstances?

A

When the entity has positive ability and intent to hold the impaired security and does not expect to recover the entire cost basis of the impaired security

When the entity has positive ability and intent to hold the impaired security and does not expect to recover the entire cost basis of the impaired security, the credit losses are recognized in earnings.

499
Q

Clara Corp. does not elect to use the fair value option to report financial assets. For marketable debt securities included in Clara’s held-to-maturity portfolio, which of the following amounts should be included in the period’s net income?
Unrealized temporary losses during the period
Gains on securities sold during the period
Permanent decline in value

A

II and III

This answer is correct because both II and III should be reported on the income statement. Unrealized gains and losses on held-to-maturity securities are not reported. Gains on securities sold always should be included as realized gains in the income statement of the applicable period. A permanent decline in value requires that the impaired security be written down to fair value. The amount of such a write-down is included in earnings as a realized loss.

500
Q

Zinc Company does not elect to use the fair value option for reporting financial assets. An unrealized gain, net of tax, on Zinc’s held-to-maturity portfolio of marketable debt securities should be reflected in the current financial statements as

A

A footnote or parenthetical disclosure only.

An unrealized gain on held-to-maturity securities is disclosed only in the notes to the financial statements. Gains are reflected in the financial statements only when they are realized (i.e., upon sale or for other than temporary declines in value). The year-end financial statements would present the held-to-maturity portfolio at cost. Parenthetical or footnote disclosure would indicate their market value.

501
Q

On July 1, Year 1, Pell Co. purchased Green Corp. 10-year, 8% bonds with a face amount of $500,000 for $420,000. The bonds mature on June 30, Year 9, and pay interest semiannually on June 30 and December 31. Using the interest method, Pell recorded bond discount amortization of $1,800 for the 6 months ended December 31, Year 1. From this long-term investment, Pell should report Year 1 interest revenue of

A

$21,800.

When the interest method of amortization is used, interest revenue is computed directly, and the amount of discount amortization is computed indirectly as follows:

Interest revenue – Stated or cash interest = Discount amortizations

In this case, the interest revenue cannot be computed directly because the effective or yield rate is not given. However, the stated or cash interest can be computed ($500,000 × 8% × 6/12 = $20,000), and the discount amortization is given ($1,800). Thus, the interest revenue can be computed indirectly, as indicated below.

Interest revenue – $20,000 = $1,800
Interest revenue = $20,000 + $1,800
Interest revenue = $21,800

502
Q

On April 1, North Company issued bonds in the market. Upon issue, South Company acquired 10% of North Company’s issue. On November 30, South sold the North Company bonds in the market, and the bonds were acquired by East Company. On December 31, which one of the companies, if any, is an investor?

A

East Company

Since East Company owns the bonds on December 31, it is the investor. North Company, the issuer of the bonds, is the investee. Since South Company did not issue the bonds and does not own the bonds on December 31, it is neither an investor nor an investee.

503
Q

On April 1, North Company issued bonds in the market. Upon issue, South Company acquired 10% of North Company’s issue. On November 30, South sold the North Company bonds in the market; the bonds were acquired by East Company. On December 31, which, if any, of the following companies is an investee?

A

North - YES
South - No
East - No

North is the investee because it issued the bonds, but neither South nor East is an investee. Since East owns the bonds on December 31, it is the investor. Since South did not issue the bonds and does not own the bonds on December 31, it is neither an investor nor an investee.

504
Q

How is the value of a stock right determined
when the per share market value of rights is
given?

A

The carrying value of the investment at the
time the rights are received is allocated
between the shares of stock and the newly
acquired rights based on the relative market
value of each set (total share and total rights).
The amount allocated to the total rights is then
divided by the number of rights received to get
the value of each right

505
Q

What entry is made by an investor who receives

stock rights?

A

An entry is made to transfer some of the cost of
the investment in the stock that “earned” the
rights to an account for the rights. The entry is:

DR: Security Stock Rights
CR: Investment

506
Q

What is the accounting treatment by an

investor when a stock dividend is received?

A

The investor adjusts the per share cost, not
total cost. Original cost is divided by the new
total number of shares (after the stock
dividend) to get the new per share cost.

507
Q

What is the accounting treatment by an

investor when a stock split occurs?

A

The investor adjusts the per share cost, not the
total cost. Original cost is divided by the new
total number of share (after the split) to get the
new per share cost.

508
Q

If stock rights are not exercised and lapse, what

entry should the investor make?

A

The investor writes off the stock rights and
recognizes a loss. The entry is:

DR: Loss on Expiration of Stock Rights
CR: Security Stock Rights

509
Q

On March 4, 2017, Evan Co. purchased 1,000 shares of LVC common stock at $80 per share.

On September 26, 2017, Evan received 1,000 stock rights to purchase an additional 1,000 shares at $90 per share. The stock rights had an expiration date of February 1, 2018. On September 30, 2017, LVC’s common stock had a market value, exrights, of $95 per share and the stock rights had a market value of $5 each.

What amount should Evan report on its September 30, 2017, Balance Sheet for investment in stock rights?

A

$4,000

The original stock investment cost is allocated to the stock and the rights based on their relative market values. Total market value of the stock is $95,000 and of the rights is $5,000. The original cost of the stock is $80,000. Thus the investment in stock rights is reported at [$5,000/($5,000 + $95,000)]$80,000 = $4,000.

510
Q

Cobb Co. purchased 10,000 shares (2% ownership) of Roe Co. on February 12. Cobb received a stock dividend of 2,000 shares on March 31 when the carrying amount per share on Roe’s books was $35 and the market value per share was $40. Roe paid a cash dividend of $1.50 per share on September 15.

In Cobb’s Income Statement for the year ended October 31, what amount should Cobb report as dividend income?

A

$18,000

Cobb has 10,000 + 2,000 or 12,000 shares at the time of the dividend. The 2,000 shares were received from the stock dividend. Thus, dividend income is 12,000($1.50) = $18,000. The stock dividend itself is not included in dividend income; rather, decreases the cost per share of the investment. However, the stock dividend does raise the number of shares that receive the cash dividend, which is recognized as income.

511
Q

Simpson Co. received dividends from its common stock investments during the year ended December 31 as follows:

A cash dividend of $8,000 from Wren Corp., in which Simpson owns a 2% interest.

A cash dividend of $45,000 from Brill Corp., in which Simpson owns a 30% interest. This investment is appropriately accounted for using the equity method.

A stock dividend of 500 shares from Paul Corp. was received on December 15 when the quoted market value of Paul’s shares was $10 per share. Simpson owns less than 1% of Paul’s common stock.

In Simpson’s Income Statement, dividend revenue should be:

A

$8,000.

Only the $8,000 dividend is included in dividend revenue. The dividend on the Brill stock is accounted for under the equity method, which treats all dividends as a reduction in the investment account.
The stock dividend is not revenue. Rather, it reduces the per share cost of the investment in Paul stock. No entry is recorded on receipt of a stock dividend.

512
Q

Plack Co. purchased 10,000 shares (2% ownership) of Ty Corp. on February 14.

Plack received a stock dividend of 2,000 shares on April 30 when the market value per share was $35. Ty paid a cash dividend of $2 per share on December 15.

In its Income Statement, what amount should Plack report as dividend income?

A

$24,000

Because Plack Co. owns only 2% of Ty Corp. stock, it does not have significant influence over Ty and will use fair value to account for its investment. Plank’s dividend will be determined by the number of shares of Ty that it owns multiplied by the amount of dividend per share. The calculation is:

2/14 Purchase 10,000 shares
4/30 Stock dividend 2,000 shares
12/15 Total shares owned 12,000
Dividend rate $ 2
Total Dividend income $24,000

The stock dividend would be recorded by Plack as a memorandum entry to adjust the per-share original cost, so it is based on the total 12,000 shares now owned.

513
Q

Band Co. uses the equity method to account for its investment in Guard, Inc. common stock. How should Band record a 2% stock dividend received from Guard?

A

As a memorandum entry, reducing the unit cost of all Guard stock owned

Under any method used to account for an investment in common stock, the investor records a stock dividend received by a memorandum entry to increase the number of shares owned. Since the cost of the investment does not change, the per share cost of the stock decreases.

514
Q

Stock dividends on common stock should be recorded at their fair value by the investor when the related investment is accounted for under which of the following methods?

A

Cost - No
Equity - No

Stock dividends are not recognized in the accounts at receipt, at fair value or any other value. Rather, they reduce the cost per share under both methods. The original cost is spread over more shares.

The investor’s percentage of the firm has not changed as a result of the stock dividend, but the investor has more shares (as do all investors). When the shares received as a dividend are sold, the reduction in cost basis increases the gain or reduces the loss.

515
Q

Clarion had the following investments in its portfolio that were purchased during year 2.

Investment Classification Cost Fair Value
12/31/Y2
Common
stock of Company X Fair value $100,000 $121,000
Bond of Company Y AFS $ 96,000 $101,000
Bond of Company Z HeldTM $ 64,000 $ 63,000

On December 31, Year 2, the amortized cost of Bond Y was $97,000, and the amortized cost of Bond Z was $63,500. Clarion does not elect the fair value option for reporting financial assets. What amount should Clarion record as an unrealized gain in its Year 2 income statement?

A

$21,000

If Clarion does not elect the fair value option for valuing its financial assets, the rules of ASC Topic 320 apply. Both the Company X stock investment and the Company Y bond available-for-sale security investment would be reported at fair value. However, only the $21,000 unrealized gain associated with the Company X stock trading security investment would be reported in earnings of the period. The unrealized gain of $5,000 on the Company Y bond available-for-sale security investment would be reported in other comprehensive income (OCI). The Company Z bond held-to-maturity security investment would be reported at amortized cost.

516
Q

Cook Company had the following debt investment portfolio of stocks that were purchased during Year 2.

Bonds Classification Cost Fair Value 12-31-Y2
Company R Available-for-sale $30,000 $32,000
Company S Trading $42,000 $46,000
Company T Available-for-sale $15,000 $18,000

Cook elects to use the fair value option for reporting all of its financial assets. What is the unrealized gain recognized on the income statement in Year 2?

A

$9,000

Cook elects to use the fair value option. Cook will value both its trading securities and available-for-sale securities at fair value and record the unrealized gains in earnings for the period. The gain is equal to $96,000 ($32,000 + $46,000 + $18,000) minus $87,000 ($30,000 + $42,000 + $15,000), or $9,000.

517
Q

On December 31, Year 1, Ott Co. had investments in marketable debt securities as follows:

	                 Cost	Market value
Mann Co.	$10,000	$ 8,000
Kemo, Inc.	9,000	10,000
Fenn Corp.	11,000	9,000
                      $30,000	$27,000

The Mann investment is classified as held-to-maturity, while the remaining securities are classified as available-for-sale. Ott does not elect the fair value option for reporting financial assets. Ott’s December 31, Year 1, balance sheet should report total marketable debt securities as

A

$29,000

ASC Topic 320 requires that held-to-maturity securities be carried at amortized cost and that available-for-sale and trading securities be carried at fair value (FV). Therefore, Ott’s investment portfolio is reported at 12/31/Y1 at the following amounts:

Bond	         Amount reported
Mann Co.	$10,000	 cost
Kemo, Inc.	10,000	 FV
Fenn Corp.	9,000	 FV
 	                $29,000
518
Q

Which, if any, of the following transfers between classifications of debt investments are possible?

Held-to-maturity to held-for-trading
Held-for-trading to held-to-maturity

A

Held-to-maturity to held-for-trading - YES
Held-for-trading to held-to-maturity - YES

Both transfers from held-to-maturity to held-for-trading classifications and from held-for-trading to held-to-maturity classifications can occur in the accounting for debt investments.

519
Q

The method of accounting for debt investments is based on the investor’s intent for holding the investment. When investor intent changes, the classification of and accounting for the debt investment changes. When debt investments are transferred between classifications, which one of the following valuation basis is most likely to be used when recording the investment in the new classification?

A

Fair value

Fair value is the valuation basis used when debt investments are transferred between classifications. Conceptually, the existing carrying value is written off and the current fair value is written on in the new classification, with any difference being an unrealized gain or loss.

520
Q

A debt security is transferred from the held-for-trading portfolio to the available-for-sale portfolio. At the transfer date, the security’s cost exceeds its fair value.

What amount is used at the transfer date to record the security in the available-for-sale portfolio?

A

Fair value, regardless of whether the decline in fair value below cost is considered permanent or temporary

Reclassifications between the two investment categories are always recorded at fair value. The reclassification is treated as if the security in the old classification was sold and the security in the new classification was purchased.

Fair value reflects a brand-new valuation and is treated as original cost from then on for the purpose of the annual year-end revaluation adjustment.

521
Q

Sun Corp. had investments in marketable debt securities costing $650,000 that were classified as available-for-sale. On June 30, Year 2, Sun decided to hold the investments to maturity and accordingly reclassified them to the held-to-maturity category on that date. The investments’ fair value was $575,000 at December 31, Year 1, $530,000 at June 30, Year 2, and $490,000 at December 31, Year 2. Sun does not elect the fair value option to account for these investments. What amount of loss from investments should Sun report in its Year 2 income statement?

A

$0

The requirement is to determine the amount of loss on investments to be reported in Sun’s Year 2 income statement. This transfer is accounted for at fair value, and any holding gains or losses on securities that are transferred to held-to-maturity from available-for-sale are reported as accumulated other comprehensive income. This amount is then amortized over the remaining life of the security as an adjustment to yield. Since cost is greater than fair value by $75,000 at 12/31/Y1 ($650,000 cost − $575,000 fair value), the following entry would be recorded:

Unrealized loss 75,000
Marketable debt securities 75,000
Then on June 30, Year 2, when Sun decides to hold the investments to maturity, an additional $45,000 will be recorded in the valuation account ($575,000 value on books − $530,000 FV) to reflect the change in FV. The following entry would be recorded:

Unrealized loss 45,000
Marketable debt securities 45,000

Each year the unrealized loss would be reported as other comprehensive income that would be closed to accumulated other comprehensive income.

522
Q

Jill Corp. had investments in marketable debt securities purchased on January 1, Year 1, for $650,000 that were classified as trading securities. On June 30, Year 2, Jill decided to hold the investments to maturity and accordingly reclassified them to the held-to-maturity category on that date. The investments’ fair value was $575,000 at December 31, Year 1, $530,000 at June 30, Year 2, and $490,000 at December 31, Year 2. Jill elects the fair value option for reporting these held-to-maturity securities. What amount of loss from investments should Jill report in its Year 2 income statement?

A

$ 85,000

An election can be made to use the fair value option when financial assets cease to qualify for fair value treatment due to specialized accounting rules. On June 30, Year 2, the trading securities were reclassified to the held-to-maturity category, and an election was made to report them at fair value. On June 30, Year 2, the held-to-maturity securities were valued at $530,000, and Jill would recognize a loss of $45,000 ($575,000 − $530,000). At December 31, Year 2, the securities declined in value an additional $40,000. Therefore, the total loss recognized in Year 2 was $85,000 ($45,000 + $40,000).

523
Q

On January 2, Year 1, Well Co. purchased 10% of Rea, Inc.’s outstanding common shares for $400,000. Well is the largest single shareholder in Rea, and Well’s officers are a majority on Rea’s board of directors. Rea reported net income of $500,000 for Year 1 and paid dividends of $150,000. Well does not elect the fair value option to report its investment in Rea. In its December 31, Year 1, balance sheet, what amount should Well report as investment in Rea?

A

$435,000

Ownership of less than 20% leads to the presumption of no substantial influence unless evidence to the contrary exists. Well’s position as Rea’s largest single shareholder and the presence of Well’s officers as a majority of Rea’s board of directors constitute evidence that Well does have significant influence despite less than 20% ownership. Therefore, the equity method is used. The investment account had a beginning balance of $400,000 (purchase price). This amount is increased by Well’s equity in Rea’s earnings (10% ownership × $500,000 income = $50,000) and decreased by Well’s dividends received from Rea (10% share × $150,000 total div. = $15,000), resulting in a balance of $435,000.

524
Q

What major transactions or events would cause
the carrying amount of an investment to
change when the cost method is used to
account for the investment?

A

The carrying amount of the investment would
change when:
Fair value becomes readily determinable
or there is an observable transaction for a
similar instrument.

The investor buys additional shares of
the subsidiary or sells some of the shares
it already owns.

525
Q

How do we account for the transfer of an

investment from held-to-maturity to held-fortrading?

A

Credit held to maturity at unamortized
cost.

Debit trading at fair value.

Recognize unrealized holding gain/loss in
net income.

526
Q

How do we account for the transfer of an

investment from held-to-maturity to availablefor-sale?

A

Credit held-to-maturity at unamortized
cost.

Debit available-for-sale at fair value.

Recognize unrealized (holding) gain or
loss to other comprehensive income.
527
Q

How do we account for the transfer of an
investment from held-for-trading to held-to-maturity
or available-for-sale?

A

Credit trading at recorded fair value.

Debit held-to-maturity or available-forsale
at current fair value.

Recognize unrealized holding gain/loss in
net income

528
Q

How do we account for the transfer of an

investment from available-for-sale to held-tomaturity?

A

Credit available-for-sale at recorded fair
value.

Debit held-to-maturity at current fair
value.

Keep unrealized holding gain/loss in
accumulated other comprehensive
income in shareholders’ equity.

Unrealized holding gain/loss at date of
transfer is amortized over remaining life
of debt.

529
Q

What are the categories of investments under
International Financial Reporting Standards
(IFRS) No. 9?

A

Under IFRS No. 9, the two categories of
investments (and other financial assets) are:

  1. Debt investments measured at amortized
    cost
  2. All other investments, including debt
    instruments not at amortized cost and all
    equity investments
530
Q

Under what conditions does International
Financial Reporting Standards (IFRS) No. 9
permit an investor to elect to measure a debt
investment at fair value that would otherwise
be measured at amortized cost?

A

An investor can elect to measure a debt
investment that would otherwise be measured
at amortized cost at fair value when the use of
fair value would eliminate or significantly
reduce a measurement or recognition
inconsistency that results from an accounting
mismatch. An accounting mismatch occurs
when assets or liabilities, or recognizing gains
or losses on them, are measured on different
bases

531
Q

Under what conditions does International
Financial Reporting Standards (IFRS) No. 9
permit an investor to elect to report gains or
losses from changes in fair value of equity
investments in other comprehensive income,
rather than through profit and loss (net
income)?

A

If the investor does not hold an equity
investment for trading purposes, the investor
may elect to report changes in fair value
through other comprehensive income rather
than through profit and loss (net income). The
election must be made when the investment is
first recognized, and it cannot be changed
subsequently

532
Q

What conditions must be met under
International Financial Reporting Standards
(IFRS) No. 9 for an investment in debt to be
classified as debt instruments measured at
amortized cost?

A

Two conditions must be met:
1. Business model test—where the entity
intends to hold the investment to collect
the contractual cash flows, not to sell the
instrument prior to its contractual
maturity to realize changes in fair value
2. Cash flow characteristic test—where the
contractual terms of the investment give
rise to cash flows on specific dates that
are solely payments of principal and
interest

533
Q

Which, if either, of the following statements concerning the transfer of investments between categories under IFRS No. 9 is/are correct?

I. Only investments in debt securities may be transferred between categories.

II. When investments are transferred between categories, financial statements of prior periods presented for comparative purposes must not be restated.

A

I only.

Statement I is correct; Statement II is not correct. Only investments in debt securities may be transferred between categories; equity securities may not be transferred between categories (Statement I). When investments are transferred between categories, financial statements of prior periods presented for comparative purposes must be restated (Statement II).

534
Q

Inco, Inc., a U.S. entity, has elected to prepare financial statements in accordance with IFRS to provide to its foreign suppliers. Inco has the following information concerning an investment in the bonds of Tryco, Inc., as of December 31

Par value $100,000
Original cost 108,000
Current premium 3,500
Fair value 105,000

Inco’s business model is to regularly invest in debt to receive the cash flow provided by interest and the repayment of principal on maturity. The bonds are not associated with any other asset or liability. Which one of the following is the amount at which Inco should report its investment in Tryco in its December 31 IFRS-based Statement of Financial Position?

A

$103,500

Under IFRS No. 9, investments in debt securities made under an entity’s business model plan to make and hold such investments solely to receive cash from interest and principal repayment, and when there is no accounting mismatch, should be reported at amortized cost. Amortized cost is par value ($100,000) plus the unamortized premium ($3,500), or $100,000 + $3,500 = $103,500, the correct answer.

535
Q

Which, if any, of the following transfers between categories is possible under IFRS No. 9 for investments in debt securities?

Amortized cost to fair value
Fair value to amortized cost

A

Amortized cost to fair value - YES
Fair value to amortized cost - YES

Under IFRS No. 9, investments in debt securities may be (1) transferred from amortized cost (when the investment originally meets both the business model test and the cash flow characteristic test) to fair value when the investment fails to continue to meet both the business model test and the cash flow characteristic test and (2) transferred from fair value to amortized cost when an investment that originally fails to meet both the business model test and the cash flow characteristic test subsequently meets both tests.

536
Q

Which of the following are possible ways that gains or losses on changes in the fair value of investments in equity securities may be reported under IFRS requirements?

In profit/loss (Income Statement)
In other comprehensive income

A

In profit/loss (Income Statement) - YES
In other comprehensive income - YES

Under IFRS, changes in fair value may be reported in profit/loss or in other comprehensive income, depending on whether or not the investment is held for trading purposes or not. If an investment in equity securities is held-for-trading purposes (i.e., to make a profit on price appreciation), changes in fair value will be reported through profit/loss. If an investment in equity securities is not held-for-trading purposes, the investor may elect to report changes in fair value through other comprehensive income.

537
Q

Inco, Inc., a U.S. entity, has elected to prepare financial statements in accordance with IFRS to provide to its foreign suppliers. Inco has the following information concerning an investment in the bonds of Tryco, Inc., as of December 31

Par value $100,000
Original cost 108,000
Current premium 3,500
Fair value 105,000

Inco normally does not invest in debt but made this investment with the expectation that it could profit from short-term decreases in the market interest rate. Which one of the following is the amount at which Inco should report its investment in Tryco in its December 31 IFRS-based Statement of Financial Position?

A

$105,000

Under IFRS No. 9, investments in debt securities that are not made under an entity’s business model plan to make and hold such investments solely to receive cash flow from interest and principal repayment should be reported at fair value. Thus, this investment should be reported at the fair value, $105,000.

538
Q

Which, if any, of the following characteristics concerning the categories of investments under IFRS No. 9 is/are correct?

I. There is a single category for debt investments and a single category for equity investments.

II. The business model test used in evaluating debt instruments for classification purposes is concerned with the investor’s intent.

A

II only

Statement II is correct; Statement I is not correct. The business model test used in evaluating debt instruments for classification purposes is concerned with the investor’s intent. Specifically, did the investor make the investment to collect cash flows from interest and return of principal, rather than to make a profit on sale of the investment (Statement II)? While there is a single category for equity investments (at fair value), there are two categories for debt investments (at amortized cost and at fair value) (Statement I).

539
Q

List the classifications of intangible assets.

A

Definite-life intangibles

Indefinite-life intangibles

540
Q

How is amortization of definite life intangibles

recorded?

A

Debit amortization expense and credit the
intangible asset (There is no accumulated
amortization contra account as with tangible
assets.)

541
Q

When can impairment of an intangible be

recovered?

A

Impairment of an indefinite- or definite-life

intangible cannot be recovered.

542
Q

Under what conditions is the residual value of a

definite-life intangible not assumed to be zero?

A

When (1) the entity has a commitment from a
third party to purchase the intangible asset at
the end of its useful life; or (2) the residual
value can be determined by reference to an
exchange transaction in an existing market for
that asset and that market is expected to exist
at the end of the asset’s useful life.

543
Q

What are intangible assets?

A

Long-term operational assets that lack physical
substance or presence but are currently used in
the operation of a business and have a useful
life extending more than one year from the
balance sheet date

544
Q

What costs can be capitalized to an intangible

asset?

A

Only external costs can be capitalized.
Examples are registration of patent, successful
legal defense of the patient, legal and
accounting fees, and design costs.

545
Q

What is the impairment test for definite-life

intangibles?

A
  1. The book value (BV) of the definite-life
    intangible is compared to the
    recoverable cost (RC) of the intangible
    asset. If BV > then RC, then
  2. BV is compared to fair value (FV). If BV >
    FV, then impairment loss = BV − FV.
546
Q

What is the impairment test for indefinite-life

intangibles?

A

One step: Book value (BV) compared to fair
value (FV). If BV > FV, then impairment loss = BV
− FV.

547
Q

What method is used to amortize intangible

assets?

A

Straight line, unless another systematic

method can be shown as more appropriate

548
Q

Which of the following types of assets would typically be reported on a company’s balance sheet as an intangible asset?

A

Cost of patent registrations

Once a project reaches technological feasibility, then the research and development cost can be capitalized. The costs of registering a patent can be capitalized as an intangible asset because if the entity is registering the patent, most likely the project has reached technological feasibility.

549
Q

Which of the following is a pair of values that are compared to determine the amount of a possible impairment loss on an intangible asset, with an indefinite life, other than goodwill?

A

Fair value, carrying value.

A possible impairment of an indefinite life intangible other than goodwill is determined by comparing the carrying value and the fair value of the asset.

550
Q

West Co. paid $50,000 for an intangible asset other than goodwill. Fair value of the asset is $55,000. West signed a contract to sell the asset for $10,000 in 10 years. What amount of amortization expense should West record each year?

A

$4,000

Unless there is evidence otherwise, amortization is on a straight-line basis. This intangible has a 10-year life and has a residual value. The annual amortization is 4,000 per year (50,000 – 10,000 = 40,000 / 10 years).

551
Q

Wind Co. incurred organization costs of $6,000 at the beginning of its first year of operations. How should Wind treat the organization costs in its financial statements in accordance with GAAP?

A

Expensed immediately.

In the past, firms capitalized and amortized organization costs. However, now, organization costs are expensed immediately. Such costs are internally generated. Typically, only costs paid to outside entities are capitalized to intangible assets, and only those intangibles with definite lives are amortized.

552
Q

Johan Co. has an intangible asset, which it estimates will have a useful life of 10 years, while Abco Co. has goodwill, which has an indefinite life. Which company should report amortization in its financial statements?

Johan
Abco

A

Johan - YES
Abco - NO

The intangible asset has a definite life and is amortized. Goodwill has an indefinite life, and is not amortized but is tested for impairment.

553
Q

Which of the following statements is correct concerning start-up costs?

A

Costs of start-up activities, including organization costs, should be expensed as incurred.

Start-up costs are expensed as incurred.

554
Q

After an impairment loss is recognized, the adjusted carrying amount of the intangible asset shall be its new accounting basis. Which of the following statements about subsequent reversal of a previously recognized impairment loss is correct?

A

It is prohibited.

All intangibles are subject to impairment, but the resulting impairment losses cannot be reversed. Although impairment losses on plant assets held for disposal can be reversed to the extent of previous losses, this is not the case for intangibles.

555
Q

Northstar Co. acquired a registered trademark for $600,000. The trademark has a remaining legal life of five years, but can be renewed every 10 years for a nominal fee. Northstar expects to renew the trademark indefinitely. What amount of amortization expense should Northstar record for the trademark in the current year?

A

$0

When the intangible asset can be renewed indefinitely, and the company has the positive ability and intent to continuously renew, then the intangible asset is an indefinite life intangible. Indefinite life intangibles are not amortized, but are tested for impairment on an annual basis.

556
Q

An entity purchases a trademark and incurs the following costs in connection with the trademark:

One-time trademark purchase price $100,000
Nonrefundable VAT taxes 5,000
Training sales personnel on the use of the new trademark 7,000
Research expenditures associated with the purchase of the new trademark 24,000
Legal costs incurred to register the trademark 10,500
Salaries of the administrative personnel 12,000

Applying IFRS and assuming that the trademark meets all the applicable initial asset-recognition criteria, the entity should recognize an asset in the amount of:

A

$115,500

The capitalizable costs for an intangible asset under IFRS 38 are essentially the same as U.S. GAAP. The cost of the asset is the cash paid to acquire the asset, including the cost to obtain legal title and control of the asset. This cost will include the purchase price ($100,000), the taxes ($5,000), and the legal costs to register the asset ($10,500).

557
Q

Grayson Co. incurred significant costs in defending its patent rights. Which of the following is the appropriate treatment of the related litigation costs?

A

Litigation costs would be capitalized if the patent right is successfully defended.

Litigation costs can be capitalized only if the defense of the patent was successful.

558
Q

On January 2, 2005, Ames Corp. signed an eight-year lease for office space. Ames has the option to renew the lease for an additional four-year period on or before January 2, 2012. During January 2005, Ames incurred the following costs:

$120,000 for general improvements to the leased premises with an estimated useful life of 10 years.

$50,000 for office furniture and equipment with an estimated useful life of 10 years.

At December 31, 2005, Ames’ intentions as to the exercise of the renewal option are uncertain. A full year’s amortization of leasehold improvements is taken for calendar year two. In Ames’ December 31, 2005 Balance Sheet, accumulated amortization should be:

A

$15,000

The appropriate amortization period for the leasehold improvements is eight years because renewal is uncertain. $120,000/8 = $15,000. This is the amount in accumulated amortization because the property has been leased only one year. The office furniture and equipment are not included in leasehold improvements because they belong to the lessee.

559
Q

On January 2, 20X4, Beal, Inc. acquired a $70,000 whole-life insurance policy on its president. The annual premium is $2,000. The company is the owner and beneficiary.

Beal charged officer’s life insurance expense as follows:

20X4	$2,000
20X5	1,800
20X6	1,500
20X7	1,100
Total	$6,400
=====
In Beal's December 31, 20X7 Balance Sheet, the investment in cash surrender value should be:
A

$1,600

The $1,600 ending cash surrender value is the difference between the total premiums paid ($8,000 = 4 × $2,000) and the total amount charged to insurance expense ($6,400). An increasing portion of the premiums on life insurance are allocated to the investment feature of life insurance each year.

560
Q

On January 1, 20X0, Nobb Corp. signed a 12-year lease for warehouse space. Nobb has an option to renew the lease for an additional 8-year period on or before January 1, 20X4.

During January 20X2, Nobb made substantial improvements to the warehouse. The cost of these improvements was $540,000, with an estimated useful life of 15 years.

At December 31, 20X2, Nobb intended to exercise the renewal option. Nobb has taken a full year’s amortization on this leasehold.

In Nobb’s December 31, 20X2 Balance Sheet, the carrying amount of this leasehold improvement should be:

A

$504,000

The remaining lease term at the end of 20X2 is nine years (the 12-year lease term began January 1, 20X0). The eight-year option is added to the term at that point to yield a revised lease term of 17 years (9 + 8). Leasehold improvements are amortized over the shorter of lease term (17 years) or useful life (15 years) because leasehold improvements revert to the lessor.

Thus, the amortization of the leasehold improvements is $36,000 ($540,000/15). At the end of 20X2, the carrying value of the leasehold improvement is $504,000 ($540,000-$36,000). A full year of amortization is warranted in 20X2 because the improvements were completed in January.

561
Q

Hull Co. bought a trademark from Roe Corp. on January 1, 20X5, for $224,000.

Hull retained an independent consultant who estimated the trademark’s remaining useful life to be 20 years. The trademark most likely will not be renewed. Its unamortized cost on Roe’s accounting records was $112,000.

In Hull’s December 31, 20X5 Balance Sheet, what amount should be reported as accumulated amortization?

A

$11,200

Twenty years is the estimated useful life. After one year, the accumulated amortization is $11,200 = $224,000/20. The purchase cost is capitalized. The book value of the previous owner is not relevant to Hull.

562
Q

On January 1, 20X4, Bay Co. acquired a land lease for a 21-year period with no option to renew.

The lease required Bay to construct a building in lieu of rent. The building, completed on January 1, 20X5, at a cost of $840,000, will be depreciated using the straight-line method. At the end of the lease, the building’s estimated market value will be $420,000.

What is the building’s carrying amount in Bay’s December 31, 20X5 Balance Sheet?

A

$798,000

The building is a leasehold improvement because it reverts to the lessor at the end of the lease. The residual value belongs to the lessor and is not relevant to the lessee. The building was completed at the beginning of the second year of the lease. Therefore, the total cost to the lessee of $840,000 is amortized over 20 years, not 21.

The carrying value of the leasehold improvement at the end of 20X5, the first year of the building’s life but the second year of the lease, is $798,000 = $840,000(19/20).

563
Q

In 20X0, Chain, Inc. purchased a $1,000,000 life insurance policy on its president, of which Chain is the beneficiary. Information regarding the policy for the year ended December 31, 20X5, follows:

Cash surrender value, 1/1/X5 $ 87,000
Cash surrender value, 12/31/X5 108,000
Annual advance premium paid 1/1/X5 40,000

During 20X5, dividends of $6,000 were applied to increase the cash surrender value of the policy. What amount should Chain report as life insurance expense for 20X5?

A

$19,000

In computing life insurance expense, the increase in cash surrender value is subtracted from the annual premium because the net cost to the firm is the premium less the increase in that investment. The investment is property of the insured firm.

The cash surrender value (an investment account) increased $21,000 during 20X5 ($108,000-$87,000). This increase is treated as a direct reduction in the current year’s insurance premium. Therefore, insurance expense is $19,000 ($40,000-$21,000) for 20X5.

What makes this question difficult is realizing that the dividends are not treated as a separate revenue; rather, they are treated as an offset against insurance expense. The reason is that the dividends are directly related to the policy. The investment aspect of whole life insurance is an integral part of the life insurance policy.

564
Q

In 20X4, Gar Corp. collected $300,000 as beneficiary of a keyman life insurance policy carried on the life of Gar’s controller, who had died in 20X4. The life insurance proceeds are not subject to income tax. At the date of the controller’s death, the policy’s cash surrender value was $90,000.

What amount should Gar report as revenue in its 20X4 Income Statement?

A

$210,000

The revenue recognized in 20X4 is $210,000, the difference between the proceeds of $300,000 and the surrender value. The build-up in surrender value has been recognized in previous years’ earnings as a reduction in insurance expense. The increase in surrender value in any year is treated as a deduction from the premium in computing insurance expense for the year. Thus, of the $300,000, $210,000 has not been recognized in earnings before 20X4.

565
Q

A company has experienced operating losses from its appliances division for the past five years. The division is the lowest level of identifiable cash flows. Having determined the division is the lowest level of identifiable cash flows, the company’s next step in performing its impairment test is to

A

Perform a recoverability test on the carrying amount of the division’s assets.

Once the lowest level of identifiable cash flows is identified for a division or unit, the next step would be to test the recoverability of the carrying amount of the division’s assets. This is done by comparing the fair value of the unit to the carrying amount to determine if further steps are necessary.

566
Q

For a public business entity, the goodwill impairment test is required to be performed

A

Any time during the fiscal year, provided that it is performed at the same time every year.

For public business entities, goodwill is not amortized; rather, it is tested for impairment at least annually or when factors indicate a potential impairment. There is no requirement as to the timing of the impairment testing other than that the testing be completed at the same time every year.

567
Q

Which of the following is not one of the qualitative factors considered to determine if it is more likely than not that the reporting unit is less than its carrying value?

A

Decline in the implied goodwill by using a discounted cash flow model.

This is a quantitative measure of the implied goodwill. The question asked which of the responses is not a qualitative factor used in the pre-step for goodwill impairment. This response is incorrect also because implied goodwill is determined by comparing the fair value of the reporting unit to the fair value of the identifiable assets - not by using a discount model.

568
Q

A company reported $6 million of goodwill in last year’s statement of financial position. How should the company account for the reported goodwill in the current year?

A

Perform a qualitative assessment to determine if it is more likely than not that the fair value of the reporting unit is less than its carrying value.

Goodwill impairment testing permits a qualitative “pre-step” test to determine if it is more likely than not that the goodwill is impaired. This pre-step can result in considerable savings to companies who do not have to complete the quantitative tests associated with testing and measuring goodwill impairment.

569
Q

Firm A is negotiating with Firm B to purchase Firm B and bring it into the corporate organization headed by Firm A. The two firms agree to the following:

Firm B’s average annual income is $900, to continue for 10 years after purchase.
Firm B’s total owner’s equity is $2,000.
Firm B’s market value of net identifiable assets is $3,500.

The average rate of return in B’s industry is 10%.
The risk adjusted rate of return for the purchase is 8%.
Compute the purchase price for B implied by this information. The present value of an annuity of $1 for 10 years at 8% is 6.71008, and at 10% is 6.14457.

A

$7,191

With the information provided, goodwill can be computed directly as the present value of excess earnings. Goodwill is then added to the market value of net identifiable assets to yield the purchase price. Excess earnings is the difference between B’s earnings and the expected earnings in B’s industry for a firm the size of B, based on market value of net assets. Given B’s size, B is expected to earn $350 per year (.10 × $3,500). Excess earnings = $900 − $350 = $550. Goodwill = present value of excess earnings = $550(6.71008) = $3,691. The implied purchase price = $3,500 + $3,691 = $7,191.

570
Q

Large purchased all of Small’s voting stock for $11 million when Small’s total owners’ equity was $4 million. The book value and market value of Small’s liabilities equal $3 million. However, the market value of Small’s total assets equals $9 million. What amount of goodwill is recorded by Large (in millions)?

A

$5

The market value of Small’s net assets is $6 ($9 − $3). Goodwill ($5) is the difference between the purchase price of $11 and the market value of Small’s net assets of $6. Goodwill is the portion of the purchase price not attributable to identifiable assets.

571
Q

Firm A purchased Firm B for $4,000 when B’s total owners’ equity was $2,000. Firm A completed the qualitative test for goodwill impairment and determined that it is more likely than not that goodwill may be impaired. B had one asset worth $500 more than the book value. One year after the purchase, Firm B’s total market value had dropped to $3,200 and the market value of its net identifiable assets was $2,000. What amount of goodwill impairment loss is recorded?

A

$300

In general, goodwill is impaired when factors and circumstances indicate that the fair value is less than the current recorded value. This generally occurs when the market value of the unit previously purchased has declined because of economic factors and events. In this question, the market value of B has declined significantly in one year. Goodwill is re-estimated the same way that it was when B was purchased, as the difference between unit market value and the market value of net identifiable assets. The new value for estimated goodwill is $1,200 (= $3,200 − $2,000). The goodwill originally recorded is $1,500 (= $4,000 − $2,000 − $500). The impairment loss is the decline in goodwill, or $300 (= $1,500 − $1,200).

572
Q

If both an asset group and goodwill in one of a company’s reporting units have to be tested for impairment, which of the following statements is correct regarding impairment testing and impairment losses?

A

The other asset group should be tested for an impairment loss before goodwill is tested.

When conducting goodwill impairment testing in a reporting unit, one must first determine the fair value of the identifiable net assets (assets minus liabilities). The fair value of the identifiable net assets is used to calculate implied goodwill to test recorded goodwill for impairment. The measurement of the fair value of the identifiable net assets is essentially measuring (and recognizing) any impairment in that asset group before the impairment testing for goodwill is completed.

573
Q

Which of the following is an intangible asset that is subject to the recoverability test when testing for impairment?

A

A patent.

The recoverability test is applied to definite-life intangible assets. The patent is the only definite-life intangible asset listed. The first step of the impairment test of a definite-life intangible is to compare the asset’s book value (BV) to the recoverable cost. Recoverable cost is the sum of the net cash flows attributable to using the asset and from the ultimate disposal. If the BV is greater than the recoverable costs, then the asset is potentially impaired. The second step is to compare the BV to the fair value (FV). If the BV is greater than the FV, the asset is written down to FV. The impairment loss equals BV – FV.
report a content error

574
Q

What is the International Financial Reporting
Standard (IFRS) treatment of development
costs?

A

Capitalize development costs.

575
Q

List the items that are included in research and

development (R&D).

A

Laboratory research

Conceptual formulation and design of
products or process alternatives

Modification of current design

Design, construction, and testing of
preproduction prototypes and models

Design of tools, jigs, molds, and dies
involving new technology

Design of a pilot plant

576
Q

List the items excluded from research and

development (R&D).

A

Engineering follow-through

Quality control and routine testing

Troubleshooting

Adaptation of existing capability to
customer’s needs

Routine design of tools, jigs, molds, and
dies

Legal work in connection with patent
applications

577
Q

What is the general rule regarding research and

development (R&D) costs?

A

Expense costs as incurred

578
Q

How are fixed assets used temporarily in a
research and development (R&D) project
accounted for?

A

Depreciation related to the time period is

included in R&D expense.

579
Q

What is the International Financial Reporting

Standard (IFRS) treatment of research costs?

A

Expense research costs.

580
Q

How are fixed assets used in a single research
and development (R&D) project with no
alternative use accounted for?

A

The entire cost is expensed as R&D

immediately.

581
Q

Define “research.”

A

The attempt to discover new knowledge aimed
at the development of new products, services,
processes, or techniques, or the significant
improvement in an existing product.

582
Q

How are assets used in several research and

development (R&D) projects accounted for?

A

They are capitalized and depreciated to R&D

expense.

583
Q

Which of the following should a company classify as a research and development expense?

A

Redesign of a product prerelease.

Redesign of a product prerelease would be considered R&D because it is discovering new technology, process, or function of the product.

Legal work on patent applications means that the research has reached technological feasibility and is no longer part of research and development (R&D) expense. At this point the R&D costs would be capitalized. Redesign of a product prerelease would be considered R&D. Routine design of tools, jigs, molds and dies are not discovery of new knowledge and not part of research and development (R&D). Redesign of a product prerelease would be considered R&D.
Periodic design changes are not discovery of new knowledge and not part of research and development (R&D). Redesign of a product prerelease would be considered R&D.

584
Q

Which of the following is a research and development cost?

A

Development or improvement of techniques and processes

R & D is the pursuit of new knowledge and the translation of that knowledge to new products and processes. Improvement in techniques and processes is also “new” in that sense and is included in R & D.

Natural resource activities are specifically excluded from the purview of R & D.
Research performed for other parties is normal business for the firm doing the research. The activities are for the benefit of another party and as such are R & D to the receiving party but not for the party doing the work.
Many firms do not have the expertise to perform research in all the areas they deem relevant to their business and employ other firms to do that work. The receiving firm records R & D expense; the firm performing the work ultimately records contract revenue and expense.
Market research is specifically excluded from the list of activities constituting R & D under ASC 730 .

585
Q

During the current year, Beta Motor Co. incurred the following costs related to a new solar-powered car:

Salaries of laboratory employees researching how to
build the new car $250,000
Legal fees for the patent application for the new car 20,000
Engineering follow-up during the early stages of commercial production (the follow-up occurred during the current year) 50,000
Marketing research to promote the new car 30,000
Design, testing, and construction of a prototype 400,000

What amount should Beta Motor report as research and development expense in its income statement for the current year?

A

$650,000

The salaries for lab employees and the design, testing, and construction of the prototype qualify for classification of R & D.

586
Q

Brand Co. incurred the following research and development project costs at the beginning of the current year:

Equipment purchased for current and future projects $100,000
Equipment purchased for current projects only 200,000
Research and development salaries for current project 400,000

Equipment has a five-year life and is depreciated using the straight-line method. What amount should Brand record as depreciation for research and development projects at December 31?

A

$ 20,000

Under U.S. GAAP, all research and development costs are expensed as incurred. Therefore, the cost of equipment for current projects and the salaries would be expensed in the current year. The equipment that can be used in future years would be depreciated over its useful life. Depreciation expense reclassified as research and development would be $100,000 / 5 years = $20,000.

587
Q

Under U.S. GAAP, how should NSB, Inc. report significant research and development costs incurred?

A

Expense all costs in the year incurred.

Under U.S. GAAP, all research and development costs are expensed as incurred.

588
Q

Which of the following is the proper treatment of the cost of equipment used in research and development activities that will have alternative future uses?

A

Capitalized and depreciated over its estimated useful life.

Equipment that is used in research and development activities and has alternative future uses should be depreciated over its estimated useful life. During the time that the equipment is used for R&D activities the depreciation will be recorded as R&D expense.

589
Q

During the current year ended December 31, Metal, Inc. incurred the following costs:

Laboratory research aimed at discovery of new knowledge $ 75,000
Design of tools, jigs, molds, and dies involving new technology 22,000
Quality control during commercial production, including routine testing 35,000
Equipment acquired two years ago, having an estimated useful life of five years with no salvage value, used in various R&D projects 150,000
Research and development services performed by Stone Co. for Metal, Inc. 23,000
Research and development services performed by Metal, Inc. for Clay Co. 32,000

What amount of research and development expenses should Metal report in its current-year income statement?

A

$150,000

Research costs are associated with the discovery of new knowledge or the development of new products, services, processes or techniques. Development costs are the translation of the research into a plan or design. The costs included in R&D are $75,000 + 22,000 and the depreciation associated with the equipment used in R&D ($150,000 / 5 years = $30,000) and the R&D services paid to a third party $23,000 = $150,000. The receipt of payment from a third party is not netted against the R&D costs.

590
Q

During 2005, Vest Co. incurred the following costs:

Testing in search for process alternatives $280,000
Routine design of tools, jigs, molds, and dies 250,000
Modification of the formulation of a process 410,000
Research and development services performed by Acme Corp. for Vest 325,000

In Vest’s 2005 income statement, research and development expense should be

A

$1,015,000.

R & D expense includes three of the four costs listed. Only routine design of tools, jigs, molds, and dies is excluded from R & D expense because it is not aimed at discovery of new knowledge or translation of knowledge into new products and processes or improvements in existing products and processes.

The other three costs meet this definition of R & D expense and sum to $1,015,000. R & D expense includes the cost of R & D services performed by other firms. The cost to Vest is conceptually the same whether Vest’s employees perform the research on facilities acquired by Vest or another firm provides the service. Either way, Vest is paying for R & D.

591
Q

During 2005, Kent Co. incurred $204,000 of research and development costs in its laboratory to develop a patent that was granted on July 1, 2005. Legal fees and other costs associated with registration of the patent totaled $41,000. The estimated economic life of the patent is 10 years.

What amount should Kent capitalize for the patent on July 1, 2005?

A

$41,000

Only the legal fees and other registration costs are capitalized to the patent account. These costs are paid to outside parties. The research and development costs are expensed under ASC 730. All research and development is expensed as incurred.

592
Q

Cody Corp. incurred the following costs during 2006:

Design of tools, jigs, molds, and dies involving new technology $125,000
Modification of the formulation of a process 160,000
Troubleshooting in connection with breakdowns during commercial production 100,000
Adaption of an existing capability to a particular customer’s need as part of a continuing commercial activity 110,000

In its 2006 income statement, Cody should report research and development expense of

A

$285,000.

Only the first two costs are research and development costs. R & D involves the search for new knowledge and translation of that knowledge toward new products and processes and improvements in existing products and processes.

The first two items are listed in ASC 730 as being included in R & D.

The last two are listed as specifically being excluded from R & D because they are routine activities or do not involve essentially new products and processes. The total R & D expense is therefore $285,000 ($125,000 + $160,000).

593
Q

In 2005, Ball Labs incurred the following costs:

Direct costs of doing contract research and development work for the government to be reimbursed by governmental unit $400,000
Research and development costs not included above were:
Depreciation $300,000
Salaries 700,000
Indirect costs appropriately allocated 200,000
Materials 180,000

What was Ball’s total research and development expense in 2005?

A

$1,380,000

The contract work performed for the government is not research and development expense to Ball but rather is normal contracting cost that will be applied against contract revenue in computing contract profit.

Research and development for Ball includes efforts aimed at discovering and translating new knowledge toward new products and services that Ball will produce and market. All four costs in the question are included in research and development.

The sum of $1,380,000 is the firm’s research and development expense for 2005 ($300,000 + $700,000 + $200,000 + $180,000).

594
Q

Heller Co. incurred the following costs in 2005:

Research and development services performed by Kay Corp. for Heller $150,000
Testing for evaluation of new products 125,000
Laboratory research aimed at discovery of new knowledge 185,000
What amount should Heller report as research and development costs in its income statement for the year ending December 31, 2005?

A

$460,000

Each of the costs is appropriately included in research and development. The sum of the three costs is $460,000 ($150,000 + $125,000 + $185,000).

Each activity represented contributes either to the discovery of new knowledge or to the translation of that new knowledge. The last two costs listed are found verbatim in the ASC 730 list of items that are included in research and development. The first cost represents purchased services. When those services are for research and development, then they are included in this category of expense.

595
Q

During 2005, Orr Co. incurred the following costs:

Research and development services performed by Key Corp. for Orr $150,000
Design, construction, and testing of preproduction prototypes and models 200,000
Testing in search for new products or process alternatives 175,000
In its 2005 income statement, what should Orr report as research and development expense?

A

$525,000

All three listed costs are included in research and development expense. Their sum is $525,000.

The first is the cost of payments to an outside party to perform R & D. This cost is as much R & D as internally incurred costs - their objective is the same.

The second and third costs meet the definition of R & D costs incurred internally. R & D includes costs of efforts to discover new knowledge and to translate that new knowledge into new products, processes, and services.

596
Q

A company’s research department incurred $1,000,000 in material, labor, and overhead costs to construct a prototype of a new product and $100,000 to test and modify the prototype. Which of the following statements correctly describes the accounting treatment of prototype costs incurred by the company?

A

Expense $1,100,000 as incurred.

All costs related to research and development are expensed as incurred. The costs associated with developing and testing a prototype are considered research and development.

597
Q

How is customer support and maintenance

accounted for?

A

Expense as incurred.

598
Q

Describe the straight-line method of

accounting for software.

A

Book value at beginning of year / Number of
years remaining in product sales life at
beginning of year

599
Q

What is meant by technological feasibility of a

software?

A

When the program model or working model of

the software is complete

600
Q

How are software production costs accounted

for?

A

Capitalize to inventory and expense as cost of

goods sold.

601
Q

How are software costs handled in
International Financial Reporting Standards
(IFRS)?

A

IFRS treats software costs the same as research

and development.

602
Q

How are costs incurred after technological

feasibility is reached?

A

Capitalize as computer software costs and

amortize.

603
Q

How are costs incurred before technological

feasibility is reached?

A

Expense as incurred.

604
Q

A collection agency spent $50,000 in staff payroll costs investigating the feasibility of developing its own software program for tracking customer contacts. After committing to funding the project, software developers were paid $200,000 to write the code, and the company incurred $70,000 in general and administrative costs related to training and software maintenance. What amount should be capitalized?

A

$200,000

The only costs that can be capitalized is the cost of software development; the $200,000 to write the code. The investigation of the feasibility and the administrative costs for training and maintenance must be expensed.

605
Q

On December 31, 2004, Byte Co. had capitalized software costs of $600,000 with an economic life of 4 years. Sales for 2005 were 10% of expected total sales of the software.

At December 31, 2005, the software had a net realizable value of $480,000.

In its December 31, 2005 balance sheet, what amount should Byte report as net capitalized cost of computer software?

A

$450,000

Costs incurred (after technological feasibility has been established) in developing software for sale, lease, or licensing are capitalized and subsequently amortized. Annual amortization is the greater of:

  1. The percentage of expected total revenues earned during the period multiplied by total capitalized amount, OR
  2. Straight-line amortization based on expected life.

For reporting purposes, capitalized cost less accumulated amortization cannot exceed net realizable value. The calculation for Byte Co. for 2005 would be:

  1. Percentage of expected total revenues = .10 × $600,000=$ 60,000 amortization
  2. Straight-line = 1 year/ 4years = .25 × $600,000= $150,000 amortization

The greater is $150,000, the amount to amortize for 2005. For reporting purposes:

Capitalized cost = $600,000

Less: Amortization = 150,000

Net book value =$450,000 less than $480,000 net realizable value

Since amortized cost is less than net realizable value, no further write down is required and Byte would report net capitalized cost at $450,000.

606
Q

Standard Co. spent $10,000,000 on its new software package that is to be used only for internal use. The amount spent is for costs after the application development stage. The economic life of the product is expected to be three years. The equipment on which the package is to be used is being depreciated over five years.

What amount of expense should Standard report on its income statement for the first full year?

A

$3,333,333

The cost of developing software for internal purposes is expensed up to the “application development stage” at which point the effort appears to be leading to a useable application. After that point, costs are capitalized. With a three-year useful life and $10 million capitalized cost, the amortization expense is one-third, or $3.33 million.

The useful life of the product is used rather than the useful life of the equipment because new software can be developed after three years for use on that equipment.

607
Q

Yellow Co. spent $12,000,000 during the current year developing its new software package. Of this amount, $4,000,000 was spent before it was at the application development stage and the package was only to be used internally. The package was completed during the year and is expected to have a 4-year useful life.

Yellow has a policy of taking a full-year’s amortization in the first year. After the development stage, $50,000 was spent on training employees to use the program.

What amount should Yellow report as an expense for the current year?

A

$6,050,000

There are three expenses to be recognized:

(1) software development costs incurred before the application development stage was reached, $4,000,000;
(2) amortization of capitalized software development costs incurred after the application development stage was reached, $8,000,000/4 = $2,000,000;
(3) $50,000 training costs.

The sum of these is $6,050,000.

Training costs are expensed as incurred. The application development stage is the point after which there is sufficient evidence of a product that software development costs are capitalized and amortized. Such costs will benefit future periods.

608
Q

Which of the following is an indication that a cloud computing arrangement includes a software license?

I. The customer has contractual right to take possession of the software at any time during the hosting period without significant penalty.

II. The cloud computing arrangement has an indefinite life because the contract is renewable indefinitely.

III. It is feasible for the customer to either run the software on its own hardware or contract with another party unrelated to the vendor to host the software.

A

I. and III.

Both option I. and III. are the criteria for determining if the cloud computing arrangement contain a software license.

Option II. is not a criterion for the cloud computing arrangement to contain a software license.

609
Q

During 2004, Pitt Corp. incurred costs to develop and produce a routine, low-risk computer software product as follows:

Completion of detail program design $13,000

Costs incurred for coding and testing to establish technological feasibility 10,000

Other coding costs after establishment of technological feasibility 24,000

Other testing costs after establishment of technological feasibility 20,000

Costs of producing product masters for training materials 15,000

Duplication of computer software and training materials from product masters (1,000 units) 25,000

Packaging product (500 units) 9,000

In Pitt’s December 31, 2004 balance sheet, what amount should be capitalized as software cost, subject to amortization?

A

$59,000

Only software development costs incurred after the point of technological feasibility is reached are capitalized as an intangible and amortized.

Technological feasibility is the point at which the firm makes the decision to continue the product development with the expectation that a workable product is possible. Costs of duplication and packaging are all product costs and, although capitalized, are debited to inventory rather than to software development costs.

The capitalized software costs for this firm are:

Other coding costs after establishment of technological feasibility $24,000

Other testing costs after establishment of technological feasibility 20,000

Costs of producing product masters for training materials 15,000

Total amount subject to amortization $59,000

610
Q

On January 1, Year 1, a company with a calendar year end began developing a software program that it intends to market and sell to its customers. The software coding was completed on March 31, Year 1, at a cost of $200,000, and the software testing was completed on June 30, Year 1, at a cost of $100,000. The company achieved technological feasibility on July 31, Year 1, at which time the company began producing product masters at a cost of $125,000. What amount should the company report for the total research and development expense for the year ended December 31, Year 1?

A

$300,000

Research and development (R&D) costs are expensed until the point the project reaches technological feasibility, at which point the costs can be capitalized. Therefore, all costs prior to July 31 are expensed as R&D. This software coding and testing ($200,000 and $100,000) should be expensed as R&D.

611
Q

Under International Financial Reporting
Standards (IFRS), at what value can you report
intangible assets?

A

Amortized cost or fair market value

612
Q

Provide examples of the class of assets you can
carry at fair market value under International
Financial Reporting Standards (IFRS).

A

Property, plant, and equipment

Identifiable intangible assets

Financial assets including investments
and financial instruments

613
Q

How much of the impairment loss can be
recovered under International Financial
Reporting Standards (IFRS)?

A

The recovery of an impairment loss is limited to
the carrying value had the impairment not
occurred.

614
Q

At what level is goodwill impairment testing
performed under U.S. generally accepted
accounting principles (GAAP) and International
Financial Reporting Standards (IFRS)?

A

Under U.S. GAAP, goodwill impairment is tested
at the reporting-unit level. Under IFRS, goodwill
impairment is tested at the cash-generating
unit

615
Q

What is a cash-generating unit?

A

The smallest identifiable group off assets that
generates cash inflows that are largely
independent of the cash inflows from other
assets or groups of assets

616
Q

Under IFRS, the test for asset impairment is to compare the carrying value of the intangible asset to its recoverable amount. Which of the following is the recoverable amount according to IFRS?

A

The greater of fair value less cost to sell or value in use.

The greater of fair value less cost to sell or value in use is the recoverable amount according to IFRS.

617
Q

After an impairment loss is recognized, the adjusted carrying amount of the intangible asset shall be its new accounting basis. Under IFRS, which of the following statements about subsequent reversal of a previously recognized impairment loss is correct?

A

It is allowed when events and circumstances change.

Under IFRS impairment losses associated with identifiable intangibles are recoverable. Impairment losses associated with goodwill are NOT recoverable.

618
Q

Under IFRS, which of the following is a criterion, other than goodwill, that must be met in order for an item to be recognized as an intangible asset?

A

The item is identifiable and lacks physical substance.

IAS 38 defines an intangible asset as a nonmonetary asset without physical substance that is identifiable. Identifiable means that the asset is 1) separable or capable of being separated or divided from the entity and can be sold or transferred and 2) arises from contractual or other legal rights, regardless of whether those rights are transferable or separable from the entity. This definition is essentially the same as under U.S. GAAP.

619
Q

Under IFRS, an entity that acquires an intangible asset may use the revaluation model for subsequent measurement only if:

A

An active market exists for the intangible asset.

An active market will provide a relevant and reliable reference to the assets value. Therefore, just like with PPE, revaluation to fair value is permitted. IAS 38 defines an active market as one that the items traded in the market are homogeneous, there are willing buyers and sellers, and prices are available to the public.

620
Q

Freight charges paid for goods held for resale.

Capitalize/Expense

A

Capitalize

Freight charges are directly connected with bringing the goods to the place of business of the buyer.

621
Q

In-transit insurance on goods held for resale purchased FOB shipping point.

Capitalize/Expense

A

Capitalize

Insurance charges are directly connected with bringing the goods to the place of business of the buyer.

622
Q

Interest on note payable for goods held for resale.

Capitalize/Expense

A

Expense

Only interest costs related to continued assets constructed for internal use or assets produced as discrete products for sale or lease should be capitalized. The informational benefit of capitalization does not justify the cost of the accounting for the interest as a product cost.

623
Q

Installation of equipment.

Capitalize/Expense

A

Capitalize

Direct cost of bringing the equipment to the buyer.

624
Q

Testing of newly purchased equipment.

Capitalize/Expense

A

Capitalize

Direct cost of converting the equipment to a usable condition.

625
Q

Cost of current year service contract on equipment.

Capitalize/Expense

A

Expense

Does not relate to bringing the equipment to the buyer or putting it into a usable condition.

626
Q

An intangible asset that is determined to have an indefinite life should be amortized over 40 years.

True/False

A

False

Intangible assets that have indefinite lives are not amortized.

627
Q

If the carrying value of the intangible asset exceeds its fair value, an impairment loss must be recognized.

True/False

A

True

The recorded values of intangibles are impaired if they are above fair value.

628
Q

The impairment test for goodwill must be performed at the end of each fiscal year for each reporting unit.

True/False

A

False

The impairment test for goodwill can be performed anytime during the year, but it must be performed at the same time every year.

629
Q

When a company borrows funds to finance a construction project and temporarily invests this cash, the interest expense should be offset against the interest expense to be capitalized.

True/False

A

False

Interest earned on temporary investments from borrowed funds used to finance construction projects should be recognized as interest revenue and not netted against interest expense during the period.

630
Q

A company may capitalize interest on inventories it regularly produces for resale to customers.

True/False

A

False

A company may not capitalize interest on funds borrowed to finance routinely produced inventory.

631
Q

Gains are never recognized when similar assets are exchanged.

A

False

Recognition of gain is required unless the transaction lacks commercial substance.

632
Q

In nonmonetary exchanges, losses are only recognized when cash is received in the transactions.

A

False

Losses on nonmonetary exchanges are recognized.

633
Q

If the fair value of the asset is unknown in a nonmonetary exchange and no gain can be computed, the asset is recorded at the fair market value of the asset given up.

A

False

In a nonmonetary exchange, if the fair value is unknown and no gain can be computed, the asset is recorded at the book value of the asset given up.

634
Q

A capital expenditure is charged against income over the useful life of the asset.

A

False

Capital expenditures are amortized over the useful life of the asset.

635
Q

Expenditures to improve the efficiency or extend the useful life of an asset should be capitalized.

A

False

Expenditures to improve capital assets should be capitalized.

636
Q

Bake Co.’s trial balance included the following at
December 31, Year 1:

Accounts payable $ 80,000
Bonds payable, due Year 2 300,000
Discount on bonds payable 15,000
Deferred income tax liability 25,000

The deferred income tax liability is not related to an asset for financial accounting purposes and is expected to reverse in Year 2. What amount should be included in the current liability section of Bake’s December 31, Year 1 balance sheet?

A

$365,000

Correct. All deferred tax assets and liabilities are reported as noncurrent. The computation is: 80,000 + 300,000 − 15,000 = $365,000.

637
Q

Acme Co.’s accounts payable balance at December 31 was $850,000 before necessary year-end adjustments, if any, related to the following information:

At December 31, Acme has a $50,000 debit balance in its accounts payable resulting from a payment to a supplier for goods to be manufactured to Acme’s specifications.

Goods shipped F.O.B. destination on December 20 were received and recorded by Acme on January 2. The invoice cost was $45,000.

In its December 31 balance sheet, what amount should Acme report as accounts payable?

A

$900,000

The $50,000 advance is not related to accounts payable, even though it was made to a supplier for which Acme would have accounts payable. It is a prepayment. Removing the $50,000 debit increases the accounts payable balance by that amount. The $45,000 shipment is not part of the inventory of Acme as of December 31 nor is it a liability (accounts payable) because title to the goods did not transfer to Acme until January 2. FOB destination means that title does not transfer until goods reach their destination. Acme treated this item correctly because it was recorded January 2. Therefore, the correct accounts payable balance is $900,000 ($850,000 before adjustment + $50,000).

638
Q

Hemple Co. maintains escrow accounts for various mortgage companies. Hemple collects the receipts and pays the bills on behalf of the customers. Hemple holds the escrow monies in interest-bearing accounts. They charge a 10% maintenance fee to the customers based on interest earned. Hemple reported the following account data:

Escrow liability beginning of year $ 500,000
Escrow receipts during the year 1,200,000
Real estate taxes paid during the year 1,450,000
Interest earned during the year 40,000

What amount represents the escrow liability balance on Hemple’s books?

A

$286,000

This question requires you to think about how liabilities are accrued. If Hemple is holding funds for a mortgage company, it is a liability for Hemple. The liability would be increased when escrow monies are deposited and decreased when there is payment made on behalf of customers. The funds are also earning interest and 10% of that interest is charged as a maintenance fee to the customer. A T-Account will help demonstrate this:

Beg Bal $ 0
\+ Receipts 1,200,000
-Taxes 1,450,000
Interest + 40,000
10% - $ 4,000
639
Q

On December 31, 20x5, special insurance costs, incurred but unpaid, were not recorded.

If these insurance costs were related to work-in-process, what is the effect of the omission on accrued liabilities and retained earnings in the December 31, 20x5 balance sheet?

Accrued liabilities
Retained earnings

A

Accrued liabilities - Understated
Retained earnings - No Effect

Accrued liabilities are understated because the insurance costs were incurred but not paid. The firm has an obligation for coverage received. The omitted journal entry is:

DR: Work in process

CR: Accrued payables

Retained earnings is unaffected because no expense has been incurred. The omission of the above entry has no effect on expenses or retained earnings.

Work in process is an asset. When work in process is completed and sold, this part of the total cost of work in process will be expensed.

640
Q

The balance in Kemp Corp.’s accounts payable account at December 31, 20X5 was $900,000 before any necessary year-end adjustment relating to the following:

Goods were in transit to Kemp from a vendor on December 31, 20X5. The invoice cost was $50,000. The goods were shipped F.O.B. shipping point on December 29, 20X5 and were received on January 4, 20X6.
Goods shipped F.O.B. destination on December 21, 20X5 from a vendor to Kemp were received on January 6, 20X6. The invoice cost was $25,000.
On December 27, 20X5, Kemp wrote and recorded checks to creditors totaling $40,000 that were mailed on January 10, 20X6.

In Kemp’s December 31, 20X5 balance sheet, the accounts payable should be

A

$990,000

The correct answer, $990,000, is the balance in the AP account at year-end, which equals: $900,000 (bal. before adjustment) + $50,000 (in transit, FOB shipping point) + $40,000 (checks not sent as of Dec. 31).

The $50,000 amount is included in AP at 12/31 because the title passed to Kemp at 12/29. The title to goods shipped FOB shipping point passes to the buyer when the goods reach the common carrier. Therefore, Kemp owned the goods and incurred a liability on 12/29.

The $25,000 amount is not included in AP at 12/31 because the title did not pass to Kemp until the goods reached the destination, which occurred after 12/31. The title of the goods shipped FOB destination passes to the buyer at the destination.

The $40,000 amount is included in AP at 12/31 because the checks were not sent as of 12/31. Kemp did not extinguish this amount of its debt as of 12/31 and should make an adjusting entry to increase both cash and AP at 12/31.

641
Q

Black Corp.’s accounts payable at December 31, 20X4, totaled $900,000 before any necessary year-end adjustments relating to the following transactions:

On December 27, 20X4, Black wrote and recorded checks to creditors totaling $400,000, causing an overdraft of $100,000 in Black’s bank account at December 31, 20X4. The checks were mailed out on January 10, 20X5.
On December 28, 20X4, Black purchased and received goods for $153,061, terms 2/10, n/30. Black records purchases and accounts payable at net amounts. The invoice was recorded and paid on January 3, 20X5.
Goods shipped F.O.B. destination on December 20, 20X4 from a vendor to Black were received January 2, 20X5. The invoice cost was $65,000.

At December 31, 20X4, what amount should Black report as total accounts payable?

A

$1,450,000

Preadjusted balance in accounts payable $900,000

Plus checks not sent to creditors until Jan. 10 (this amount was debited to accounts payable and must be reversed because the checks have not been sent - accounts payable has not been reduced as of December 31) 400,000

Plus goods received Dec. 28, at net: .98($153,061)(the firm records purchases at net of 2% discount) 150,000
Equals ending accounts payable $1,450,000

There is no liability at December 31, 20X4 for the goods shipped FOB destination because title does not pass until the goods reach the destination, which did not occur until January.

The firm must include the Dec. 28 receipt of goods in accounts payable because the firm has received the goods.

642
Q

Which of the following is generally associated with payables classified as accounts payable?

Periodic payment of interest
Secured by collateral

A

Periodic payment of interest - No
Secured by collateral - No

Accounts payable is also labeled: accounts payable, trade. The accounts payable account is used only for routine trade payables, typically for purchases of inventory and supplies.

Interest accrued is recorded in accrued interest payable, and secured debt is recorded in other specifically-labeled liability accounts.

643
Q

Kew Co.’s accounts payable balance at December 31, 20X2 was $2,200,000 before considering the following data:

Goods shipped to Kew F.O.B. shipping point on December 22, 20X2 were lost in transit. The invoice cost of $40,000 was not recorded by Kew. On January 7, 20X3, Kew filed a $40,000 claim against the common carrier.
On December 27, 20X2, a vendor authorized Kew to return, for full credit, goods shipped and billed at $70,000 on December 3, 20X2. The returned goods were shipped by Kew on December 28, 20X2. A $70,000 credit memo was received and recorded by Kew on January 5, 20X3.
Goods shipped to Kew F.O.B. destination on December 20, 20X2 were received on January 6, 20X3. The invoice cost was $50,000.

What amount should Kew report as accounts payable in its December 31, 20X2 balance sheet?

A

$2,170,000

Preadjusted balance $2,200,000

Plus cost of goods in transit. The goods became the property of Kew at the shipping point (FOB shipping point). Kew owes the vendor for the goods. 40,000

Less credit for returned goods shipped back to vendor before year-end (70,000)

Total accounts payable $2,170,000

The goods in transit FOB destination are not yet the property of Kew because the title does not pass until the goods reach the destination.

644
Q

Kemp Co. must determine the December 31, 20X2, year-end accruals for advertising and rent expenses. A bill for $500 in advertising expenses was received January 7, 20X3, comprising costs of $375 for advertisements in December 20X2 issues, and $125 for advertisements in January 20X3 issues of the newspaper.

A store lease, effective December 16, 20X1, calls for fixed rent of $1,200 per month, payable one month from the effective date and monthly thereafter. In addition, rent equal to 5% of net sales over $300,000 per calendar year is payable on January 31 of the following year. Net sales for 20X2 were $550,000.

In its December 31, 20X2 balance sheet, Kemp should report accrued liabilities of

A

$13,475

Portion of the $500 advertising bill relating to December 20X2 $375

Lease rental ($1,200 × 1/2 of December) $600

Additional rental based on sales (.05)($550,000 − $300,000) $12,500

Total current liabilities $13,475

645
Q

Gar, Inc.’s trial balance reflected the following liability account balances at December 31, 20X5:

Accounts payable	$19,000
Bonds payable, due 20X6	34,000
Deferred income tax payable	4,000
Discount on bonds payable	2,000
Dividends payable on 2/15/X6	5,000
Income tax payable	9,000
Notes payable, due 1/19/X7	6,000

The deferred income tax payable is based on temporary differences that will reverse in 20X7 and 20X8.

In Gar’s December 31, 20X5 balance sheet, the current liabilities total was

A

$65,000

Accounts payable	$19,000
Bonds payable, due 20X6	34,000
Discount on bonds payable	(2,000)
Dividends payable on 2/15/X6	5,000
Income tax payable	9,000
Current liabilities at 12/31/X5	$65,000

Deferred taxes are classified as non-current.

646
Q

Rabb Co. records its purchases at gross amounts but wishes to change to recording purchases net of purchase discounts. Discounts available on purchases recorded from October 1, Year 3 to September 30, Year 4, totaled $2,000. Of this amount, $200 is still available in the accounts payable balance.

The balances in Rabb’s accounts as of and for the year ended September 30, Year 4, before conversion are:

Purchases $100,000
Purchase discounts taken 800
Accounts payable 30,000

What is Rabb’s accounts payable balance as of September 30, 20X4 after the conversion?

A

$29,800

Only the discounts still available on accounts yet to be paid can be deducted from the accounts payable balance, which now stands at gross.

The firm owes $30,000 at gross, which means that if none of the cash discounts available are taken, the firm will pay $30,000. The net counterpart of that amount is $29,800 ($30,000 − $200). If the firm pays all its remaining accounts within the cash discount period, it would pay only $29,800.

That is the assumption underlying the net method.

647
Q

Mill Co.’s trial balance included the following account balances at December 31, Year 3:

Accounts payable	$15,000
Bonds payable, due Year 4	25,000
Discount on bonds payable, due Year 4	3,000
Dividends payable 1/31/Y4	8,000
Notes payable, due Year 5	20,000

What amount should be included in the current liability section of Mill’s December 31, Year 3 balance sheet?

A

$45,000

Each item in the list, except the notes payable, due more than one year from the Year 3 balance sheet should be included in current liabilities. Except the notes payable, each item is due within one year of the Year 3 balance sheet. The discount reduces the net bond liability.

Thus, current liabilities total $45,000 = $15,000 + ($25,000 − $3,000) + $8,000.

648
Q

In its Year 5 financial statements, Cris Co. reported an interest expense of $85,000 in its income statement and a cash amount of $68,000 paid for interest in its cash flow statement. There was no prepaid interest or interest capitalization at either the beginning or end of Year 5. The accrued interest at December 31, Year 4 was $15,000.

What amount should Cris Co. report as accrued interest payable in its December 31, Year 5 balance sheet?

A

$32,000

An analysis of the accrued interest payable account leads to the correct ending balance:

Beginning balance + Interest expense - Interest payments = Ending balance
$15,000 + $85,000 - $68,000 = $32,000

649
Q

List the three key elements of liabilities.

A
  1. Present obligation to transfer assets or
    provide services.
  2. The obligation is unavoidable.
  3. Result of past transactions or events.
650
Q

How are current liabilities valued and

recorded?

A

Normally recorded at face value

651
Q

What do liabilities represent?

A

The represent outsider claims to a firm’s assets
or enforceable claims for services to be
rendered by the firm.

652
Q

How should a short-term note payable
refinanced every six months on a continuous
basis be classified?

A

The classification of this note is current.

653
Q

How are nonmonetary liabilities paid?

A

Payable in services or nonmonetary assets

654
Q

What account is credited in subsequent
monthly property tax accrual entries after the
tax bill is paid?

A

Prepaid property taxes

655
Q

Define “sales taxes payable.”

A

Account recognized for sales tax collected from

customers

656
Q

During the current year, Casual Wear Co. had total retail sales of $800,000 and collected a 5% state sales tax on all sales. At the end of the prior year, Casual Wear had $4,500 in sales taxes that had not been remitted to state authorities. During the current year, Casual Wear remitted $39,500 in state sales tax. What amount should be recorded in Casual Wear’s current-year financial statements?

A

$5,000 in sales tax payable

The ending liability balance is calculated as follows:
Beginning balance $4,500
Collections of sales tax ($800,000 × .05) 40,000
Less sales tax remitted (39,500)
Ending balance $5,000

657
Q

As of December 15, Year 1, Aviator had dividends in arrears of $200,000 on its cumulative preferred stock. Dividends for Year 1 of $100,000 have not yet been declared. The Board of Directors plans to declare cash dividends on its preferred and common stock on January 16, Year 2. Aviator paid an annual bonus to its CEO based on the company’s annual profits. The bonus for Year 1 was $50,000, which will be paid on February 10, Year 2. What amount should Aviator report as current liabilities on its balance sheet at December 31, Year 1?

A

$ 50,000

Only the bonus is a liability of the firm as of 12/31/Year 1. That amount was earned and granted in Year 1 and thus is recognized in the Year 1 balance sheet because it is not due for payment until Year 2. Dividends are not liabilities until declared. There is no unpaid declared dividend at 12/31/Year 1.

658
Q

Kent, Co. filed a voluntary bankruptcy petition on August 15, 20X5 and the statement of affairs reflects the following accounts:

                                        Book Val   Current Val Assets:		 Assets pledged with  fully secured creditors	  $ 300,000	$370,000 Assets pledged with  partially secured creditors	180,000	120,000 Free assets	                      420,000	320,000
                                       $ 900,000	$810,000
   ==========	========== Liabilities:	 Liabilities with priority	$ 70,000 Fully secured creditors	260,000 Partially secured creditors	200,000 Unsecured creditors	540,000
                                     $1,070,000 ========== Assume that the assets are converted to cash at the estimated current values and the business is liquidated. What amount of cash will be available to pay unsecured non-priority claims?
A

$360,000

The partially secured creditors receive payment from the assets pledged for partially secured creditors. Any remaining claims fall under the unsecured non-priority category.

The correct computation is:

Free assets $320,000
Assets pledged with fully secured creditors $370,000
Less payments to fully secured creditors (260,000)
Equals pledged assets available for unsecured claims 110,000
Total cash available for unsecured claims 430,000
Less priority claims (70,000)
Equals amount available to pay unsecured non-priority claims $360,000

The partially secured creditors receive payment from the assets pledged for partially secured creditors. Any remaining claims fall under the category of unsecured non-priority.

659
Q

Hudson Hotel collects 15% in city sales taxes on room rentals, in addition to a $2 per room, per night, occupancy tax.

Sales taxes for each month are due at the end of the following month, and occupancy taxes are due 15 days after the end of each calendar quarter. On January 3, 20X5 Hudson paid its November 20X4 sales taxes and its fourth quarter 20X4 occupancy taxes. Additional information pertaining to Hudson’s operations is:

20X4 Room Rentals Room Nights
October $100,000 1,100
November 110,000 1,200
December 150,000 1,800

What amounts should Hudson report as sales taxes payable and occupancy taxes payable in its December 31, 20X4 balance sheet?

A

Sales taxes - $39,000
Occupancy taxes - $8,200

Sales taxes payable at the end of 20X4 are .15($110,000 + $150,000) = $39,000 because November and December sales taxes were not yet paid as of 12/31/X4.

Occupancy taxes payable are $2(1,100 + 1,200 + 1,800) = $8,200 because the fourth quarter taxes were not paid as of 12/31/X4.

660
Q

On January 1, year 1, Alpha Co. signed an annual maintenance agreement with a software provider for $15,000 and the maintenance period begins on March 1, year 1. Alpha also incurred $5,000 of costs on January 1, year 1, related to software modification requests that will increase the functionality of the software. Alpha depreciates and amortizes its computer and software assets over five years using the straight-line method. What amount is the total expense that Alpha should recognize related to the maintenance agreement and the software modifications for the year ended December 31, year 1?

A

$13,500

This question has two costs that occurred during the year. You are asked how much of these costs would be recognized in year 1. The $15,000 of maintenance cost is for a 1 year period beginning March 1. The maintenance cost would be allocated 1/12 evenly over the life of the service period or $1,250 a month × 10 months in year 1 = 12,500. The $5,000 modification to the software has increased its functionality and therefore should be capitalized and amortized over the life of the software $5,000 / 5 years = $1,000/year. The total expense recognized in year 1 would be $12,500 + 1,000 = $13,500.

661
Q

What payroll taxes are paid only by the

employer?

A

State and federal unemployment taxes are paid

only by employer.

662
Q

What payroll taxes are paid in equal amounts

by the employer and the employee?

A

Federal Insurance Contributions Act
(FICA) tax
Medicare

663
Q

How is the total employer expense for payroll

computed?

A

Gross pay + Employer payroll taxes + Employer

portion of employee fringe benefits

664
Q

What basis of accounting is used for
recognizing expense for compensated
absences?

A

The basis is accrual.

665
Q

What is the meaning of “accumulate” in the

context of compensated absences?

A

Benefits carry over to future periods although

there may be limits.

666
Q

When is the expense associated with
compensated absences accrued, even if the
benefits do not vest or accumulate or if the
obligation is not attributable to services
rendered as of the balance sheet date?

A

When it is probable that benefits will be paid,

and the amount is estimable

667
Q

What is the exception to the requirement that

compensated absence expense be accrued?

A

Sick pay benefits

668
Q

What is the most likely recognition of the effect
of a pay raise between the time of recognition
of compensated absence liability and
payment?

A

Increase in expense in the period of payment

669
Q

When is an amount less than the total benefit

earned by employees accrued?

A

When not all earned benefits are expected to

be paid

670
Q

Doren Co.’s officers’ compensation expense account had a balance of $490,000 at December 31, 20X4, before any appropriate year-end adjustment relating to the following:

No Salary accrual was made for the week of December 25-31, 20X4. Officers’ salaries for this period totaled $18,000 and were paid on January 5, 20X5.
Bonuses to officers for 20X4 were paid on January 31, 20X5 in the total amount of $175,000.
The adjusted balance for officers’ compensation expense for the year ended December 31, 20X4, should be:

A

$683,000.

Total compensation expense should include the two adjusting items. Therefore, the total expense is $490,000 + $18,000 + $175,000 = $683,000. The two adjusting items are not included in the expense account balance because the adjustments have not yet been made. The accrued, but unpaid, salaries, as well as the bonuses, relate to 20X4. Officer bonuses are another form of employee compensation.

671
Q

Gavin Co. grants all employees two weeks of paid vacation for each full year of employment. Unused vacation time can be accumulated and carried forward to succeeding years and will be paid at the salaries in effect when vacations are taken or when employment is terminated.

There was no employee turnover in 2005.

Additional information relating to the year ended December 31, 2005 is as follows:

Liability for accumulated vacations at December 31, 2004 $35,000

Pre-2005 accrued vacations taken from January 1, 2005 to 30 September 2005 (the authorized period for vacations) 20,000

Vacations earned for work in 2005 (adjusted to current rates) 30,000

Gavin granted a 10% salary increase to all employees on October 1, 2005, its annual salary increase date. For the year ended December 31, 2005, Gavin should report vacation pay expense of

A

$31,500

The total vacation pay expense for 2005 is $31,500. This is the sum of two amounts:

(1) the amount earned in 2005, plus
(2) the increase in cost from earlier periods owing to wage increases in 2005.

These two amounts are:

(1) $30,000 as given in the problem; this amount is already updated for the most current rate
(2) $1,500 = ($35,000 − $20,000).10 = the amount of vacation pay yet to be disbursed on benefits earned before 2005; the liability for this amount is increased by the 10% pay increase.

The increase in pay rate on the pre-2005 benefits is treated as an estimate change. Therefore, it is handled in current and future years. Retroactive application does not apply in this case.

672
Q

Bloy Corp.’s payroll for the pay period ended October 31, 2005 is summarized as follows:

                      Fed Inc            Amt of wages 
                                            subject to payroll taxes Department  Payroll	Total Wages Taxwheld  F.I.C.A.	 
                                                                  Unemployment Factory	22,000	       3,000	16,000	2,000 Sales	18,000	       2,000	8,000	   - Office	60,000	       7,000	56,000	18,000 Total	$100,000     $12,000	$80,000	$20,000 =========	=========	========	========= Assume the following payroll tax rates:

F.I.C.A. for employer and employee 7% each

Unemployment 3%

What amount should Bloy accrue as its share of payroll taxes in its October 31, 2005 balance sheet?

A

$6,200

This question asks for the ending payroll tax liability. This amount is the employer’s share of FICA at 7% and the unemployment tax at 3%. Both have maximum wage limits. The $6,200 ending payroll tax liability is computed as $80,000(.07) + $20,000(.03).

673
Q

The following information pertains to Rik Co.’s two employees:

Name Weekly salary
Number of weeks
worked in 2005
Vacation
rights vest or
accumulate
Ryan $800 52 Yes
Todd 600 52 No

Neither Ryan nor Todd took the usual two-week vacation in 2005. In Rik’s December 31, 2005, financial statements, what amount of vacation expense and liability should be reported?

A

$1,600

Only Ryan’s vacation rights vest or accumulate and cause a liability to be reported. This amount is $1,600 (two weeks vacation × $800).

Todd’s rights do not vest or accumulate and therefore fail to meet the criteria of FAS 43. These criteria require accrual of the liability if the benefits accumulate or vest, the benefits are earned, and the payment is probable and estimable. Ryan’s rights meet the criteria.

From the firm’s point of view, because Todd’s rights do not vest or accumulate (carry over to a future year), then Todd loses those rights. The firm has no obligation to pay those benefits, and therefore no liability or expense is accrued at the end of 2005 for Todd.

674
Q

At December 31, 2004, Taos Co. estimates that its employees have earned vacation pay of $100,000. Employees will receive their vacation pay in 2005.

Should Taos accrue a liability at December 31, 2004 if the rights to this compensation accumulated over time or if the rights are vested?

Accumulated
Vested

A

Accumulated - YES
Vested - YES

When compensated absences either accumulate OR vest, then the liability should be accrued. Benefits accumulate if they can be carried over to future years.

For example, assume an employee earns four weeks’ vacation per year, but does not take a vacation for two years. If the employee can take an eight-week vacation in the third year, the benefits are said to accumulate (firms usually place restrictions on the total time that can be accumulated).

Benefits vest if they are no longer contingent on continued employment. This means that if an employee retires, he or she will receive their vested vacation pay.

Either way, through accumulation or vesting, it is probable that the vacation compensation will be paid. Therefore, a liability has been incurred as of the balance sheet date.

675
Q

North Corp. has an employee benefit plan for compensated absences that gives employees ten paid vacation days and ten paid sick days per year.

Both vacation and sick days can be carried over indefinitely. Employees can elect to receive payment in lieu of vacation days; however, no payment is given for sick days not taken.

At December 31, 2004, North’s unadjusted balance of liability for compensated absences was $21,000. North estimated that there were 150 vacation days and 75 sick days available at December 31, 2004. North’s employees earn an average of $100 per day.

In its December 31, 2004 balance sheet, what amount of liability for compensated absences is North required to report?

A

$15,000

The liability must be accrued only for the vacation pay, because it is probable that paid vacations will be taken. Therefore, the liability is $15,000 (150 days × $100 per day).

The firm may, but is not required to, accrue a liability for sick days. If the employees were routinely paid for sick days not taken, then sick days would be required to be accrued. For this firm, there is no payment for sick days not taken, therefore there is no requirement to accrue this cost.

It may be argued that illness is the condition that mandates payment of sick pay. Illness cannot be predicted and therefore is not required to be accrued.

676
Q

Dana Co.’s officers’ compensation expense account had a balance of $224,000 at December 31, 20X4 before any appropriate year-end adjustment relating to the following:

No salary accrual was made for December 30-31, 20X4. Salaries for the two-day period totaled $3,500.

20X4 officers’ bonuses of $62,500 were paid on January 31, 20X5.

In its 20X4 income statement, what amount should Dana report as officers’ compensation expense?

A

$290,000

Both adjustments are included in compensation expense because they are costs of services rendered by employees in 20X4. The firm has incurred a liability and therefore an expense as of 12/31/X4 for each of these items. $290,000 = $224,000 + $3,500 + $62,500.

677
Q

If the payment of employees’ compensation for future absences is probable, the amount can be reasonably estimated, and the obligation relates to rights that accumulate, the compensation should be

A

Accrued if attributable to employees’ services already rendered.

Only costs that are attributable to employee service already rendered can be accrued. The firm has received no benefit for services that employees have not yet rendered. The firm owes employees nothing for future services and therefore has no liability for these amounts and no cost or expense should be recognized.

678
Q

Under state law, Acme may pay 3% of eligible gross wages or it may reimburse the state directly for actual unemployment claims.

Acme believes that actual unemployment claims will be 2% of eligible gross wages and has chosen to reimburse the state. Eligible gross wages are defined as the first $10,000 of gross wages paid to each employee. Acme had five employees each of whom earned $20,000 during 20X4.

In its December 31, 20X4 balance sheet, what amount should Acme report as accrued liability for unemployment claims?

A

$1,000

The wage limit on unemployment tax is $10,000. Thus, the total accrued liability, which is also the unemployment tax amount, is 5($10,000)(.02) = $1,000.

679
Q

What account is credited in subsequent
monthly property tax accrual entries after the
tax bill is paid?

A

Prepaid property taxes

680
Q

The beginning of the current year, a firm invested $30 million in a natural resources site. This amount was applied to the acquisition of the mineral rights, exploring for the resource (full-costing method is used), and development. In addition, the firm must bring the property back to its original state three years from today. Two estimates of the future cost for that future effort are: (1) $6 million with 30% probability, and (2) $4 million with 70% probability. 6% is the appropriate risk adjusted rate of return. The present value of $1 in three years at 6% is 0.83962. By the end of the current year, the firm had removed 20% of the total estimated resource in the deposit. Compute depletion and accretion expense for the current year.

Depletion
Accretion expense

A

Depletion - $6,772,450
Accretion expense - $231,735

Asset retirement obligation beginning balance = $6,000,000(.30) + $4,000,000(.70) = $3,862,252. This is the present value of the expected future cost of reclaiming the property. The risk-adjusted rate of return is used because the probabilities account for the uncertainty of the cash flow amounts. This beginning balance is added to the $30 million figure for a total of $33,862,252 capitalized depletion base. Depletion is 20% of that amount or $6,772,450. Accretion expense is the growth in the asset retirement obligation for the year or .06($3,862,252) = $231,735.

681
Q

The recording of an asset retirement obligation for a natural resources development site increases which of the following for the firm involved in the site?
Liability
Depletion base

A

Liability -YES
Depletion base - YES

An asset retirement obligation is recorded when a firm has a probable and estimable future cost to reclaim the property exploited at the end of the project life. The amount is the fair value or present value of future cash payments to be made. The asset retirement obligation is recorded as a debit to the natural resources account (depletion base) and a credit to a liability.

682
Q

A firm’s natural resource exploitation site will require an expenditure of $5 million to reclaim the site for environmental purposes. That expenditure is expected to be made five years from now. The present value today of that amount is $3.5 million. Because of this obligation, by what amount will (1) total depletion on the site increase and (2) how much accretion expense will be recognized, over the five years (in millions)?
1
2

A

1 - $ 3.5
2 - $ 1.5

The natural resources account is increased by the asset retirement obligation, which is the present value of the $5 to be paid later, or $3.5. Therefore, total depletion over the venture’s life increases by that amount. The growth in the obligation over time is the accretion expense. The $3.5 amount will grow to $5 in five years, at which time the expenditure of that amount is made. The journal entry to record the asset retirement obligation is a debit to the natural resources account of $3.5 and a credit to asset retirement obligation of $3.5.

683
Q

A company completes construction of a $400 million offshore oil platform and places it into service on January 1. State law requires that the platform be dismantled and removed at the end of its useful life, which is estimated to be 10 years. The company estimates that the cost of dismantling the platform will be $20 million. The discounted value of the liability is $9 million using the company’s credit-adjusted, risk-free rate. The company has already capitalized the $400 million construction cost of the platform. What amounts should the company record as liability and expense when the asset is placed into service?

A

Liability, $9,000,000; expense, $0

There is liability recorded for dismantling the platform on the day the asset is placed in service. At the time an obligation is probable and estimable, it is accrued as a liability. No expense is recognized at on the day the asset is placed into service. The accretion of the $9 million liability to $20 million is recognized as an expense over the assets 10-year life.

684
Q

Choose the best description of accretion expense associated with an asset retirement obligation.

A

Growth in asset retirement obligation

Accretion expense is simply the increase in the asset retirement obligation over time. The asset retirement obligation is initially recorded at present value or fair value, and over time grows with interest until it reaches its future value—the amount due. Accretion expense is similar to the interest cost component of pension expense—the growth in projected benefit obligation. It is caused by the fact that the asset retirement obligation is recorded at present value but not paid until later.

685
Q

What is the meaning of “remote” in the context

of contingent liabilities?

A

Based on professional judgment, the
probability of occurrence is considered to be
very low or as the word implies, remote.

686
Q

What is the meaning of “probable” in context of

contingent liabilities?

A

Based on professional judgment, the
probability of occurrence is considered very
high or a near certainty.

687
Q

Define “contingency.”

A

An existing condition (at the balance sheet
date) involving uncertainty as to a possible gain
or loss that will be resolved when a future event
occurs or fails to occur

688
Q

Describe the accounting treatment when a loss
contingency is probable and can be reasonably
estimated.

A

Record loss and liability.

If only a range is estimable, must
recognize the liability for the lowest
amount in the range

689
Q

Describe the accounting treatment when a loss

contingency is remote.

A

Can be disclosed in footnotes but is not

required

690
Q

List the contingency’s probability of occurrence

categories.

A

Probable
Reasonably possible
Remote

691
Q

Describe the accounting treatment when a loss
contingency is probable but not reasonably
estimable.

A

Disclose in footnotes

692
Q

Describe the accounting treatment when a loss

contingency is reasonably possible.

A

Disclose in footnotes.

693
Q

What is the meaning of “reasonably possible”

in the context of contingent liabilities?

A

Based on professional judgment, the
probability of occurrence is neither very high
nor remote.

694
Q

Which of the following methods should a company use to account for a contingent liability when the loss is probable but not reasonably estimated?

A

The liability should only be disclosed in the notes to the financial statements.

A loss that is probable to occur but cannot be reasonably estimated is disclosed in the notes to the financial statements.

695
Q

Hill Corp. began production of a new product. During the first calendar year, 1,000 units of the product were sold for $1,200 per unit. Each unit had a two-year warranty. Based on warranty costs for similar products, Hill estimates that warranty costs will average $100 per unit. Hill incurred $12,000 in warranty costs during the first year and $22,000 in warranty costs during the second year. The company uses the expense warranty accrual method. What should be the balance in the estimated liability under warranties account at the end of the first calendar year?

A

$88,000

The beginning warranty liability in year 1 is $100,000 (1,000 units × $100) less the first year warranty costs of $12,000, equals the first year ending balance $88,000.

696
Q

Which of the following information about threatened litigation should not be considered to determine whether an accrual is appropriate prior to an issuance of a company’s financial statements?

A

The period in which the threatened litigation became known to management.

This question is stated in the null form (what is not considered). So, let’s review what must be considered to determine if an accrual is appropriate. A contingency is accrued if it is probable to occur, estimable, and an event or transaction has occurred. This response refers to the period that management becomes aware of the litigation. This is not one of the factors we would take into consideration to determine if an accrual is necessary, therefore this is the correct response.

697
Q

Martin Pharmaceutical Co. is currently involved in two lawsuits. One is a class-action suit in which consumers claim that one of Martin’s best selling drugs caused severe health problems. It is reasonably possible that Martin will lose the suit and have to pay $20 million in damages. Martin is suing another company for false advertising and false claims against Martin. It is probable that Martin will win the suit and be awarded $5 million in damages. What amount should Martin report on its financial statements as a result of these two lawsuits?

A

$0

A contingent liability is recognized only when occurrence is probably and estimable. This class-action suit is reasonably possible (a 50/50 chance) but not probable (a higher threshold). Therefore, a liability for the class-action suit would not be accrued. Contingent assets are not recognized until the amount is actually received, even if the outcome is probable and estimable. Therefore, no asset is accrued for the suit where Martin may be awarded damages.

698
Q

Hudson Corp. operates several factories that manufacture medical equipment. The factories have a historical cost of $200 million. Near the end of the company’s fiscal year, a change in business climate related to a competitor’s innovative products indicated to Hudson’s management that the $170 million carrying amount of the assets of one of Hudson’s factories may not be recoverable. Management identified cash flows from this factory and estimated that the undiscounted future cash flows over the remaining useful life of the factory would be $150 million. The fair value of the factory’s assets is reliably estimated to be $135 million. The change in business climate requires investigation of possible impairment. Which of the following amounts is the impairment loss?

A

$35 million

Under U.S. GAAP, impairment testing is a two step process. The first step compares the assets’ carry value (CV) to its undiscounted cash flows (UCF). In this problem the CV > UCF; therefore the asset is potentially impaired and we must go to the second step. The second step compares the assets CV to its fair value (FV). In this problem the FV < CV and the asset is written down to its FV. $170 million − $135 million = $35 million impairment loss.

699
Q

Management can estimate the amount of the loss that will occur if a foreign government expropriates some company assets.

If expropriation is reasonably possible, a loss contingency should be:

A

Disclosed but not accrued as a liability.

A reasonably possible loss contingency is disclosed in the footnotes, but not recognized as a liability.

Only when the contingent loss is both probable and estimable is the loss accrued (recognized).

700
Q

East Corp. manufactures stereo systems that carry a two-year warranty against defects. Based on past experience, warranty costs are estimated at 4% of sales for the warranty period.

During 2005, stereo system sales totaled $3,000,000, and warranty costs of $67,500 were incurred.

In its income statement for the year ended December 31, 2005, East should report warranty expense of:

A

$120,000

Warranty expense is recognized in the year of sale under the accrual accounting system.

Warranties are a part of the selling effort, and the associated expense should be recognized when the liability is probable and estimable (in 2005). The actual repairs reduce the liability recognized when the expense was recorded (in the year of sale).

The $120,000 of warranty expense in 2005 = .04(sales in 2005) = .04($3,000,000). The relevant entries for 2005 are:

Warranty expense 120,000
Warranty liability 120,000

Warranty liability 67,500
Cash, parts, etc. 67,500

701
Q

Snelling Co. did not record an accrual for a contingent loss, but disclosed the nature of the contingency and the range of the possible loss.

How likely is the loss?

A

Reasonably possible.

Remote contingent losses may be disclosed in the footnotes, but there is no requirement to do so. Probable contingent losses are accrued. Certain losses are no longer contingent losses. When a loss is reasonably possible, it is footnoted. It is most likely that the loss is reasonably possible when a range of losses is disclosed.

702
Q

During 2004, Gum Co. introduced a new product carrying a two-year warranty against defects. The estimated warranty costs related to dollar sales are 2% within 12 months following the sale and 4% in the second 12 months following the sale.

Sales and actual warranty expenditures for the years ended December 31, 2004 and 2005 are as follows:

Sales	Actual warranty expenditures
2004	$150,000	$2,250
2005	250,000	           7,500
               $400,000	$9,750
                 =========	=========
What amount should Gum report as estimated warranty liability in its December 31, 2005, balance sheet?
A

$14,250

At Dec. 31, 2005, the total warranty liability accrued for the two years is 6% of sales (2% + 4%). This total is $24,000 (.06 × $400,000). Subtracting $9,750 of actual warranty expenditures to the end of 2005 yields the $14,250 ending warranty liability.

703
Q

In May 2000, Caso Co. filed suit against Wayne, Inc. seeking $1,900,000 in damages for patent infringement.

A court verdict in November 2003 awarded Caso $1,500,000 in damages, but Wayne’s appeal is not expected to be decided before 2005. Caso’s counsel believes it is probable that Caso will be successful against Wayne for an estimated amount in the range between $800,000 and $1,100,000, with $1,000,000 considered the most likely amount.

What amount should Caso record as income from the lawsuit in the year ended December 31, 2003?

A

$ -0-

This is a gain contingency. These items are not recognized in the financial statements until the contingency is removed. Thus, no income is recognized in 2003.

704
Q

In 2003, a personal injury lawsuit was brought against Halsey Co.

Based on counsel’s estimate, Halsey reported a $50,000 liability in its December 31, 2003, balance sheet. In November 2004, Halsey received a favorable judgment, requiring the plaintiff to reimburse Halsey for expenses of $30,000. The plaintiff has appealed the decision, and Halsey’s counsel is unable to predict the outcome of the appeal.

In its December 31, 2004, balance sheet, Halsey should report what amounts of asset and liability related to these legal actions?

Asset
Liability

A

Asset - $ 0
Liability - $ 0

The contingent liability at the end of 2003 no longer exists.

It is not probable, given the facts in the question, that Halsey will be required to make any payment in the lawsuit. The favorable judgment indicates a contingent gain (asset). Contingent gains are not recognized in the accounts, but only footnoted.

705
Q

At December 31, 2004, Date Co. awaits judgment on a lawsuit for a competitor’s infringement of Date’s patent. Legal counsel believes it is probable that Date will win the suit and indicated the most likely award together with a range of possible awards.

How should the lawsuit be reported in Date’s 2004 financial statements?

A

In note disclosure only.

This is a gain contingency. Gain contingencies are footnoted at most, not accrued. To recognize gain contingencies in the accounts would violate the conservatism constraint.

706
Q

What is the underlying concept that supports the immediate recognition of a contingent loss?

A

Conservatism.

A contingent loss has not occurred as of the balance sheet date, but since it is probable and estimable, and would result in lower income and net assets, the loss should be recognized. A contingent gain that also is probable and estimable is not recognized. Thus, it is only the direction of the effect of the item that causes the accounting treatment to be different.

This can only be explained by conservatism: under conditions of uncertainty, report lower earnings and net assets. Uncertain gains are not allowed to be recognized because they may raise the expectations of investors unnecessarily. Uncertain gains may not be realized.

707
Q

Vadis Co. sells appliances that include a three-year warranty. Service calls under the warranty are performed by an independent mechanic under a contract with Vadis. Based on experience, warranty costs are estimated at $30 for each machine sold.

When should Vadis recognize these warranty costs?

A

When the machines are sold.

At the point of sale, Vadis has committed to service the products it sells. The firm has incurred a recognized obligation at that point because it is both probable and estimable (FAS 5).

The cost of the warranty, therefore, is recognized in the year of sale. The cost (expense) is the temporary account that measures the reduction in net assets from operations (earnings) caused by the increase in the obligation.

A less acceptable explanation is that the warranty cost or expense should be matched against the sales it helped to produce. Either explanation leads to the same result, however.

708
Q

Rollin Corporation purchased 100 bonds issued by Boyle Corp. and classifies the investment as trading securities. Rollin should report these trading securities at

A

Fair value, with holding gains and losses included in earnings.

Trading securities are debt securities purchased and held principally for the purpose of generating gains on resale. Trading securities should be reported on the balance sheet at fair value, with holding gains and losses included in earnings.

709
Q

Which of the following best describes the definition of a debt security?

A

Securities that represent the right of the holder to receive from the issuer a principal amount at a specified future date and to receive interest as payment for providing use of funds

A debt security provides the right for the holder to receive principal and interest from the issuer.

710
Q

Which of the following statements related to securities is not true:

A

Unrealized gains and losses related to available-for-sale securities are reported in earnings.

This response is not true. Only debt securities can be classified as trading, available-for-sale, or held-to-maturity; equity securities cannot be placed in one of these classifications. The unrealized gains and losses of available-for-sale securities are recorded in OCI.

711
Q

Desno Corporation reports on a calendar-year basis. Its December 31, Year 1, financial statements were issued on February 3, Year 2. The auditor’s report was dated January 22, Year 2. The following information pertains to Desno’s aggregate equity securities portfolio:

Cost $500,000
Market value, 12/31/Y1 400,000
Market value, 1/22/Y2 350,000
Market value, 2/3/Y2 300,000

How much should be reported on Desno’s balance sheet at December 31, Year 1, for marketable equity securities?

A

$400,000

This answer is correct because an enterprise’s financial statements should not be adjusted for changes in market prices with respect to marketable equity securities when such changes occur after the date of the financial statements but prior to their issuance. The question does not indicate the level of economic influence of these holdings. However, equity securities should be reported at fair value whenever it is readily determinable. Thus, the fair value at 12/31/Y1 of $400,000 is reported.

712
Q

On January 1, Year 1, Grade Company paid $300,000 for 20,000 shares of Medium Company’s common stock, which represents a 15% investment in Medium. Grade does not have the ability to exercise significant influence over Medium. Medium declared and paid a dividend of $1 a share to its stockholders during Year 1. Medium reported net income of $260,000 for the year ended December 31, Year 1, and had a market value of $300,000 at December 31, Year 1. The balance in Grade’s balance sheet account “Investment in Medium Company” at December 31, Year 1, should be

A

$300,000.

Grade Company will account for this investment at fair value. This method is used because fair value is readily determinable and Grade owns less than 20% of Medium and cannot exercise significant influence over the company.

713
Q

On March 14, Apple Corporation purchased 6,000 shares of Pear Inc. for $25 per share. There is no readily determinable fair value of Pear. On June 30, Pear declared an annual dividend of $0.40 per share. On August 14, Apple sold 4,000 shares of Pear for $29 per share less a brokerage fee of $225. The journal entry at the date of sale would include

A

a credit to gain on the sale of investments for $15,775.

The entries related to the investment in Pear are as follows:
DR: Investment in Pear 150,000
CR: Cash 150,000
(to record the purchase of Pear ($25 × 6,000)
DR: Cash 2,400
CR: Dividend income 2,400
(to record dividend income ($0.40 × 6,000))
DR: Cash 115,775
CR: Investment in Pear 100,000
CR: Gain on sale 15,775
(to record the sale of 4,000 shares of Pear at $29 per share)

Cash = (($29 × 4,000) – 225 = 115,775)

Investment in Pear = ($25 × 4,000 = 100,000)

Gain = (115,775 – 100,000 = 15,775)

714
Q

When the equity method is used to account for investments in common stock, which one of the following affect(s) the investor’s reported investment income?

Change in market value of investee’s common stock
Cash dividends from investee

A

Change in market value of investee’s common stock - No
Cash dividends from investee - No

Under the equity method of accounting, changes in the market value of the investee’s common stock are ignored. Further, cash dividends received by the investor from the investee are not recognized as investment income; rather they are recognized as a debit to cash and a reduction in (credit to) the investor’s investment in investee account.

715
Q

Anchor Co. owns 40% of Main Co.’s common stock outstanding and 75% of Main’s noncumulative preferred stock outstanding. Anchor exercises significant influence over Main’s operations. During the current period, Main declared dividends of $200,000 on its common stock and $100,000 on its noncumulative preferred stock. What amount of dividend income should Anchor report on its Income Statement for the period related to its investment in Main?

A

$ 75,000

Anchor Co. would report 75% of Main’s dividend on its noncumulative preferred stock as dividend income. That calculation would be: .75 × $100,000 = $75,000. Anchor Co. would not report 40% of Main’s dividend on its common stock as dividend income because, since it can exercise significant influence over Main’s operations, it must account for its common stock investment in Main using the equity method of accounting. Under the equity method, common stock dividends received from an investee are recognized as a reduction in the investment in the investee. So, Anchor’s entry to recognize the $80,000 (.40 × $200,000) common stock dividend from Main would be:

DR: Cash $80,000
CR: Investment in Main $80,000

716
Q

Lee, Inc. acquired 30% of Polk Corp.’s voting stock on January 1, Year 1 for $100,000. During Year 1, Polk earned $40,000 and paid dividends of $25,000.

Lee’s 30% interest in Polk gives Lee the ability to exercise significant influence over Polk’s operating and financial policies. During Year 2, Polk earned $50,000 and paid dividends of $15,000 on April 1 and $15,000 on October 1.

On July 1, Year 2, Lee sold half of its stock in Polk for $66,000 cash.

Before income taxes, what amount should Lee include in its Year 1 Income Statement as a result of the investment?

A

$12,000

$12,000 = .30($40,000). Under the equity method, the investor recognizes its share of investee earnings in its own income. The equity method is used because Lee has significant influence over Polk.

717
Q

Sage, Inc. bought 40% of Adams Corp.’s outstanding common stock on January 2, Year 1 for $400,000. The investment gave Sage significant influence over Adams.

The carrying amount of Adams’ net assets at the purchase date totaled $900,000. Fair values and carrying amounts were the same for all items except for plant and inventory, for which fair values exceeded their carrying amounts by $90,000 and $10,000, respectively. The plant has an 18-year life. All inventory was sold during Year 1. Goodwill, if any, is expected to have a useful life of 40 years. During Year 1, Adams reported net income of $120,000 and paid a $20,000 cash dividend.

What amount should Sage report in its Income Statement from its investment in Adams for the year ended December 31, Year 1?

A

$42,000

This is a question on the full equity method.

Goodwill on the purchase = price of investment - 40%(fair value of Adams’ net assets)
= $400,000 -.40($900,000 + $90,000 + $10,000)
= $400,000 - $400,000 = 0 (there is no goodwill to amortize)
Investment income:

40% of Adams’ income: .40($120,000) $48,000
Less 40% of excess of fair value of inventory over book value at acquisition. The inventory is sold, and therefore, the cost of goods sold of the investor must be increased by this amount: .40($10,000) (4,000)
Less depreciation on 40% of the excess of fair value over book value of equipment: .40($90,000)/18 (2,000)
Equals amount Sage reports as investment income $42,000

718
Q

In its financial statements, Pulham Corp. uses the equity method of accounting for its 30% ownership of Angles Corp. At December 31, Year 2, Pulham has a receivable from Angles.

How should the receivable be reported in Pulham’s Year 2 financial statements?

A

The total receivable should be disclosed separately.

Although the equity method is often called a “one-line” consolidation, intercompany receivables remain separate from the investment account. Intercompany profit or loss is eliminated but that affects the income recognized by Pulham, not the receivable.

719
Q

Pare, Inc. purchased 10% of Tot Co.’s 100,000 outstanding shares of common stock on January 2, Year 1, for $50,000.

On December 31, Year 1, Pare purchased an additional 20,000 shares of Tot for $150,000. There was no goodwill as a result of either acquisition, and Tot had not issued any additional stock during Year 1. Tot reported earnings of $300,000 for Year 1.

What amount should Pare report in its December 31, Year 1, Balance Sheet as investment in Tot?

A

$200,000

Once the investor has acquired a sufficient percentage of the stock to use the equity method, it is prospectively applied. In this question, the equity method becomes the required method only at the very end of the year, and Pare would begin applying the equity method beginning on December 31. The ending balance of the investment account is: $50,000 (original investment) + $150,000 (second investment).

720
Q

On January 10, Year 1, Wayne, Inc., purchased 5,000 of Jason bonds at $60 par per bond. The purchase is a long-term investment and is appropriately reflected in Wayne’s balance sheet in an available-for-sale securities portfolio at December 31, Year 1. The fair value of Wayne’s investment in Jason’s bonds are as follows:

                                               Fair value Date	                           Per bond	   Total December 15, Year 1	     $47	   $235,000 December 31, Year 1	     46	    230,000

On December 15, Year 1, Wayne determined that there had been an other-than-temporary decline in the fair value. What amount should Wayne record as a loss in its income statement for the year ended December 31, Year 1?

A

$70,000

The loss realized from an other-than-temporary decline in the market value of available-for-sale securities is measured as the difference between their cost and market value on the balance sheet date of the reporting period for which the assessment is made.
1/10/Y1 Carrying value 5,000 shares × $60 = $300,000
12/31/Y1 Market value 5,000 shares × $46 to be recognized = 230,000
Realized loss to be recognized $ 70,000

721
Q

The following information pertains to Lark Corp.’s available-for-sale debt securities portfolio:

                      Year 2	         Year 1 Cost	     $200,000	      $200,000 Fair value	240,000  	180,000

Lark does not elect to use the fair value option for reporting financial assets. Differences between cost and fair values are considered to be temporary. The decline in fair value was properly accounted for at December 31, Year 1. What is the amount of unrealized gain (loss) on debt securities, a component of other comprehensive income, for the year ended to December 31, Year 2?

A

$60,000

The amount by which an available-for-sale (AFS) portfolio’s aggregate carrying value exceeds its fair value should be recognized as an unrealized loss and shown as other comprehensive income. Note that at the end of each year, the unrealized gain (loss) is closed to accumulated other comprehensive income, not retained earnings. In future years, the portfolio’s aggregate fair value increases, the unrealized loss is removed from the accumulated other comprehensive income account, and eventually an unrealized gain may be established. Lark Corp’s journal entries for Year 1 and Year 2 were

Year 1:		
Unrealized loss on AFS	20,000	
  Investment in AFS		20,000
Year 2:		
Investment in AFS	60,000	
  Unrealized gain in AFS		60,000

Therefore, the unrealized gain (loss) account would increase by $60,000 in Year 2 for the recovery in fair value of the previously recognized unrealized loss of $20,000 in Year 1 plus the additional $40,000 increase in fair value above cost.

722
Q

During Year 1, Anthony Company purchased debt securities as a long-term investment. Pertinent data are as follows:

     Security	  Cost	  Fair value at 12/31/Y1
          A	    $ 20,000	$ 17,000
          B	      40,000	30,000
          C	      90,000	92,000
                   $150,000	$139,000

Anthony appropriately carries these securities at fair value, and the decline in value of Security B is not considered to be temporary. The change in value of securities A and C is considered to be temporary. Anthony does not elect to use the fair value option in reporting financial assets. The amount of loss on these securities that will appear on Anthony’s balance sheet as a component of “Accumulated other comprehensive income” at 12/31/Y1 should be

A

$ 1,000.

Since these equity securities were not purchased with the intent to sell them in the near term for a quick profit, they must be classified as available-for-sale securities. Any unrealized gain or loss on available-for-sale securities is to be reported net as a separate component of stockholders’ equity entitled “Accumulated other comprehensive income.” However, in the case of impairment, the security is written down to fair value and the amount of the write-down is reported in earnings as a realized loss. Thus, the $10,000 loss on Security B would go to the income statement while the $1,000 ($3,000 loss on A − $2,000 gain on C) unrealized loss on the remaining portfolio would appear separately in the stockholders’ equity section of Anthony’s balance sheet at 12/31/Y1.

723
Q

On January 2, Year 1, Winn Company purchased as a long-term investment 5,000 bonds of Pyle Corp. at par for $70 per bond. On December 31, Year 1, the fair value of the bonds was $75 per bond. On December 18, Year 2, Winn needed additional cash for operations and sold all 5,000 bonds for $100 per bond. Winn’s income tax rate was 40% for Year 2. For the year ended December 31, Year 2, Winn should include in its income from continuing operations a realized gain on disposal of long-term investment of

A

$150,000

Since these securities were not purchased with the intent to sell in the near term for a quick profit, they must be classified as available-for-sale securities. A realized gain on the disposal of such securities equals the excess of the proceeds (5,000 × $100 = $500,000) over the carrying value before recognition of any unrealized gains or losses (which equals cost) of the investment [5,000 × $70 = $350,000]. Therefore, the realized gain is $150,000 ($500,000 − $350,000). Note that the tax rate (40%) is not used; components of income from continuing operations are reported gross with income tax expense deducted from the total income from continuing operations.

724
Q

On April 1, North Company issued bonds in the market. Upon issue, South Company acquired 10% of North Company’s issue. On November 30, South sold the North Company bonds in the market; the bonds were acquired by East Company. On December 31, which, if any, of the following companies is an investee?

A

North - YES
South - NO
East - NO

North is the investee because it issued the bonds, but neither South nor East is an investee. Since East owns the bonds on December 31, it is the investor. Since South did not issue the bonds and does not own the bonds on December 31, it is neither an investor nor an investee.

725
Q

On July 1, Year 7, Hilltop Company purchased as a long-term investment Essex Company’s 10-year 9% bonds, with a face value of $100,000, for $95,200. Interest is payable semiannually on January 1 and July 1. The bonds mature on July 1, Year 11. Hilltop uses the straight-line method of amortization. What is the amount of interest receivable and amortization of bond discount that Hilltop should report in its income statement for the year ended December 31, Year 7?

A

$4,500 and $600

Hilltop purchased the bonds on July 1, so it should recognize interest receivable and bond amortization only for the last 6 months of the year. The interest receivable (cash interest to be received) for a half-year is ($100,000 × 9% × 6/12) = $4,500. The bond discount is $100,000 – $95,200, or $4,800. Since the bonds will be outstanding 48 months from the date of purchase, the bond discount amortization for the 6 months of Year 7 in which Hilltop held the investment is ($4,800 × 6/48) = $600.

726
Q

On January 1, Year 1, Weaver Company purchased as a long-term investment $500,000 face value of Park Corporation’s 8% bonds for $456,200. The bonds were purchased to yield 10% interest. The bonds mature on January 1, Year 7, and pay interest annually on January 1. Weaver intends to hold the bonds until their scheduled maturity. Weaver uses the interest method of amortization and does not elect the fair value option for reporting financial assets. What amount should Weaver report on its December 31, Year 1, balance sheet as long-term investment?

A

$461,820

Held-to-maturity investments should be reported at amortized cost on the balance sheet. The carrying value of long-term investments on December 31, Year 1, will be the carrying value on January 1, Year 1, plus the discount amortization. Discount amortization is the difference between interest revenue and interest receivable. Interest revenue is the book value of the bonds times the yield rate of interest ($456,200 × .10 = $45,620). Interest receivable is the face value of the bonds times the face rate of interest ($500,000 × .08 = $40,000). The adjusting entry on December 31, Year 1, will be

Interest receivable 40,000
Bond investment (long term) 5,620
Interest revenue 45,620

Thus, the carrying value of the bonds on December 31, Year 1, is $461,820 ($456,200 + $5,620). The above entry assumes that the bonds were recorded net when purchased. If a discount of $43,800 was recorded, the $5,620 debit would be to the discount account.

727
Q

On December 31, Year 1, Clark Company, an investment banker, purchased marketable debt securities with the intent to sell them for a quick profit. Pertinent data are as follows:

Bond Cost Market value at 12/31/Y2
W $24,000 $26,000
X 36,000 33,000
Y 72,000 65,000

On December 31, Year 2, Clark reclassified its investment in security Y from trading to available-for-sale because Clark intends to retain security Y as a long-term investment. Assume Clark does not elect the fair value option for reporting financial assets. What total amount of loss on these securities should be included in Clark’s income statement for the year ended December 31, Year 2?

A

$8,000

Unrealized losses on trading securities are reported in the income statement. The amount of the unrealized loss at 12/31/Y2 is determined by comparing the total carrying value of the portfolio at 1/01/Y2 to its total market value at 12/31/Y2. The carrying value at 1/01/Y2 is its cost of $132,000 ($24,000 + $36,000 + $72,000), and the market value at 12/31/Y2 is $124,000 ($26,000 + $33,000 + $65,000). Therefore, $8,000 total unrealized loss on trading securities is recognized in income. The unrealized gain or loss on trading securities on the date that they are transferred from that category will have already been recognized in earnings and shall not be reversed.

728
Q
Assume Flag Corp holds equity securities in Red Banner Co., there is a readily determinable fair value of Red Banner stock, and Flag has no significant influence. Flag also holds securities of Star Co. and has significant influence in Star. The market price of the stock of both companies increased during the year. How will Flag's investment account be affected by the increase in market price of each? Assume the fair value option is not elected.
Red Banner  	  Star  
A. 	 No effect 	 No effect 
B. 	 No effect 	 Increase 
C. 	 Increase 	 No effect 
D. 	 Increase 	 Increase
A

Row C

There is a readily determinable fair value for Red Banner, and there is no significant influence. Therefore, Red Banner is accounted for at fair value. The investment in Star is accounted for using the equity method, and it is not adjusted for changes in the fair value of the stock. Therefore, an increase in the fair value of Red Banner would require an increase in the investment, and there is no effect on the investment in Star under the equity method.

729
Q

A debt security in an available-for-sale securities portfolio is transferred to a held-to-maturity securities portfolio. The security should be transferred between the corresponding portfolios at

A

The fair value at date of transfer, regardless of its cost.

Any transfer of securities between categories of investments should be accounted for at fair value. In addition, any unrealized holding gain or loss on securities transferred from available-for-sale to held-to-maturity continues to be reported as “Accumulated other comprehensive income” in stockholders’ equity but is amortized over the remaining life of the security.

730
Q

Assume a company does not elect the fair value option for reporting financial assets. Realized gains from the sale of marketable debt securities should be included in net income of the period of sale when the marketable debt securities portfolio of which they are a part is classified as

Available-for-sale
Held-to-maturity

A

Available-for-sale - YES
Held-to-maturity - YES

Realized gains and losses shall be included in the determination of net income of the period in which they occur for both available-for-sale and held-to-maturity securities.

731
Q

An increase in the cash-surrender value of a life insurance policy owned by a company would be recorded by:

A

Decreasing annual insurance expense.

A portion of the premium is allocated to the cash surrender value, which is an asset to the insured firm. Thus, the insurance expense is reduced by the amount of the premium so allocated.

For example, if the annual premium is $4,000, and the increase in cash surrender value is $800 for the year, insurance expense is $3,200 for the year.

732
Q

A company recently acquired a copyright that now has a remaining legal life of 30 years. The copyright initially had a 38-year useful life assigned to it. An analysis of market trends and consumer habits indicated that the copyrighted material will generate positive cash flows for approximately 25 years. What is the remaining useful life, if any, over which the company can amortize the copyright for accounting purposes?

A

25 years

This copyright has a definite life; the question is what is the length of that life? The life assigned to the intangible asset is the shorter of its legal and useful life. The useful life is shorter than the legal life, so this copyright is amortized over 25 years.

733
Q

On January 2, 20X5, Ral Co. leased land and a building from an unrelated lessor for a 10-year term. The lease has a renewal option for an additional 10 years, but Ral has not reached a decision with regard to the renewal option. In early January of 2005, Ral completed the following improvements to the property:

Description Estimated life Cost
Sales office 10 years $47,000
Warehouse 25 years 75,000
Parking lot 15 years 18,000

Amortization of leasehold improvements for 20X6 should be:

A

$14,000

The shorter of the lease term and useful life of the leasehold improvements is used for amortization because leasehold improvements revert to the lessor at the end of the lease term. A 10-year lease term is used because renewal is uncertain.

The leasehold improvements were capitalized at the beginning of 20X5. Annual amortization (and therefore amortization for 20X6) is $14,000 ($47,000 + $75,000 + $18,000)/10. Ten years is used for each asset because each has a useful life of at least 10 years and the lease term is 10 years as of the beginning of 20X6. The assets cannot be amortized over a period greater than 10 years because the lease will be concluded in 10 years.

734
Q

A company is completing its annual impairment analysis of the goodwill included in one of its cash generating units (CGUs). The recoverable amount of the CGU is $32,000. The company noted the following related to the CGU:

                      Goodwill   Patents  Other assets Total Historical cost$15,000	$10,000	$35,000	  $60,000

Depreciation
and amortization 0 3,333 11,667 15,000
Carrying amount,
December 31 $15,000 $6,667 $23,333 $45,000

Under IFRS, which of the following adjustments should be recognized in the company’s consolidated financial statements?

A

Decrease goodwill by $13,000.

Under IFRS goodwill impairment is measured in a one-step process. The carrying value of the CGU is compared to the recoverable amount. If the CV > recoverable amount the goodwill is impaired. The impairment loss is the recoverable amount - the CV. In this case $32,000 − $45,000 = ($13,000) loss.

735
Q

During 2004, Pitt Corp. incurred costs to develop and produce a routine, low-risk computer software product as follows:

Completion of detailed program design $13,000
Costs incurred for coding and testing to establish technological feasibility 10,000
Other coding costs after establishment of technological feasibility 24,000
Other testing costs after establishment of technological feasibility 20,000
Costs of producing product masters for training materials 15,000
Duplication of computer software and training materials from product masters (1,000 units) 25,000
Packaging product (500 units) 9,000

In Pitt’s December 31, 2004 balance sheet, what amount should be reported in inventory?

A

$34,000

Only the last two costs in the list are inventoried. The software development is complete when the product masters are produced.

The duplication costs ($25,000) and packaging costs ($9,000) are debited to inventory for a total of $34,000. These are costs necessary to bring the asset into salable condition. This cost is expensed when product is sold.

All the costs listed above the last two are aimed at developing the software. These costs are not debited to inventory. There is no product in which to inventory these costs until the development is complete.

736
Q

Describe the accounting treatment when a gain

contingency is probable.

A

Disclose in a footnote.

737
Q

What is the accounting treatment of excess
actual loss over recognized loss on a
contingent liability?

A

Additional loss in the period of lawsuit
resolution (estimated loss was recognized
previously)

738
Q

Describe the accounting treatment when a gain

contingency is possible.

A

Disclose in a footnote.

739
Q

What is the source of contingent liabilities not
requiring a current claim by an outside entity
for recognition?

A

Unasserted claim

740
Q

What account is debited when premiums are

redeemed by customers?

A

Premium liability

741
Q

In what period is a premium expense

recognized?

A

Year of sale

742
Q

Describe the accounting treatment when a gain

contingency is remote.

A

Disclosing in a footnote is not recommended

743
Q

Eagle Co. has cosigned the mortgage note on the home of its president, guaranteeing the indebtedness in the event that the president should default. Eagle considers the likelihood of default to be remote.

How should the guarantee be treated in Eagle’s financial statements?

A

Disclosed only.

In the interest of conservatism and disclosure, the guarantee should be disclosed. It is not required to be accrued because the probability is remote that the firm will have to pay the note.

744
Q

On January 17, 2005, an explosion occurred at a Sims Co. plant, causing extensive property damage to area buildings.

Although no claims had yet been asserted against Sims by March 10, 2005, Sims’ management and counsel concluded that it is likely that claims will be asserted and that it is reasonably possible Sims will be responsible for damages. Sims’ management believed that $1,250,000 would be a reasonable estimate of its liability. Sims’ $5,000,000 comprehensive public liability policy has a $250,000 deductible clause.

In Sims’ December 31, 2004, financial statements, which were issued on March 25, 2005, how should this item be reported?

A

As a footnote disclosure indicating the possible loss of $250,000.

This contingent liability is not probable but only reasonably possible. Only probable contingent liabilities are accrued. Reasonably possible contingent liabilities are footnoted.

The estimated loss is only $250,000 because the policy easily covers the estimated loss. Only the deductible of $250,000 would have to be paid by Sims.

745
Q

On January 3, 2005, Ard Corp. owned a machine that had cost $60,000. The accumulated depreciation was $50,000, estimated salvage value was $5,000, and fair market value was $90,000.

On January 4, 2005, this machine was irreparably damaged by Rice Corp. and became worthless. In October 2005, a court awarded damages of $90,000 against Rice in favor of Ard. At December 31, 2005, the final outcome of this case was awaiting appeal and was, therefore, uncertain.

However, in the opinion of Ard’s attorney, Rice’s appeal will be denied.

At December 31, 2005, what amount should Ard accrue for this gain contingency?

A

$ -0-

This is a gain contingency. Gain contingencies are not accrued in the accounts. At most they are footnoted.

This is an example of conservatism. Probable and estimable contingencies are recognized in the accounts if they are losses, but not if they are gains.

746
Q

Invern, Inc. has a self-insurance plan.

Each year, retained earnings is appropriated for contingencies in an amount equal to insurance premiums saved less recognized losses from lawsuits and other claims. As a result of a 2005 accident, Invern is a defendant in a lawsuit in which it will probably have to pay damages of $190,000.

What are the effects of this lawsuit’s probable outcome on Invern’s 2005 financial statements?

A

An increase in both expenses and liabilities.

The information about self-insurance (which means no insurance) is irrelevant to the problem except that if the firm loses the lawsuit, there will be no insurance coverage. This is a contingent liability. It is probable, and the amount is estimable.

Therefore, expenses (or a loss) and a liability are recognized for $190,000.

747
Q

In 2003, a contract dispute between Dollis Co. and Brooks Co. was submitted to binding arbitration.

In 2003, each party’s attorney indicated privately that the probable award in Dollis’ favor could be reasonably estimated. In 2004, the arbitrator decided in favor of Dollis.

When should Dollis and Brooks recognize their respective gain and loss?

Dollis’ gain
Brooks’ loss

A

Dollis’ gain - 2004
Brooks’ loss - 2003

Both the gain and loss are contingent items at the end of 2003. Contingent losses are recognized when probable and estimable—2003 in this case. Contingent gains are not recognized until realized— 2004 in this case.

748
Q

Grim Corporation operates a plant in a foreign country. It is probable that the plant will be expropriated. However, the foreign government has indicated that Grim will receive a definite amount of compensation for the plant.

The amount of compensation is less than the fair market value but exceeds the carrying amount of the plant.

The contingency should be reported:

A

In the notes to the financial statements.

This is a gain contingency. The possible gain is the difference between the compensation amount and the carrying value of the plant. However, the gain is contingent on receipt of the compensation. Gain contingencies are reported only in the notes and are not recognized in the financial statements.

749
Q

During 2005, Tedd Co. became involved in a tax dispute with the IRS. At December 31, 2005, Tedd’s tax advisor believed that an unfavorable outcome was probable.

A reasonable estimate of additional taxes was $400,000 but could be as much as $600,000.

After the 2005 financial statements were issued, Tedd received and accepted an IRS settlement offer of $450,000.

What amount of accrued liability should Tedd have reported in its December 31, 2005 balance sheet?

A

$400,000

When a contingent loss is both probable and estimable, it must be accrued. When only a range of amounts can be estimated, rather than a point estimate, the lowest amount in the range is accrued.

In this case, the low end of the range is $400,000, and that amount is accrued. The footnotes will describe the entire range to indicate the firm’s maximum exposure to loss. The actual settlement offer amount was unknown before the statements were issued.

750
Q

At December 31, 2005, Creole Co. was suing a competitor for patent infringement. The award from the probable favorable outcome could be reasonably estimated.

Creole’s 2005 financial statements should report the expected award as a:

A

Disclosure by footnote only.

Contingent gains are gains that are dependent on the outcome of a future event. This is a contingent gain because the outcome of the suit is not yet known. Contingent gains are not recognized in the accounts, but rather are footnoted at most.

751
Q

On November 1, 2004, Beni Corp. was awarded a judgment of $1,500,000 in connection with a lawsuit. The decision is being appealed by the defendant, and it is expected that the appeal process will be completed by the end of 2005.

Beni’s attorney feels that it is highly probable that an award will be upheld on appeal, but that the judgment may be reduced by an estimated 40%.

In addition to footnote disclosure, what amount should be reported as a receivable in Beni’s balance sheet at December 31, 2004?

A

$ -0-

This is a contingent asset or gain. Contingent assets are not recognized in the accounts.

This is a classic case of conservatism. If all the data were the same except that a loss was expected, a loss would be accrued in the accounts.

752
Q

During 2005, Smith Co. filed suit against West, Inc. seeking damages for patent infringement.

At December 31, 2005, Smith’s legal counsel believed that it was probable that Smith would be successful against West for an estimated amount in the range of $75,000 to $150,000, with all amounts in the range considered equally likely. In March 2006, Smith was awarded $100,000 and received full payment thereof.

In its 2005 financial statements, issued in February 2006, how should this award be reported?

A

As a disclosure of a contingent gain of an undetermined amount in the range of $75,000 to $150,000.

Contingent gains are not recognized in the accounts. At most, footnote disclosure is considered acceptable reporting. This is the best answer because no amount in the range of possible values is more likely than any other. The $100,000 amount was not known when the financial statements were published.

753
Q

During 2003, Manfred Corp. guaranteed a supplier’s $500,000 loan from a bank.

On October 1, 2004, Manfred was notified that the supplier had defaulted on the loan and filed for bankruptcy protection. Counsel believes Manfred will probably have to pay between $250,000 and $450,000 under its guarantee.

As a result of the supplier’s bankruptcy, Manfred entered into a contract in December 2004 to retool its machines so that Manfred could accept parts from other suppliers. Retooling costs are estimated to be $300,000.

What amount should Manfred report as a liability in its December 31, 2004, balance sheet?

A

$250,000

The retooling costs are not part of the liability, but are rather a response to changing business conditions. They most likely would be capitalized and amortized over their useful life. The liability is a contingent liability.

The amount depends on the outcome of the bankruptcy proceedings. When a range of values is estimated with no one value being more probable than the others, the lowest amount is accrued. Thus, $250,000 is accrued as of the end of 2004.

754
Q

For a range of equally probable amounts for a
provision, what amount is used for recognition
under International Financial Reporting
Standards (IFRS)?

A

Midpoint of the range

755
Q

What is meant by “more likely than not” with

respect to the probability for a provision?

A

Greater than 50%

756
Q

List examples of U.S. contingent liabilities
equivalent to provisions under International
Financial Reporting Standards (IFRS).

A

Warranty liability

Premium liability

757
Q

What is the definition of “provision” for
International Financial Reporting Standards
(IFRS)?

A

Liability that is uncertain in terms of timing and

amount but is not of uncertain existence

758
Q

When are contingent liabilities recognized
under International Financial Reporting
Standards (IFRS)?

A

They are not recognized.

759
Q

Choose the correct statement about international accounting standards as they relate to contingent liabilities and similar items.

A

A provision that has a reasonably possible chance of requiring the outflow of benefits is treated as a contingent liability.

A provision is a present obligation. This is one of the ways a liability can be treated as a contingent liability under international standards. If the provision involved a probable outflow, then it would be recognized, but would not be a contingent liability.

760
Q

Which of the following is a recognized liability for both international accounting standards and U.S. standards?

A

Obligation to provide rebates to customers, 90% probability of occurring.

For international accounting standards, this is a recognized provision. For U.S. standards, it is a recognized contingent liability.

761
Q

Which of the following is not a contingent liability under international accounting standards?

A

A provision with a 60% chance of requiring an outflow of benefits, amount is estimable.

A probable (> 50%) outflow of benefits is implied, and the amount is estimable. This is a recognized liability for international accounting standards, not a contingent liability.

762
Q

A firm considers its regular warranty liability to be an existing liability of uncertain amount. At year-end, the firm estimates that the amount required to extinguish its warranty liability in the future is in the range of $20 to $60 million, with no amount more likely than any other. Under the two sets of standards, what amount will be recognized?

International
U.S.

A

International - 40
U.S. - 20

International accounting standards recognize the midpoint, whereas U.S. standards recognize the low point.

763
Q

Is a noninterest-bearing note issued at a

premium or discount?

A

Discount

764
Q

What is the amount of the periodic payment for

an installment note issued at discount?

A

Face value divided by the annuity factor for the

term of the note and the stated rate on the note

765
Q

What is the amount of interest recognized for a
period on an installment note (one requiring
equal periodic payments that include both
principal and interest)?

A

Product of the effective rate at date of issuing
the note and the principal balance at the
beginning of the period

766
Q

What causes a premium on a note?

A

The yield rate is less than the stated rate.

767
Q

Define “discount on note” for a note exchanged

for cash and other privileges

A

The amount of interest revenue recognized

over the term of the note

768
Q

What is the amount of interest recognized for a
period on a note calling for a face amount due
at maturity, issued with an effective interest
rate not equal to the stated rate?

A

Product of the effective rate at date of issuing
the note and the principal balance at the
beginning of the period

769
Q

What is the distinction between notes payable

and accounts payable?

A
  1. Time period is usually extended.

2. Notes have an interest element.

770
Q

When is the straight-line method not allowed

for notes payable accounting?

A

Installment notes, and when the yield and

stated rates are greatly different

771
Q

List the two different methods of recording a

note for which a discount or premium occurs.

A
  1. Gross method

2. Net method

772
Q

What is the total interest expense recognized

on a noninterest-bearing note?

A

Total payments less amount borrowed

773
Q

What is the reported amount of a note calling
for a face amount due at maturity, issued with
an effective interest rate not equal to the stated
rate?

A

Present value of remaining cash flows

discounted at the effective rate

774
Q

What is the principal amount of a noninterestbearing

note?

A

Present value of the face amount discounted at

the yield rate on the note

775
Q

List the two different methods of amortizing a

discount or premium on a note.

A
  1. Effective interest method

2. Straight-line method

776
Q

What causes a discount on a note?

A

The yield rate is greater than the stated rate.

777
Q

What is the net note balance for a note issued

at a discount?

A

Face value less unamortized discount

778
Q

What is the amount borrowed on an

installment note issued at discount?

A

Product of the periodic payment and the
annuity factor for the term of the note and the
yield rate on the note

779
Q

Define “interest-bearing note payable.”

A

A note in which the interest element is explicitly

stated

780
Q

Pane Co. had the following borrowings on its books at the end of the current year:

$100,000, 12% interest rate, borrowed five years ago on September 30; interest payable March 31 and September 30.

$75,000, 10% interest rate, borrowed two years ago on July 1; interest paid April 1, July 1, October 1, and January 1.

$200,000, noninterest bearing note, borrowed July 1 of current year, due January 2 of next year; proceeds of $178,000.

What amount should Pane report as interest payable in its December 31 balance sheet?

A

$ 4,875

The interest payable is comprised of the following:

$100,000, 12% note (100,000 × .12 × 3/12) $3,000
$75,000, 10% note (75,000 × .10 × 3/12) 1,875
$4,875

There is no interest accrued on the noninterest bearing note because it is for less than one year.

781
Q

On September 30, World Co. borrowed $1,000,000 on a 9% note payable. World paid the first of four quarterly payments of $264,200 when due on December 30. In its December 31, balance sheet, what amount should World report as note payable?

A

$758,300

The first payment included interest of $22,500 (.09 × .25 × $1,000,000). Note that interest rates are always expressed for an annual period. Only 25% of year elapsed from Sept. 30 to the end of the year. The rest of the payment ($241,700 = $264,200 − $22,500) is principal. The note payable balance at Dec. 31 therefore is $758,300 ($1,000,000 − $241,700).

Expanded explanation:

The note payable has a stated interest rate of 9%,
payments are made quarterly,
first payment was issued after 3 months on December 30 in the amount of $264,200, and
the question is asking for the note payable amount to be recorded on the balance sheet on December 31 of the same year.
The payment made on December 30 is part interest, part principal. The amount to be recorded on the balance sheet on December 31 is principal only.

Interest for quarter 1 = $1,000,000 × 9% × ¼ = $22,500

Payment of $264,200 = $22,500 interest + x principal
Solve for principal amount = $241,700
Remaining note to be shown on balance sheet on December 31 is $1,000,000 ‒ $241,700 principal payment = $758,300
The next payment would be calculated in a similar manner; however, interest would be paid on the remaining principal balance of $758,300.

782
Q

A company issued a short-term note payable to a bank with a stated 12 percent rate of interest . The bank charged a .5% loan origination fee and remitted the balance to the company.

The effective interest rate paid by the company in this transaction would be

A

More than 12.5%

The .5% loan origination fee reduces the proceeds to the borrower AND increases the total interest to be paid by the same amount. The effect is to raise the interest rate above 12.5%. Assume the loan amount is $1,000 before the loan origination fee. Therefore, the net amount loaned is $995 [1 − .005($1,000)]. However, the full $1,000 must be paid at maturity. The total interest to be paid is thus increased by the $5 origination fee ($1,000 − $995).

For simplicity, assume the loan is for a full year. Then total interest paid is: .12($1,000) + $5 = $125.

The effective rate of interest for the year then is: $125/$995 = .1256. This exceeds 12.5%.

783
Q

At December 31, Year 1, a $1,200,000 note payable was included in Cobb Corp.’s liability account balances. The note is dated October 1, Year 1, bears interest at 15%, and is payable in three equal annual payments of $400,000. The first interest and principal payment was made on October 1, Year 2. In its December 31, Year 2 balance sheet, what amount should Cobb report as accrued interest payable for this note?

A

$30,000

The $30,000 answer is correct and equals ($1,200,000 − $400,000)(.15)(3/12). On 10/1/year 2, the first $400,000 principal payment was made. That left $800,000 of principal balance remaining on 10/1/year 2. The interest on that amount is computed as shown above.

784
Q

On December 30, 2005, Bart, Inc. purchased a machine from Fell Corp. in exchange for a non-interest bearing note requiring eight payments of $20,000. The first payment was made on December 30, 2005, and the others are due annually on December 30. At date of issuance, the prevailing rate of interest for this type of note was 11%. Present value factors are as follows:

Period Present value of
ordinary
annuity of 1 at 11% Present value of
annuity in
advance of 1 at 11%
7 4.712 5.231
8 5.146 5.712

On Bart’s December 31, 2005 balance sheet, the note payable to Fell was

A

$94,240

The note payable balance at the end of 2005 is the present value of a $20,000 ordinary annuity for seven periods.

The first payment reduced the liability immediately by the amount of the payment because it was due at the date the liability originated. The remaining seven payments begin one year from the December 31, 2005 balance sheet, making the annuity an ordinary annuity. Thus, the liability balance on that date is: $20,000(4.712) = $94,240.

785
Q

Seco Corp. was forced into bankruptcy and is in the process of liquidating assets and paying claims. Unsecured claims will be paid at the rate of forty cents on the dollar.

Hale holds a $30,000 noninterest-bearing note receivable from Seco collateralized by an asset with a book value of $35,000, and a liquidation value of $5,000.

The amount to be realized by Hale on this note is

A

$15,000

Bankruptcy law specifies that secured creditors are to be satisfied before any assets are paid to unsecured creditors. Hale is a secured creditor for the $5,000 liquidation value. A liquidation value is paid at the liquidation of the firm and thus acts as a secured debt. The remaining claim of $25,000 ($30,000 − $5,000) is unsecured and at the 40% rate yields an additional claim of $10,000, for a total amount to be realized of $15,000.

786
Q

Mann Corp.’s liability account balances at June 30, 20X3 included a 10% note payable in the amount of $3,600,000.

The note is dated October 1, 20X2, and is payable in three equal annual payments of $1,200,000 plus interest. The first interest and principal payment was made on October 1, 20X3.

In Mann’s June 30, 20X4 balance sheet, what amount should be reported as accrued interest payable for this note?

A

$180,000

On October 1, 20X3, the first payment was made. This payment included all interest due to that point, as well as the first $1,200,000 principal payment.

Thus, on that date, the liability balance is $2,400,000 (the remaining two principal payments). June 30, 20X4 is 9 months after this first payment date. The accrued interest for 9 months is $180,000 = $2,400,000(.10)(9/12).

787
Q

Loeb’s Corp. frequently borrows from the bank in order to maintain sufficient operating cash. The following loans were at a 12% interest rate, with interest payable at maturity. Loeb repaid each loan on its scheduled maturity date.

Date of loan Amount Maturity date Term of loan
11/1/X4 $ 5,000 10/31/X5 1 Year
2/1/X5 15,000 7/31/X5 6 Months
5/1/05 8,000 1/31/X6 9 Months
Loeb records interest expense when the loans are repaid. As a result, the interest expense of $1,500 was recorded in 2005. If no correction is made, by what amount would 2005 interest expense be understated?

A

$540

First loan:

Interest expense included in the $1,500 = $5,000(.12) = $600

Interest expense that should have been included in 20X5 interest expense = $5,000(.12)(10/12) = 500

Overstatement of 20X5 interest expense = $100

Second loan: (no error because the term is completely within 20X5 so the interest recognized equals the interest paid)

Third loan:

Interest expense included in the $1,500 = $0

(the loan and therefore the interest was not paid in 20X5) = 0

Interest expense that should have been included in 20X5 interest expense = $8,000(.12)(8/12) = 640

Understatement of 20X5 interest expense = $640

Net understatement = $540

788
Q

On October 1, 2005, Fleur Retailers signed a 4-month, 16% note payable to finance the purchase of holiday merchandise. At that date, there was no direct method of pricing the merchandise, and the note’s market rate of interest was 11%.

Fleur recorded the purchase at the note’s face amount. All of the merchandise was sold by December 1, 2005. Fleur’s 2005 financial statements reported interest payable and interest expense on the note for three months at 16%. All amounts due on the note were paid February 1, 2006.

Fleur’s 2005 cost of goods sold for the holiday merchandise was

A

Understated by the difference between the note’s face amount and the note’s October 1, 2005 present value.

The note (and merchandise) should have been recorded at its present value using the market interest rate of 11%. This rate is lower than the stated rate of 16% implying that the present value of the note (face value and interest payments at 16%) using 11% exceeds the face value of the note.

Thus, the merchandise was recorded at an amount which understated its market value. All the merchandise was sold before the end of the year causing cost of goods sold to be similarly understated.

789
Q

On October 1, 20X4, Fleur Retailers signed a 4-month, 16% note payable to finance the purchase of holiday merchandise.

At that date, there was no direct method of pricing the merchandise, and the note’s market rate of interest was 11%. Fleur recorded the purchase at the note’s face amount. All of the merchandise was sold by December 1, 20X4.

Fleur’s 20X4 financial statements reported interest payable and interest expense on the note for three months at 16%. All amounts due on the note were paid February 1, 20X5.

As a result of Fleur’s accounting treatment of the note, interest, and merchandise, which of the following items was reported correctly?

12/31/X4 retained earnings
12/31/X4 interest payable

A

12/31/X4 retained earnings - No
12/31/X4 interest payable - YES

Interest expense on notes should reflect the market interest rate at the date of issuing the note. This firm is recording interest expense at the 16% stated rate rather than the 11% market rate. Thus, interest expense is incorrectly recorded.

Also, the merchandise and note should have been recorded at the present value of the note using the market interest rate. Thus, cost of goods sold is incorrectly measured. The result of these two effects is that income and retained earnings are misstated.

However, interest payable reflects the stated rate which determines the cash amount to be paid. Interest payable is correctly stated.

790
Q

On September 1, 20X3, Brak Co. borrowed on a $1,350,000 note payable from the Federal Bank.

The note bears interest at 12% and is payable in three equal annual principal payments of $450,000. On this date, the bank’s prime rate was 11%. The first annual payment for interest and principal was made on September 1, 20X4.

At December 31, 20X4, what amount should Brak report as accrued interest payable?

A

$36,000

Although the question is not completely clear, interest is paid at the time each principal payment is made. Thus, on 9/1/X4, after the principal payment of $450,000 (and interest as well) is made, the remaining note balance is $900,000 ($1,350,000 − $450,000). Note that only the principal payment reduces the note balance. Interest for four months to 12/31/X4 is recorded in accrued interest payable: $36,000 ($900,000 × .12 × 1/3 year).

791
Q

Which of the following is reported as interest expense?

A

Imputed interest on noninterest-bearing note

A noninterest-bearing note actually causes interest to be paid by the maker. The term noninterest-bearing means that the note carries no stated rate of interest. The face value of the note includes interest however, and exceeds the principal amount of the note (the amount borrowed).

For example, a firm might borrow $4,000 and sign a $5,000 noninterest-bearing note. When the firm pays the $5,000 at maturity, it will be paying $1,000 in interest. The term “imputed” interest means that in computing interest expense, the amount of interest implied in the note (difference between face value and principal) is used to compute the interest rate for recognizing interest expense over the term of the note.

792
Q

On March 1, 20X4, Fine Co. borrowed $10,000 and signed a two-year note bearing interest at 12% per annum compounded annually. Interest is payable in full at maturity on February 28, 20X6.

What amount should Fine report as a liability for accrued interest at December 31, 20X5?

A

$2,320

The accrued interest covers the period from the borrowing to 12/31/X5 because no interest has yet been paid. The interest is also compounded (this is a stumbling point easily missed).

The 20X5 ending balance in accrued interest payable therefore includes interest on 20X4’s accrued interest:

20X4: $10,000(.12)(10/12) $1,000
20X5: ($10,000 + $1,000)(.12)(12/12) 1,320
Total accrued interest payable, December 31, 20X5 $2,320

793
Q

Describe three general aspects about the

valuation of all long-term liabilities.

A
  1. Initially recorded at the present value of
    future cash flows.
  2. Interest and amortization are recognized
    at the market interest rate on the date
    the liability was established.
  3. Interest expense equals the liability
    balance at the beginning of the period
    times the market rate of interest the date
    the liability was recorded.
794
Q

Define “bond.”

A

A financial debt instrument that typically calls
for the payment of periodic interest (although a
zero-coupon bond pays no interest), with the
principal being due at some time in the future

795
Q

Define “bond date.”

A

The first possible issuance date

796
Q

Define “issuance date.”

A

The date the bonds are actually issued

797
Q

Define “maturity date.”

A

The date the maturity value is paid, the end of

the bond term

798
Q

Define “secured bonds.”

A

Bonds that have a claim to specific assets

799
Q

Define “serial bonds.”

A

Bonds that mature at regular or staggered

intervals

800
Q

When are bonds sold at a premium?

A

When stated rate > market rate

801
Q

When are bonds sold at a discount?

A

When stated rate < market rate

802
Q

What method is required for premium/discount

amortization?

A

Effective interest method

803
Q

When the effective interest method of amortization is used for bonds issued at a premium, the amount of interest payable for an interest period is calculated by multiplying the

A

Face value of the bonds at the beginning of the period by the contractual interest rate.

The amount of interest paid each period on a bond issue is the product of the total face value and the contractual rate. A 4%, $1,000 bond pays $40 interest per year, for example. This answer is the same regardless of the amortization method used.

804
Q

On January 2, Vole Co. issued bonds with a face value of $480,000 at a discount to yield 10%. The bonds pay interest semiannually. On June 30, Vole paid bond interest of $14,400. After Vole recorded amortization of the bond discount of $3,600, the bonds had a carrying amount of $363,600. What amount did Vole receive upon issuing the bonds?

A

$360,000

The carrying value of the bonds after the first interest payment is $363,600 which reflects $3,600 of discount amortization. Because discount amortization increases the carrying value of the bond by reducing the discount, the carrying value (and amount received at issuance) must have been $3,600 less or $360,000 ($363,600 − $3,600). As discount is amortized, the carrying value gradually increases to the maturity (face) value over the bond term.

805
Q

On January 1, 20X5, Korn Co. sold to Kay Corp. $400,000 of its 10% bonds for $354,118 to yield 12%. Interest is payable semiannually on January 1 and July 1.

What amount should Korn report as interest expense for the six months ended June 30, 20X5?

A

$21,247

Interest expense = $354,118(.12/2) = $21,247

The yield rate is applied to the beginning book value of the bonds. The interest expense is for 1/2 a year. This requires that 1/2 of the yield rate be applied. The beginning book value is used because that is the debt 6 months before the interest is recognized.

Interest is defined as the true yield rate multiplied by the debt balance at the beginning of the interest period.

806
Q

What type of bonds in a particular bond issuance will not all mature on the same date?

A

Serial bonds

Serial bonds mature according to a schedule. For example, after 20 years, 10% of the bonds may be retired at the end of each of the next 10 years. The bond term ends at the end of the 30th year.

807
Q

A company issued a bond with a stated rate of interest that is less than the effective interest rate on the date of issuance. The bond was issued on one of the interest payment dates. What should the company report on the first interest payment date?

A

An interest expense that is greater than the cash payment made to bondholders.

When the yield rate (effective interest rate) exceeds the stated or coupon rate, the bond sells at a discount. For example, the only way a 5% bond can yield 6% is to sell below face value. The discount represents interest expense over and above the periodic cash interest paid because the full face value is paid at maturity. The discount is recorded as debit contra account to bonds payable. This extra amount of interest is recognized by amortizing the discount recorded at issuance. The journal entry for periodic interest is: dr. Interest Expense, cr. Discount, cr. Cash. In this way, interest expense exceeds the cash interest paid at each interest payment date.

808
Q

On January 1, 2004, Oak Co. issued 400 of its 8%, $1,000 bonds at 97 plus accrued interest.

The bonds are dated October 1, 2003 and mature on October 1, 2013. Interest is payable semiannually on April 1 and October 1. Accrued interest for the period October 1, 2003 to January 1, 2004, amounted to $8,000.

On January 1, 2004, what amount should Oak report as bonds payable, net of discount?

A

$388,000

The net book value of the bonds on the issuance date is $388,000 (.97 × $400,000). The three months of accrued interest collected on issuance increases the proceeds but does not affect the net bond liability. Accrued interest is reported separately from the net bond liability.

809
Q

When a bond is purchased, the present value of the bond’s expected net future cash inflows discounted at the market rate of interest provides what information about the bond?

A

Price.

The bond price is the present value of the future cash payments to be paid by the issuer over the bond term. These payments are (1) the face value paid at maturity and (2) the interest payments. Each interest payment is the product of the coupon rate for the appropriate portion of a year (usually six months) and the face value. The stream of interest payments is an annuity. Both the single payment (face value) and the annuity are discounted at the yield or market rate of interest. The result is the bond price.

810
Q

The following information pertains to Camp Corp.’s issuance of bonds on July 1, 20X5:

Face amount	$800,000	
Term	10 years	
Stated interest rate	6%	
Interest payment dates	Annually on July 1	
Yield	9%
                                                        At 6%	At 9% Present value of 1 for 10 periods	   0.558	0.422 Future value of 1 for 10 periods	     1.791	2.367 Present value of ordinary annuity  of 1 for 10 periods	                            7.360	6.418

What should be the issue price for each $1,000 bond?

A

$807

The issue price for one $1,000 face value bond is the present value of all future payments discounted at the yield rate of 9%.

Issue price = $1,000(.422) + .06($1,000)(6.418) = $807

811
Q

On January 1, year 1, Boston Group issued $100,000 par value, 5% five-year bonds when the market rate of interest was 8%. Interest is payable annually on December 31. The following present value information is available:

                                                   5%	              8% Present value of $1 (n = 5)	   0.78353	0.68058 Present value of an ordinary  annuity (n = 5)	                   4.32948	3.99271

What amount is the value of net bonds payable at the end of year 1?

A

$90,064

The bonds were issued at a discount because the market interest rate exceeds the stated rate. The net liability at the end of year 1 reflects the issue price at the beginning of the year plus the discount amortization for year 1. Discount is a contra bond payable account. When it is amortized, the contra account is reduced, thus increasing the net bond liability. The bond price is $100,000(.68058) + .05($100,000)(3.99271) = $88,022. Interest expense for year 1 is .08($88,022) = $7,042. The journal entry for the first interest payment is: dr. Interest expense 7,042; cr. Discount 2,042; cr. Cash 5,000. Net liability at end of year 1 = $88,022 + $2,042 = $90,064.

812
Q

On June 1, Greendale Corp. issued $700,000, five-year bonds at 8%, with interest payable annually on May 31. The bonds sold for $728,700 when the market rate of interest was 7%. Greendale uses the effective interest method for amortizing premiums on bonds payable. What is the balance of the premiums on bonds payable account immediately following the first interest payment?

A

$23,709

The initial balance of the premium is $28,700 ($728,700 issue price – $700,000 total face value). The premium balance is a credit because the premium is an adjunct account adding to the total bond liability. The journal entry for the first interest payment is:

Debit: Interest expense 728,700(.07) 51,009
Debit: Premium (derived amount) 4,991
Credit: Cash 700,000(.08) 56,000

The debit to the premium is a reduction in the balance. The firm recognizes less interest expense than the amount paid because the premium is not returned to the bondholders. The issuing firm pays only face value at maturity.

The balance in the premium account after this payment is $23,709 (= $28,700 – $4,991).

813
Q

How many months of interest are collected at
issuance when bonds are issued between
interest dates?

A

Number of months between the most recent

interest payment date and the date of issuance

814
Q

Define “bond issue costs.”

A

The costs of printing, registering, and

marketing the bonds

815
Q

How are bond issue costs accounted for?

A

The are treated as a direct reduction from the
debt carrying value and amortized to interest
expense over the term of the bonds.

816
Q

Define “stated rate.”

A

Rate listed on the bond and used to calculate

accrued interest and cash interest payments

817
Q

What is the length of a bond term when bonds

are issued between interest dates?

A

Period of time from issuance date to maturity

date

818
Q

Define “bond proceeds.”

A

The sum of the bond price and any accrued

interest

819
Q

On July 1, Year 1, Howe Corp. issued 300 of its 10%, $1,000 bonds at 99 plus accrued interest.

The bonds are dated April 1, year 1 and mature on April 1, year 11. Interest is payable semiannually on April 1 and October 1.

What amount did Howe receive from the bond issuance?

A

$304,500

The amount received is the price of the bonds plus interest from April 1, the bond date, to July 1, the issue date:

Amount received = .99(300)($1,000) + .10(3/12 year)(300)($1,000) = $304,500

The second term in the above calculation uses 3/12 of a year, which is the portion of a year between April 1 and July 1.

820
Q

A bond issued on June 1, Year 1, has interest payment dates of April 1 and October 1. The bond interest expense for the year ended December 31, year 1 is for a period of

A

Seven months.

The bonds have been outstanding seven months by the end of year 1. The firm has borrowed money for seven months. Therefore, seven months’ interest should be recognized in year 1.

Only six months of interest was PAID in year 1 because the bonds were issued after April 1 (one of the two interest payment dates per year), but that is not what the question asks.

821
Q

On October 1, Year 1, Brock, Inc. issued 200 of its 10%, $1,000 bonds at 101 plus accrued interest. The bonds are dated July 1, Year 1, and mature on July 1, Year 11. Interest is payable semiannually on January 1 and July 1.

At the time of issuance, Brock received cash of

A

$207,000.

The proceeds on a bond issue equal the total bond price plus accrued interest since the last interest date. The proceeds of $207,000 = 200($1,000)(1.01) + 200($1,000)(.10)(3/12).

The 3/12 factor is the time between the bond date (or previous interest date) and the issuance date.

822
Q

On January 31, Year 4, Beau Corp. issued $300,000 maturity value, 12% bonds for $300,000 cash. The bonds are dated December 31, Year 3, and mature on December 31, Year 10. Interest will be paid semiannually on June 30 and December 31.

What amount of accrued interest payable should Beau report in its September 30, Year 4 balance sheet?

A

$9,000

The interest payable is for the period July 1-September 30 (Year 4), a three-month period, or 1/4 of a year. Thus, the accrued interest payable is $9,000 ($300,000 × .12 × 1/4). The interest payment on June 30 paid all accrued interest to that date.

823
Q

A bond issued on June 1, 20X5, has interest payment dates of April 1 and October 1.

The bond interest expense for the year ended December 31, 20X5, is for a period of

A

Seven months.

Interest expense is recognized only during the bond term, which began June 1 when the bonds were issued. The bonds were outstanding 7 months in 20X5. Seven months of interest is recognized in 20X5.

824
Q

On December 31, 20X5. Cobb issued 2,000 of its 10%, $1,000 bonds at 99. The issuance price established a bond discount of $20,000. In connection with the sale of these bonds. Cobb paid the following expenses:

Legal and accounting fees $45,000
Printing of the prospectus 55,000
Underwriting fees 85,000

In Cobb’s December 31, 20X5, balance sheet, bond issue costs total

A

$185,000.

Legal and accounting fees $ 45,000
Printing of the prospectus 55,000
Underwriting fees 85,000
Total bond issue costs $185,000

Bond issue costs are those costs incurred to facilitate the issuance of the bonds. All of the above costs contribute to that effort.

825
Q

Dixon Co. incurred costs of $3,300 when it issued, on August 31, 20X5, five-year debenture bonds dated April 1, 20X5. Dixon uses the straight-line method to amortize bond issue costs. By what amount is 20X5 interest expense increased by the amortization of bond issue costs?

A

$240

There are four years and seven months in the bond term (5 years less the 5 months from April 1 to August 31) or a total of 55 months. Thus, the 20X5 amortization of bond issue costs, is $240 [(4/55)$3,300]. The bonds were outstanding four months in 20X5.

826
Q

On September 1, Year 1, Cobb Co. issued a note payable to the National Bank in the amount of $900,000, bearing interest at 12%, and payable in three equal annual principal payments of $300,000.

On this date, the bank’s prime rate was 11%. The first payment for interest and principal was made on September 1, Year 2.

At December 31, Year 2, Cobb should record accrued interest payable of

A

$24,000.

As of 12/31/Year 2, the first $300,000 principal installment has been paid, along with interest. This payment was made 9/1/Year 2. The remaining principal outstanding on that date is $600,000 ($900,000 − $300,000). Thus, accrued interest on 12/31/Year 2 is $24,000 = 600,000(.12)(4/12).

827
Q

On June 30, 20X5, Huff Corp. issued at 99, 1000 of its 8%, $1,000 bonds. The bonds were issued through an underwriter to whom Huff paid bond issue costs of $35,000.

On June 30, 20X5, Huff should report the bond liability at

A

$955,000.

This answer is correct. The $955,000 is the $1,000,000 face value less the $10,000 discount. The discount is computed as (1.00 − .99)($1,000,000) = $10,000, which is the face value less the bond price. The $990,000 is then reduced by the bond issue costs of $35,000. Another way to compute the net bond liability at issuance is to apply the unit bond price to the total face value: $990,000 = .99($1,000,000).

828
Q

On November 1, 20X5, Mason Corp. issued $800,000 of its 10-year, 8% term bonds dated October 1, 20X5. The bonds were sold to yield 10%, with total proceeds of $700,000 plus accrued interest. Interest is paid every April 1 and October 1. What amount should Mason report for interest payable in its December 31, 20X5 balance sheet?

A

$16,000

One month of accrued interest was collected from the bondholders at issuance for the period October 1—November 1, and interest for the next two months to December 31 was accrued.

Total accrued interest is $16,000 = $800,000(.08)(3/12).

829
Q

On April 1, 20X4, Hill Corp. issued 200 of its $1,000 face value bonds at 101 plus accrued interest. The bonds were dated November 1, 20X3, and bear interest at an annual rate of 9% payable semiannually on November 1 and May 1. What amount did Hill receive from the bond issuance?

A

$209,500

The total proceeds on the bond issue equal the total bond price plus accrued interest. The latter amount is the cash interest between the bond issuance date (April 1, 20X4), and the most recent interest payment date before the bond issuance date (November 1, 20X3), a period of 5 months. The most recent interest payment date in this case is the bond date.

Total bond price: 200($1,000)(1.01) $202,000
Plus accrued interest (5/12)($200,000)(.09) 7,500
Equals bond proceeds $209,500

830
Q

On January 2, Year 4, Gill Co. issued $2,000,000 of 10-year, 8% bonds at par. The bonds, dated January 1, Year 4, pay interest semiannually on January 1 and July 1. Bond issue costs were $250,000.

What amount of bond issue costs are unamortized at June 30, Year 5, assuming straight-line method?

A

$212,500

The bond term is 10 years. At 6/30/05, 8 1/2 years remain in the bond term. Thus, $212,500 [$250,000(8.5)/10] remain unamortized.

831
Q

On July 1, Year 5, Eagle Corp. issued 600 of its 10%, $1,000 bonds at 99 plus accrued interest. The bonds are dated April 1, Year 5 and mature on April 1, Year 15. Interest is payable semiannually on April 1 and October 1.

What amount did Eagle receive from the bond issuance?

A

$609,000

The total amount received, which is called proceeds on the bond issue, is:

.99($1,000)(600) + .10(3/12)(600)($1,000) = $609,000.

The first factor is the total bond price, exclusive of accrued interest. The second factor is the accrued interest since 4/1/Y5.

When bonds are issued between interest dates, the cash interest since the most recent past interest payment date must be collected from the bondholders because a full six months’ interest is paid on the following interest date.

832
Q

What is the international applicability of the

fair value option?

A

The option is limited to liabilities that are part
of a group with financial assets managed
together.

833
Q

What is the balance sheet effect of the fair

value option applied to financial liabilities?

A

Report liability at fair value.

834
Q

How is total interest expense for a bond issue
using an effective interest bond amortization
schedule (assume a premium) computed?

A

Sum of the cash interest column less sum of

amortization of premium column

835
Q

How is interest expenses on the current line of
an effective interest bond amortization
schedule computed, assuming semiannual
interest payments?

A

Multiply one-half the yield rate at date of
issuance by the book value of the bond issue on
the line above the current line.

836
Q

What is the income statement effect of the fair

value option applied to financial liabilities?

A

Recognize gain or loss for the change in the fair
value adjustment of the liability during the
period.

837
Q

What is the international treatment of debt

issue costs?

A

Reduction in the proceeds from the debt (same

as U.S, standards).

838
Q

Is the fair value option for financial liabilities

required, and to what securities is it applied?

A

It is an option (not required) and can be applied

to any and all financial liabilities.

839
Q

Foley Co. is preparing the electronic spreadsheet below to amortize the discount on its 10-year, 6%, $100,000 bonds payable. Bonds were issued on December 31 to yield 8%. Interest is paid annually. Foley uses the effective interest method to amortize bond discounts.

   A	   B	       C	         D	               E 1	Year	Cash  Interest  Discount         Carrying
             paid expense  amortization  amount 2	1	 	 	 	                         $86,580 3	2	$6,000	 

Which formula should Foley use in cell E3 to calculate the carrying amount of the bonds at the end of Year 2?
A

E2+D3

This question is a basic discount amortization scenario. The carrying amount of the bond will increase by the amount of the discount amortization each period until the carrying amount is $100,000 at maturity. Adding cells E2+D3 together will add the discount amortized for the period to the carrying amount of the bond, giving the new carrying amount at the end of the period.

840
Q

On January 1, year 2, Southern Corporation received $107,720 for a $100,000 face amount, 12% bond, a price that yields 10%. The bonds pay interest semiannually. Southern elects the fair value option for valuing its financial liabilities. On December 31, year 2, the fair value of the bond is determined to be $106,460. Southern recognized interest expense of $12,000 in its year 2 income statement. What was the gain or loss recognized on the year 2 income statement to report this bond at fair value?

A

$1,260 gain

Interest expense can be measured using various methods. In this situation, Southern recognized interest expense by debiting interest expense for $12,000 and crediting cash for $12,000, which represents the coupon interest paid on the bond. Therefore, interest expense was recognized on the income statement as a separate line item. The change in fair value from January 1, year 2, to December 31, year 2, would, therefore, be recognized as a gain or loss to revalue the bond’s carrying value to fair value. The change in value is calculated as beginning of year carrying value of $107,720 less end of year carrying value of 106,460, or $1,260. Since the value of the liability decreased, this indicates a gain of $1,260 that would be recognized on the year 2 income statement.

841
Q

On July 1, year 2, Marseto Corporation borrows $100,000 on a 10%, five-year interest-bearing note. At December 31, year 2, the fair value of the note is determined to be $97,500. Marseto elects the fair value option for reporting its financial liabilities. On its December 31, year 2 financial statements, what amounts should be presented for this note?

Interest Expense
Note Payable
Gain (Loss)

A

Interest Expense - $ 5,000
Note Payable - $ 97,500
Gain (Loss) - $ 2,500

If the fair value option is elected for reporting financial liabilities, the liability would be reported on the balance sheet at its fair value of $97,500, with a resulting gain of $2,500 ($100,000 − $97,500) on the income statement. The interest expense for the period would be calculated as $100,000 × 10% × 6/12 = $5,000.

842
Q

On February 1, year 1, Blake Corporation issued bonds with a fair value of $1,000,000. Blake prepares its financial statements in accordance with IFRS. What methods may Blake use to report the bonds on its December 31, year 1 statement of financial position?

I. Amortized cost.
II. Fair value method.
III. Fair value through profit or loss.

A

I and III only.

IFRS provides that financial liabilities may be reported at amortized cost or at the fair value through profit or loss (FVTPL). If FVTPL is elected, the resulting gain or loss is recognized in profit or loss for the period.

843
Q

How does an entity show intent to refinance

short-term obligations?

A

The intent must be proven, possibly in the form

of board of directors’ meeting minutes.

844
Q

List the three ways to meet the “ability to

refinance” requirement

A
  1. Actually refinance before issuance of the
    financial statements.
  2. Enter into a noncancelable refinancing
    agreement supported by a viable lender.
  3. Issue equity securities replacing the debt.
845
Q

List the criteria for reclassifying current

liabilities to long term.

A

The intent to refinance the short-term
obligation must be proven.
The firm must demonstrate the ability to
refinance the obligation.

846
Q

A company has the following liabilities at year end:

Mortgage note payable; $16,000 due within 12 months $355,000
Short-term debt that the company is refinancing with long-term debt 175,000
Deferred tax liability arising from depreciation 25,000
What amount should the company include in the current liability section of the balance sheet?

A

$16,000

Only the principal portion of the mortgage note is current. The short-term debt has been refinanced and reclassified as noncurrent. The deferred tax liability relating to depreciation is noncurrent.

847
Q

Witt Corp. has outstanding at December 31, 20x4 two long-term borrowings with annual sinking fund requirements and maturities as follows:

Sinking fund requirements	Maturities
20x4	$1,000,000	$ -
20x5	1,500,000	2,000,000
20x6	1,500,000	2,000,000
20x7	2,000,000	2,500,000
20x8	2,000,000	3,000,000
                $8,000,000	$9,500,000
               ==========	==========
In the notes to its December 31, 20x4, balance sheet, how should Witt report the above data?
A

The combined aggregate of $17,500,000 of maturities and sinking fund requirements detailed by year should be disclosed.

FASB accounting standards require the disclosure of the aggregate amount of maturities and sinking fund requirements for all long-term debt for each of the five years following the balance sheet date. The detail for each year is disclosed by the amount of both the sinking fund requirement and the maturity.

848
Q

On December 31, 20x4, Largo, Inc. had a $750,000 note payable outstanding, due July 31, 20x5.

Largo borrowed the money to finance construction of a new plant. Largo planned to refinance the note by issuing long-term bonds. Because Largo temporarily had excess cash, it prepaid $250,000 of the note on January 12, 20x5.

In February 20x5, Largo completed a $1,500,000 bond offering. Largo will use the bond offering proceeds to repay the note payable at its maturity and to pay construction costs during 2005.

On March 3, 20x5, Largo issued its 2004 financial statements.

What amount of the note payable should Largo include in the current liabilities section of its December 31, 20x4 balance sheet?

A

$250,000

The portion of the note paid between the balance sheet date and the date of issuing the financial statements is classified as current because current assets were used to retire that part of the note.

Even though proceeds from the bond issue may be used to replenish these funds, the $250,000 payment nonetheless reduced current assets. The remainder of the note is classified as noncurrent because the firm fully intends to refinance the remaining $500,000 with a long-term liability, and has shown its ability to do so by actually issuing bonds for the purpose of refinancing before issuing the financial statements.

Thus, the requirements for reclassifying the remaining $500,000 portion of the current note payable as noncurrent have been met.

849
Q

Cali, Inc., had a $4,000,000 note payable due on March 15, 20x6. On January 28, 20x6, before the issuance of its 20x5 financial statements, Cali issued long-term bonds in the amount of $4,500,000. Proceeds from the bonds were used to repay the note when it came due.

How should Cali classify the note in its December 31, 20x5, financial statements?

A

As a noncurrent liability, with separate disclosure of the note refinancing

The note was refinanced (replaced) with a long-term liability (bonds) between the balance sheet date and the balance sheet issuance date. This is one of the conditions under which a current liability can be reclassified as long term at the balance sheet date. Another is to enter into an irrevocable refinancing agreement.

The details of the refinancing must be disclosed in the notes. The note is not to be classified as a current liability because it will cause no reduction in current assets or increase in current liabilities during the coming period.

850
Q

Included in Lee Corp.’s liability account balances at December 31, 20x4, were the following:

14% note payable issued October 1, 20x4, maturing September 30, 20x5 $125,000
16% note payable issued April 1, 20x3, payable in six equal annual installments of $50,000 beginning April 1, 20x3 200,000
Lee’s December 31, 20x4 financial statements were issued on March 31, 20x5.

On January 15, 20x5, the entire $200,000 balance of the 16% note was refinanced by issuance of a long-term obligation payable in a lump sum. In addition, on March 10, 20x5, Lee consummated a noncancelable agreement with the lender to refinance the 14%, $125,000 note on a long-term basis, on readily determinable terms that have not yet been implemented.

Both parties are financially capable of honoring the agreement, and there have been no violations of the agreement’s provisions.

On the December 31, 20x4 balance sheet, the amount of the notes payable that Lee should classify as short-term obligations is

A

$0

Current liabilities that are refinanced before the issuance of the financial statements can be reclassified as noncurrent provided they meet the requirements.

Essentially, as long as the firm actually refinances the liability on a noncurrent basis before issuing the financial statements, or it enters into a non-cancelable agreement to do so (again before issuing the financial statements), then the liability can be reclassified. Thus, both listed liabilities will be classified as noncurrent in the 20x4 balance sheet.

851
Q

A company has outstanding accounts payable of $30,000 and a short-term construction loan in the amount of $100,000 at year end. The loan was refinanced through issuance of long-term bonds after year end but before issuance of financial statements. How should these liabilities be recorded in the balance sheet?

A

Current liabilities of $30,000, long-term liabilities of $100,000

The accounts payable is a current liability. The refinancing resulted in reclassifying the construction loan (which would otherwise also be current) as a noncurrent liability. The replacement of the loan with a noncurrent liability (bonds) took place before the financial statements were issued, thus meeting the requirements for refinancing on a long-term basis.

852
Q

Ames, Inc. has $500,000 of notes payable due June 15, 20x3.

Ames signed an agreement on December 1, 20x2, to borrow up to $500,000 to refinance the notes payable on a long-term basis with no payments due until 20x4. The financing agreement stipulated that borrowings may not exceed 80% of the value of the collateral Ames was providing.

At the date of issuance of December 31, 20x2, financial statements, the value of the collateral was $600,000 and is not expected to fall below this amount during 2003.

In Ames’ December 31, 20x2, balance sheet, the obligation for these notes payable should be classified as

Short term
Long term

A

Short term - $ 20,000
Long term -$ 480,000

Ames has successfully refinanced a short-term note on a long-term basis before the issuance of the 20x2 financial statements.

The borrowings on a long-term basis are limited to $480,000 (.80 × $600,000 collateral provided by Ames). Thus, $480,000 of the note is reclassified as long term, and the remaining $20,000 of the note is classified as current.

853
Q

On December 31, year 1, Taylor, Inc. signed a binding agreement with a bank for the refinancing of an existing note payable scheduled to mature in February, year 2. The terms of the refinancing included extending the maturity date of the note by three years. On January 15, year 2, the note was refinanced. How should Taylor report the note payable in its December 31, year 1 balance sheet?

A

A long-term liability.

This is a short-term note that is scheduled to mature within one year. This type of note is excluded from current liabilities and shown as a long-term liability if both the following conditions are met:

1) The company intends to refinance and,
2) The company demonstrates an ability to refinance.

One way a company can demonstrate the ability to refinance is by entering into a financing agreement that permits the company to refinance on a long term basis. Taylor has entered in to such an agreement.

854
Q

Verona Co. had $500,000 in short-term liabilities at the end of the current year. Verona issued $400,000 of common stock subsequent to the end of the year, but before the financial statements were issued. The proceeds from the stock issue were intended to be used to pay the short-term debt. What amount should Verona report as a short-term liability on its balance sheet at the end of the current year?

A

$100,000

There is no net use of current assets to liquidate the $400,000 amount of the liabilities because the stock issuance provided the necessary cash. Only the remaining $100,000 is classified as current.

855
Q

On December 31, 20x2, Paxton Co. had a note payable due on August 1, 20x3. On January 20, 20x3, Paxton signed a financing agreement to borrow the balance of the note payable from a lending institution to refinance the note. The agreement does not expire within one year, and no violation of any provision in the financing agreement exists. On February 1, 2003, Paxton was informed by its financial advisor that the lender is not expected to be financially capable of honoring the agreement. Paxton’s financial statements were issued on March 31, 20x3. How should Paxton classify the note on its balance sheet at December 31, 20x2?

A

As a current liability because the lender is not expected to be financially capable of honoring the agreement.

This is a short-term note that is scheduled to mature within one year. This type of note is excluded from current liabilities and shown as a long-term liability if both the following conditions are met:

1) The company intends to refinance and,
2) The company demonstrates an ability to refinance.

The financial statements were issued after Paxton was informed that the financial institution would not be capable of honoring the commitment. Therefore, there is no intention or ability in existence when the financial statements were issued. As a result, this would be shown as a current liability.

856
Q

Willem Co. reported the following liabilities at December 31, 20x1:

Accounts payable-trade $ 750,000
Short-term borrowings 400,000
Mortgage payable, current portion $100,000 3,500,000
Other bank loan, matures June 30, 20x2 1,000,000
The $1,000,000 bank loan was refinanced with a 20-year loan on January 15, 20x2, with the first principal payment due January 15, 20x3. Willem’s audited financial statements were issued February 28, 20x2.

What amount should Willem report as current liabilities at December 31, 2001?

A

$1,250,000

The $1,000,000 loan was successfully refinanced on a long-term basis and therefore was moved to the noncurrent liability category. The refinancing took place before the financial statements were issued, thus meeting the requirements for reclassification on a long-term basis. The remaining items are all current: $750,000 accounts payable + $400,000 short term borrowings + $100,000 current portion of mortgage payable = $1,250,000 total current liabilities.

857
Q

List the conditions that must exist for debt to

be extinguished.

A

Debtor pays creditor and is relieved of
obligation.
Debtor is legally released from being
primary obligor.

858
Q

How are gains/losses from extinguishment of

debt reported on the income statement?

A

Recognized as components of income from

continuing operations

859
Q

List the steps to retire debt on the books

A
Record interest, amortization of
discount/premium, issue costs to date of
retirement.
Remove debt and related accounts.
Record gain or loss.
860
Q

When determining whether a debt
extinguishment results in a gain or a loss, how
is that calculated?

A

Gain/Loss = Reacquisition price - Net Carrying

Amount

861
Q

When determining whether a debt
extinguishment results in a gain or a loss, what
are the financial statement adjusting items?

A
Reacquisition items to be accounted for:
Debt issue costs
Any unamortized discount/premium
Difference between debt's face value and
reacquisition amount
862
Q

A 15-year bond was issued in year 5 at a discount. During year 15, a 10-year bond was issued at face amount with the proceeds used to retire the 15-year bond at its face amount.

The net effect of the year 15 bond transactions was to increase long-term liabilities by the excess of the 10-year bond’s face amount over that of the 15-year bond’s:

A

Carrying amount.

The proceeds of the second bond issue, which equal the second issue’s carrying value and face value, were used to retire the first issue at that same amount.

However, the first issue has a lower carrying value than the face value because of the unamortized discount. Therefore, the net increase in long-term debt is the amount of the unamortized discount, which also equals the difference between the carrying values of the two issues.

863
Q

On January 1, year 1, Fox Corp. issued 1,000 of its 10%, $1,000 bonds for $1,040,000. These bonds were to mature on January 1, year 11 but were callable at 101 any time after December 31, year 4. Interest was payable semiannually on July 1 and January 1.

On July 1, year 6, Fox called all of the bonds and retired them.

The bond premium was amortized on a straight-line basis. Before income taxes, Fox’s gain or loss in year 6 on this early extinguishment of debt was

A

$8,000 gain

The portion of the bond term that remains is 4 1/2 years as of July 1, year 6, because the bonds have been outstanding for 5 1/2 years as of that date. Therefore, the book value of the bonds on July 1, year 6 equals the face value of the bonds ($1,000,000) plus the unamortized bond premium of $18,000 = (4.5/10)$40,000, for a total of $1,018,000.

The gain on the bond extinguishment is the difference between the book value and the amount paid to extinguish the bonds: $1,018,000 − 1.01($1,000,000) = $8,000. The gain results because it cost Fox less to retire the bonds than the book value of the bonds.

864
Q

Gains or losses from the early extinguishment of debt, if material, should be

A

Recognized in income from continuing operations in the period of extinguishment.

The gain or loss on retirement of debt is included in income from continuing operations.

865
Q

On June 30, 2005, Town Co. had outstanding 8%, $2,000,000 face amount, 15-year bonds that matured on June 30, 2015. Interest is payable on June 30 and December 31.

The unamortized balances in the bond discount and deferred bond issue costs accounts on June 30, 2005 were $70,000 and $20,000, respectively. On June 30, 2005, Town acquired all these bonds at 94 and retired them.

What net carrying amount should be used in computing gain or loss on this early extinguishment of debt?

A

$1,910,000

The journal entry for retirement:

Dr. Bonds payable	2,000,000	
  Cr.Bond discount		70,000
  Cr.Bond issue costs		20,000
  Cr.Cash .94($2,000,000)		1,880,000
  Cr.Gain		30,000

The gain is the difference between (1) the net bond liability less the unamortized bond issue costs and (2) the amount paid to retire the bonds:

Net bond liability: $2,000,000 − $70,000 $1,930,000
Less unamortized bond issue costs (20,000)
Net carrying amount to use in computing gain or loss $1,910,000
Less amount paid to retire bonds (1,880,000)
Gain $30,000
The correct answer is $1,910,000.

866
Q

On June 2, year 1, Tory, Inc. issued $500,000 of 10%, 15-year bonds at par. Interest is payable semiannually on June 1 and December 1. Bond issue costs were $6,000. On June 2, year 6, Tory retired half of the bonds at 98.

What is the net amount that Tory should use in computing the gain or loss on the retirement of debt?

A

$248,000

The question asks for the book value amount to be compared to the price paid for the bonds retired when the gain or loss on retirement is computed.

The net book value includes the unamortized bond issue costs. The amount of unamortized issue costs relating to the portion of the bond issue retired increases the loss or decreases the gain. The remaining portion of the bond term for the portion of the bond issue retired is the period for which the bond issue costs were not amortized.

Number of semiannual periods in bond term: 15(2) = 30
Number of semiannual periods remaining at 6/2/year 6 = 10(2) = 20
Remaining unamortized bond issue costs: $6,000(20/30) = $4,000
Net amount to compare to price paid for bonds, to determine gain or loss on retirement, on one-half the bond issue: (1/2)($500,000 − $4,000) = $248,000. A journal entry recording the retirement of one-half the issue helps show why $248,000 is the correct answer.

Bonds payable	250,000	
Bond issue costs		2,000
Cash .98($250,000)		245,000
Gain		3,000
The gain equals the net value of two accounts removed from the books ($248,000) less the amount paid to retire the bonds.
867
Q

(Note: This AICPA question has been amended.)

On January 1, year 15, Hart, Inc. redeemed its 15-year bonds of $500,000 par value for 102.

They were originally issued on January 1, year 3 at 98 with a maturity date of January 1, year 18.

The bond issue costs relating to this transaction were $20,000. Hart amortizes discounts, premiums, and bond issue costs using the straight-line method.

What amount of loss should Hart recognize on the redemption of these bonds?

A

$16,000

The journal entry for retirement:

Dr. Bonds payable 500,000
Dr. Loss 16,000
Cr. Bond Discount 2,000 .02($500,000) (3/15)
Cr. Bond Issue Costs 4,000 $20,000(3/15)
Cr. Cash 510,000 $500,000(1.02)

The bond term is 15 years. Retirement is 3 years before maturity. Therefore, under the straight-line method, 3/15 of both the total bond discount and bond issue costs would remain unamortized at the retirement date. These amounts are removed along with the face value of the bonds (bonds payable account).

The original discount was 2% of $500,000. The bond issue costs are removed because they no longer have any future benefit.

868
Q

Weald Co. took advantage of market conditions to refund debt. This was the fifth refunding operation carried out by Weald within the last four years.

The excess of the carrying amount of the old debt over the amount paid to extinguish it should be reported as a(an)

A

Part of continuing operations.

This gain is included in income from continuing operations, as would other gains and losses such as on equipment disposal.

869
Q

On July 31, year 1, Dome Co. issued $1,000,000 of 10%, 15-year bonds at par and used a portion of the proceeds to call its 600 outstanding 11%, $1,000 face-value bonds, due on July 31, year 15, at 102. On that date, the unamortized bond premium relating to the 11% bonds was $65,000.

In its year 1 income statement, what amount should Dome report as a gain or loss, before income taxes, from the retirement of the bonds?

A

$53,000 gain

Book value of bonds retired $600,000 + $65,000 $ 665,000
Less total market value of bonds retired 600($1,000)(1.02) (612,000)
Equals gain on retirement $ 53,000
The unamortized premium is a component of the book value at retirement. A premium increases the net book value of the bonds because more was paid in than the face value when the bonds were originally issued.

When a liability is retired for less than its book value, a gain is recorded because the firm reduces its liabilities more than the reduction in its cash or other assets used for retirement.

870
Q

Describe the post-restructure interest rate for a
troubled debt restructure that modifies the
terms of the original debt such that the sum of
restructured cash flows is greater than the
book value of the original debt.

A

Rate that equates the book value of the original
debt with the present value of restructured
cash flows

871
Q

Describe the debtor’s recording of a settlement

restructure

A
Gain = Book value of debt + Unpaid
accrued interest – Market value of
consideration transferred
Gain/loss on disposal of assets
transferred
Remove debt from books.
Record any stock issued at market value.
872
Q

What is the amount of interest to be recognized
after a troubled debt restructure modifies the
terms of the original debt such that the sum of
restructured cash flows is greater than the
book value of the original debt?

A

Difference between the sum of restructured
cash flows and the book value of the original
debt

873
Q

What aspect of a debt restructuring must be

present for the restructuring to be “troubled”?

A

Creditor makes a concession, and debtor must

be in financial difficulty.

874
Q

What is the subsequent accounting treatment
for restructured cash flows in a troubled debt
restructure that modifies the terms of the
original debt such that the sum of restructured
cash flows is less than the book value of the
original debt?

A

They are all treated as principal payments.

875
Q

What is the amount of interest to be recognized
after a troubled debt restructure that modifies
the terms of the original debt such that the sum
of restructured cash flows is less than the book
value of the original debt?

A

No interest is recognized; all payments are

considered principal payments.

876
Q

List the two categories of modification of terms
debt restructures for international accounting
standards.

A
  1. Significant modification

2. Not significant modification

877
Q

What is the international accounting standard
treatment of settlement troubled debt
restructures?

A

Same as U.S. accounting but is considered an

extinguishment

878
Q

For a troubled debt restructuring involving only a modification of terms, which of the following items specified by the new terms would be compared to the carrying amount of the debt to determine if the debtor should report a gain on restructuring?

A

The total future cash payments

In a troubled debt restructuring involving only a modification of terms, the debtor will recognize a gain only if the total undiscounted future cash payments for principal and interest under the new terms are less than the current amount payable for principal and accrued interest.

When the future payments under the new terms are less than the current obligation, the debtor writes down the carrying amount of the liability by the amount of the difference and thus recognizes a gain.

879
Q

In a modification of the terms, troubled debt restructure of type II (sum of new flows > book value of debt), what amount of gain is recognized by the debtor?

A

No gain is recognized.

Although the debtor has an economic gain (the creditor is making a concession), GAAP requires that the debtor compute the new rate of interest based on the restructured cash flows and recognize interest expense over the note term. No gain is recognized by the debtor.

880
Q

On 12/31/x1, DInc. owed CInc. the full face value of a 10%, $350,000 note that requires interest payments annually on Dec. 31. DInc. paid the interest due 12/31/x1, but is experiencing financial problems and requested that the loan agreement be restructured. Three years remain in the note term as of today. The two parties agree to the following restructuring agreement:

DInc. will pay no more interest.

DInc. will pay $196,270 one year from today, and that same amount again two years from today (total of two payments of $196,270).

What amount of interest expense will DInc. recognize on 12/31/x2 when the firm makes the first of two payments of $196,270?

Factor for PV Ordinary Annuity for 2 periods at 8% = 1.78326
Factor for PV Annuity Due for 2 periods at 8% = 1.92593
Factor for Single Sum for 2 periods at 8% = .85734

A

28,000

The sum of restructured flows (2 × $196,270) exceeds $350,000, which requires that a new interest rate (m) be computed, as follows: $350,000 = $196,270(pva, m, 2). $350,000/$196,270 = (pva, m, 2) = 1.78326. The new rate (m) corresponds to 8%. Interest expense at the end of 20x2 is computed as .08($350,000) = $28,000.

881
Q

The following information pertains to the transfer of real estate pursuant to a troubled debt restructuring by Knob Co. to Mene Corp. in full liquidation of Knob’s liability to Mene:

Carrying amount of liability liquidated $150,000
Carrying amount of real estate transferred 100,000
Fair value of real estate transferred 90,000
What amount should Knob report as an ordinary gain (loss) on transfer on disposal?

A

($10,000)

The real estate transferred in settlement of the debt is worth $10,000 less than its carrying value. Thus, an ordinary loss is recognized on the transfer. It is as if Knob first sold the real estate for $90,000 causing a loss of $10,000, and then used the cash for settlement of the debt.

882
Q

Nu Corp. agreed to give Rand Co. a machine in full settlement of a note payable to Rand. The machine’s original cost was $140,000. The note’s face amount was $110,000. On the date of the agreement,

the note’s carrying amount was $105,000, and its present value was $96,000.
The machine’s carrying amount was $109,000, and its fair value was $96,000.
What amount of net gain (or losses) should Nu recognize?

A

($4,000)

The net loss listed is the difference between the carrying amount of the liability and the carrying amount of the machine. Nu has a gain of $9,000 on the note settlement, which is the difference between the liability carrying value ($105,000) and the fair value of the consideration given to extinguish the debt ($96,000). Nu also has a disposal loss on the machine. An ordinary loss of $13,000 is recognized and equals the difference between the machine’s carrying value ($109,000) and its fair value ($96,000).

883
Q

A debtor and a creditor have negotiated new terms on a note. How can you determine whether the restructuring is a troubled debt restructure?

A

If the present value of the restructured flows using the original interest rate is less than the book value of the debt at the date of the restructure.

This is one of the ways to determine if a restructuring is troubled. Under the terms of this answer, the creditor is receiving a stream of cash flows with a present value less than what is currently owed and is making a concession.

884
Q

The following information pertains to the transfer of real estate pursuant to a troubled debt restructuring by Knob Co. to Mene Corp. in full liquidation of Knob’s liability to Mene:

Carrying amount of liability liquidated $150,000
Carrying amount of real estate transferred 100,000
Fair value of real estate transferred 90,000
What amount should Knob report as a gain (loss) on restructuring of payables?

A

$60,000

The gain on debt restructure recognized by the debtor Knob is the difference between the book value of the debt ($150,000) and the market value of the asset transferred in settlement ($90,000). Thus, the gain equals $60,000.

It represents the increase in net worth, measured at market value, from extinguishing debt at less than its carrying value. The carrying value of the asset transferred is not relevant to the determination of the restructuring gain.

885
Q

Choose the correct statement regarding the accounting treatment of troubled debt restructures (TDRs) under international accounting standards (IAS).

A

Settlements are treated the same way as under U.S. standards.

Both sets of standards treat settlements as extinguishments with a gain to the debtor for the difference between debt book value and fair value of consideration paid.

886
Q

Wood Corp., a debtor-in-possession under Chapter 11 of the Federal Bankruptcy Code, granted an equity interest to a creditor in full settlement of a $28,000 debt owed to the creditor. At the date of this transaction, the equity interest had a fair value of $25,000.

What amount should Wood recognize as a gain on restructuring of debt?

A

$3,000

The gain is the difference between the debt book value of $28,000 and the fair value of the equity interest transferred ($25,000) or $3,000.

887
Q

On December 30, 20x4, Hale Corp. paid $400,000 cash and issued 80,000 shares of its $1 par value common stock to its unsecured creditors on a pro rata basis pursuant to a reorganization plan under Chapter 11 of the bankruptcy statutes. Hale owed these unsecured creditors a total of $1,200,000. Hale’s common stock was trading at $1.25 per share on December 30, 20x4.

As a result of this transaction, Hale’s total stockholders’ equity had a net increase of

A

$800,000

The market value of the stock issued to the creditors is $100,000 (80,000 × $1.25). The fair value of consideration paid to settle the debt therefore is $500,000 ($400,000 cash + $100,000 of stock). The gain on settling the debt therefore is $700,000 ($1,200,000 − $500,000 total consideration).

The gain increases owners’ equity by way of net income. The issuance of stock is recorded at market value, $100,000. Thus, the total owners’ equity increase is $800,000 ($700,000 + $100,000). Note that this amount is also the difference between the amount of debt retired ($1,200,000) and cash paid ($400,000).

888
Q

During year 2, Colt Co. experienced financial difficulties and was likely to default on a $1,000,000, 15%, three-year note dated January 1, year 1, payable to Cain National Bank.

On December 31, year 2, the bank agreed to settle the note and unpaid year 2 interest of $150,000 for $820,000 cash payable on January 31, year 3.

What is the amount of gain, before income taxes, from the debt restructuring?

A

$330,000

Gain = book value of note plus interest − cash paid Gain = $1,000,000 + $150,000 − $820,000 = $330,000.

889
Q

Define “debt covenant.”

A

Restriction on debtor and possible responses

by creditor

890
Q

Define covenant-lite loan.

A

Loan with less stringent restrictions

891
Q

What is the classification of liabilities that are

due on demand?

A

Current liability

892
Q

What is the classification of a liability callable
on demand if the debt covenant is violated and
it is probable that the debtor will cure
violation?

A

Noncurrent liability

893
Q

Define “debt covenant compliance.”

A

Steps taken by a debtor to meet the restriction

and reporting such compliance

894
Q

What is the classification of a liability subject to

a subjective acceleration clause?

A

Current if it is possible the debt will be called;
noncurrent if a remote chance exists of calling
the debts

895
Q

Give an example of a response by a creditor if

the debt covenant is violated by the debtor.

A

Require the debtor to pay the debt or refinance

the debt.

896
Q

List some examples of specific attributes in a

covenant.

A

Minimum current ratio

Maximum debt to equity ratio

897
Q

Choose the correct statement concerning the classification of a liability when a firm is subject to a debt covenant

A

If the covenant includes a subjective acceleration clause and there is only a remote chance that debt will be called, then the liability is classified as noncurrent.

It must be at least possible that the liability will be called in order for the classification to be downgraded to current.

It must be probable that the violation will be cured for the classification to be noncurrent.

Whether to call the debt is the option of the creditor in this case. If the violation is waived (forgiven), then the debt will not be called and the liability can remain in the noncurrent category.

Liabilities may be callable on demand only if the debtor violates a debt covenant, for example.

898
Q

A firm’s debt to equity ratio (total debt to total owners’ equity) cannot exceed 3.0 without allowing a major creditor to call a loan to the firm. The ratio is currently at the maximum before any of the transactions are listed. Which of the following transactions would not subject the firm to an immediate call by the creditor?

A

Retire a different loan by issuing common stock.

This transaction reduces total liabilities and increases OE by the same amount. The numerator of the ratio is reduced and the denominator is increased. Both factors cause the ratio to decrease below the maximum.

A deferred tax liability increase causes the numerator of the ratio to increase, thus increasing the ratio. It can also be argued that an increase in the deferred tax liability is associated with an increase in income tax expense, which decreases earnings, retained earnings, and owners’ equity. The decrease in the denominator also serves to increase the ratio.

Any treasury stock purchase reduces the total OE by the amount paid. There is no effect on debt. The denominator of the debt to equity is reduced, causing the ratio to increase.

899
Q

Which of the following actions helps a firm to maintain compliance with a debt covenant that includes a minimum current ratio and a minimum retained earnings balance: (1) refinancing current debt on a long-term basis, (2) appropriating retained earnings, (3) purchasing treasury stock, (4) declaring cash dividends.

A

1 and 2

Both actions are appropriate. Refinancing current debt on a long-term basis reduces current liabilities (increases noncurrent debt) and increases the current ratio. Appropriating retained earnings is an action that typically signals a future reduction in dividends, albeit on a temporary basis. As a result, total retained earnings is maintained at a higher level.

However, declaring cash dividends increases current liabilities, decreases the current ratio, and reduces retained earnings. Both quantitative measures used in the covenant deteriorate as a result.

However, purchasing treasury stock reduces current assets and the current ratio.

900
Q

Which of the following accounting strategies (for financial reporting purposes) is the least likely for a firm that is currently only marginally fulfilling the quantitative measures (all involving earnings) of its debt covenants?

A

Changing to the successful efforts method of accounting for natural resource exploration costs.

This accounting change would cause earnings to be reduced, possibly significantly. The successful efforts method expenses the cost of all unsuccessful exploration efforts immediately. Full costing capitalizes the cost of all efforts of expensing that take place when the resource is sold. Thus, the full costing method is the method of choice. This firm should use accounting methods that increase earnings. Higher earnings improve several quantitative measures used in debt covenants, including any measure of earnings, owners’ equity balances, debt to equity ratio, and rates of return such as return on assets and return on equity.

Straight-line depreciation consistently results in the highest annual earnings. This is the reason most firms use some form of this method. This choice causes earnings to be reduced the least amount, thus reducing the risk of noncompliance.

For most firms, FIFO yields the lowest cost of goods sold, and therefore the highest earnings. Increasing earnings reduces the risk of noncompliance for this firm.

The specific method first capitalizes the interest on specific debt. With interest rates on the decline, interest on lower rate debt is capitalized and more is expensed, relative to the weighted average method, which capitalizes at the average rate over all debt. The weighted average method would capitalize more interest on older (higher interest) rate debt, thus reducing the current amount of interest expense and increasing earnings. Increasing earnings reduces the risk of noncompliance for this firm.

901
Q

A firm is required by its creditors to maintain a 2.00 (or greater) current ratio in order to maintain compliance with a debt covenant. The current ratio of the firm is currently at the minimum before any of the transactions are listed. Which of the following actions would cause the firm to fall out of compliance?

A

Declare cash dividends.

This transaction increases current liabilities, thus reducing the current ratio. The current ratio is current assets divided by current liabilities. There is no effect on current assets.

This transaction increases current assets and therefore the current ratio. The current ratio is current assets divided by current liabilities. There is no effect on current liabilities.

This transaction reduces both current assets and current liabilities by the same amount. Because current liabilities are less than current assets, the denominator of the current ratio decreases by a larger percentage causing the ratio to increase. The current ratio is current assets divided by current liabilities.

Both current assets and liabilities (dividends payable) are reduced by the amount paid, but current liabilities are reduced a greater percentage, which causes the current ratio to increase.

902
Q

When are mandatorily redeemable financial

instruments classified as debt?

A

When they are obligations to repurchase equity
shares and require the issuer to settle the
obligation by transferring assets

903
Q

How should an obligation to issue fixed number

of shares be classified?

A

The classification is equity.

904
Q

How is an obligation to issue shares of a fixed

dollar value valued?

A

The fixed dollar value agreed upon by the

parties

905
Q

How is an obligation to issue a fixed number of

shares valued?

A

Fair value of shares at the time of agreement

906
Q

How is an option written by a firm allowing
other entities to purchase the firm’s stock at a
fixed price classified?

A

The classification is liability.

907
Q

Which aspect of a compound financial
instrument with characteristics of both debt
and equity is measured first under
International Financial Reporting Standards
(IFRS)?

A

The aspect is debt

908
Q

How should an obligation to issue shares of a

fixed dollar value be classified?

A

The classification is liability

909
Q

Early in 20x3, Shifter, Inc. wrote put options for 1,000 shares of its common stock. Purchasers of the options can sell Shifter stock back to Shifter for $20 per share on 12/31/x3. The estimated fair value of each option is $2 at the time of sale. At 12/31/x3, the share price is $15 and the options are exercised. As a result, Shifter

A

Recognizes a $3,000 loss.

Shifter paid $5,000 more for the treasury stock than its fair value: 1,000 shares × ($20 − $15). The $2,000 fee (1,000 × $2) offsets that loss yielding a net loss of $3,000.

910
Q

Allam, Inc. contracted for services to be provided over a period of time with full payment in Allam’s $2 par common stock when the service is completed. At the time of the agreement, Allam stock was trading at $20 per share. The agreed-upon total value of the contract is $20,000. When the service was completed, Allam’s stock price was $25 per share. Therefore, Allam

A

Increases the common stock account $1,600.

The value of the stock to be issued is $20,000. At time of issuance, the stock price is $25. Therefore, 800 shares are issued ($20,000/$25). The par value of the stock is $2, requiring a credit of $1,600.

911
Q

Choose the correct statement concerning transactions involving the issuance of shares in payment of obligations for goods and services.

A

When the value of shares to be issued is fixed, the number of shares to be issued is variable.

The value of the shares to be issued is the value of the contract or the agreed-upon value of the goods and services for the two parties. If the stock price changes between the time of contracting and the delivery of the goods or services, the number of shares changes in order to provide that fixed value.

912
Q

A firm selling put options to sell the firm’s stock

A

Increases a liability for the fair value of the options.

The liability will be extinguished when the option is exercised or when it expires.

913
Q

Renwood, Inc. contracted for services to be provided over a period of time in return for 2,000 shares of Renwood’s $5 par common stock when the service is completed. At the time, Renwood stock was selling for $10 per share. When the service was completed, Renwood’s stock price was $12 per share. Therefore, Renwood

A

Increases contributed capital in excess of par $10,000.

The total owners’ equity increase of $20,000 (2,000 shares × $10) is recorded at signing. Of that amount, the common stock account will receive $10,000 (2,000 shares × $5 par). Therefore, the remainder ($10,000) is allocated to contributed capital in excess of par. Subsequent changes in stock price do not change the total amount of OE recorded.

914
Q

List the two main owners’ equity categories.

A
  1. Earned

2. Contributed

915
Q

List the main types of ownerships in business

organizations.

A
  1. Soleproprietorship
  2. Partnership
  3. Corporation
916
Q

How is the number of shares outstanding

determined?

A

It is equal to the number of shares currently

held by stockholders.

917
Q

Define “dividends in arrears.”

A

Unpaid dividends for a particular year on

cumulative preferred stock

918
Q

Define “legal capital.”

A

The par value of the stock or the stated value of

the stock issued

919
Q

Define “authorized shares.”

A

The total number of shares that may be issued

920
Q

What is the primary measurement basis for

contributed capital?

A

The historical value of direct investments made

in the firm by investors

921
Q

List the major owners’ equity accounts for a

corporation.

A
Preferred stock
Common stock
Additional paid-in capital, preferred
Additional paid-in capital, common
Retained earnings
Treasury stock
Accumulated other comprehensive
income
922
Q

List the types of common stock rights.

A

Voting
Dividend
Preemptive

923
Q

What purpose does legal capital serve?

A

It establishes the minimum investment.
It provides protection for creditors.
(Dividends may not be paid from legal
capital.)

924
Q

What is the number of common shares issued?

A

Number of shares ever issued by the firm but
not retired = Number of outstanding shares +
Number of treasury shares

925
Q

What does owners’ equity represent?

A

It represents the residual interest in the net
assets of an entity that remains after deducting
its liabilities.

926
Q

List the types of preferred stock rights.

A

Nonvoting
Dividend preferences
Liquidation preferences

927
Q

How is the number of shares in the treasury

determined?

A

It is equal to the number of shares purchased

by the issuing firm and not yet reissued.

928
Q

Of the 125,000 shares of common stock issued by Vey Corp., 25,000 shares were held as treasury stock at December 31, 20X4. During 20X5, transactions involving Vey’s common stock were as follows:

January 1 through October 31 − 13,000 treasury shares were distributed to officers as part of a stock compensation plan.

November 1 - A 3-for-1 stock split took effect.

December 1 - Vey purchased 5,000 of its own shares to discourage an unfriendly takeover. These shares were not retired.

At December 31, 20X5, how many shares of Vey’s common stock were issued and outstanding?

A

Shares Issued - 375,000
Shares Outstanding - 334,000

Issued shares include outstanding shares and treasury shares. Treasury shares are issued but not outstanding. Stock splits are applied to all outstanding and treasury shares because a split reduces the par value of each share of issued stock, and increases the number of shares in inverse proportion.

Assume the stock has a par value of $3 before the split. After the 3-for-1 split in this question, the par value is $1, but the number of shares is tripled. Treasury shares must be adjusted for splits because treasury shares typically are reissued. No shares were retired.

Therefore, the number of issued shares at the end of the year is three times the number at the beginning of the year: 125,000(3) = 375,000 shares issued at December 31, 20X5.

Number of shares outstanding at December 31, 20X5 = (100,000 + 13,000)3 − 5,000 = 334,000.

Only 100,000 shares were outstanding at the beginning of the year because the 125,000 issued shares includes the 25,000 in the treasury. The split took effect after 13,000 of the 25,000 treasury shares at the beginning of the year were issued. The 5,000 treasury shares purchased at December 1 already reflect the split and are not adjusted further.

929
Q

Jones Co. had 50,000 shares of $5 par value common stock outstanding at January 1. On August 1, Jones declared a 5% stock dividend followed by a two-for-one stock split on September 1. What amount should Jones report as common shares outstanding at December 31?

A

105,000

A 5% stock dividend increases outstanding shares by 5%, and a 2-for-1 split doubles outstanding shares. The number of outstanding shares at year-end therefore is 105,000 = 50,000(1.05)(2). Each subsequent dividend or split compounds the previous change.

930
Q

Kamy Corp. is in liquidation under Chapter 7 of the Federal Bankruptcy Code. The bankruptcy trustee has established a new set of books for the bankruptcy estate. After assuming custody of the estate, the trustee discovered an unrecorded invoice of $1,000 for machinery repairs performed before the bankruptcy filing.

In addition, a truck with a carrying amount of $20,000 was sold for $12,000 cash. This truck was bought and paid for in the year before the bankruptcy.

What amount should be debited to estate equity as a result of these transactions?

A

$9,000

The debit to estate equity is the additional expense and loss that had not been previously recorded.

This amount is $9,000 ($1,000 repair cost + $8,000 loss on the truck). The $8,000 loss is the difference between the $20,000 carrying amount and the $12,000 proceeds.

931
Q

Rudd Corp. had 700,000 shares of common stock authorized and 300,000 shares outstanding at December 31, Year 1. The following events occurred during Year 2:

January 31	Declared 10% stock dividend
June 30	Purchased 100,000 shares
August 1	Reissued 50,000 shares
November 30	Declared 2-for-1 stock split
At December 31, Year 2, how many shares of common stock did Rudd have outstanding?
A

560,000

Stock dividends and splits increase the number of shares outstanding on the date of distribution by the percentage effect implied by the dividend (10%) or split (100% or multiply by 2). The number of shares outstanding at 12/31/Year 1 = [300,000(1.10) - 100,000 + 50,000]2 = 560,000.

Treasury shares reduce the number of shares outstanding, and reissuance increases the number of shares outstanding. The total number of shares outstanding just before the split (280,000) is doubled in a 2-for-1 split.

932
Q

An entity authorized 500,000 shares of common stock. At January 1, year 2, the entity had 110,000 shares of common stock issued and 100,000 shares of common stock outstanding. The entity had the following transactions in year 2:

March 1 Issued 15,000 shares of common stock
June 1 Resold 2,500 shares of treasury stock
September 1 Completed a 2-for-1 common stock split
What is the total number of shares of common stock that the entity has outstanding at the end of year 2?

A

235,000

The number of shares outstanding at the end of year 2 = (100,000 + 15,000 + 2,500)2 = 235,000. The beginning outstanding shares of 100,000 is augmented by the issuance of previously unissued stock, and by the reissuance of treasury stock. Stock splits are applied retroactively to all changes in outstanding shares occurring before the split. The split is a nonsubstantive change in shares. Each share after the split is worth half of one share before the split.

933
Q

Which of the following errors could result in an overstatement of both current assets and stockholders’ equity?

A

Holiday pay expense for administrative employees is misclassified as manufacturing overhead.

This error reduces expenses because part of the holiday pay will be held back in ending inventory, which is a current asset. Thus, net income, and therefore OE are overstated, as well as ending inventory, which is a current asset.

An understatement of accrued sales expenses. This error understates expenses and current liabilities. OE is overstated but current assets are unaffected.

Noncurrent note receivable principal is misclassified as a current asset. This error overstates current assets, but OE is unaffected.

Annual depreciation on manufacturing machinery is understated. This error overstates OE, but current assets are unaffected.

934
Q

At December 31, 20X5, Salo Corp.’s balance sheet accounts increased by the following amounts compared with those at the end of the prior year:

Assets	$178,000
Liabilities	54,000
Capital stock	120,000
Additional paid-in capital	12,000
The only charge to retained earnings during 20X5 was for a dividend payment of $26,000. Net income for 20X5 amounted to
A

$18,000

The change in total owners’ equity for the year is $124,000, which equals the difference between the increase in assets and liabilities for the year ($178,000 − $54,000). Contributed capital increased $132,000 ($120,000 + $12,000). The firm paid dividends of $26,000. Thus:

$132,000 + net income − $26,000 = $124,000

net income = $124,000 − $132,000 + $26,000

= $18,000

935
Q

Nest Co. issued 100,000 shares of common stock. Of these, 5,000 were held as treasury stock at December 31, 20X4. During 20X5, transactions involving Nest’s common stock were as follows:

May 3 - 1,000 shares of treasury stock were sold.
August 6 - 10,000 shares of previously unissued stock were sold.
November 18 - A 2-for-1 stock split took effect.
Laws in Nest’s state of incorporation protect treasury stock from dilution. At December 31, 20X5, how many shares of Nest’s common stock were issued and outstanding?

A

Shares Issued -220,000
Shares Outstanding - 212,000

Treasury shares are considered issued, but not outstanding. At December 31, 20X5:

Number of shares issued = [100,000 (beginning) + 10,000 (new issuance)]2 = 220,000.

Number of shares outstanding =

[95,000 (beginning) + 1,000 TS reissuance + 10,000 (new issuance)]2 = 212,000.

936
Q

List the requirements for stock sold on a

subscription basis.

A

Contract stating:
Specifying share price
Number of shares
Payment dates

937
Q

List the alternatives to par value when a stock

does not have such value.

A

Stated value

No par value

938
Q

How is stock issued for nonmonetary

consideration valued?

A

Fair value of stock or consideration, whichever

is more reliable

939
Q

What is the classification of the stock

subscriptions’ receivable account?

A

Contra owners’ equity (contra common stock

subscribed)

940
Q

What value is added to the contributed capital
account when a no par stock has a stated
value?

A

Contributed capital in excess of stated value

common

941
Q

Define “par value.”

A

The minimum legal issue price for capital stock
in most states; this value appears on the stock
certificate.

942
Q

Describe the journal entry to record initial

payment of stocks sold on subscriptions.

A

DR: Cash
DR: Subscription Receivable
CR: Common Stock Subscriptions
CR: Additional Paid in Capital (contract pricepar).

943
Q

What is the basis of allocation for stock basket

sale proceeds?

A

Fair value of individual stocks in the basket

944
Q

Describe the journal entry for subsequent

payments on stock sold on subscriptions.

A

DR: Cash

CR: Subscriptions Receivable

945
Q

On September 1, 20X4, Hyde Corp., a newly formed company, had the following stock issued and outstanding:

Common stock, no par, $1 stated value, 5,000 shares originally issued at $15 per share.
Preferred stock, $10 par value, 1,500 shares originally issued for $25 per share.

Hyde’s September 1, 20X4 statement of stockholders’ equity should report

A

Common stock - $5,000
Preferred stock - $15,000
Additional Paid-in capital - $92,500

Common stock: 5,000($1) = $5,000 (only par is recorded in common stock)

Preferred stock: 1,500($10) = $15,000 (only par is recorded in preferred stock)

Additional paid-in capital:

Common: 5,000($15 − $1) = $70,000

Preferred: 1,500($25 − $10) = 22,500

Total additional paid-in capital $92,500

946
Q

When collectability is reasonably assured, the excess of the subscription price over the stated value of the no par common stock subscribed should be recorded as

A

Additional paid-in capital when the subscription is recorded

This is one of the few examples of a recognized executory contract. Given that collectability is not an issue, the recording of a stock subscription is essentially the same as the entry for issuing stock for cash, except that a receivable stands in place of cash, and common stock subscribed stands in place of common stock.

Common stock subscribed is an owners’ equity account that is replaced by common stock upon issuance. Any additional paid-in capital is recorded when the contract is signed or recorded, just as if cash were received at that point.

947
Q

On July 1, 20X5, Cove Corp., a closely-held corporation, issued 6% bonds with a maturity value of $60,000, together with 1,000 shares of its $5 par value common stock, for a combined cash amount of $110,000.

The market value of Cove’s stock cannot be ascertained. If the bonds were issued separately, they would have sold for $40,000 on an 8% yield to maturity basis.

What amount should Cove report for additional paid-in capital on the issuance of the stock?

A

$65,000

The amount of the proceeds allocated to the stock is $70,000 ($110,000 − $40,000). When only one of the two securities has a known market value, that value is allocated to that security and the remaining proceeds are allocated to the security without a known market value.

The total par value of 1,000 shares of $5 par stock is $5,000. Therefore, $65,000 ($70,000 − $5,000) is recorded in additional paid-in capital on common stock.

948
Q

On March 1, 20X5, Rya Corp. issued 1,000 shares of its $20 par value common stock and 2,000 shares of its $20 par value convertible preferred stock for a total of $80,000.

At this date, Rya’s common stock was selling for $36 per share, and the convertible preferred stock was selling for $27 per share.

What amount of the proceeds should be allocated to Rya’s convertible preferred stock?

A

$48,000

The total proceeds are allocated to the two securities based on relative market values.

Market value of common: 1,000($36) = $36,000
Market value of preferred: 2,000($27) = 54,000
Total market value $90,000
Allocation of proceeds to preferred = ($54,000/$90,000)$80,000 = $48,000

949
Q

On April 1, 20X4, Hyde Corp., a newly formed company, had the following stock issued and outstanding:

Common stock, no par, $1 stated value, 20,000 shares originally issued for $30 per share.
Preferred stock, $10 par value, 6,000 shares originally issued for $50 per share.

Hyde’s April 1, 20X4 statement of stockholders’ equity should report

A

Common stock - $20,000
Preferred stock - $60,000
Additional paid-in capital - $820,000

Common stock: 20,000($1) = $20,000. Only par value is credited to common stock.

Preferred stock: 6,000($10) = $60,000. Only par value is credited to preferred stock.

Additional paid-in capital (the amount received on issuance in excess of par):

Common: 20,000($30 − $1) = $580,000

Preferred: 6,000($50 − $10) = 240,000

Total $820,000

950
Q

An individual contracts for the purchase of 200 shares of $10 par common stock at a subscription price of $15. After making payments totaling $1,200, the subscriber defaults. Shares are issued in proportion to the amount of cash paid by the investor. The summary journal entry to record the net effect of these two transactions includes:

A

Credit paid in capital in excess of par on common, $400

The net effect of the transactions is to receive cash of $1,200 and issue stock for that amount at $15/share; $1,200/$15 = 80 shares fully paid. Required net changes in balances are (1) common stock, 80($10) = $800, (2) PIC-CS, 80($15 − $10) = $400, (3) cash $1,200. The share purchase contract receivable account is opened and then closed for the same amount. There is no ending balance in that account.

951
Q

When preferred stock is called and retired, which account or aggregate category of accounts can be increased?

Total Owners’ Equity
Retained Earnings

A

Total Owners’ Equity - No
Retained Earnings - No

When a firm retires preferred stock, cash is paid to the shareholders reducing total owners’ equity. Retained earnings can never be increased when shares are retired, redeemed, or converted into another class of stock.

952
Q

500 shares of 6%, $100 par convertible preferred stock were issued at $103 per share. Each share is convertible into 20 shares of $5 par common stock. The journal entry to record conversion includes which of the following?

A

Cr. paid in capital in excess of par, common $1,500.

The contributed capital accounts for the preferred are closed and the total amount is transferred to the common stock accounts resulting from issuance upon conversion. The total par value of common stock is first credited to the common stock account. If there is a credit remainder, it is recorded in paid in capital in excess of par, common. If there is a debit remainder, retained earnings is debited. In this case, there is no remainder. The journal entry is:

DR: Preferred stock 500($100) 50,000
DR: PIC-preferred 500($103 − $100) 1,500
CR: Common stock 500(20)($5) 50,000
CR: PIC-common 1,500

953
Q

500 shares of 6%, $100 par callable preferred stock are called at $101. The shares were issued at $103 per share. The journal entry to record the retirement includes which of the following?

A

Cr. paid in capital from retirement of preferred stock, $1,000.

The $2 difference multiplied by 500 shares yields $1,000 paid in capital kept by the firm. The journal entry is:

DR: Preferred stock 500($100) 50,000
DR: PIC-preferred 500($103 − $100) 1,500
CR: PIC-retirement of preferred 1,000
CR: Cash 500($101) 50,500

954
Q

Cross Corp. had outstanding 2,000 shares of 11% preferred stock, $50 par. On August 8, 1992, Cross redeemed and retired 25% of these shares for $22,500. On that date, Cross’ additional paid-in capital from preferred stock totaled $30,000.

To record this transaction, Cross should debit (credit) its capital accounts as follows:

Preferred stock
Additional paid-in capital
Retained earnings

A

Preferred stock - $25,000
Additional paid-in capital - ($2500)
Retained earnings - 0

The journal entry for retirement:

Preferred stock 2,000(.25)($50) 25,000
Additional paid-in capital, preferred stock $30,000(.25) 7,500
Additional paid-in capital from retirement of preferred stock 10,000
Cash 22,500
The additional paid-in capital from retirement of preferred stock is the net difference between the other amounts in the entry.

When preferred stock is retired, the par value of the stock retired is removed from the preferred stock account, and the pro-rata share of the additional paid-in capital from original issuance is removed. If the total of these two amounts exceeds the amount paid to redeem and retire the stock, as is the case here, additional paid-in capital from retirement of preferred stock is credited. The net effect on additional paid-in capital is an increase of $2,500 ($10,000 − $7,500).

Retained earnings is unaffected.

955
Q

What is the accounting treatment for the

conversion of preferred stock?

A
Preferred stock accounts are transferred
to common stock accounts.
If total preferred stock value is less than
common stock par value, retained
earnings are debited.
956
Q

Under what condition is retained earnings
debited on conversion of preferred stock to
common stock?

A

Total recorded value of preferred stock is less

than par value of common stock on conversion.

957
Q

What is the accounting treatment for the

retirement of preferred stock?

A
All related owner's equity accounts are
removed
Debit differences go to retained earnings
Credit differences go to contributed
capital
958
Q

For what amount is Preferred Stock Additional
Paid-in Capital debited when called or
redeemed?

A

Amount recorded from original issuance.

959
Q

What is the effect of treasury stock transactions

on earnings?

A

There is no effect.

960
Q

Describe the accounting treatment of

purchases of stock under the par value method.

A
  1. Treasury stock is debited at par;
  2. Additional Paid in Capital (APIC) is
    debited by amount credited when stock
    was originally issued;
  3. Cash is credited.
961
Q

What is the effect on the treasury stock account
under the par method when donated stock is
received?

A

The effect on the treasury stock account is an

increase for the par value of the stock received.

962
Q

What accounts may reflect different balances
under the cost and par method for the same
firm?

A

Treasury stock
Paid-in capital in excess of par-common
Paid-in capital from treasury stock
Retained earnings

963
Q

What is the effect on the treasury stock account
under the cost method when donated stock is
received?

A

The effect on the treasury stock account is an

increase for the fair value of the stock received.

964
Q

How can retained earnings be affected by

treasury stock transactions?

A

Retained earnings can be decreased (as a last

resort) but never increased.

965
Q

Describe the cost method for accounting for

treasury stock.

A

Purchases are debited at cost
Reissuances debit cash, credit treasury
stock at cost, and contributed capital
from treasury stock is plugged.

966
Q

Describe the accounting treatment of
reissuance of stock under the par value
method.

A

Treasury stock is credited at par.
The remainder of the entry is treated like
a stock issuance.

967
Q

How is treasury stock presented on the balance

sheet under the par value method?

A

It is reported as a subtraction from the
common stock account, at par, in the balance
sheet.

968
Q

List the methods for accounting for treasury

stock.

A

Cost method

Par value method

969
Q

When is paid in capital from treasury stock

decreased under the cost method?

A

When treasury stock is reissued for less than

cost

970
Q

Under the cost method of accounting for
treasury stock, how is treasury stock presented
on the balance sheet?

A

Treasury stock is subtracted at the very bottom
of the owners’ equity section of the balance
sheet.

971
Q

When is paid in capital from treasury stock

increased under the par method?

A

When treasury stock is purchased for less than

the original issue price

972
Q

What is the relationship of total owners’ equity

for cost and par method?

A

They are equal, although certain components
of owners’ equity would show different
balances.

973
Q

What is the effect on owners’ equity when
treasury stock is purchased and subsequently
reissued at a price in excess of cost (using the
cost method)?

A

Owners’ equity is increased by the difference in

purchase cost and reissuance price.

974
Q

During the current year, Onal Co. purchased 10,000 shares of its own stock at $7 per share. The stock was originally issued at $6. The firm sold 5,000 of the treasury shares for $10 per share. The firm uses the cost method to account for treasury stock. What amount should Onal report in its income statement for these transactions?

A

$0

Income is not affected by treasury stock transactions. When a firm transacts with its owners acting as owners, it cannot profit or report a negative income. In this case, the $3 difference between the $10 reissue price of the treasury stock and its $7 cost is credited to an owners’ equity account as paid-in capital from treasury stock transactions. The firm’s net worth has increased as a result of its treasury stock purchase and reissuance, but the “gain” is not recognized as earnings.

975
Q

In 20X3, Fogg, Inc. issued $10 par value common stock for $25 per share. No other common stock transactions occurred until March 31, 20X5, when Fogg acquired some of the issued shares for $20 per share and retired them.

Which of the following statements correctly states an effect of this acquisition and retirement?

A

Additional paid-in capital is decreased

Additional paid-in capital is reduced when stock is retired because the additional paid-in capital from treasury stock transactions is closed.

The other three alternative answers cause income to change or retained earnings to increase. Retained earnings can never be increased through transactions with owners, nor can income be affected by such transactions.

976
Q

On incorporation, Dee Inc. issued common stock at a price in excess of its par value. No other stock transactions occurred except treasury stock was acquired for an amount exceeding this issue price.

If Dee uses the par value method of accounting for treasury stock appropriate for retired stock, what is the effect of the acquisition on the following?

Net common stock
Additional paid-in capital
Retained earnings

A

Net common stock - Decrease
Additional paid-in capital - Decrease
Retained earnings - Decrease

Under the par value method, when treasury stock is purchased and retired at a price exceeding the original issue price, the following entry is made. No additional paid-in capital from treasury stock transactions exists. Therefore, retained earnings is debited. Common stock par Additional paid-in capital original issue price - par Retained earnings acquisition price - original issuance price Cash acquisition price. Thus, all three accounts listed in the question are decreased.

977
Q

On December 31, 20X4, Pack Corp.’s Board of Directors canceled 50,000 shares of $2.50 par value common stock held in treasury at an average cost of $13 per share. Before recording the cancellation of the treasury stock, Pack had the following balances in its stockholders’ equity accounts:

Common stock	$540,000
Additional paid-in capital	750,000
Retained earnings	900,000
Treasury stock, at cost	650,000
In its balance sheet at December 31, 20X4, Pack should report common stock outstanding of
A

$415,000

When treasury stock is cancelled (retired), the common stock account is reduced by the par value of the common stock cancelled. The ending common stock account balance is $415,000 = $540,000 − 50,000($2.50).

978
Q

In 20X4, Seda Corp. acquired 6,000 shares of its $1 par value common stock at $36 per share. During 20X5, Seda issued 3,000 of these shares at $50 per share. Seda uses the cost method to account for its treasury stock transactions.

What accounts and amounts should Seda credit in 20X5 to record the issuance of the 3,000 shares?

Treasury stock
Additional paid-in capital
Retained earnings
Common stock

A

Treasury stock - $ 108,000
Additional paid-in capital - $ 42,000
Retained earnings - $ 0
Common stock - $ 0

Under the cost method, when treasury stock is reissued at a price in excess of its cost, additional paid-in capital from treasury stock is credited for the difference. Treasury stock is credited for cost. The journal entry is:

DR: Cash 3,000($50) 150,000
CR: Treasury stock 3,000($36) 108,000
CR: Additional paid-in capital
from treasury stock transactions 42,000

979
Q

At its date of incorporation, Glean, Inc. issued 100,000 shares of its $10 par common stock at $11 per share. During the current year, Glean acquired 30,000 shares of its common stock at a price of $16 per share and accounted for them by the cost method. Subsequently, these shares were reissued at a price of $12 per share.

There have been no other issuances or acquisitions of its own common stock.

What effect does the reissue of the stock have on the following accounts?

Retained earnings
Additional paid-in capital

A

Retained earnings - Decrease
Additional paid-in capital -No Effect

Under the cost method, additional paid-in capital from treasury stock transactions is credited when treasury stock is reissued at a price in excess of cost. This account is to be debited before retained earnings when the opposite occurs: reissue treasury stock at less than cost (as happened in the question).

However, only one treasury stock reissuance has occurred. Therefore, there is no additional paid-in capital from previous treasury stock transactions to draw on so the $4 difference between the purchase price and reissuance price is debited to retained earnings (a decrease). There is no effect on additional paid-in capital. The entry for reissuance is:

DR: Cash $12(30,000) 360,000
DR: Retained earnings 120,000
CR: Treasury stock $16(30,000) 480,000

980
Q

On December 1, 20X4, Line Corp. received a donation of 2,000 shares of its $5 par value common stock from a stockholder. On that date, the stock’s market value was $35 per share. The stock was originally issued for $25 per share.

By what amount would this donation cause total stockholders’ equity to decrease?

A

$0

The shares are considered donated treasury shares. Treasury stock and a gain or revenue account are increased by the market value of the stock received in donation (FAS 116). The increase in the treasury stock account decreases the owners’ equity, but the gain or revenue increases the owners’ equity by the same amount. Therefore, there is no net effect on the owners’ equity.

981
Q

List the journal entry to record payment of

property dividend.

A

DR: Dividends Payable

CR: Cash or Asset

982
Q

Define “scrip dividend.”

A

A dividend in the form of an interest bearing

note payable

983
Q

On what date are dividend liabilities

established?

A

Declaration date

984
Q

List the journal entry to record a scrip dividend

declaration.

A

DR: Retained earnings

CR: Scrip dividend payable

985
Q
List the journal entry to record a declaration of
property dividends (assuming asset fair value
exceeds book value).
A

DR: Retained Earnings (at FV at declaration
date) DR: Asset (FV – Book value) CR: Dividends
Payable (FV) CR: Gain on Disposal (FV – Book
value)

986
Q

List the journal entries to record cash dividend

declaration and payment.

A

DR: Retained Earnings (or Dividends)
CR: Dividends Payable
DR: Dividends Payable
CR: Cash

987
Q

Define “liquidating dividends.”

A

A return of capital rather than a return on
capital
Reduces contributed capital account
instead of retained earnings

988
Q

What types of dividends require establishing a

liability?

A

Those dividends involving a distribution of

assets

989
Q

List the entry to record a scrip dividend

payment.

A

DR: Scrip dividend payable
DR: Interest expense
CR: Cash

990
Q

A property dividend should be recorded in retained earnings at the property’s

A

Market value at date of declaration.

The date of declaration is the date on which the firm has made the commitment to pay the dividend. The market value on this date is the value that was considered when the board made the decision to distribute a property dividend and thus is the appropriate measure of the sacrifice to the firm.

991
Q

Bal Corp. declared a $25,000 cash dividend on May 8, 20X5, to stockholders of record on May 23, 20X5, payable on June 3, 20X5. As a result of this cash dividend, working capital

A

Decreased on May 8.

It is at declaration that a dividend has its effect on the value of the firm and on working capital. Retained earnings are decreased (or a holding account called Dividends, which is closed to retained earnings, may be recorded), and dividends payable are increased. Dividends payable are a current liability, causing working capital to decrease.

Working capital equals current assets, less current liabilities. The payment of a dividend does not affect working capital, because both cash and the dividend payable are reduced. Both current assets and current liabilities are reduced by the same amount.

992
Q

East Corp., a calendar-year company, had sufficient retained earnings in 20X3 as a basis for dividends, but was temporarily short of cash.

East declared a dividend of $100,000 on April 1, 20X3 and issued promissory notes to its stockholders in lieu of cash. The notes, which were dated April 1, 20X3, had a maturity date of March 31, 20X4 and an interest rate of 10%.

How should East account for the scrip dividend and related interest?

A

Debit retained earnings for $100,000 on April 1, 20X3, and debit interest expense for $7,500 on December 31, 20X3.

Retained earnings is reduced only by the amount of the dividend otherwise payable in cash, in this case $100,000. Interest on the notes is recognized as interest expense, not as a part of the dividend. 12/31/X3 is three-fourths of the way into the note term. Thus, .75(.10)($100,000) or $7,500 of interest expense should be recognized on this date.

993
Q

When a company declares a cash dividend, retained earnings is decreased by the amount of the dividend on the date of

A

Declaration.

A legal liability comes into existence at declaration. The firm has committed itself to paying resources to shareholders from retained earnings on that date.

994
Q

Ole Corp. declared and paid a liquidating dividend of $100,000. This distribution resulted in a decrease in Ole’s
Paid-in capital
Retained earnings

A

Paid-in capital - YES
Retained earnings - No

A liquidating dividend is a return of capital. Its source is not earnings, and, therefore, it is not retained earnings. The firm is liquidating part of its permanent capital. The usual account to debit for a liquidating dividend is additional paid-in capital.

995
Q

Mio Corp. was the sole stockholder of Plasti Corp.

On September 30, 20X4, Mio declared a property dividend of Plasti’s 2,000 outstanding shares of $1 par value common stock, distributable to Mio’s stockholders.

On that date, the book value of Plasti’s stock was $1.50 per share. Immediately after the distribution, the market value of Plasti’s stock was $4.50 per share.

What amount should Mio report in its 20X4 financial statements as gain on disposal of the Plasti stock?

A

$6,000

When a property dividend is distributed, any unrealized holding gain or loss on the property is first recognized. The distribution of the property is a disposal and thus calls for the recognition of any holding gain or loss in earnings.

The market value exceeded the carrying value of the investment in Plasti stock by $3.00 per share ($4.50 − $1.50) on the distribution date. This is the gain recognized per share of stock distributed. With 2,000 shares distributed, the total gain is $6,000 ($3.00 × 2,000).

The gain that is recognized on the distribution of the property dividend is the same as that which would have resulted had Mio first sold the Plasti stock for a $6,000 gain and then distributed the proceeds from the sale of the stock as the dividend. The net effect on shareholders’ equity would be the same either way.

996
Q

In 20X5, Elm Corp. bought 10,000 shares of Oil Corp. at a cost of $20,000. On January 15, 20X6, Elm declared a property dividend of the Oil stock to shareholders of record on February 1, 20X6, payable on February 15, 20X6. During 20X6, the Oil stock had the following market values:

January 15 $25,000
February 1 26,000
February 15 24,000
The net effect of the foregoing transactions on retained earnings during 20X6 should be a reduction of

A

$20,000

The property dividend is recorded at market value, with a debit of $25,000 to retained earnings at declaration. A gain of $5,000 on the securities is recognized as a gain on disposal (as if it were sold). The net effect is a decrease in retained earnings of $20,000.

997
Q

Godart Co. issued $4.5mn notes payable as a scrip dividend that matured in five years. At maturity, each shareholder of Godart’s 3mn shares will receive payment of the note principal, plus interest. The annual interest rate was 10%.

What amount should be paid to the stockholders at the end of the fifth year?

A

$6.75mn

Instead of paying $4.5mn in dividends at declaration, the firm decided to issue notes due in five years, calling for the principal amount ($4.5mn), plus five years of simple interest to be paid. The note does not call for compounding.

Therefore, the amount due at maturity is $4.5mn + (5 years)(.10)($4.5mn) = $6.75.

998
Q

At December 31, 20X4 and 20X5, Apex Co. had 3,000 shares of $100 par, 5% cumulative preferred stock outstanding. No dividends were in arrears as of December 31, 20X3. Apex did not declare a dividend during 20X4. During 20X5, Apex paid a cash dividend of $10,000 on its preferred stock.

Apex should report dividends in arrears in its 20X5 financial statements as a(an)

A

Disclosure of $20,000.

The annual preferred stock dividend is $15,000 (3,000 × $100 × 5%). Total dividends in arrears at the end of 2005 are therefore $20,000 (2 years × $15,000 − $10,000 paid). Dividends in arrears are footnoted only. They are not recognized as a liability until they are declared.

999
Q

How is a large stock dividend accounted for?

A

Capitalize retained earnings or paid-in capital

at par value.

1000
Q

How is a stock split accounted for?

A

No accounting entry is needed.

1001
Q

How is a small stock dividend accounted for?

A

Capitalize retained earnings at market price.

1002
Q

What is a small stock dividend?

A

Percentage of dividend is less than 20% to25%

1003
Q

What is a large stock dividend?

A

Percentage of dividend is greater than 20% to

25%

1004
Q

What is the date used to establish market price

for small stock dividends?

A

Declaration date

1005
Q

Define “stock dividend.”

A

A distribution by a firm of its stock to its
shareholders in proportion to their existing
holdings

1006
Q

The following stock dividends were declared and distributed by Sol Corp:

Percentage of common shares outstanding at declaration date Fair value Par value
10 $15,000 $10,000
28 40,000 30,800
What aggregate amount should be debited to retained earnings for these stock dividends?

A

$45,800

Small stock dividends (less than 25%) are capitalized at the fair value of stock issued and large stock dividends (greater than 25%) are capitalized at the par value of stock issued.

The total amount capitalized (debited) to retained earnings, therefore, is $15,000 for the 10% stock dividend (fair value), plus $30,800 for the 28% stock dividend (par value) for a total of $45,800.

1007
Q

The following format was used by Gee, Inc. for its 20X5 statement of owners’ equity:

                          Common   Additional    Retained
                        stock, $1     paid-in capital  earnings
Balance at 1/1/X5	$90,000	$800,000	$175,000
Additions and deductions:			
100% stock dividend			
5% stock dividend			
\_\_\_\_\_\_\_\_\_\_	\_\_\_\_\_\_\_\_\_\_	\_\_\_\_\_\_\_\_\_\_
Balance at 12/31/X5			
==========	==========	==========
When both the 100% and the 5% stock dividends were declared, Gee's common stock was selling for more than its $1 par value.

How would the 5% stock dividend affect the additional paid-in capital and retained earnings amounts reported in Gee’s 2005 statement of owners’ equity?

Additional paid-in capital
Retained earnings

A

Additional paid-in capital - Increase
Retained earnings - Decrease

The stock dividend is a “small” stock dividend because it is less than 20% - 25%. Small stock dividends are capitalized at market value, which exceeds par in this case. Retained earnings is reduced by the market value of the shares issued, common stock is increased by the par value of stock issued, and additional paid-in capital is increased by the difference between market value and par value times the number of shares issued.

1008
Q

Pugh Co. reported the following in its statement of stockholders’ equity on January 1, 20X4:

Common Stock, $5 par value, authorized 200,000 shares, issued 100,000 shares	$ 500,000
Additional paid-in capital	1,500,000
Retained earnings	516,000
2,516,000
Less treasury stock, at cost, 5,000 shares	40,000
Total stockholders' equity	$2,476,000
========
The following events occurred in 20X4:

May 1 − 1,000 shares of treasury stock were sold for $10,000.

July 9 − 10,000 shares of previously unissued common stock were sold for $12 per share.

October 1 - The distribution of a 2-for-1 stock split resulted in the common stock’s per share par value being halved.

Pugh accounts for treasury stock under the cost method. Laws in the state of Pugh’s incorporation protect shares held in treasury from dilution when stock dividends or stock splits are declared.

In Pugh’s December 31, 20X4 statement of stockholders’ equity, the par value of the issued common stock should be

A

$550,000

Treasury stock transactions do not affect shares issued, because treasury shares are included in issued shares. The only event during the year affecting the total par value of common stock issued is the July 9 issuance of shares that were not issued before.

The stock split does not change total par of shares issued, because par is cut in half and the number of shares is doubled. Treasury shares are protected, meaning they are also doubled and have their par cut in half. The total par of issued common stock at year-end is $550,000:

$500,000 from January 1

Plus $50,000 from July 9 issuance: 10,000 × $5

Equals $550,000 total par of issued shares.

1009
Q

A company whose stock is trading at $10 per share has 1,000 shares of $1 par common stock outstanding when the board of directors declares a 30% common stock dividend. Which of the following adjustments should be made when recording the stock dividend?

A

Retained earnings is debited for $300.

This is a large stock dividend (> 25%); therefore retained earnings is debited for par value. The amount is the par value of the shares distributed in the dividend, or 1,000(.30)($1) = $300. The credit is to common stock for the shares issued.

1010
Q

Wood Co. owns 2,000 shares of Arlo, Inc.’s 20,000 shares of $100 par, 6% cumulative, non-participating preferred stock and 1,000 shares (2%) of Arlo’s common stock.

During 20X5, Arlo declared and paid dividends of $240,000 on preferred stock. No dividends had been declared or paid during 20X4. In addition, Wood received a 5% common stock dividend from Arlo when the quoted market price of Arlo’s common stock was $10 per share.

What amount should Wood report as dividend income in its 20X5 income statement?

A

$24,000

The annual preferred dividend commitment is $120,000 (20,000 × $100 × .06).

The amount paid in 20X5 ($240,000) covers both 20X5, and 20X4 (dividends in arrears). Wood owns 10% of Arlo’s preferred stock and, therefore, received $24,000. This amount is recognized as revenue in 20X5.

Dividends in arrears are not recognized until received. The stock dividend is not treated as revenue, but rather reduces the cost per unit of Wood’s investment in Arlo’s common stock.

1011
Q

Long Co. had 100,000 shares of common stock issued and outstanding at January 1, 20X6. During 20X6, Long took the following actions:

March 15 - Declared a 2-for-1 stock split, when the fair value of the stock was $80 per share.

December 15 - Declared a $.50 per share cash dividend.

In Long’s statement of stockholders’ equity for 20X6, what amount should Long report as dividends?

A

$100,000

Total shares receiving a dividend equal 200,000, which is twice the number of shares at the beginning of the year.

The stock split doubled the number of shares outstanding. Therefore, the total cash dividend is $.50(200,000) = $100,000. This amount will appear in the statement of stockholders’ equity as dividends declared.

1012
Q

On May 18, 20X4, Sol Corp.’s board of directors declared a 10% stock dividend.

The market price of Sol’s 3,000 outstanding shares of $2 par value common stock was $9 per share on that date. The stock dividend was distributed on July 21, 20X4, when the stock’s market price was $10 per share.

What amount should Sol credit to additional paid-in capital for this stock dividend?

A

$2,100

This question is difficult, because authoritative sources and textbooks disagree as to the market value that should be used to value stock dividends. Small stock dividends (less than 25%) are measured at the market value of the stock issued. But the market value can be measured at declaration or at issuance.

The answer listed as correct uses the market value on the declaration date. The journal entry to record the dividend assuming the declaration and distribution occurred in the same fiscal period:

Dr. Retained earnings .10(3,000)($9)	2,700	
Cr.Common stock .10(3,000)($2)
600
Cr.Additional paid-in capital
2,100
1013
Q

The following format was used by Gee, Inc. for its 20X5 statement of owners’ equity:

                          Common   Additional    Retained                                
                       stock, $1 par paid-in cap  earnings \_\_\_\_\_\_\_\_\_\_\_	\_\_\_\_\_\_\_\_\_\_\_	\_\_\_\_\_\_\_\_\_\_\_ Balance at 1/1/X5	$90,000	$800,000	$175,000 Additions and deductions:			 100% stock dividend			 5% stock dividend			 \_\_\_\_\_\_\_\_\_\_	\_\_\_\_\_\_\_\_\_\_\_	\_\_\_\_\_\_\_\_\_\_\_ Balance at 12/31/X5			 ==========	==========	========== When both the 100% and the 5% stock dividends were declared, Gee's common stock was selling for more than its $1 par value.

How would the 100% stock dividend affect the additional paid-in capital and retained earnings amounts reported in Gee’s 20X5 statement of owners’ equity?

Additional paid-in capital
Retained earnings

A

Additional paid-in capital - No Change
Retained earnings - Decrease

A 100% stock dividend is a “large” stock dividend because it exceeds 20% - 25%. Large stock dividends are capitalized at par value. Retained earnings is reduced by the par value of the shares issued, and common stock is increased by the par value of stock issued. There is no effect on additional paid-in capital because the entire decrease in retained earnings is recorded in common stock. A large stock dividend permanently capitalizes the par value of the issued shares into common stock.

1014
Q

Is there any additional participation to
preferred stock if total dividends are not
sufficient to provide common stock with
dividends based on the fully participating
preferred percentage?

A

There is no additional participation.

1015
Q

Is there any additional participation to
preferred stock if total dividends are not
sufficient to provide the partially participating
preferred percentage to both common stock
and preferred stock?

A

The remainder of dividends after the basic
preferred percentage is provided to common, is
multiplied by the ratio: total preferred par
value to total par value of both classes of stock.

1016
Q

What dividends are paid before any other

dividends are paid?

A

Preferred stock dividends in arrears

1017
Q

List the order of dividend payment when
partially participating preferred stock is
outstanding.

A
  1. Preferred: Any dividends in arrears
  2. Preferred: Current-period dividends
  3. Common: Preferred percentage × Total
    par outstanding
  4. Preferred: Additional percentage
  5. Common: Remainder
1018
Q

List the order of dividend payment if
nonparticipating preferred stock is
outstanding.

A
  1. Preferred: Dividends in arrears (if
    cumulative)
  2. Preferred: Current-period dividend
  3. Common: Remainder
1019
Q

List the order of payment for partially

participating stocks.

A
  1. Dividends in arrears
  2. Current p/s dividend
  3. Preferred stock receives up to an
    additional percentage
1020
Q

Arp Corp.’s outstanding capital stock at December 15, 20X5 consisted of the following:

30,000 shares of 5% cumulative preferred stock, par value $10 per share, fully participating as to dividends. No dividends were in arrears.
200,000 shares of common stock, par value $1 per share.
On December 15, 20X5, Arp declared dividends of $100,000. What was the amount of dividends payable to Arp’s common stockholders?

A

$40,000

Participating dividends are allocated according to total par value after each class of stock receives the preferred dividend percentage.

Total par value of preferred stock outstanding is $300,000 (30,000 × $10), and total par value of common stock outstanding is $200,000 (200,000 × $1). Total par value of all stock outstanding is therefore $500,000. Preferred is three-fifths of total par value outstanding ($300,000/$500,000) and common is two-fifths ($200,000/$500,000) of total par value outstanding.

20X5 preferred requirement: 30,000(.05)($10) = $15,000
Preferred percentage to common: .05(200,000)($1) = 10,000 –> $10,000
Subtotal $25,000
Remaining dividends: $100,000 − $25,000 = $75,000
Participation to preferred, based on total par value:
$75,000(3/5) = $45,000
Participation to common, based on total par value:
$75,000(2/5) = 30,000 ———————> 30,000
Total common dividends $40,000

1021
Q

The owners’ equity section of a firm includes (1) $10,000 of 8%, $100 par cumulative preferred stock, and (2) $40,000 of $5 par common stock. There is additional paid-in capital on both issues. The preferred participates up to an additional 4% and there are two years of dividends in arrears as of the beginning of the current year. If the firm pays $7,100 in dividends, what amount is allocated to common?

A

$4,400

Preferred represents one-fifth of total par value outstanding ($10,000/$50,000).

To Preferred
Arrearage .08(2)($10,000) = $1,600
Current year .08($10,000) = 800

To Common
Matching .08($40,000) = $3,200

Dividends remaining = $7,100 − $1,600 − $800 − $3,200 = $1,500, which is insufficient to provide 4% participation to both classes of stock [.04($50,000) = $2,000]. The remainder is allocated according to total par value.

To Preferred
Participation 1/5($1,500) = 30
To Common
4/5($1,500) = 1,200

Total PS$2,700 CS $4,400

1022
Q

The owners’ equity section of a firm includes (1) $10,000 of 8%, $100 par cumulative preferred stock, and (2) $40,000 of $5 par common stock. There is additional paid-in capital on both issues. The preferred participates up to an additional 4% and there are two years of dividends in arrears as of the beginning of the current year. If the firm pays $30,000 in dividends, what amount is allocated to common?

A

$27,200

Preferred represents one-fifth of total par value outstanding ($10,000/$50,000).

To Preferred
Arrearage .08(2)($10,000) = $1,600
Current year .08($10,000) = 800

To Common
Matching .08($40,000) = $3,200

Dividends remaining = $30,000 − $1,600 − $800 − $3,200 = $24,400, which exceeds .04($50,000), so there is enough for both classes of stock to participate at 4%.

To Preferred
Participation .04($10,000 = 400

To Common
Remaining $24,400 − $400 = 24,000

Total PS$2,800 CS$27,200

1023
Q

The owners’ equity section of a firm includes (1) $10,000 of 8%, $100 par cumulative preferred stock, and (2) $40,000 of $5 par common stock. There is additional paid-in capital on both issues. The preferred is fully participating and there are two years of dividends in arrears as of the beginning of the current year. If the firm pays $30,000 in dividends, what amount is allocated to common?

A

$22,720

Preferred represents one-fifth of total par value outstanding ($10,000/$50,000).

To Preferred
Arrearage .08(2)($10,000) = $1,600
Current year .08($10,000) = 800

To Common
Matching .08($40,000) = $3,200

Dividends remaining = $30,000 − $1,600 − $800 − $3,200 = $24,400, which is allocated according to total par value.

To Preferred
Participation one-fifth($24,400) = 4,880

To Common
four-fifths($24,400) = 19,520

Total PS$7,280 CS $22,720

1024
Q

Describe the accounting entry to record stock
rights issued to outside parties for service at
issuance.

A

Record an expense and an owners’ equity
account equal to the difference between the
market price and exercise price times the
number of shares under option.

1025
Q

What is the effect of retained earnings

appropriations on assets?

A

Retained earnings appropriations have no

effect on assets.

1026
Q

Define “retained earnings appropriation.”

A

Management’s formal communication that a
portion of retained earnings has been declared
off-limits for dividends

1027
Q

What is often used to convey preemptive rights

regarding stocks?

A

Stock rights

1028
Q

Define “restrictions on retained earnings.”

A

External constraints placed on a certain portion

of retained earnings by an external party

1029
Q

List the reasons for appropriating retained

earnings.

A

Financial planning
Legal requirement
Contractual obligation

1030
Q

How should restrictions and appropriations on

retained earnings be reported?

A

They should be disclosed in footnotes.

1031
Q

Describe the accounting entry to record stock
rights issued to outside parties for services at
exercise of rights.

A

Record the stock issuance at the exercise price
and remove the OE account credited at
issuance of the rights.

1032
Q

Zinc Co.’s adjusted trial balance at December 31, 20X5 includes the following account balances:

Common stock, $3 par $600,000
Additional paid-in capital 800,000
Treasury stock, at cost 50,000
Net unrealized loss on available-for-sale debt securities 20,000
Retained earnings: appropriated for uninsured earthquake losses 150,000
Retained earnings: unappropriated 200,000

What amount should Zinc report as total stockholders’ equity in its December 31, 20X5 balance sheet?

A

$1,680,000

This response is correct because each item listed belongs in owners’ equity. The treasury stock and net unrealized loss on the AFS debt securities are negative items (debits), but the rest are positive items (credits).

Therefore, total owners’ equity is $1,680,000 = $600,000 + $800,000 − $50,000 − $20,000 + $150,000 + $200,000.

1033
Q

In September Year 1, West Corp. made a dividend distribution of one right for each of its 120,000 shares of outstanding common stock.

Each right was exercisable for the purchase of one-hundredth of a share of West’s $50 variable-rate preferred stock at an exercise price of $80 per share. On March 20, Year 5, none of the rights had been exercised, and West redeemed them by paying each stockholder $0.10 per right.

As a result of this redemption, West’s stockholders’ equity was reduced by

A

$12,000

The dividend did not affect total OE, because no resources were expended or received. The payment of $0.10 per right reduces OE by a total of 120,000($0.10) = $12,000, because this amount of cash was paid.

1034
Q

In September Year 1, Cal Corp. made a dividend distribution of one right for each of its 240,000 shares of outstanding common stock.

Each right was exercisable for the purchase of one-hundredth of a share of Cal’s $50 variable-rate preferred stock at an exercise price of $80 per share. On March 20, Year 6, none of the rights had been exercised, and Cal redeemed them by paying each stockholder $0.10 per right.

As a result of this redemption, Cal’s stockholders’ equity was reduced by

A

$24,000

None of the rights was exercised. The net effect on stockholders’ equity is the $24,000 of cash paid to the shareholders: 240,000($.10).

1035
Q

The following trial balance of Mint Corp. at December 31, 20X5, has been adjusted except for income tax expense.

TRIAL BALANCE

December 31, 20X5
Dr. Cr.
_______ ______
Dr. Cash $600,000
Dr.Accounts receivable, net $3.5mn
Dr.Cost in excess of billings on long-term contracts $1.6mn
Cr.Billings in excess of costs on long-term contracts $700,000
Dr.Prepaid taxes 450,000
Dr.Property, plant, and equipment, net 1.48mn
Cr.Note Payable - non-current 1.62mn
Cr.Common stock 750,000
Cr.Additional paid-in capital 2mn
Cr.Retained earnings - unappropriated 900,000
Cr.Retained earnings - restricted for note payable 160,000
Cr.Earnings from long-term contracts 6.68mn
Dr.Costs and expenses 5.18mn
____________ ____________
$12.81mn $12.81mn
=========== ===========
Other financial data for the year ended December 31, 20X5, are:

Mint uses the percentage-of-completion method to account for long-term construction contracts for financial statement and income tax purposes. All receivables on these contracts are considered to be collectible within 12 months.
During 20X5, estimated tax payments of $450,000 were charged to prepaid taxes. Mint has not recorded income tax expense. There were no temporary or permanent differences, and Mint’s tax rate is 30%.
In Mint’s December 31, 20X5 balance sheet, what amount should be reported as total retained earnings?

A

$2.11mn

The retained earnings balance in an adjusted trial balance does not reflect earnings for the year. Retained earnings are not adjusted for net income until the accounts are closed, which occurs after the trial balance is prepared. Therefore, the retained earnings accounts listed in the trial balance do not reflect current earnings.

Retained earnings per trial balance ($900,000 + $160,000) $1.06mn
Plus earnings, less costs and expenses $6.68mn − $5.18mn − $450,000 (tax) 1.05mn
Total ending retained earnings $2,110,000
The trial balance does not reflect the income tax expense. This amount was listed as prepaid taxes, but must be reclassified to income tax expense. When firms make estimated tax payments, the debit should be to income tax expense. Income tax has not been recorded. The estimated payments approximately cover the tax liability for the year that has ended. Therefore, the taxes cannot be said to be prepaid.

1036
Q

Cricket Corp. issued, without consideration, rights allowing stockholders to subscribe for additional shares at an amount greater than par value, but less than both market and book values.

When the rights are exercised, how are the following accounts affected?

Retained earnings
Additional paid-in capital

A

Retained earnings - Not Affected
Additional paid-in capital - Increased

The exercise is treated as would be any issuance of stock. The exercise price determines the total proceeds, the common stock account is credited for the total par of stock issued, and additional paid-in capital is credited for the remainder. The exercise price exceeds par value, so additional paid-in capital is increased. Retained earnings are unaffected.

1037
Q

On November 2, 20X3, Finsbury, Inc. issued warrants to its stockholders, giving them the right to purchase additional $20 par value common shares at a price of $30.

The stockholders exercised all warrants on March 1, 20X4. The shares had market prices of $33, $35, and $40 on November 2, 20X3, December 31, 20X3, and March 1, 20X4, respectively.

What were the effects of the warrants on Finsbury’s additional paid-in capital and net income?

Additional paid-in capital
Net income

A

Additional paid-in capital - Increased in 20X4
Net income -No effect

The issuance of warrants to shareholders does not require a journal entry, because no resources are expended or received. Therefore, in 20X3, there is no effect on owners’ equity.

When the warrants are exercised in 20X4, the shareholders pay $30 per share for the stock purchased under the warrants. This issuance is recorded as a normal issuance at $30, even though that is not the market price at the date of issuance. Additional paid-in capital is increased by $10, the difference between the $30 exercise price and $20 par. Issuing warrants or stock has no effect on earnings. A firm does not profit on transactions with owners.

1038
Q

A retained earnings appropriation can be used to

A

Restrict earnings available for dividends.

The purpose of appropriations is to restrict dividends and communicate that restriction to users of the financial statements. It is merely a partitioning of retained earnings into two parts: (1) available for dividends, and (2) unavailable.

A firm need not record an appropriation in order to restrict dividends. However, it helps alert stockholders to the possibility that dividends may be curtailed, and informs them of the reason for that curtailment.

1039
Q

The following trial balance of Trey Co. at December 31, 20X5 has been adjusted, except for income tax expense.

Dr.	Cr.
\_\_\_\_\_\_\_\_\_\_	\_\_\_\_\_\_\_\_\_\_
Dr.Cash	$550,000	
Dr.Accounts receivable, net	1,650,000	
Dr.Prepaid taxes	300,000	
Cr.Accounts payable		$ 120,000
Cr.Common stock		500,000
Cr.Additional paid-in capital		680,000
Cr.Retained earnings		630,000
Dr.Foreign currency translation adjustment	430,000	
Cr.Revenues		3.6mn
Dr.Expenses	2.6mn	
\_\_\_\_\_\_\_\_\_\_	\_\_\_\_\_\_\_\_\_\_
$5.53mn	$5.53mn
==========	==========
Additional information:

During 20X5, estimated tax payments of $300,000 were charged to prepaid taxes. Trey has not yet recorded income tax expense. There were no differences between financial statement and income tax income, and Trey’s tax rate is 30%.
Included in accounts receivable is $500,000 due from a customer. Special terms granted to this customer require payment in equal semi-annual installments of $125,000 every April 1 and October 1.
In Trey’s December 31, 20X5 balance sheet, what amount should be reported as total retained earnings?

A

$1.33mn

Retained earnings at December 31, 20X5 would be the sum of the beginning retained earnings of $630,000 and the net income. Net income is revenue ($3.6mn) - expense ($2.6mn) or $1mn.

However, this did not include taxes. Taxes on $1mn (at the 30% tax rate) are $300,000. This is exactly the same as the pre-paid taxes, so no additional liability exists. The pre-paid just need to be transferred to an expense category.

This makes income $700,000 and ending retained earnings $1.33mn. No adjustment need be made for the accounts receivable, unless Trey is handling this transaction differently for tax (installment method) than for book purposes, which is contrary to the facts presented in the problem.

1040
Q

Define “common stockholders’ equity.”

A

Total owner’s equity after preferred dividend

claims are removed

1041
Q

Define “book value per share.”

A

Common stockholders’ equity per share of
outstanding common stock at the end of the
period

1042
Q

What is the effect of dividends in arrears on

book value per share?

A

Decrease in book value per share.

1043
Q

What is the effect on book value per share of a

transaction that increases earnings?

A

Increase in book value per share

1044
Q

Hoyt Corp.’s current balance sheet reports the following stockholders’ equity:

5% cumulative preferred stock, par value $100 per share; 2,500 shares issued and outstanding $250,000
Common stock, par value $3.50 per share; 100,000 shares issued and outstanding 350,000
Additional paid-in capital in excess of par value of common stock 125,000
Retained earnings 300,000
Dividends in arrears on the preferred stock amount to $25,000. If Hoyt were to be liquidated, the preferred stockholders would receive par value plus a premium of $50,000. The book value of common stock is

A

$7.00

Book value per share of common stock is the portion of owners’ equity that would remain for common shareholders after the preferred claim was paid, divided by the number of common shares outstanding.

The preferred dividend claim includes the liquidation preference ($50,000 premium above par, plus par value) and the $25,000 dividends in arrears. All preferred stock dividends in arrears must be paid on liquidation before common shareholders receive any cash.

Total owners’ equity is $1.025mn, the sum of the four items taken from the owners’ equity section of the balance sheet ($250,000 + $350,000 + $125,000 + $300,000).

Book value per share of common stock = $7.00 = ($1.025 − $250,000 − $50,000 premium − $25,000 dividends in arrears)/100,000.

1045
Q

When a company goes through a quasi-reorganization, its balance sheet carrying amounts are stated at:

A

Fair value.

Assets and liabilities are revalued to market or fair value to provide a fresh-start valuation.

Usually, there are overvalued assets that have contributed to the operating losses and resulting retained-earnings deficit. The write-downs of overvalued assets further increase the retained-earnings deficit, but allow a new beginning for the firm. The contributed capital of the firm is reduced, permanently recording the losses, and retained earnings are increased to a zero balance.

1046
Q

Grid Corp. acquired some of its own common shares at a price greater than both their par value and original issue price, but less than their book value. Grid uses the cost method of accounting for treasury stock.

What is the impact of this acquisition on total stockholders’ equity and the book value per common share?

Total stockholders’ equity
Book value per share

A

Total stockholders’ equity - Decrease
Book value per share - Increase

The purchase of treasury stock at any price decreases total owners’ equity under the cost method because treasury stock is a contra OE account. When the purchase price per share is less than book value per share, then the denominator decreases by a greater percentage than does the numerator, and book value per share increases.

For example, assume for simplicity that there is only common stock outstanding. Total owners’ equity and number of shares before the treasury stock purchase is $40,000 and 4,000, respectively. Book value per share then is $10. The firm purchases 200 shares of treasury stock for $8 (less than book value). The new book value per share is:

($40,000 − $1,600)/(3,800) = $10.11. Book value per share has increased.

1047
Q

Boe Corp.’s stockholders’ equity at December 31, 20X4 was as follows:

6% non-cumulative preferred stock, $100 par (liquidation value $105 per share) $100,000
Common stock, $10 par 300,000
Retained earnings 95,000
At December 31, 20X4, Boe’s book value per common share was

A

$13.00

Book value per share of common stock is common stockholders’ equity per share of common stock. The portion of owners’ equity allocated to preferred stock for this ratio is measured as the liquidation value per share. Therefore, the book value per share equals the net assets of the corporation per share that would be distributed to common shareholders on liquidation of the company, if the market value equaled book value for all assets and liabilities. The preferred shareholders would be paid the liquidation value per share first.

This firm has 1,000 shares of preferred stock outstanding: $100,000/$100 par; and 30,000 shares of common stock ($300,000/$10 par). It also has total owners’ equity of $495,000 ($100,000 + $300,000 + $95,000).

For this firm, book value per share is $13.00 = [$495,000 − 1,000($105)]/30,000. Had there been dividends in arrears, they would also be subtracted from total owners’ equity in the numerator. However, the preferred stock is non-cumulative, so there could be no dividends in arrears.

1048
Q

The stockholders’ equity section of Brown Co.’s December 31, 20X5 balance sheet consisted of the following:

Common stock, $30 par, 10,000 shares authorized and outstanding $300,000
Additional paid-in capital 150,000
Retained earnings (deficit) (210,000)
On January 2, 20X6, Brown put into effect a stockholder-approved quasi-reorganization by reducing the par value of the stock to $5 and eliminating the deficit against additional paid-in capital. Immediately after the quasi-reorganization, what amount should Brown report as additional paid-in capital?

A

$190,000

Reducing the par value to $5 creates $250,000 of additional paid-in capital: ($30 − $5)10,000 shares = $250,000. The common stock account is now $50,000: ($5)10,000.

Additional paid-in capital now stands at $400,000 ($150,000 + $250,000).

After absorbing the deficit in retained earnings, $190,000 remains in additional paid-in capital: ($400,000 − $210,000). Retained earnings are now zero.